Vous êtes sur la page 1sur 638

Number Systems

Module 1
Algebra
Since the dawn of civilization, man tried to have a count of his belongingsgoods, stones, animals, trees, etc. Whenever the animals were taken out of the enclosure for grazing, for each animal taken out, a scratch was put on the ground/stone. Thus, the number of animals taken out for grazing were equal to the number of scratches. (This is analogous to tally marks used now a days). On return, for every one animal returning to the enclosure a scratch was erased. In this way, even without knowing the counting, they used to save their belongings. Slowly the civilizations advanced and first came the counting numbers (natural numbers). You will be happy and proud to know that the present system of numeration, including numerals 0, 1, 2, 3, ..., 9 and place value system were the discoveries of our ancient Indians to the world. From India, these numbers reached the reign of Arabian king Al-mansur whose wise man Al-khowarizmi, translated the works of Indian scholars and mathematicians. From Arabia, the numerals reached the western world. Therefore, these are called Hindu-Arabic Numerals. You know that Algebra is generalised form of Arithmetic in which variables are used for numbers. Aryabhatt (476 AD) and Brahmgupta (578 AD) were the first Indian Mathematicians who used variables for numbers and called them Yavat-Tawat. They illustrated the sum, difference, product and division of expressions using variables and even found their squares, cubes, square-roots, cube-roots. Aryabhatt and Brahmgupta worked on solving linear, quadratic and indeterminate equations also. They called the method of solving indeterminate equations as Chakrawal and gave Avyakat Ganit to algebra. Bhaskaracharya and Mahaviracharya also contributed a lot to this, especially ratio and proportion and extended the works of previous Mathematicians on equations and indices and surds. The name Beejganit was given to Algebra by Bhaskaracharya. The credit of calling this as Algebra goes to Al-khowarizmi, the wise man of Al-Mansur. In this module, we shall study about number system, polynomials, factorisation of algebraic expressions, simplificaiton of rational expressions, solving linear and quadratic equations, indices and surds, Arithmetic and Geometric Progressions.

Mathematics

1
Number Systems
1.1 INTRODUCTION One of the greatest inventions in the history of civilization is the creation of NUMBERS. You can imagine the state of confusion, if you did not know about natural numbers or counting numbers. The introduction of those numbers enabled us to answer the question How many? You may recall your familiarity with the concepts of natural numbers, whole numbers, integers, fractions and rational numbers. These helped us to count, to know about the number zero representing nothingness, parts of a whole, describe opposites like profit and loss, rise and fall, going towards east and west etc. But one could have easily got along without the introduction of these numbers except the natural numbers. However, the working out of problems became easier with the introduction of numbers upto rational numbers. All these numbers mentioned above are rational numbers. Recall that a natural number is a rational number, a whole number is a rational number, a fraction is also a rational number and so also is an integer. At this stage, it is pertinent to ask ourselves a question. Are all numbers rational numbers ? Is it possible to solve all mathematical and life problems with the help of rational numbers? The answer to this question is an emphatic NO. For example, given a unit of length, we are not in a position to exactly measure all distances if we have with us rational numbers only. Likewise, it is not possible to find out the square root of an arbitrary natural number as a rational number, as for example, you know that 2 is not a rational number. To overcome such difficulties, it is essential to extend to the system of real numbers. In this lesson, you will be able to recall all that you may be knowing about rational numbers but also extend the same to the system of real numbers. This is a big leap forward in the study of Mathematics. 1.2 OBJECTIVES After studying this lesson, the learner will be able to :
z

illustrate the extension of system of numbers from natural numbers to real (rational and irrational) numbers. identify different types of numbers. express an integer as a rational number

z z

Number Systems

express a rational number as a terminating or non terminating but recurring decimal and vice-versa. find a rational number between two given numbers. represent a rational number on the number line cite examples of irrational numbers represent 2 , 3 , 5 on the number line

z z z z z z z

find an irrational number between two given numbers. round off a rational or irrational number to a given number of decimal places. perform the four fundamental operations of Arithmetic i.e., addition, subtraction, multiplication and division on real numbers.

1.3 EXPECTED BACKGROUND KNOWLEDGE


z z z

Concept of Natural numbers, whole numbers, fractions, integers and rational numbers. Prime and composite numbers and co-prime numbers. HCF and LCM of two or more natural numbers.

1.4 RATIONAL NUMBERS You may recall your familiarity with rational numbers. For example 1 , 2 , 3 , 1 , 4 , 4 , 0 , 9 , 22 , ... 2 5 7 4 4 are all rational numbers. 1, 2, 3, 4, ... are counting numbers or Natural Numbers. With the help of these you are able to answer the question How many ? If we introduce another number zero into the system of natural numbers, we get the system of Whole Numbers as 0, 1, 2, 3, 4, ... Notice that 0 is not a natural number but it is a whole number. All natural numbers are whole numbers as well. By introducing into the system of whole numbers the opposite (negatives) of natural numbers, we get the numbers ... 4, 3, 2, 1, 0, 1, 2, 3, 4, 5, ... called integers. Similarly, the numbers 1 , 2 , 6 , 5 , ... 2 3 4 7

Mathematics

are called fractions. You may have a look at some numbers like 2 , 5 , 6 , ... 3 8 21 These numbers do not fall into the categories mentioned above. If we include these numbers as well in our system of numbers, we get what is known as the system of Rational Numbers. Thus,

A number of the form numbers.

p where p and q are integers and q 0 is called a rational q

Here p is called the numerator and q the denominator of the rational number. You may notice that all natural numbers, whole numbers, fractions, integers are rational numbers but a rational number may not be a natural number, a whole number, a fraction or 11 an integer. For example, the number is not a natural number. It is not a whole number. 7 11 It is neither a fraction nor an integer. But certainly is rational number. Can you justify 7 11 p this statement ? Surely, you may see that is a number of the form where p = 11, q 7 q = 7 and q 0. A rational number whose numerator and denominator are both positive integers or both negative integers is a positive rational number. Similarly, if in a rational number, one out of its numerator and the denominator is positive and the other is negative is a negative rational number. Thus, for examples 2 , 7 are positive rational numbers 3 11 and 2 , 11 are negative rational numbers. 7 7
0 is the zero rational number. 1

Notice that 0 =

It may be noted that


2 4 6 2 = = = , and 3 6 9 3 33 11 11 22 = = = 7 14 7 21

Number Systems

14.1 Rational Number in its Lowest Terms Consider the rational number
24 . Here its numerator and denominator are 24 and 36. But these 36 two natural numbers have a Highest Common Factor, namely 12 and so the number could as well as be written as 2 12 2 or simply 3 12 3 12 2 is said to be rational number in the lowest terms. Similarly, the rational number , could 48 3 easily be rewritten as 1 12 1 = 4 12 4

The numerator and denominator of a rational number in lowest terms are co-prime numbers. Note : The number
12 12 is as well written as . 48 48

CHECK YOUR PROGRESS 1.1 1. From the following numbers, pick out (a) natural numbers (b) integers which are not natural numbers (c) whole numbers which are not natural numbers (d) fractions which are not natural numbers (e) numbers which are neither whole numbers, nor integers not fractions 1 , 7 , 3 , 7 , 4 , 0 , 23 , 2 , 15 , 11 . 8 3 7 3 6 2. Express the following integers as rational numbers : (i) 4 (ii) 23 (iii) 0
21 6

(iv) 1

(v) 1

3. Express the following rational numbers in their lowest terms : (i)


15 36

(ii)

44 14

(iii)

(iv)

8 12

1.5 DECIMAL REPRESENTATION OF A RATIONAL NUMBER You may be familiar with the process of representing a rational number in the form of a decimal. We illustrate this process with the help of a few examples.

Mathematics

Example 1.1 : Represent the numbers

1 3 , , in the decimal form. 4 5

Solution : By actual division, we know that (i)


1 = 0.25 4

4) 1.0 (0.25 8 20 20 0 5) 3.0 (0.6 30 0

(ii)

3 = 0.6 5

In the cases above, we see that in the process of repeated division by the denominator, we end up after a finite number of steps, when we get the remainder as zero. But this is not always so. To illustrate this we consider another example. Example 1.2 : Express the following rational numbers (i)
2 3

(ii)

6 7

in the decimal form. Solution : (i)


2 = 0.6666 ..... 3

Please note that in this case, the process of repeated division never comes to an end. In fact at every step, we get the remainder 2. (ii)
6 = 0.857142 857142 ..... 7

In this example, we find that the remainder at each step keeps changing till we arrive at a stage that the remainder repeats and then on further division the quotient will start repeating. In examples, 1.1 and 1.2 above, we see that either the remainder will be zero after a finite number of steps or it will start repeating after a finite number of steps. In the first case we say that the decimal is terminating and in the other we say that the decimal is non-terminating but repeating. In fact we have the following important statement.

3) 2.0 (0.66... 18 20 18 20 18 2 7) 6.0 (0.857142 56 40 35 50 49 10 7 30 28 20 14 6

Number Systems

A rational number is either a terminating decimal or a non-terminating but recurring (repeating) decimal. In the following examples, we try to represent a decimal (terminating or non-terminating but repeating decimal) in the form

p . q

p Examples 1.3 : Express (i) 0.72, (ii) 0.125 in the form q .


Solution : (i) (ii) 0.72 = 0.125 = 72 = 18 100 25 125 = 1 1000 8

Example 1.4 : Express (i) 0.33....., (ii) 0.234 234 .... in the form Solution : (i) You may write a = 0.33 .... Multiply both sides by 10 to get 10a = 3.33 .... Subtracting (1) from (2), we get 9a = 3.33.... 0.33.... =3 Thus, (ii) Let a= 0.33 = 1 3 1 3

p . q

...(1) ...(2)

b = 0.234234....

...(1)

Multiplying both sides by 1000 to get 1000 b = 234.234.... Subtracting (1) from (2), we get 999 b = 234 b=
234 26 = . 999 111

...(2)

The examples help us to assign at a result which can be stated in the following form.

Mathematics

A terminating decimal or a non-terminating but recurring decimal is a rational number. Note : Recurring decimals such as 0.66..., 0.234234..., 3.142857142857... are also written as 0.6 , 0. 234 , 3.142857 . In fact a digit or a group of digits which repeat are put below a bar to indicate that these repeat again and again. CHECK YOUR PROGRESS 1.2 1. Represent the following rational numbers in the decimal form : (i)
21 40

(ii)

16 25

(iii)

13 8

(iv)

15 6

(v)

91 35

2. Represent the following rational numbers in the decimal form : (i)


5 7

(ii)

5 6

(iii)

15 11

(iv)

27 13

3. Represent the following decimals in the form (i) 2.3 (ii) 7.12

p : q
(iv) 8.146

(iii) 0.315

p 4. Represent the following decimals in q form :


(i) 0.333... (ii) 3.12 (iii) 0.315315...

1.6 RATIONAL NUMBERS BETWEEN ANY TWO RATIONAL NUMBERS Given two rational numbers, can you find a rational number between them ? To answer this question, we consider the following examples : Example 1.5 : Find a rational number between two rational numbers 1 3 + Solution : Consider the number 2 4 2
1 3 and . 2 4

Number Systems

2+3 5 You may simplify this to get 4 i.e., 8 2 Surely, 5>1 3 5 5 1 3 5 and > . You may verify this by computing and which come 8 2 4 8 8 2 4 8 5 is a rational number such that 8

out to be both positive numbers, Thus,


1 5 3 < < 2 8 4

Think for a while, what we have done. We added the two numbers and divided their sum by 2. The resulting number is between the two given numbers. It will be greater than the smaller of the two numbers and less than the larger number. Let us now consider the following question : How many rational numbers lie between two given rational numbers ? Can you guess the answer ? In fact you can find as many rational numbers between two given numbers as you like. Can you think of as to how you may find many rational numbers lying between two rational numbers ? Example 1.6 : Find a rational number between 0.12 and 0.13. Solution : Consider the number
012 . + 013 . 2

= 0.125 It is a number greater than 0.12 and less than 0.13. Can you find some other rational numbers between 0.12 and 0.13 ? How many such like numbers you can find ? These are some of interesting aspects of the study of rational numbers. CHECK YOUR PROGRESS 1.3 1. Find a rational number between the following rational numbers (i) 7 8 and 8 7 (ii) 2 and 3 3 1 (iii) , 4 3

10

Mathematics

2. Find two rational numbers between the following rational numbers : 2 1 (i) and 3 2 (i) 0.23 and 0.24 2 1 (ii) and 3 4 (ii) 7.31 and 7.32

3. Find two rational numbers between the following rational numbers

1.7 THE NUMBER LINE Rational numbers can be represented on a line which is known as the number line in the following manner. Consider a line as shown in Fig 1.1. We fix a point O on its line. We choose a convenient unit of length and mark points on the line on both sides of O at fixed distances equal to the unit of length chosen. These points represent the number 0, 1, 2, 3, 4, .... 1, 2, 3, 4, ..... 7 on the line. We as shown in the Fig 1.1. Suppose now we have to represent the number 4 divide the distance between 1 and 2 into four equal parts and mark the point after three equal parts as shown in the figure.

Fig. 1.1

In this way we can represent any positive rational numbers by a point on the number line to the right of the point O. Similarly if we were to represent the number
1 on the number line, we divide the line segment 2

between 1 and 0 into two equal parts and mark the point

1 as shown in the Fig 1.1. In 2 this manner you may see that any given rational number can be represented by a point on the number line. You may also note that the number 0 is represented by the point O on the number line.

Example 1.7 : Represent the number 1.2 on the number line. Solution :

Fig. 1.2

Consider the number line with points on it marked as 2, 1, 0, 1, 2 as shown in Fig. 1.2 above. Divide the line segment between 1 and 2 into 10 equal parts. Mark a point P as 1.2, as is shown

Number Systems

11

in the Fig, 1.2, 2 steps ahead of the point representing the number 1. P is the point representing the number 1.2 on the number line. You may note that any rational number whether in the form be represented by a point on the number line. CHECK YOUR PROGRESS 1.4 1. Represent the following numbers by points on the number line (i)
3 2

p or in the decimal form can q

(ii)

1 3

(iii) 1.5

(iv) 1.3

2. Find the numbers corresponding to the points O, P, Q and R on the number line as shown in the Fig. 1.3.

Fig. 1.3

1.8 IRRATIONAL NUMBERS In section 1.5, you have seen that when a rational number is represented as a decimal, then either this decimal is terminating or it is a non-terminating but repeating decimal. The question that arises is this. Are there decimals which are neither terminating nor non-terminating but repeating decimals ? The answer to this question is Yes. Consider for example the decimal 0.101001000100001 ... This decimal has been written in such a manner that it has a definite pattern and so you can keep on writing it indefinitely. But this pattern is such that no block of digits repeats again and again. It is an example of a non-terminating and non-repeating decimal. Similarly, you may consider the decimal 0.123456789 10 11 12 13 ... Can you write the next nine digits in the decimal ? Note that, all that has been done is that we have written successively all natural numbers in the ascending order. The next six digits will be 14 15 16. Note once again you can continue this process endlessly. Thus, there is a definite pattern to write the digits but no block of digits is repeating again and again. This is another example of a non-terminating and non-repeating decimal.

12

Mathematics

The two examples we have discussed above indicate that there are decimals which are not rational numbers. Why ? We conclude that if we have to possess in our fold all decimals, namely. (i) terminating decimals (ii) non-terminating but repeating decimals (iii) non-terminating and non-repeating decimals then the system of rational numbers is not adequate and so we must extend this system to include numbers which are not rational i.e., irrational numbers.

p Note that the name rational is derived from the word 'ratio', as a number q , q 0 is the ratio
of two integers p and q. Also number which is not a ratio of integers p and q, q 0 is named as an irrational number. 1.8.1 Inadequacy of rational numbers You may recall that we would not have been able to answer the question 'how many' ?, if we did not know counting numbers or natural numbers. Let us now try to examine what we shall not be able to do if we have in our possession the rational numbers only. We put to ourselves the following question. Can we measure the length of any given line segment in terms of a prescribed unit of length, with the help of rational numbers ? The answer to this question is an emphatic No, as is clear from the following example. Consider a square ABCD of each side of unit length. The diagonal AC of this square has its length 2 units. You also know that 2 is not a rational number as there is no rational number whose square is 2.

Fig. 1.4

We conclude that we cannot exactly measure the length of the line segment AC in terms of rational numbers, if the unit of length given to us is AB. It is this inadequacy which necessitates the extension of the system of rational numbers to a system which includes in it rational as well as irrational numbers. There is yet another point which necessitates the extension of rational numbers. We discuss the same in the following paragraph. Recall that you have seen in section 1.7, that to each rational number these corresponds a unique point on the number line. Let us now consider the converse of it. Given a point on the number line, will it always correspond to a rational number ? The answer to this question is also a 'NO', as is clear from the following example.

Number Systems

13

On the number line mark the points corresponding to the number 0 and 1, as shown in Fig 1.5.

Fig. 1.5

Construct a square on the line segment. The diagonal of the square has length as centre and radius OA, draw an arc cutting the number line in P. Then OP =

2 . With O 2 . Thus, the

point P on the number line corresponds to the number 2 which is irrational. Thus, there is a point namely P, on the number line which does not correspond to a rational number. How many such points are those on the line which do not correspond to rational numbers ? In fact there are many such points and their number is infinite i.e. there are infinitely many points on the number line which do not correspond to rational numbers. In fact, if P corresponds to the number

2 as in Fig 1.6, we construct a rectangle with one

side OP and the other side 1. The diagonal of the rectangle will be OQ = 3 . With O as centre and OQ as radius draw an arc cutting the number line in the point R. Then the point R corresponds to the irrational number

3.

Fig. 1.6

Similarly, by constructing the rectangle with sides 2 and 1 as shown in Fig 1.7 and with O as centre and diagonal OA = corresponds to

5 , draw an arc cutting the number line in P. This point P

5 , again an irrational number.

Fig. 1.7

Once you have the points on the number line corresponding to irrational number

2 , 3 , 5 etc,

p 1 2 3 you can get points on the line corresponding to number like 2 2 , 3 3 , 4 5 , q 7 , etc.

14

Mathematics

p where q is any rational numbers. All these points correspond to irrational numbers. This discussion leads us to the following conclusion.
The number line consisting of points corresponding to rational numbers has gaps on it. In other words it is not complete. But the number line consisting of points corresponding to all rational and irrational numbers is without any gaps and therefore is complete. This number line is called the real line and the one corresponding to rational numbers is known as the rational line. We may therefore say that the rational line is not complete whereas the real line is complete. We have thus extended the system of rational numbers to include in it all irrational numbers as well. This system is called the Real Number System. The system of numbers consisting of all rational and irrational numbers is called the system of real numbers or the Real Number System.

CHECK YOUR PROGRESS 1.5 1. Write down the next three digits in the following numbers : (i) 0.246 ... (ii) 1.35 ... (iii) 3.6912 ... (iv) 0.11011110 ... 2. Represent the following numbers on the number line : (i)
1 2 3

(ii) 1 + 2

(iii) 3

(iv) 2 + 3

1.9 IRRATIONAL NUMBER BETWEEN TWO GIVEN NUMBERS Recall that you had found rational numbers between two given rational numbers. In the following examples we illustrate how do we find irrational numbers between any two numbers. Example 1.8 : Find an irrational number between the numbers 1 and 2. Solution : Consider the number 1 2 . It is the square root of the product of the two numbers 1 and 2. You may refer to Fig 1.5 and see that the point P corresponding to 1 and 2 and so

2 lies between the points

2 is such that 2 is greater than 1 and less than 2. So an irrational number lying between the number 1 and 2.

1 2 i.e.,

2 is

Number Systems

15

Example 1.9 : Find an irrational number lying between the number


3+ 2 2

2 and 3.

Solution : Consider the number

We have As

3+ 2 3+ 2 2 2 3 2 2 = = 2 2 2

2 is less than 2, it is less than 3.


3 2 is a positive real number 2
3+ 2 is greater than 2

2.
3+ 2 . 2

Also, you can verify than 3 is greater than the number


3+ 2 . 2

The required number is

CHECK YOUR PROGRESS 1.6 1. Find an irrational number between the following pairs of numbers : (i) 2 and 3 (iii) (ii)

3 and 2 2 and 8

2 and

(iv)

2. How many irrational numbers are there between 1 and 2 ? Give three examples of irrational numbers between these two numbers. 1.10 ROUNDING OFF NUMBERS TO A GIVEN NUMBER OF DECIMAL PLACES Quite often it is convenient to write the approximate value of a real number to a specified number of decimal places. We illustrate this process with the help of the following examples. Example 1.10 : Express the number 2.178473 approximately by rounding it off to three places of decimal. Solution : We look up the 4th place after the decimal point. In this case, it is 4 which is less than 5. So The approximate value of 2.178473, upto three places of decimal is 2.178.

16

Mathematics

Example 1.11 : Find the approximate value of 2.178473... correct upto 4 places of decimal. Solution : The fifth place of decimal (one after the fourth) is 7. So, we add to 4 the fourth place 1, because 7 is greater than 5. The required approximate value of the number is 2.1785 Thus, we observe that, to round off a number to some decimal places, we observe the next digit in the decimal part of number. (i) If the digit is less than 5, we ignore it and give the answer. (ii) If the digit is 5 or more than 5, we add 1 to the preceding digit to get the required number.

CHECK YOUR PROGRESS 1.7 1. Write down the approximate value of the numbers, correct upto 4 places of decimals (i) 0.7777 ... (iii) 1.01101 ... (iv) 3.142857 LET US SUM UP
z z

(ii) 7.325444 ... (iv) 12.34567 ... (iv) 3.14159 ...

The Real Number System consists of rational and irrational numbers. With the help of real numbers, we can measure exactly any length in terms of a unit of length. There are gaps on the rational line. The real line is complete i.e., there are no gaps on it. A rational number, when represented as a decimal is either terminating or non-terminating but repeating decimal. An irrational number is a non-terminating and non-repeating decimal. Between any two given numbers, there lie an infinite number of rational as well as irrational numbers. TERMINAL EXERCISE

z z z

z z

p 1. Express the decimal, in the form q


(i) 0.3125 (ii) 0.1111 ... (iii) 0.9999 ...

Number Systems

17

2. Express the following numbers as non-terminating but repeating decimals : (i)


2 7

(ii)

4 13

(iii)

6 11

(iv) 1

3. Find two rational and two irrational numbers between the following pairs of numbers : (i) 2 and 3 (iii) 1.2 and 1.3 (ii) (iv)

3 and 2 2 and 3

4. Represent the following numbers on the number line : (i)


3 2

(ii) 1.333... (iv) 2 + 5

(iii) 2 2

5. Find the value of the following numbers correct upto 3 places of decimal : (i)
2 3

(ii)

(iii) 1.7326

(iv) 3.142857

(iv) 0.99999...

18

Mathematics

ANSWERS Check Your Progress 1.1 1. (a) 1, 7 15 (b) 3, 0, 23, 3 (ii) 23 1 22 7 (c) 0 0 1 7 2 (d) 7 , 2 , 11 8 7 6 1 1 2 3 (e) None 1 1

2. (i)

4 1 5 12

(iii)

(iv)

(v)

3. (i)

(ii)

(iii)

(iv)

Check Your Progress 1.2 1. (i) 0.525 2. (i) 0.714285 3. (i) 23 10 1 3 (ii) 0.64 (ii) 0.833 168 (ii) 25 (ii) 103 33 (iii) 1.625 (iii) 1.363 (iii) 63 200 35 11 (iv) 2.5 (iv) 2.076923 (iv) 4073 500 (v) 2.6

4. (i)

(iii)

Check Your Progress 1.4 1. 0, 1.5, 2.5, 1.5 Check Your Progress 1.5 1. (i) 0.24681012... (ii) 1.357911... (iii) 3.6912151821... (iv) 0.110 11110 1111110 111111110... Check Your Progress 1.6 1. (i)

(ii)

3+2 2

(iii) 2 +2 2

2+ 3 2

(iv)

3 2 2

2. Infinite number,

2,

1+ 2 , 2

Number Systems

19

Check Your Progress 1.7 1. (i) 0.7778 (v) 3.1429 Terminal Exercise 1. (i) 5 16 (ii) 1 9 (iii) 1 (iii) 0.54 (iii) 1.733 (iv) 1 (iv) 3.143 (v) 1.0 (ii) 7.3254 (vi) 3.1416 (iii) 1.0110 (iv) 12.3457

2. (i) 0.285714 5. (i) 0.667

(ii) 0.307692 (ii) 1.414

20

Mathematics

2
Indices (Exponents)
2.1 INTRODUCTION You have learnt, how to multiply two or more real numbers. You can easily write the product 5 5 5 = 125 or 7 7 7 7 = 2401. In 5 5 5, five is being multiplied by itself three times. In 7 7 7 7, seven is being multiplied by itself 4 times. Think of the problem when 11 is multiplied by itself 10 times or 15 is multiplied by itself 20 times. Isnt it difficult to write 15 15 ... 20 times? In this lesson we shall try to overcome this difficulty by the introduction of Exponential notation. We shall explain its meaning, state and derive laws of exponents. We shall learn the application of laws of exponents. We shall express real numbers as product of powers of prime numbers. In the end of the lesson we shall also learn how to express very large and very small numbers using scientific notation. 2.2 OBJECTIVES After studying this lesson, the learner will be able to :
z z z z

write a repeated multiplication in exponential notation and vice-versa. identify the base and the exponent of a number in the exponential notation. express a natural number as a product of powers of prime numbers uniquely. state the laws of exponents. explain the meaning of a0, am and a .
p q

z z z

express very large and very small numbers using scientific notation. simplify expressions involving exponents using laws of exponents.

2.3 EXPECTED BACKGROUND KNOWLEDGE


z z z

Multiplication of numbers Division of numbers Prime Numbers

Indices (Exponents)

21

z z

Rational Numbers Four fundamental operations on rational numbers.

2.4 EXPONENTIAL NOTATION Let us consider the following products : (i) 5 5 (ii) 3 3 3 3 (iii) 4 4 4 4 4 4 (iv) 6 6 6 6 6 In (i) we have to multiply 5 by itself 2 times, in (ii) multiply 3 by itself 4 times, in (iii) multiply 4 by itself 6 times, and in (iv) multiply 6 by itself 5 times. We write 5 5 as 52, 3 3 3 3 as 34, 4 4 4 4 4 4 as 46, and 6 6 6 6 6 as 65. 52 is read as five raised to the power 2 or second power of five In 52, the number 5 is called the base and 2 is called the exponent or index. Similarly 34 is read as three raised to the power 4, or fourth power of three. 3 is the base and 4 is the exponent. Again in 46, 4 is the base and 6 is the exponent and in 65, 6 is the base and 5 is the exponent. Similarly (2) (2) (2) (2) = (2)4 where 2 is the base and 4 is the exponent. The notation for writing the product of a number by itself several times is called the exponential notation. You can now write 8 8 8 ... 11 times as 811 and (3) (3) (3) ... 7 times as (3)7. Can you identify base and exponent in 811 ? Of course, yes. In 811, the base is 8 and exponent is 11. Write the base and exponent of (3)7 Base : Exponent :

22

Mathematics

Exponential notation can also be used for writing the product of a rational number by itself several times. For example, two. Here 3 3 is written as 4 4 3I FH 4 K
2

. It is read as

3 raised to the power 4

3 is base and 2 is the exponent. 4

Similarly

666 = 5 5 5

FH 6 I 5K

and

1I F 1I F 1I F 1I FH 2 1I K H 2 K H 2 K H 2 K = FH 2 K

In general, if a is any rational number, then a a a ..... m times = am, a is called the base and m is called the exponent or index. a a = a2 is known as square of a and a a a = a3 is known as cube of a. Note. The plural of the word index is indices. Example 2.1 : Evaluate each of the following : (a)

FH 2 I 3K

(b)

3I FH 5 K

(c)

3I FH 7 K

Solution : (a)

(b)

FH 2 I 3K 3I FH 5 K

=
4

2 2 = 22 = 4 3 3 3 3 9 3333 3 3 3 3 = 81 = 5 5 5 5 5 5 5 5 625

=
3

(c)

b 3g b3g 3I F 3I F 3I 3I FH 7 = 27 K H 7 K H 7 K = b3g7 K = FH 7 77 343

Example 2.2 : Express each of the following in the exponential form : (a) 7 7 7 7 7 7 7 (b) (c) 5 5 5 11 11 11

FH 4 I F 4I F 4I F 4I F 4I 9K H 9K H 9K H 9K H 9K
(b) (c) 5I FH 11 K FH 4 I F 4I F 4I F 4I F 4I F 4I 9K H 9K H 9K H 9K H 9K = H 9K 5 5 5 = 11 11 11
3

Solution : (a) 7 7 7 7 7 7 7 = (7)7

Indices (Exponents)

23

Example 2.3 : Express each of the following in exponential notation and write down the base and exponent in each case : (a) 2187 (b) 25 81 (c) 8 27 3 2187 3 3 3 3
3

Solution : (a) 2187 = 3 3 3 3 3 3 3 = 37. Base = 3, Exponent = 7. (b) 25 55 = 5 5 = 5 = 81 99 9 9 9

FH IK

729 243 81 27 9 3

5 and Exponent =2 Base = 9 (c) 2 2 2 8 = 2 2 2 = 27 3 3 3 3 3 3 2 Base = and Exponent = 3. 3

b g b g b g F I F I F I = F 2I H K H K H K H 3K

1 in exponential form. Write down the base and the exponent. Example 2.4 : Express 243 Solution : 1 = 1 243 243 =

b1g b1g b1g b1g b1g


3 3 3 3 3
5

3 3 3 3

243 81 27 9 3

b1g = FG 1IJ = H 3K 3
5 5

Here Base =

1 and Exponent = 5. 3

Example 2.5 : Simplify 3I FH 2 I F K H 3 4K


2

3I FH 2 I F K H 3 4K
2 3

Solution :

= =

2 2 33 32 4 3

4 27 3 = 9 64 16 Example 2.6 : Find the reciprocal of each of the following and express them in exponential form : (a) 52 (b) 3I FH 7 K
4

(c)

FH 4 I 5K

24

Mathematics

Solution : (a) Reciprocal of 52 = (b) 3I FH 7 K


4

1 1 = 1 = 2 55 5 5

FH IK
4

34 74

Reciprocal of

3I FH 7 K

(c)

FH 4 I 5K

b4g =
59

74 = 7 3 34

FH IK

F 4I Reciprocal of H 5 K
From the above, we observe that

FH IK b g
59 = 5 9 4 4

p If q is any non zero rational number and m is any positive integer, then the

F pI reciprocal of H q K

F qI is H p K

CHECK YOUR PROGRESS 2.1 1. Write the base and exponent in each of the following : (a) 37 8I FH 13 K
2 3

(b)

73 2I FH 11 K
4

2I (c) F H 11 K

5I (d) F H 9 K

20

2. Evaluate each of the following : (a) (b)


3

(c)

5I FH 2 K

(d)

FH 43IK

3. Simplify each of the following : (a) (b)

FH 2 I F 5I 5 K H 3K FH 2 I F 1I K H 3 2K
3

4. Find the reciprocal of each of the following : (a) (3)4 (b) (5)3 (c) 3I FH 7 K
4

2.5 PRIME FACTORISATION You may recall how to write the factors of a given natural number, in finding the HCF and LCM of numbers. Also, given a natural number, it is either the number 1 or it is a prime number or it is a composite number. Any composite number can be expressed as a product of prime numbers. Let us consider the following examples.

Indices (Exponents)

25

(a)

2 2 2

24 12 6 3

24 = 2 2 2 3 = 23 31

(b)

2 2 5 7

980 490 245 49 7


980 = 2 2 5 7 7 = 22 51 72.

(c)

3 3 7 11

7623 2541 847 121 11


7623 = 3 3 7 11 11 = 32 7 112

From the examples given above, we see that any given natural number can be expressed as a product of powers of prime factors. This product is also unique apart from the order of occurrence of factors. This result is known as the Fundamental Theorem of Arithmetic which is formally stated as follows. Any natural number other than one, is expressible as a product of powers of prime numbers in a unique manner apart from the order of occurrence of factors. Example 2.7 : Express each of the following in exponential form : (a) 16200 Solution : (a) 2 2 2 3 3 3 3 5 16200 8100 4050 2025 675 225 75 25 5
16200 = 2 2 2 3 3 3 3 5 5

(b) 49392

= 23 34 52

26

Mathematics

(b)

2 2 2 2 3 3 7 7

49392 24696 12348 6174 3087 1029 343 49 7

49392 = 2 2 2 2 3 3 7 7 7 = 24 32 73.

CHECK YOUR PROGRESS 2.2 1. Express the following as products of powers of prime numbers : (a) 39 (d) 320 (b) 36 (e) 216 (c) 91 (f) 3024

2. Express in the exponential form each of the following : (a) 243 729 (d) 64 (b) 1024 (e) 1331 4096 (c) 1296

2.6 LAWS OF EXPONENTS Let us study the following : (a) 22 23 = (2 2) (2 2 2) = 2 2 2 2 2 = 25 = 22+3 (b) (3)3 (3)4 = [(3) (3) (3)] [(3) (3) (3) (3)] = (3) (3) (3) (3) (3) (3) (3) = (3)7 = (3)3+4 (c)

FG 2 IJ FG 2 IJ H 5K H 5K
2

= =

FG 2 2 IJ FG 2 2 2 2 IJ H 5 5K H 5 5 5 5K
2 2 2 2 2 2 5 5 5 5 5 5
6 2+4

F 2I F 2I = G J =G J H 5K H 5K

Indices (Exponents)

27

(d)

FG 2 IJ FG 2 IJ H 5K H 5K
3

LMFG 2 IJ FG 2 IJ FG 2 IJ OP LMFG 2 IJ FG 2 IJ FG 2 IJ FG 2 IJ OP NH 5 K H 5 K H 5 K Q NH 5 K H 5K H 5K H 5K Q F 2 IJ FG 2 IJ FG 2 IJ FG 2 IJ FG 2 IJ FG 2 IJ FG 2 IJ = G H 5K H 5K H 5K H 5K H 5K H 5K H 5K
7 3+ 4

F 2I F 2I = G J = G J H 5K H 5K

From the above, we observe that in each case the base is the same but the indices, get added. Thus, we have the following : Law 1 : If a is any rational number and m and n are two positive integers, then am an = am+n 3 Example 2.8 : Find the value of 4 3 Solution : 4 3I FH IK FH 4 K
2 5

FH IK
2

3 and 4

FH IK
5

3 have the same base . Therefore to find the product, we 4

add the exponents

3I F 3I 3I FH 4 K H 4 K = FH 4 K

2 +5

3I = F H 4 K

2187 = 16384

Let us the study the following :


35 3 3 3 3 3 = 3 3 3 = 33 = 352 (a) 35 32 = 2 = 3 3 3

(b) (2)7 (2)3

b2g = b 2 g

7 3

b 2 g b 2 g b 2 g b 2 g b 2 g b 2 g b 2 g b 2 g b 2 g b 2 g

= (2) (2) (2) (2) = (2)4 = (2)73

28

Mathematics

3 e2 j 3I F 3I F (c) H K H K = 2 2 3 e2 j
6 4

6 4

3 3 3 3 3 3 2 2 2 2 2 2 = 3333 2 2 2 2 3I = F 3I FH 2 K H 2K
2 6 4

= 5I F 5I (d) F H 7 K H 7K
3

3 3 = 2 2 5 7 = 5 7 =
3

5 5 5 e j = e 7 j e 7j e 7j 5 5 e 7 j e 7j e j
2 1 3 2

5I = F 5I FH 7 K H 7K

From the above, we observe that Law 2 : If a is any non-zero rational number and m, n are positive integers such that m > n, then am an = amn Example 2.9 : Find the value of

FH 7 I FH 7 I 5K 5K
11

Solution :

FH 7 I FH 7 I 5K 5K
11

FH 7 I 5K

118

= 7 5

FH IK

343 125

Let us study the following :

55 1 1 (a) 52 53 = 5 5 5 = 1 = 3 2 5 5
(b) (3)3 (3)5 =

b3g b3g b3g b3g b3g b3g b3g b3g b g b g b g


=
1 1 = 3 3 3
2

b3g

1
5 3

3I F 3I (c) F H2 K H 2K
2

33 2 2 = 3333 2 2 2 2

1 = 1 2 = 33 3 2 2 2

1 3 e2 j

ej

42

Indices (Exponents)

29

2 j e 2 j e 2 j e 3 3 3 I F 2I (d) F = H 2 K H K 3 3 2 2 2 2 e 2 3j e 3j e 3j e 3j e 3j
3 5

e j e j e j

1 = 1 2 2 2 3 3 3

e j

1 5 3 2 3

From the above, we observe that Law 3 : If a is any non-zero rational number and m, n are positive integers such that m < n, then

1 am an = n m a
5 I F 5 I FH 11 K H 11K
7 9

Example 2.10 : Find the value of 5 I F 5 I FH 11 K H 11K


7 9

Solution :

1 5 e11 j

97

e j

1 5 11

= Let us study the following :

1 = 121 . 25 25 121

(a) (22)3 = 22 22 22 = 22+2+2 = 26 = 223 (b) [(5)3]4 = (5)3 (5)3 (5)3 (5)3 = (5)3+3+3+3 = (5)12 = (5)34.

LF 2 I O 2 I F 2 I F 2 I F 2 I F 2 I (c) MH 9 K P = F N Q H 9K H 9K H 9K H 9K H 9K I I = FH 2 I = F = F H2 H2 9K 9K 9K LF 4 I O 4 I F 4 I F 4 I F 4 I (d) MH 3 K P = F N Q H 3K H 3K H 3K H 3K I I = FH 4 I =F = F H 4 H 4 3K 3K 3K
3 5 3 3 3 3 3 3+ 3+ 3+ 3+ 3 15 2 4 2 2 2 2 2+ 2+2+2 8

35

24

30

Mathematics

From the above we observe that Law 4 : If a is any rational number and m and n are two positive integers, then (am)n = amn = amn

LF 1I O Example 2.11 : Find the value of MH 3K P . N Q LF 1I O 1I = F 1I = 1 = 1 Solution : MH 3K P = F N Q H 3K H 3K 3 729


3 2 3 2
3 2 6

2.6.1 ZERO EXPONENT You have seen that so far we have established four laws of exponents, which were for positive integers. Do you know, what will happen when the index is negative or zero? We shall take up negative exponents at a later stage. We will now see what happens when exponent is zero. You are aware that if a rational number is divided by itself, the result is 1. i.e., 22 =1 7 7 5I F 5I FH 9 K H 9K = 1 Again Now 23 23 = 1 23 23 = 233 = 20 20 = 1 Similarly, (a) (5)4 (5)4 = 3 I F 3 I (b) F H 13 K H 13K
3

...(i) ...(ii)

From (i) and (ii), we get

b5g b5g b5g b5g = 1 = (5) b5g b5g b5g b5g

44

= (5)0

3 3 3 13 13 13 = 1 = 3 = 13 3 3 3 13 13 13

FH IK

3 3

3I FH 13 K

3 j e 3 j e 7 7 3I F 3I F 3I and (c) F = 3 3 = 1 = H 7K H 7 K H 7 K e 7 j e 7j
2 2

22

= 3 7

FH IK

Indices (Exponents)

31

From the above, we get Law 5 : If a is any rational number other than zero, then Example 2.12 : Find the value of : (a) a0 = 1

FH 7 I 2K
0

Solution : (a)

FH 7 I 2K

= 1,

5I (b) F H 19 K 5I (b) F H 19 K =1
0

CHECK YOUR PROGRESS 2.3 1. Simplify and express the result in the exponential form : (a) (3)3 (3)4 (b)
5

FH 2 I F 2I 3K H 3K
4

(c)

FH 2 I F 2I F 2I 5K H 5K H 5K
2

2. Simplify and express the result in the exponential form : (a) (11)8 (11)6 (c) (b)
11

5I F 5I FH 7 K H 7K
8

FH 4 I FH 4 I 3K 3K
18

3. Simplify and express the result in the exponential form : (a) (36)3 (b)

LMF 5I OP NH 9 K Q

4 5

LF 3 I O (c) MH 17 K P N Q
(b) 9 I F 9 I FH 11 K H 11K
0 15

3 4

4. Simplify and express the result in the exponential form : (a) 11I F 15 I FH 13 K H 17 K
8 0

5. Which of the following are true statements ? (a) 2I 2I FH 2 I F = F K H K H 5 5 5K LMF 4 I OP = F 4 I NH 9 K Q H 9 K


2 3 6

(b)

5 4

(c)

(d)

3I F 3I 3I FH 11 K H 11K = FH 11 K LMF 9 I OP = F 9 I NH 19 K Q H 19 K
3 3

6 5

30

32

Mathematics

(e)

FH 8 I 3K

=0
4 0

(f) 8I = F H 25 K
4

FH 4 I 5K

16 = 25

(g)

8 I F 7 I FH 25 K H 19 K

2.7 NEGATIVE INTEGERS AS EXPONENTS In the previous sections, we have been taking non-negative integers as exponents. Now we shall try to assign meaning to negative integers as exponents. We know that : The reciprocal of 3 is 1 . We write it as 31 and read it as 3 raised to the power (1). 3 1 . We write it as (4)1 and read it as 4 raised to the power 4

The reciprocal of 4 is (1).

4 7 and are 1 and 1 respectively. We write them as 5 9 45 7 9 1 1 4 7 7 4 and and read them as raised to the power (1) and raised to the 9 5 9 5 power (1) respectively. Similarly, the reciprocals of

FH IK

FH IK

Similarly reciprocal of 32 is (2). The reciprocal of (5)4 is (4). The reciprocal of power (5). From the above we get, 7I FH 18 K
5

1 . We write it as 32 and read it as 3 raised to the power 32

b5g
is

. We write it as (5)4 and read it as 5 raised to the power

b g

1 . We write it as 7 5 18 7 18

FH IK

and read it as

7 raised to the 18

If a is any non-zero rational number and m is any positive integer, then the reciprocal of am i.e.,

1 is written as am and is read as a raised to the power (m). Thus, am 1 = am am

Indices (Exponents)

33

Example 2.13 : Convert the negative exponents to positive exponents in the following :

I (a) F H4 7K
Solution. (a)

I (b) F H 5 6K ej
1 = 1 73 = 7 = 3 4 43 43 4 3 7 7
=

FH 4 I 7K FH 5 I 6K

FH IK

(b)

e j b g
7

1 5 6

1 67 = 6 = 7 7 5 5 5 67

FH IK b g

6 = 5

FH IK

From the above example, we get the following result :

p If q is any non zero rational number and m is any positive integer, then

Fp I q HK

qm q = pm = p
3

FI HK

Example 2.14 : Simplify

FH 2 I FH 5 I 3K 3K
= 3I FH 2 K 3I FH 5 K
3

Solution :

FH 2 I 3K FH 5 I 3K FH 2 I FH 5 I 3K 3K
3

and

3 I F 3I FH 2 K H 5K
3

3 I F 3I FH 2 K H 5K
3

3 3 3 55 75 = 2 2 2 3 3 8

2.8 LAWS OF EXPONENTS FOR INTEGRAL EXPONENTS After attaching meaning to negative integers as exponents of rational numbers, let us see whether the rules of exponents learnt earlier in this chapter hold for negative exponents also or there is any need to modify them. Let us study the following examples :

34

Mathematics

Example 2.15 : Show that (a) 3I F 3 I 3I FH 5 K H 5K = FH 5 K


5 4 5+ 4

(c)

FH IK FH IK
3 4
3

3 4

FH IK
3 4

3+ 7

FH 2 I 3K LF 3 I (d) MH 11K N
(b)

2 3

FH IK

FH 2 I 3K

2 3

2 3

OP = F 3 I Q H 11K

2 3

(e) (5 3)3 = 53 33. Solution : (a) 3I F 3 I FH 5 K H 5K


5 4

ej

1 3 5 5 3 5

FH IK

1 3 e5 j

5 4

3I b =F H 5K
1 2 3
3

5 4

3I = F H5 K

5+ 4

(b)

FH 2 I FH 2 I 3K 3K
2

e j e j
2

1 2 3

2 e 2 e j 3j e 3j

1 2+3 2 3
2 3

= 3I F 3I FH 4 K H 4K
3 7

FH 2 I 3K
1 3 4

2+3

b g

= 2 3

FH IK
7

(c)

e j e j 1 F 3I H K 3 e 4 j 4
3 73

1 3 4

(d)

LMF 3 I OP NH 11K Q

2 3

3I 3I FH 4 K = FH 4 K LMF 11I OP = F 11 I NH 3 K Q H 3 K
2 3 2

3+ 7

2 3

1123 323 = = = 32 3 1123

FG 3 IJ H 11K

2 3

Indices (Exponents)

35

(e)

(5 3)3 =

b5 3g

53

1 = 53 33 33

From the above examples we see that Laws 1 to 5 hold good for negative exponents as well. In general, if a, b be any non-zero rational numbers and m, n be any integers, then 1. 2. 3. 4. am an = am+n am an = amn (am)n = amn (a b)m = am bm

CHECK YOUR PROGRESS 2.4 4I FH 11 K


1

1. Express as a rational number :

2. Express as a power of a rational number with positive exponent : (a)

FH IK
2 9

(b) 127 124

(c)

LMF 5 I OP NH 17 K Q LMF 7 I OP NH 4 K Q
5

2 5

3. Express as a power of a rational number with negative exponent (a)

FH 7 I 8K

(b)

(53)4

2 5

(c)

4. Simplify each of the following : (a) 3I F 3I FH 2 K H 2K


3 5

(b)

FH 2 I FH 2 I 3K 3K
3

(c)

3I F 3 I FH 5 K H 5K

5. Which of the following statements are true ? (a) am an = amn (b) (am)n = amn (c) am bm = (ab)m (d) am an = amn (e) am a0 = am

36

Mathematics

2.9 MEANING OF a . We know that am an = am+n for all integral values of m and n. If a is a positive rational number, and q is a natural number, how should we define a q . Let us multiply a
1 1 q, 1

p q

q times.
1 1 1

i.e.,

1 + 1 + 1 + ..... q times a q a q a q ..... a q = aq = a aq q q = q times


1 1

In other words, qth power of a q = a. We state that a q is the qth root of a and is written as
q

a . Let us consider an example to illustrate this point. 3 3 3 3 3


1 5 1 5 1 5 1 5 1 5

1+1+1+ 1+ 1 5 5 5 5 5 1 5

=3

5 5

= 31 = 3

or

1 5

is the fifth root of 3 which is written as

Now we define a rational power of a. If a is a positive real number, p is an integer and q is a natural number, then

a
Now

p q

ap

a a a ... q factors = a

p q

p q

p q

p p p q + q + q ... q times p q

=a

pq q

=a

= q ap .

Thus, a q means the qth root of ap. We observe that if the exponent is a rational number, its numerator denotes the index and the denominator denotes the root. Thus, 4 3 means cube root of 42.
2

Indices (Exponents)

37

For rational exponents, the laws of exponents are as follows : (i) am.an = am+n (ii) a m a n = amn (iii) (am)n = amn (iv) (ab)m = am bm

F aI (v) G J H bK

am bm

Let us take the following examples and verify the above laws : Example 2.16 : Find the value of : (a) 16

b g

1 4

(b)

b243g
1 4 4 (2 )

2 5
1

(c)

FH 16 I 81K

3 4

Solution : (a)

b16g

1 4

= (2 2 2 2) 4 = = 2
4 1 4

= 21 = 2.
2 5

(b) (c)

b243g b g d i 2 2 2 2I FH 16 I = F K H 81 3 3 3 3 K L 2 I OP = F 2 I e j = MF NH 3K Q H 3K 3 I = 27 I = FH 2 = F H2 K 8 3K
= 3 3 3 3 3 = 35
3 4 3 4 3 4 4 4 3 4 3 3

2 5

2 5

=3

5 2 5

= 32 = 9

CHECK YOUR PROGRESS 2.5 1. Simplify each of the following : (a) 8

bg

2 3

(b)

FG 64 IJ H 125K
13 I FH 16 K

2 3

2. Simplify each of the following : (a) 125

b g

1 3

25

1 2

(b)

3 4

13 I F H 16 K

7 4

38

Mathematics

2.10 SCIENTIFIC NOTATION Let us consider the following problem : Can you calculate what powers of 10 should be multiplied to 1.7 to get (i) 17 Clearly, in (i) (ii) (iii) (iv) Similarly, (ii) 170 17 = 1.7 101 170 = 1.7 102 1700 = 1.7 103 17000 = 1.7 104 170000 = 1.7 10 5 1700000 = 1.7 106 and so on. Scientists and Engineers often find it convenient to write numbers in this form. A number is said to be in scientific notation, when it is expressed as a number between 1 and 10, multiplied by an integral power of 10. Sometimes it is also referred to as writing a number in standard form. In symbols, A 10n is in scientific notation, where A is terminating decimal lying between 1 and 10 i.e. 1 < A < 10 and n is an integer. Now we shall see how to write a number less than 1 in the scientific notation. Let us write 0.00017 in the scientific notation. 17.0 = 1.7 10 = 1.7 101 1.7 = 1.7 1 = 1.7 100 0.17 = 1.7
1 = 1.7 101 10 1 = 1.7 102 100 1 = 1.7 103 1000 1 = 1.7 104 and so on. 10000 1 = 3.1 105. 105

(iii) 1700

(iv) 17000 ?

0.017 = 1.7

0.0017 = 1.7

0.00017 = 1.7

Similarly,

0.000031 = 3.1

Indices (Exponents)

39

From the above, we observe that a number written in scientific notation has the following characteristics : (a) There is only one digit to the left of decimal point. (b) The exponent of 10 is a positive integer equal to the number of places, the decimal point has been moved when the given number is greater than or equal to 10 (c) The exponent of 10 is zero when the given number is greater than or equal to 1 but less than 10 (d) The power of 10 is a negative integer equal to the number of places the decimal point has been moved when the given number is less than 1. Example 2.17 : Write each of the following in scientific notation : (a) Seven thousand (b) 0.57 (b) 143000 (d) 0.00031 (b) 143000 = 1.43000 105 = 1.43 105 (c) 0.57 = 5.7 101 (d) 0.00031 = 3.1 104 Example 2.18 : It is said that the distance of earth from the sun is 149000000 km. Express it in the scientific notation. Solution : The distance of earth from the sun = 149000000 km = 1.49 108 km CHECK YOUR PROGRESS 2.6 1. Express each of the following numbers in scientific notation (a) 15 lakh (b) 72 106 (b) 3730000 (d) 317 104

Solution : (a) Seven thousand = 7000 = 7.0 103

2. Express each of the following numbers in scientific notation : (a) 0.00079 (c) 0.00000567 (b) 0.11 (d) 33 105

3. The diameter of the earth is said to be 13000 km. Write it in scientific notation. 4. It is said that distance of moon from the earth is 380000 km. Express it in scientific notation.

40

Mathematics

5. An aeroplane flies at 7.0 105 metres per hour. How long will it take to cover a distance of 3.5 104 km? 2.11 LET US SUM UP
z z

a a a ... m times = am is the exponential form, where a is the base and m is exponent. Laws of exponents are am an = am+n
mn am an = a (ab)m = am.bm

FH a I bK

am bm

(am)n = amn a0 = 1 am = a
p q

1 am

z z

ap

Very large and very small numbers in scientific notation are written as A 10n, where 1 < A < 10 and n is any integer TERMINAL EXERCISE

1. Express each of the following in the exponential form : (a) 3 3 3 5 5 7 7 7 7 (b) 5 I F 5 I F 5 I F 5 I FH 11 K H 11K H 11K H 11K

2. Simplify each of the following : (a) 3 I F 1I F 8 I FH 4 K H 3K H 27 K


3 2

(b)

3I FH 7 K

35 1 27 5

FH IK

3. Simplify and express the result in the exponential form : (a) (8)2 (6)3 (15)2 (b) 37 I F 37 I FH 19 K H 19 K
26 20

(c)

LMF 9 I OP NH 43K Q

5 7

Indices (Exponents)

41

4. Simplify each of the following : (a) 50 + 30 + 170 31 5. Simplify each of the following : (a) (35)12 (35)3 (c) (b) (101)6 (101)4
4

(b) (70 30) (70 + 30).

FH 2 I FH 2 I 9K 9K
3

6. Find x so that 7. Find x so that

FH 7 I FH 7 I = FH 7 I 8K 8K 8K
3 11 9

3 I F 3 I FH 13 K H 13K
2

3I FH 13 K

2 x +1

8. Expressing as a product of primes, write the answers of each of the following in exponential form : (a) 19440000 (c) 605000 (b) 172872 (d) 35591400

9. Write each of the following numbers in the scientific notation : (a) 50 lakh (c) 4500000 (b) 3030000 (d) 720 106

10. Express each of the following numbers in the scientific notation : (a) 0.00029 (c) 860 105 (b) 0.00000399 (d) 301 104.

11. The star sirus is about 8.1 1013 km from the earth. Assuming that light travels at 3.0 105 km per second, find how long, light from sirus takes to reach the earth. 12. The number of haemoglobin molecules in a single red cell is 270000000. Express it in scientific notation.

42

Mathematics

ANSWERS Check Your Progress 2.1 1. (a) Base = 3, Exponent = 7 (c) Base = 2. (a) 3. (a) 4. (a) 512 2197 20 27
2 , Exponent = 8 11

(b) Base = 7, Exponent = 3 (d) Base = (c) 3125 32


5 , Exponent = 20. 9

(b)

16 14641

27 (d) 64

2 (b) 27
4

FH 1 I 3K

(b)

FH 1 I 5K

(c)

FH 7 I 3K

Check Your Progress 2.2 1. (a) 3 13 (d) 26 5 2. (a) 35 3 (d) 2 (b) 22 32 (e) 23 33 (b) 210
6

(c) 7 13 (f) 24 33 7 (c) 24 34

FH IK

113 (e) 12 2

Check Your Progress 2.3 1. (a) 37

2. (a) (11)2

I (b) F H2 3K 5I (b) F H7 K
(b)

(c)
3

(c)
20

FH 2 I 5K FH 4 I 3K

3. (a) 318 11I FH 13 K


8

4. (a)

5I FH 9 K 9I (b) F H 11 K

(c)
15

3I FH 17 K

12

5.(b), (d) and (g) Check Your Progress 2.4 1. 11 4

Indices (Exponents)

43

2. (a) 3. (a) 4. (a)

FH 9 I 2K FH 8 I 7K

(b) 123 (b)

(c)
12

FH 1 I 5K

(c) (c)

FH 17 I 5K FH 4 I 7K
25 9

10

10

9 4 5. b, c and d

2 (b) 3

Check Your Progress 2.5 1. (a) 4 2. (a) 1 Check Your Progress 2.6 1. (a) 1.5 106 2. (a) 7.9 104 3. 1.3 104 km Terminal Exercise 1. (a) 33 52 74 2. (a) 1 72 (b) (b) (b) 5I FH 11 K 1 105 37 I FH 19 K
6 4

(b) (b)

25 16 13 16 (c) 7.2 107 (c) 5.67 106 5. 5.0 101 hour. (d) 3.17 106 (d) 3.3 104

(b) 3.73 106 (d) 1.1 101 4. 3.8 105 km

3. (a) 29 35 52 4. (a) 0 5. (a) (35)15 6. 8 (c) 23 54 112 9. (a) 5.0 106 10. (a) 2.9 104 11. 7.5 104 hours

(c)

9I FH 43 K

35

(b) 0 (b) (101)2 7. 6 (b) 23 32 74 (d) 23 34 52 133. (b) 3.03 106 (b) 3.99 106 (c) 4.5 106 (c) 8.6 103 12. 2.7 108. (d) 7.2 108 (d) 3.01 102 (c)
2 9

8. (a) 54 35 27

44

Mathematics

3
Radicals (Surds)
2.1 INTRODUCTION In the last lesson, you have learnt the laws of exponents for positive integral indices and negative integral indices. You have also learnt the meaning of numbers of the type a . In this lesson we shall study about a special type of numbers a which are irrational numbers. In the last lesson we gave meaning to the number a q as the qth root of a. In this lesson we shall call q a or n x a radical, q or n as index and a or x as the radicand. We shall also name this as surd. We shall also discuss the laws of radicals. We shall find the simplest (lowest) form of a radical. We shall find the rationalising factor of a radical and rationalise the denominator of a radical and also simplify expressions involving radicals. 3.2 OBJECTIVES After studying this lesson, the learner will be able to :
z z z z z z z z z z z

p q

p q

identify radicals from a given set of irrational numbers identify index and radicand of a surd state the laws of radicals (surds) express a given surd in the simplest form classify similar and non-similar surds reduce surds of different orders to those of the same order perform the four fundamental operations on surds arrange the given surds in ascending/descending order of magnitude find a rationalising factor of a given surd rationalise the denominator of a given surd simplify expressions involving surds.

Radicals (Surds)

45

3.3 EXPECTED BACKGROUND KNOWLEDGE


z z z z

Four fundamental operations on numbers Prime numbers, order relation in numbers Laws of exponents Meaning of a0, am and a .
p q

3.4 SURD You have already studied in lesson 1 that numbers of the type 2 , 3 , 5 , ... are irrational numbers. Now we shall study irrational numbers of a particular type called radicals or surds. Definition. A surd is defined as a positive irrational number of the type n x , where it is not possible to find exactly the nth root of x, where x is a positive rational number. Thus a number
n

x is a surd if and only if

(a) it is an irrational number (b) it is a root of positive rational number In the surd n x , the symbol is called the radical sign. The index n is called the order of the surd and x the radicand. When the order is not mentioned it is taken as 2. For example, in the 3 5 , order of the surd is 3 and 5 is the radicand. In 7 , the order is 2 and the radicand is 7. Similarly,
6

8,

32 and

50 are all surds, but

8 is not a surd as its value 2 is rational.

Again 2 + 2 , though it is an irrational number, is not a surd because it is the square not of an irrational number. Similarly 5 + 3 , are not surds, as the radicands are not rational numbers. We may repeat, that surd is an irrational number in which the radicand is a positive rational number. Note. If n is a positive integer and a be real number, then if a is irrational, Again if n a is rational, then also n a is not a surd. 3.5 PURE AND MIXED SURDS A surd which has unity as its rational factor, other factor being irrational is called a pure surd. For example
n

a is not a surd.

27 , 5 112 and 3 50 are pure surds. A surd having rational factor other than unity along with irrational factor is called a mixed surd.
4

4 For example 2 3 , 3 3 5 and 4 7 are mixed surds. 5 It may be noted that in the cases of conversion, type of the radicand should be kept unchanged.

46

Mathematics

3.6 ORDER OF SURDS In the surd 7 3 2 , 7 is called coefficient of the surd, 3 is order of the surd and 2 is radicand. When there is no coefficient in a surd, it is assumed that coefficient is 1. Example 3.1 : State which of the following are surds and which are not : (a) (c)
4

25 81

(b) (d)
3

50 128

Solution : (b)

25 = 5 which is a rational number. 25 is not a surd.


2 55 = 5 2

50 =

50 is an irrational number
Hence

50 is a surd.

(c)

81 =
4

3 3 3 3 = 3, which is a rational number.

(d)
3

81 is not a surd
3

128 =

4442

= 4 3 2 , which is an irrational number.


3

128 is a surd.

Example 3.2 : Identify index and radicand of each of the following surds : (a) (c)
5 6

17 123

(b) (d)

82
11

517

Solution : (a) Here index is 5 and radicand is 17 (b) Here index is 2 and radicand is 82 (c) Here index is 6 and radicand is 123 (d) Here index is 11 and radicand is 517. Example 3.3 : Identify the pure and mixed surds from the following. (i)

21

(ii)

18

(iii) 3 125

(iv) 2 7 141

(v) 5 5 1125

Solution : Pure surds (i) and (ii); mixed surds (iii), (iv) and (v)

Radicals (Surds)

47

3.7 LAWS OF RADICALS We state here some laws of radicals which are used to simplify surds : (i) (ii)
n

a
n

=a

b = n ab

(iii)

n n

a =n a b b

where a and b are positive rational numbers and n is a positive integer. Example 3.4 : Using laws of radicals, find which of the following are surds and which are not : (a) (c)
3

10 40 2 3 4 10 40 =
= =

(b) (d)

15 4 10 48 27

Solution : (a)

400 20 20

d 20 i

= 20

which is a rational number.

10 40 is not a surd.

(b) 2 15 4 10 =

2 15 4 10 2 2 15 4 4 10 60 160

3 8 which is an irrational number.


= 2 15 4 10 is a surd

48

Mathematics

(c)

2 3 4 =

24
=
3

222

= 2, which is a rational number. (d)


3

2 3 4 is not a surd
48 27 = = = 48 27 16 9 4 which is again a rational number 3

48 27 =

48 27 is not a surd.
CHECK YOUR PROGRESS 3.1

1. Write the index (order) and the radicand in each of the following : (a)
5

125

(b)

343

(c)

19

2. State which of the following are surds ? (a) (d)


3

49 500
1+ 3

(b) (e)

625 5 45
7 169

(c)

216

(f) 3 2 5 6

(g)

(h)

3. Identify pure and mixed surds, out of the following : (i)

15

(ii) 2 12

(iii) 3 3 7

(iv)

35

3.8 LAWS OF SURDS It has been seen that the surds can be expressed as numbers with fractional exponents. Laws of indices studied in the last lesson are applicable to them also. We recall these laws here. (i)
n

y = n xy

or

x1 n y1 n = xy

b g

1n

Radicals (Surds)

49

(ii)

x =n x y y

or

x1 n = x y y1 n

FI HK

1n

(iii) (iv) (v)

mn

x = mn x = n m x

or or or

dx i dx i
dx i

1n 1m

= x1 mn = x1 m

d i

1n

xm = xm n x p = mn x pn

m 1n

= xm n = xp
m

p 1m

= x pn

mn

= x pn

d i

1 mn

In these results x and y are positive rational numbers and m, n and p are positive integers. Let us illustrate these laws in the following examples : (a)
3

2 3 7 = 2 3 . 7 3 = ( 2 7) 3 = 14 3 = 3 14
17 17

b5g (b) b9g


(c) (d)
5 3

FG 5 IJ H 9K

17

=7

5 9
15

3 =

5 13 3

= 31 3

d i

= 31 15 = 15 3 = 53 3 = 3 5 3

3 43 = 4

d i

15

= 4 3 5 = 4 9 15 = 4 9

d i

1 15

= 15 4 9 = 35 4 33

You must have observed from the above, that order of a surd can be changed by multiplying the index of the surd and index of the radicand by the same positive integer. For example
3

2 =

2 2 = 6 4 and

3 = 8 32 = 8 9 .

3.9 SIMILAR (OR LIKE) SURDS Two surds are similar (or like) if they can be reduced to the same irrational factors, whatever coefficients they may have. For example 2 5 and 7 5 are similar surds. be expressed as 5 3 and 2 3 OR as similar surds. 3.10 SIMPLEST (LOWEST FORM) OF A SURD A surd is said to be in its simplest (lowest) form if it has (a) the smallest possible index of the radical (b) no fraction under the radical sign

75 and

12 can

50

Mathematics

(c) no factor of the form an where a is a positive integer is under the radical sign of index n. Thus a pure surd of index n may be reduced to a rational multiple of nth root of a positive integer, no prime factor of which occurs to an exponent as high as n. Such a surd is said to be in lowest term. Thus for example
3

125 = 18 =

125 12 18 12

5 3 12 is the simplest (lowest) form of the given surd. 6


5

Similarly, the simplest form of the surd

3 2 2 is

12 .
3 32 4

Example 3.5 : Express as a pure surd in the simplified form. (a) 2 7 Solution : (a) (b)
2 7 = 34 5 =
4

(b) 3 4 5
2 2 7 = 4 7 = 28

(c)

34 .

5 = 4 34 5

= = (c)
3 32 = 4

4 4

81 5 405
3I FH 4 K
2

32

9 32 = 18 16

Example 3.6 : Express as a mixed surd in the simplest form : (a)

128

(b)

567

(c)

320

(d)

250

Solution : (a)

128 =
=

64 2
8 8 2 = 82 2

=8 2 (b)
4

567 =

81 7 = 4 34 7

= 34 7

Radicals (Surds)

51

(c)

320 =

64 5 = 6 2 6 5

= 265 (d)
3

250 =
=

3
3

125 2
53 2

= 53 2 (e)
3

27 = 25

27 5 3 3 = 5 25 5 5
CHECK YOUR PROGRESS 3.2

1. State which of the following are pairs of similar surds : (a) (c)
2, 8 75 , 48

(b) 5 3 , 3 18 (d)
20 , 125

2. Express as a pure surd : (a) 7 3 (b) 33 2 (c) 5 5 (d)


5 12 2

3. Express as a mixed surd in the simplest form : (a)

50

(b)

81

(c)

128

(d)

270

(e)

512

3.11 ADDITION AND SUBTRACTION OF SURDS We have studied addition and subtraction of rational numbers. Now we shall perform these operations on surds. We can add and subtract similar surds in the same way as we added and subtracted like terms of an algebraic expression. For example, 5 3 + 7 3 = 5 + 7

b g

3 = 12 3 and 8 5 3 5 = 8 3 5 = 5 5

b g

Thus for adding or subtracting the surds we change them to similar surds before performing addition and subtraction. For example

50 + 72
=

55 2 + 6 6 2

= 5 2 + 6 2 = 5+ 6 2 = 11 2

b g

52

Mathematics

And

48 + 3 162
=
3

2 2 2 6 + 3 3 3 3 6

= 2 3 6 +33 6 = 2+3 = 53 6 Similarly, =

b g

98 18 7 7 2 3 3 2

= 7 2 3 2 = 73 2 = 4 2 Example 3.7 : Simplify each of the following : (a) 4 3 + 6 27 (b) 45 6 3 216 Solution : (a) 4 3 + 6 27 = 4 3 + 6 3 3 3 = 4 3 +63 3 = 4 3 + 18 3 = 4 + 18 = 22 3 (b) 45 6 3 216 = 45 6 3 6 6 6 = 45 6 3 6 6 = 45 6 18 6 = 45 18 6 = 27 6 Example 3.8 : Simplify : 2 3 250 + 8 3 16 3 3 54 + 4 32 Solution : 2 3 250 + 8 3 16 3 3 54 + 4 32

b g

Radicals (Surds)

53

= 2 3 5 5 5 2 + 8 3 2 2 2 2 3 3 3 3 3 2 + 4 2 2 2 2 2 = 2 5 3 2 + 8 2 3 2 3 3 3 2 + 2 4 2 = 10 3 2 + 16 3 2 9 3 2 + 2 4 2 = 10 + 16 9

2 + 24 2 = 17 3 2 + 2 4 2

Example 3.9 : Show that 8 45 8 20 + 245 3 125 = 0 Solution : 8 45 8 20 + 245 3 125 = 8 3 3 5 8 2 2 5 + 5 7 7 3 5 5 5 = 8 3 5 8 2 5 + 7 5 3 5 5 = 24 5 16 5 + 7 5 15 5 = 24 + 7 5 16 + 15 5 = 31 5 31 5 = 0 CHECK YOUR PROGRESS 3.3 Simplify each of the following : 1. 2.

175 + 28
32 + 50 + 128

3. 3 50 + 4 18 4. 5. 6.
3

108 75

500 80
24 + 3 81 2 3 3

7. 3 50 4 8 + 7 18 8. 6 3 54 2 3 16 + 3 128 9. 2 3 40 + 3 3 625 4 3 320 10. 12 18 + 6 20 6 147 + 3 50 + 8 45

54

Mathematics

3.12 MULTIPLICATION AND DIVISION IN SURDS In the last section, you have seen that operation of addition and subtraction can be performed when the surds are similar surds. Similarly two surds can be multiplied or divided if they are of the same order. You have also studied that order of a surd can be changed by multiplying or dividing the index of the surd and index of the radicand by the same positive number. Thus, before multiplying or dividing, we change them to the surds of the same order. However if the given surds are of the same order, we can perform the operation directly. For example

3 2 = 3 2 = 6 12 = 6 2

Q 3 and 2 both are surds of order 2

and

12 2 =

Q 12 and 2 are surds of order 2

But if we have to multiply

3 by

2 , we proceed as follows :

The order of the surds are 2 and 3. LCM of 2 and 3 is 6 We shall change both the surds to surds of order 6. and
3

3 =

6 6

33 = 6 27 22 = 6 4

2 =

33 2 =
=

6 6
6

27 6 4 27 4 =
27 = 6 27 4 4
6

108

and

3 = 2

Example 3.10 : (a) Multiply 5 3 16 and 11 3 40 (b) Multiply 5 3 16 and 5 4 3 Solution : (a) 5 3 16 11 3 40 = 5 11 2 3 2 2 3 5 = 55 2 2 3 2 3 5 = 220 3 2 5 = 220 3 10

Radicals (Surds)

55

(b) 17 3 5 5 4 3 The surds 17 3 5 and 5 4 3 are surds of order 3 and 4 respectively. LCM of 3 and 4 = 12 We change both the surds to the surds of order 12 Now and
17 3 5 = 17 12 54 = 17 12 625
12 12 5 4 3 = 5 33 = 5 27

17 3 5 5 4 3 = 17 12 625 5 12 27

= 85 12 625 27 = 85 12 16875 Example 3.11 : Divide 15 3 13 by 6 6 5


15 3 13 5 . 6 132 = 66 5 2 65

Solution :

5 6 169 5 = 2 65 2

169 5

Example 3.12 : Simplify and express in the simplest form 2 50 3 32 4 18 Solution : 2 50 3 32 4 18 = 2 5 5 2 3 4 4 2 4 3 3 2 = 2 5 2 3 4 2 4 3 2 = 10 2 12 2 12 2 = 1440 2 2 2 = 2880 2 3.13 COMPARISON OF SURDS It is difficult to calculate actual value of surds. That is why we cannot tell which of the two surds is greater in value. To know this we change both the surds to surds of same order. Then we can compare them by the value of the radicands alongwith their coefficients.

56

Mathematics

Example 3.13 : Which is greater

1 or 2

1 ? 3

Solution : The two surds are of orders 2 and 3. LCM of 2 and 3 is 6 and
Q

FG 1 IJ H 2K FG 1IJ H 3K
6

12

=
13

FG 1IJ H 2K FG 1IJ H 3K
1 9
1 3

=6
2

1 8 1 9

=6

1 1 > 8 9

i.e.

1 > 8
1 > 2

Example 3.14 : Arrange in ascending order : Solution : The surd


3

2,

3 and

2,

3 and

5 are of orders 3, 2 and 6 respectively.


6 6

LCM of 3, 2 and 6 is 6.
3

2 = 3 = 5 = 4 < 2 <

6 6

22 = 33 =

4 27

and Now

6 6 3

6
6 6

5
5<
6

27

5< 3
3

Ascending order of given surds is

2 , 6 5, 3

CHECK YOUR PROGRESS 3.4 1. Multiply 2. Multiply 3. Divide 4. Divide


3 3

32 and 5 3 4 3 and
3 3 3

5 5

135 by 24 by

320 5 or
3

5. Which is greater

4 ?

Radicals (Surds)

57

6. Which is smaller :

10 or

9 ?

7. Arrange in ascending order


3

2,

3,

8. Arrange in descending order


3

2,

3,

3.14 RATIONALISATION OF SURDS Consider the following products (a) 31 2 . 31 2 = 3 (b) 32 3 . 31 3 = 3 (c) 57 11 . 54 11 = 5 (d) 7 3 4 . 71 4 = 7 We observe that on multiplying the two surds we get the result as a rational number. In such cases each surd is called a rationalising factor the other. Thus (i) (ii) (iii) and (iv)
3

3 is a rationalising factor of 3 is a rationalising factor of


3

3 9 and vice versa


11 7 5

11 4 5

is a rationalising factor of
4

and vice versa

7 is a rationalising factor of

7 3 and vice versa.

Thus we see here, how to multiply a surd by another surd in such a way that the product is a rational number. The process of converting the surds to rational numbers is called rationalisation. Thus to rationalise, we search a factor which when multiplied by the given surd gives us a rational number. For example, the rationalising factor of

x is

x and rationalising factor of

3 + 2 is

3 2
Note : 1. The quantities x y and x + y are called conjugate surds. Their sum and product are always rational 2. Rationalisation is usually used to rationalise the denominator of a rational expression involving irrational surds.

58

Mathematics

Example 3.15 : Find the rationalising factor of Solution :

27

27 =

3 3 3

= 3 3 Rationalising factor =

3
5

Example 3.16 : Find the rationalising factor of Solution :


5

800

800 =
=

5
5

2 2 2 2 2 5 5
25 52

= 2 5 52 Rationalising factor is
5

53 or

125

Example 3.17 : Rationalise the denominator of 3+ 5 = 3 5 3+ 5 3+ 5 3 5 3+ 5

3+ 5 3 5

Solution :

[Multiplying and dividing by

3 + 5 i.e.

by rationalising factor of 3 5 ]

3+ 5
2

2 2

d 3i d 5i

= 3+5+ 2 3 5 35

= 8 + 2 15 = 4 15 2 Example 3.18 : Rationalise 4+3 5 43 5 4+3 5 43 5 = 4+3 5 4+3 5 43 5 43 5

Solution :

Radicals (Surds)

59

d4 + 3 5 i = b4 g d 3 5 i
2 2

16 + 45 + 24 5 16 45

= 61 + 24 5 29 61 24 5 = 29 29 Example 3.19 : If a = 3 2 and b = 3+ 2 a= = 3 2 3+ 2 3 2 3 2 3+ 2 3 2 3+ 2 , show that a + b = 10 3 2

Solution :

= 3+ 2 2 6 3 2 = 5 2 6 and b= = = 3+ 2 3 2 3+ 2 3+ 2 3 2 3+ 2
3+ 2 + 2 6 3 2

...(i)

= 5+ 2 6 Adding (i) and (ii), we get a + b = 10 Example 3.20 : Rationalise

...(ii)

1 3 2 +1
1

Solution :

1 3 2 +1

3 2 +1

d d

3 2 1 3

i 2i 1

60

Mathematics

3 2 1 3 2
2

i b1g

3 2 1 3+ 2 2 6 1 3 2 1 42 6 3 2 1 4 + 2 6 42 6 4+2 6
4 3 4 2 4 + 2 18 2 12 2 6

b 4g d 2 6 i
2

4 34 2 4+6 2 4 32 6 16 24 2 2 42 6 8

2 2 6 4
2+ 6 2 4

Example 3.21 : If numbers. Solution :

3+ 2 2 = a + b 2 , find the values of a and b given, a and b are rational 3 2

3+ 2 2 3+ 2 2 3+ 2 = 3 2 3 2 3+ 2
= 9+6 2 +3 2 +4

b3g d 2 i
2

13 + 9 2 7

Radicals (Surds)

61

= a=

13 9 + 2 = a+b 2 7 7 13 9 and b = 7 7

...(Given)

CHECK YOUR PROGRESS 3.5 1. Find the rationalising factor of each of the following : (a) (c)
3
7

49
54

(b) (d)
3

2 +1
x 2 + 3 y 2 + 3 xy

2. Simplify by rationalising the denominator in each of the following : (a)

2 5 11 5 11 + 5

(b)

3 17 3 +1 3 1

(c)

235 3 7

(d)

(e)

3. Simplify :

2+ 3 2 3 + 2 3 2+ 3
4. Rationalise the denominator of

1 3 2 1
1 5. If a = 3 + 2 2 , find the value of a + a

6. If

2 + 5 7 = x + 7y , find rational number x and y. 25 7

LET US SUM UP
z

An irrational number In
n

x is called a surd if x is a rational number.

z z

x , n is called index and x is called radicand

A surd having unity as its rational factor, other factor being irrational is called a pure surd

62

Mathematics

z z z z

A surd having factor other than unity along with irrational factor is called a mixed surd The order of the surd is the number that indicates the root. The order of
n

x is n

Laws of radicals (a > 0, b > 0)


n

a
n

=a

b = n ab

a na = n b b
z

Operations on surds
x1 n y1 n = xy x1 n x = 1 n y y

b g

1n

FG IJ HK

1n

dx i
dx i
m
z z

1n 1m

= x1 mn = x1 m

d i

1n

m 1n

= xm n

x a = mn x an or x a

d i

1m

= xa

= x an mn = x an

d i

1 mn

Surds are similar if they have the same irrational factor. In the simplest form of a surd, the radical has the smallest possible index with no fraction under the radical sign and no factors of the form an under the radical sign Similar surds can be added and subtracted Orders of surds can be changed by multiplying index of the surd and index of the radicand but the same number Surds of the same order are multiplied and divided. To compare surds, we change the surds to surds of the same order. Then they can be compared by their radicands

z z

z z

z z

x + y is called rationalising factor of x y and vice versa.


If the product of two surds is rational, then each is called rationalising factor of the other.

Radicals (Surds)

63

TERMINAL EXERCISE 1. State which of the following are surds : (a)

25 289

(b)

12

729

(c)

5 +1

(d)

625

2. Express as a pure surd : (a) 2 3 5 (b) 3 4 4 (c) 5 5 2 (d) 7 3 7

3. Express as a mixed surd in the simplest form (a)


4

405

(b)

320

(c)

128

(d)

686

4. Which of the following are pairs of similar surds : (a) (c)


6

112 , 216 ,

343 250

(b) (d)
3

125 , 135 ,
3

500 625

5. Simplify each of the following : 5 1 +4 3 (a) 3 48 2 3 (c) (b)

63 + 28 175

8 + 32 50 6. Which is greater ?
2 or
3

(d) 4 3 81 + 5 3 375 3 3 192 (b)


3

(a)

6 or

7. Arrange in the descending order (a)


3,
3

4,

(b)

2,

3,

8 Arrange in the ascending order :


3

16 ,

12 ,

320

9. Simplify by rationalising the denominator : (a) 5 6 5 (b) 12 7+ 3 (c) 3 1 3 +1 (d) 3 2 2 3+ 2 2

10. Simplify each of the following by rationalising the denominator : (a) 11. If 1 1+ 2 3 (b) 1 7 + 6 13

5+ 2 3 = a + b 3 , find the values of a and b where a, b are rational numbers. 7+4 3

1 12. If x = 7 + 4 3 , find the value of x + x

64

Mathematics

ANSWERS Check Your Progress 3.1 1. (a) Index 5, radicand 125 (b) Index 6, radicand 343 (c) Index 2, radicand 19 2. a, c, d, f and h 3. (i), (iv) are pure surds and (ii), (iii) are mixed surds. Check Your Progress 3.2 1. a, c, d 2. (a) 3. (a) (e)

147
5 2 44 2

(b)

54

(c)

125

(d)

75

(b) 3 3 3

(c) 2 5 4

(d) 3 3 10

Check Your Progress 3.3 1. 7 7 4. 2. 17 2 5. 6 5 8. 18 3 2 3. 27 2 6. 3 3 3 9. 3 3 5

7 28 2 10. 36 5 42 3 + 51 2 Check Your Progress 3.4 1. 10 3 4 4. 7.


6

2. 5. 8.

675

3. 3 6.
5

27 200
3, 3 2 , 3 4

4
4,
4

10

3,

Check Your Progress 3.5 1. (a)


3

(b)

2 1

(c) 53 7

(d)

x 3 y

Radicals (Surds)

65

2. (a) (e) 3. 14 6. x =

2 5 5

(b)

51 17

(c)

2 3 245 7

(d) 8 55 3

2+ 3
4. 2 + 2 + 6 4 179 20 ,y= 171 171 5. 6

Terminal Exercise 1. (b), (d) 2. (a) 3. (a)


3

40

(b)

324
5

(c)

6250
3

(d)

2401
3

3. 4 5

(b) 2

10

(c) 4

(d) 7

4. a, b, d 5. (a) 6. (a) 7. (a) (b) 8.


3

91 3 6
3

(b) 0 (b)
3 3 3

(c)

(d) 25 3 3

3 3, 3,
6

4, 4,

5 2

16 ,

320 ,

12

9. (a) 10. (a) (b)

5 6+ 5

(b) 3 7 3

(c) 2 3

(d) 17 12 2

2+ 2 + 6 4 7 6 + 6 7 + 546 84 12. 14

11. a = 11, b = 6

66

Mathematics

4
Algebraic Expressions and Polynomials
4.1 INTRODUCTION If you go to the market to purchase pens for your class and suppose each pen is available for Rs 10, then for one pen, you will have to pay Rs 10 two pens, you will have to pay Rs 10 2 or Rs 20 three pens, you will have to pay Rs 10 3 or Rs 30 In general for p pens, you will have to pay Rs 10 p or Rs 10p. Here p can take different values from one onwards. Therefore, p represents an unknown quantity, called a variable. Variables are generally denoted by alphabets x, y, z, a, b, c, p, q etc. Thus, a variable p or x, takes different values which you assign to it and behaves like that (those) numbers (s). The branch of mathematics which deals with variables and four fundamental operations on them is called Algebra. In this lesson, you will learn about some basic concepts of algebra, algebraic expressions and polynomials and four fundamental operations on them. 4.2 OBJECTIVES After studying this lesson, the learner will be to able to :
z z z z z

identify a variable cite examples of algebraic expressions classify algebraic expressions into monomials, binomials and trinomials perform four fundamental operations on algebraic expressions evaluate an algebraic expression for given values of the variables

Algebraic Expressions and Polynomials

67

z z z z z

understand and identify a polynomial as a special case of an algebraic expression write examples of polynomials in one and two variables define a polynomial in one variable, say x state the degree of a polynomial perform four fundamental operations on polynomials

4.3 EXPECTED BACKGROUND KNOWLEDGE


z

Different number systems and four fundamental operations on them

4.4 IDENTIFYING A VARIABLE Consider numbers of the type

4 , 14 ,

2 ,

3 , 2

4 , 15

3x ,

21y , 8

21y 3 4 , z contain unknown x, y and are real numbers 3x , You know that 4 , 14 , 2 , , 2 15 8 z respectively and therefore do not have a fixed value like 4, 14 etc. Their values will depend on x, y and z respectively.
Thus, x, y and z are called variables. Can you pick up variables from the following ? 13 , 14 , 15 , x , p 4 4 y , 3z , 4a , 7 b , 17

You can see that p, x, y, z, a and b are variables and all others are constants, as their values do not change. A variable is a number which can have different values whereas a constant has a fixed value. 4.5 ALGEBRAIC EXPRESSIONS Consider expressions of the form 3 , etc. x + 4 , y 3 , z , 3z + 6 , 2x + y 4 The expressions of the above type, which involve variables and constants connected by operations of addition, subtraction, multiplication and division are called Algebraic Expressions. An algebraic expression is a number or a combination of numbers including variables joined by the four fundamental operations.

68

Mathematics

Term : When one or more of the symbols + or occur in an algebraic expression, they separate the algebraic expression into parts, each of which is called a term. Thus, x 3y has two terms, namely, x and 3y; x 3y + 3 has three terms, namely, x, 3y and 3. 3x has only one term, namely 3x. Similarly 4x2 + 3 has two terms 4x2 and 3 and x has only one term. 3

Thus a term contains either a variable or a constant or both variables(s) and constant connected by the operation of multiplication. Coefficient : The constant in a term is called the coefficient of the variable. For example, in 4x + 3y, there are two terms 4x and 3y and the coefficient of x in 4x is 4 and coefficient of y in 3y is 3.
2 2 1 Similarly, the coefficient of x2 in x is because x = 1 x 2 . 3 3 3 3

CHECK YOUR PROGRESS 4.1 1. Identify variables and constants in the following : 2x, 41, 4y, 3t, 17 2. Write the terms and the coefficients of the variables in the terms, in the following algebraic expressions : (i) 3x2 4y 5 (iii) 3x + 4 y z 2 x 8 (v) 7 y 4.6 TYPES OF ALGEBRAIC EXPRESSIONS 1. Monomial : An algebraic expression which has only one term is called a monomial. For example. 4x is a monomial. 2. Binomial : An algebraic expression which has two terms is called a binomial. For example 4x 3y is a binomial : 3. Trinomial : An algebraic expression which has three terms is called a trinomial. For example, 2x 3y + z is a trinomial. (ii) 2x+y 3

(iv) x2 + 2xy + y2

Algebraic Expressions and Polynomials

69

CHECK YOUR PROGRESS 4.2 Classify the following algebraic expressions into monomials, binomials and trinomials. 4y2 3y, 3x, x + y + z, 3 2 x + 2x + 1, 5x 2y2 , p 2

4.7 LIKE AND UNLIKE TERMS The terms of the algebraic expression having the same variable (s) and the same exponent(s) of the variables are said to be like terms. For example, in the algebraic expression 3xy 9x + 8xy 7x + 2x2, the terms 3xy and 8xy, 9x and 7x are like terms. The terms of the algebraic expression 2x2 3xy + 9y2 7yz are all unlike i.e., there are no two like terms in the given algebraic expression. CHECK YOUR PROGRESS 4.3 Write the like terms in the following algebraic expression : (i) 3x2 4xy + 5x2 9 (ii) xyz 5xy + 6zx + 15xyz 1 (iii) x3 + y3 3xy + 6yx 4y3 (iv) abc + bcd + cda 4.8 OPERATIONS ON ALGEBRAIC EXPRESSIONS Let us now define the four fundamental operations of addition, subtraction, multiplication and division on algebraic expressions. 4.8.1 Addition To add two or more algebraic expressions, the following steps are to be followed : Step 1 : Group the like terms of the given algebraic expressions together. Step 2 : Add the like terms grouped together, to get the sum. Let us take some examples. Example 4.1 : Add 3x + 4 and 2x2 7x 2 Solution : (3x + 4) + (2x2 7x 2) = 2x2 + (3x 7x) + (4 2)

70

Mathematics

= 2x2 10x + 2

(3x + 4) + (2x2 7x 2) = 2x2 10x + 2

Algebraic expressions can be added more conveniently and accurately if : (i) the given algebraic expressions are so arranged that the like terms are in one column; and (ii) the coefficients of each column (i.e. of the group of like terms) are added. Example 4.2 : Add 5x + 3y 3 and 2x + y + 7 4 4 Solution : 5x + 3y 3 4 2x + y + 7 4 + 3 7 (5 2)x + (3 + 1)y + 4 + 4 = 3x + 4y + 1

FH

IK

(5x + 3y

3 7 ) + (2x + y + ) = 3x + 4y + 1 4 4

Example 4.3 : Add 17x + 10 + 9xy and 3 7 x 18xy 7y 3 y Solution : 17x + 10 + 9xy y 7 3 + 3 x + 7y 18xy
3 I 1 + 9 18 xy FH17 7 I x + FH10 + 7 Ky b g 3K

44 73 = 3 x + 7 y 9 xy

F17x + 10 + 9xyI + F 3 7 x 18xyI H y K H 7y 3 K


44 73 = 3 x + 7y 9xy

Algebraic Expressions and Polynomials

71

Example 4.4 : Add

3 3 x3 + x2 + x + 1 and x 4 x 3x + 1 2 2

Solution :

3 x3 + x2 + x + 1 2 + x4 x 2
3

3x + 1

x4 + 3 1 x 3 + x 2 + 1 3 x + 2 2 2 or x4 + x3 + x2 2x + 2

FH

IK

b g

CHECK YOUR PROGRESS 4.4 Add the following pairs of algebraic expressions : (i) (ii) 2 x2 + x + 1 3 2 1 and x + x + 5 3 7 4 7 x3 x2 + 1 and 2x2 + x 3 5

z z (iii) xy + y + zx and 3xy y (iv)

4 y 3x 3y x + + z and xz + 2 y z xz 2 y

4.8.2 Subtraction In order to subtract one algebraic expression from another, we go through the following three steps : Step 1 : Arrange the given algebraic expressions so that the like terms are in one column. Step 2 : Change the sign (from + to and to +) of each term of the algebraic expression to be subtracted. Step 3 : Add the coefficients of each column separately. Let us understand the procedure by means of some examples. Example 4.5 : Subtract :
2 2 2 2 (i) 4 x + 3x + from 9 x 3x 3 7

72

Mathematics

2 2x 2 3x (ii) 5x + y 1 from 7 x 13 y

Solution : (i)

9 x 2 3x 2 7 4 x 2 + 3x + 2 3 + 2I b9 + 4gx + b3 3gx + FH 2 7 3K
2 2 20 = 13x 6x 21

(9x2 3x x 7 x 2 13 3 y x 5x 2 1 + 2 y +
2

2 ) 7

FH 4x

2 20 + 3x + 2 = 13x 6x 21 3

IK

(ii)

b7 5gx + b13 + 1g b3 + 2g x y
x = 2 x 2 12 5 y

CHECK YOUR PROGRESS 4.5 Subtract : (i) 7x3 3x2 + 2 from x2 5x + 2

y 2y + 7 + x2 (ii) x + 3 from 3x y z y z
(iii) ax2 + 2hxy + by2 from cx2 + 2gxy + dy2 48.3 Multiplication In order to multiply two algebraic expressions, each of the terms of one algebraic expression is multiplied by each term of the other algebraic expression and the result simplified by adding the like terms.

Algebraic Expressions and Polynomials

73

Let us take some examples : Example 4.6 : Multiply 2n + 3 by n2 3n + 4 Solution : (2n + 3) (n2 3n + 4) = 2n(n2 3n + 4) + 3(n2 3n + 4) = 2n n2 + 2n (3n) + 2n 4 + 3 n2 + 3(3n) + 3 4 = 2n3 6n2 + 8n + 3n2 9n +12 = 2n3 3n2 n + 12

(2n + 3)(n2 3n + 4) = 2n3 3n2 n + 12 9 by x + 7 . x x = 2 x 2 x + 7 3x x + 7 9 x + 7 x x x x

Example 4.7 : Multiply 2x2 3x


2 9 Solution : 2 x 3x x

FH

IK FH x + 7 I xK

FH

IK FH

IK FH

IK

2 2 7 7 9 9 7 = 2 x x + 2 x 3x x 3x x x x x x x

b g

b g

b g

= 2 x 3 + 14 x + 3x 2 21 + 9 63 x2 = 2 x 3 + 3x 2 + 14 x 12 63 x2

FH 2x

3x 9 x

IK FH x + 7 I xK

= 2 x 3 + 3x 2 + 14 x 12 63 x2

4.8.4 Division Try to recall the process when you divided 20 by 3 6 Divisor 3) 20 Quotient Dividend

18 2

Remainder

20 = 3 6 + 2

Algebraic expressions are also divided in the same way. The steps involved in the division process are explained with the help of the following example :

74

Mathematics

Example 4.8. Divide 2x2 + 5x + 3 by 2x + 3 Step 1 : Divide the first term of the dividend by the first term of the divisor and write the result as the first term of the quotient. 2x + 3) 2x2 + 5x + 3 2x2 2x = x 2x + 3) 2x2 x + 5x + 3

( (

Step 2 : Multiply the divisor by the first term of the quotient and subtract the result from the dividend.

(2x + 3)x = 2x2 + 3x x 2 2x + 3) 2x + 5x + 3( 2x2 + 3x 2x + 3

Step 3 : Divide the first term of the resultant dividend obtained in Step 2 by the first term of the divisor and write the result as the second term of the quotient.

2x 2x = 1

x+1 2x + 3) 2x2 + 5x + 3( 2x2 + 3x 2x + 3

Step 4 : Multiply the divisor by the second term of the quotient and subtract the result from the resultant dividend of Step 3.

x+1 2x + 3) 2x2 + 5x + 3( 2x2 + 3x 2x + 3 2x + 3 0

Step 5 : Repeat this process i.e. Step 3 and 4 till you get the remainder zero or the highest exponent of the remainder is less than that of the highest exponent in the divisor in the above example, Remainder = zero, Quotient = x + 1

Algebraic Expressions and Polynomials

75

Let us take one more example. Example 4.9 : Divide x4 + 2x3 + Solution : x2 + 1 3
2 1 1 x by x2 + 3 3 3

x2 + 2x

IJ K

1 3

x4 + 2x3 +

2x 1 3 3

IJ K

1 x4 + x2 3
2x3 1 x2 + 2 x 1 3 3 3 2x3 + 1 x2 1 3 3 1 x2 1 3 9 + + Quotient = x2 + 2x 2 9 2x 3

1 2 and remainder = 3 9

Thus you see that, on dividing an algebraic expression by another, the remainder may not always be zero. CHECK YOUR PROGRESS 4.6 1. Find the product of (x 1) and (x2 + x + 1) : 2. Find the product of

FH x

+ 2 x+ 5 3 6

IK

and

FH x 7 I 4K

x 3 y + 2z 3. Find the product of 2 y 4 z + 7 and 2x 4. Find the remainder when x2 x + 1 is divided by x + 1 5. Find the quotient and remainder when 6x2 5x + 1 is divided by 2x 1.

76

Mathematics

4.9 EVALUATING AN ALGEBRAIC EXPRESSION We can evaluate any algebraic expression for given values(s) of the variable(s) occurring in it. Let us understand the steps involved by evaluating 3x2 x + 2 for x = 2 Step 1 : Substitute the given value(s) in place of the variable(s). i.e., 3 22 2 + 2 Step 2 : Simplify the numerical result obtained in Step 1. 3 22 2 + 2 = 3 4 2 + 2 = 12 2 + 2 = 12 or 3x2 x + 2 = 12 when x = 2. Let us take another example. Example 4.10 : Evaluate : (i) (3x2 3x + 1) (x 1) for x = 3 (ii) (3x2 1) + (4x3 4x 3) for x = 1 Solution : (i) Substituting 3 for x in (3x2 3x + 1)(x 1) we get, (3 32 3 3 + 1) (3 1) = 2(3 9 9 + 1) = 2[19] = 38 (ii) Substituting 1 for x in (3x2 1) + (4x3 4x 3) we get, 3 (1)2 1 + [4(1)3 4(1) 3] = 3 1 + [4 + 4 3] = 2 3 = 1. CHECK YOUR PROGRESS 4.7 Evaluate each of the following algebraic expression for the indicated values(s) of the variables : 1. x2 + 3x 5 at x = 4 2. 2 x3 + 4 x2 7 at x = 1 3 5 5

1 2 x2 7 3. 3x + 4 2 at x = 3 x

Algebraic Expressions and Polynomials

77

x2 4. y + 3xy 11 at x = 2 , y = 1
5. 3xyz x3 y3 + z3 at x = 2 , y = 1 , z = 3. 4.10 POLYNOMIALS Polynomial : An algebraic expression in which the variable(s) does (do) not occur in the denominator and the exponents of the variable (or variables) are whole numbers and the co-efficients of different terms are real numbers is called a polynomial.
3 1 a b For example 5 , 3x y , and x 2 y 2 are all polynomials whereas x 3 x is not a 3 3

polynomial. Also

x + y is not a polynomial.

CHECK YOUR PROGRESS 4.8 1. Which of the following algebraic expressions are polynomials ?
1 2 x 3 + 1 5x 2 y 2 , 4 3y, 5 x + y , , 3x3 y3z, 3x 3 + 4 y 2 + 2 . 3 3 x z

4.11 GENERAL FORM OF A POLYNOMIAL IN ONE VARIABLE An expression of the form a0xn + a1xn1 + a 2 x n 2 + .... + an1x + an where a0, a1, a2, ..., an are real constants, x is a variable and n is a non-negative integer is called a polynomial in the variable x. 4.12 DEGREE OF A POLYNOMIAL : The degree of a polynomial in one variable is the greatest exponent of the variable occurring in the various terms of the polynomial. For example, the degree of the polynomial 2x3 9x + 3 is 3. Similarly the degree of is 1 and the degree of 3 is zero. The degree of a constant is taken as zero. x 9 4

78

Mathematics

Note : A polynomial in one variable is written in decreasing powers of the variable and this is called the standard form of the polynomial. For example, 4 x 3 +
5 4 x + 3x + 4 is written as 2

5 4 x + 4 x 3 + 3x + 4 in standard form. 2

CHECK YOUR PROGRESS 4.9 Write the following polynomials in standard form and specify their degree : (i) 5x2 + 1 + 2x (ii) 3x4 + x5
3 2 x +1 2

(iii) 4x3 7x2 x 2 4.13 OPERATIONS ON POLYNOMIALS We have already learnt four fundamental operations on algebraic expressions. As polynomials are special types of algebraic expressions, the same rules and steps, stated in case of operations on algebraic expressions, hold good in case of polynomials also. Let us illustrate these with the help of examples Example 4.11 : Add 7x2 3x + 4 and 3x3 + 5x2 4x +
7 3

Solution : Writing the polynomials in standard form in columns, with like terms coming in the same column, we have + 7x2 3x + 4 3x3 + 5x2 4x +
7 3

3x3 + (7 + 5)x2 (3 + 4)x + 4 + = 3x3 + 12x2 7x +

FG H

7 3

IJ K

19 3

7 19 2 3 2 2 2 Thus, 7 x 3x + 4 + 3x + 5x 4 x + 3 = 3x + 12 x 7 x + 3

i FH

IK

Algebraic Expressions and Polynomials

79

Example 4.12 : Subtract 3x 5x2 + 7 + 3x3 from 2x2 5 + 11x x3 Solution : Rewriting the polynomials in standard form in columns, we have x3 + 2x2 + 11x 5 + 3x3 5x2 + 3x + 7 +

(1+ 3)x3 + (2 + 5)x2 + (11 3)x (5 + 7) = 4x3 + 7x2 + 8x 12 Thus, (2x2 5 + 11x x3) (3x 5x2 + 7 +3x3) = 4x3 + 7x2 + 8x 12 Example 4.13 : Find the product of (2x2 + 7x + 2) and (3x 1) Solution : (2x2 + 7x + 2) (3x 1) = 3x (2x2 + 7x + 2) 1(2x2 + 7x + 2) = 6x3 + 21x2 + 6x 2x2 7x 2 = 6x3 + (21 2)x2 + (6 7)x 2 = 6x3 + 19x2 x 2 Thus, (2x2 + 7x + 2) (3x 1) = 6x3 + 19x2 x 2

Example 4.14 : Find the product of (0.2x2 + 0.7x + 3) and (0.5x2 3x) Solution : (0.2x2 + 0.7x + 3) (0.5x2 3x) = 0.5x2 (0.2x2 + 0.7x + 3) 3x(0.2 x2 + 0.7x + 3) = 0.1x4 + 0.35 x3 + 1.5x2 0.6x3 2.1x2 9x = 0.1x4 + (0.35 0.6)x3 + (1.5 2.1)x2 9x = 0.1x4 0.25x3 0.6x2 9x Thus, (0.2x2 + 0.7x + 3) (0.5x2 3x) = 0.1x4 0.25 x3 0.6x2 9x

80

Mathematics

Example 4.15 : Divide x3 1 by x 1 Solution : x1 x2 + x + 1

) x3 1
x3 x2 1 x2 x1 x1 + 0 x2 + x +

Quotient is x2 + x + 1, Remainder = 0

Example 4.16 : Divide 2x3 3x2 + 5x 11 by 2x 5 Solution : 2x 5

7 45 x2 + x + 2 4

) 2x3 3x2 + 5x 11 (
2x3 10x2 + 7x2 + 5x 11 7x2 + 45 x 11 2 45 x 225 2 4 +
181 4

35 x 2

45 2 7 Quotient = x + x + 2 4
Remainder =
181 . 4

Algebraic Expressions and Polynomials

81

CHECK YOUR PROGRESS 4.10 1. Add 2x3 + 7x2 5x + 7 and 2x 2 + 7x3 3x 7 2. Add

7 3 7 3 2 2 2 3 x + x 3x + and x3 + x2 3x + 5 5 3 5 3 5

3. Subtract 2x3 + 7x 5x2 + 2 from 5x + 7 3x2 + 5x3 4. Subtract 12x3 3x2 11x +13 from 5x3 + 7x2 + 2x 4 5. Find the product of (x2 + 1) and (x3 1) 6. Find the product of

FG 2 x H3

5 + x 3 and (3x2 + 4x + 1) 4

IJ K

7. Find the remainder and quotient for each of the following : (i) Divide x4 1 by x 1 (ii) Divide x3 3x2 + 5x 8 by x 2 LET US SUM UP
z z

An unknown quantity, which can have various values, is called a variable. An expression which has one or more terms involving variable joined by + or sign is called an algebraic expression. An algebraic expression with one term is called a monomial, with two terms, a binomial and with three terms is called a trinomial. The terms of an expression having the same variable with same exponent are called like terms. The terms which are not like are called unlike terms. Four fundamental operations on algebraic expressions yield another algebraic expression. The process of substituting a numerical value of the variable in the algebraic expression is called evaluating the algebraic expression for that value of the variable. An expression of the type a0xn + a1xn1 + ... + an, where a0, a1, ..., an are real numbers, x is a variable and n is a non-negative integer is called a polynomial in x of degree n in standard form. The degree of a polynomial in one variable is the greatest exponent of the variable occurring in the various terms of the polynomial. The degree of a constant is zero. Four fundamental operations on polynomials yield another polynomial. In case of division of polynomial, the degree of remainder is less than that of divisor.

z z

z z

82

Mathematics

TERMINAL EXERCISE 1. Identify monomials, binomials and trinomials in the following : 2x + 3, 3z, 4x + 3z + 2, 3z2 9, 5x5, 3xyz 2. Identify like terms and unlike terms in the two algebraic expressions given below : (i) 3 x3 + 5x 2 7 x + 4 3 and 5x + 7 x 3 x 2 30 2 2

3 5x 2 3 7 3 (ii) 2 x + yz 3x 12 xyz and 17 xyz + x 3 + 5xy 2 3x


3. Perform the indicated operations on the following algebraic expressions 3 (i) Add 3x2 + 7x + 4y and 6x 2 + x 3 x 7 y 2 (ii) Add
3x 2 2x 2 + 7 x3 3x 2 yz and 2 x3 + 5x 2 yz y y

(iii) Subtract 3x3 5x4 + 6x2 2x + 1 form 2x + 3x4 2x3 + 5x2 1 (x3

(iv) Multiply

dx + 1) by

+ x +1 y

4. Perform the indicated operation on the following polynomials and find the value of resulting polynomial at x = 2 (i) Add 7x4 + 3x3 2x + 1 and 3x4 + 2x 4x3 1 (ii) Subtract 3x2 4x3 + 7x 1 from 6x3 + 3x2 + 6x 1 (iii) Multiply (3x2 + 2x + 1) by (2x + 3) (iv) Multiply (0.5 x2 3x 0.7) by (0.7x 0.3) 5. Find remainder and quotient for each of the following : (i) Divide (x6 1) by x 1 (ii) Divide x3 + 1 by (x + 1) (iii) Divide x6 + 5x4 + 3x2 1 by x2 2

Algebraic Expressions and Polynomials

83

ANSWERS Check Your Progress 4.1 1. Variables : 2x, 4y, 3t ; constants : 41, 17 2. (i) Terms : 3x2, 4y ; coefficients are 3, 4 respectively (ii) Terms : 2x 2 , y ; coefficients are , 1 respectively 3 3

5z 5 ; coefficients are 3, 4, respectively 2 2 (iv) Terms : x2, 2xy, y2; coefficients are 1, 2, 1 respectively (iii) Terms : 3x, 4y, x 8 1 (v) Terms : 7 , y ; coefficients are , 8. 7 Check Your Progress 4.2 1. Monomials : 3x, p Binomials : 4y2 3y, 5x 2y2 3 Trinomials : x + y + z, x2 + 2x + 1 2 Check Your Progress 4.3 (i) 3x2, 5x2 (iii) y3, 4y3 ; 3xy, 6yx (ii) xyz, 15xyz ; 6xz, 7zx (iv) no like terms involved

Check Your Progress 4.4 (i) 23 x 2 + 5 x + 6 21 4 7 x3 + x2 + x 2 5 y 2x (iv) xz + y (ii)

(iii) 4xy + zx Check Your Progress 4.5 (i) 7x3 + 4x2 5x (iii) (c a)x2 + 2(g h)xy + (d b)y2 Check Your Progress 4.6 1. x3 1 3. 1 3 yz 7 y xz 3 2 + + z + 14 z 4 8x 2 x y 2

2 x 3y 2 (ii) y z + 10 + x

13 x 35 2. x 3 x 2 12 3 24 4. 3

5. Quotient : 3x 1, remainder ; 0 (zero)

84

Mathematics

Check Your Progress 4.7 1. 23 4. 1 Check Your Progress 4.8 1. 2 x3 + 1 , 5x2 y2, 3x3 y3z 3 19 2. 15 5. 54 3. 2195 36

Check Your Progress 4.9 (i) 5x2 + 2x + 1; 2 (iii) 4x3 7x2 x 2; 3 Check Your Progress 4.10 1. 9x3 + 5x2 8x 3. 3x3 + 2x2 2x + 5 5. x5 + x3 x2 1 7. 2. 3x3 + x2 6x + 2 4. 7x3 + 10x2 + 13x 17 77 x 3 10 x 2 43 x 3 6. 2 x 4 + 12 3 4 (ii) x5 + 3x4 3 2 x + 1; 5 2

(i) Quotient : x3 + x2 + x + 1, remainder : 0 (zero) (ii) Quotient : x2 x + 3 ; remainder : 2.

Terminal Exercise 1. Monomials : 3z, 5x5, 3xyz Binomials : 2x + 3, 3z2 9 Trinomial : 4x + 3z + 2 3 3 2. (i) Like terms : x 3 ,7 x 3 ; 5x2, x 2 ; 7x, 5x; 4, 30. 2 2 3 , 7 3 (ii) Like terms : ; 3x3, x 3 ; 12xyz, 17 xyz 2 x 3x 2

5x 2 , 5xy Unlike terms : yz


3. 11 (i) x 3 + 9 x 2 + x 3y 2 (iii) 4. 5. 8x4 5x3 x2 + 4x 2
3 x 2 (ii) 9 x + y + 2 x yz 2

(i) 10x4 x3 (iii) 6x3 + 13x2 + 8x + 3; 119

+ x4 + x3 + x2 + x + 1 (iv) y 3 (ii) 10x x ; 78

dx

(iv) 0.35 x3 0.36 x2 + 0.41 x + 0.21; 2.39

(i) Quotient : x5 + x4 + x3 + x2 + x + 1 ; remainder : 0(zero) (ii) Quotient : x2 x + 1 ; remainder : 0(zero) (iii) Quotient : x4 + 7x2 + 17 ; remainder : 33

Special Products and Factorisation

85

5
Special Products and Factorisation
5.1 INTRODUCTION You have already studied different number systems, algebraic expressions and polynomials in a previous lesson. Try to recall how you would find the square of a number, say 1032. Simply you have to multiply 103 with itself. Is this the only method ? No! Here is another method. Method 1 : Direct Multiplication 103 103 309 000 103 10609 or 1032 = 10609 or The answer is the same in both cases. Notice carefully the expression in the box in the second method. This is the most important step in this calculation. i.e., 1032 = (100 + 3)2 = square of the first term + two times the product of the first and the second terms + square of the second term. This can easily be generalised to the square of any binomial (or an algebraic expression with two terms). Now, how would you find the cube of a number, say, 993 ? 1032 = 1002 + 2 100 3 + 32 1032 = 10000 + 600 + 9 1032 = 10609 Method 2 : Breaking up 103 103 = 100 + 3 1032 = (100 + 3)2 = (100 + 3) (100 + 3) = 100 100 + 100 3 + 3 100 + 3 3

86

Mathematics

Direct multiplication will surely make the calculations lengthy and difficult, but if you write, 99 = 100 1 993 = (100 1)3 Thus, you can easily calculate 993 if you know the formula for the cube of a binomial. In this lesson, you will study these as well as many other useful results which will make calculations simpler and also help you to understand polynomials better. 5.2 OBJECTIVES After studying this lesson, the learner will be able to :
z z z z z z z z

write formulae for special products calculate squares and cubes of numbers using special products factorise a given algebraic expression and a polynomial in one variable factorise a given quadratic polynomial by splitting the middle term determine the HCF and LCM of two or more polynomials cite examples of rational expressions in one or two variables express a given rational expression in the simplest form perform four fundamental operations on rational expressions.

5.3 EXPECTED BACKGROUND KNOWLEDGE


z z z z

Knowledge of four fundamental operations on numbers and algebraic expressions GCD and LCM of numbers Polynomials and operations on them Area of squares and rectangles.

5.4 SQUARE OF A BINOMIAL (a) Consider a binomial say, x + y Let us find (x + y)2 (x + y)2 = (x + y) (x + y) = x (x + y) + y(x + y) = x.x + x.y + y.x + y.y = x2 + 2xy + y2 ...(i)

Special Products and Factorisation

87

The square of a binomial is equal to the sum of the square of the first term, two times the product of both the terms and the square of the second term. or (x + y)2 = x2 + 2xy + y2 ...(i)

You can easily make a model and verify this formula geometrically. Method Step 1 : Take a square piece of cardboard ABCD and mark points P and Q on sides AB and AD, respectively, in such a way that AP = AQ = x (say) and PB = QD = y (say) [See Fig. 5.1]
Fig. 5.1

Step 2 : Through P draw a line l parallel to AD, intersecting DC at R. Step 3 : Through Q draw a line m parallel to AB, intersecting BC at S. l and m divide ABCD into two squares and two rectangles. (See Fig. 5.2) Area of ABCD = x2 + xy + y2 + xy
Fig. 5.2

[Using the formulae for the areas of a square and a rectangle] (x + y)2 = x2 + 2xy + y2

(b) Consider the square of the binomial (x y) There are many ways of finding this product. Two methods are given below: Method 1 : Direct Multiplication (x y)2 = (x y) (x y) = x(x y) y(x y) = x.x + x (y) y.x y. (y) = x2 xy xy + y2 (x y)2 = x2 2xy + y2 ...(ii)

88

Mathematics

Method 2 : Using (x + y)2 x y = x + (y) or or (x y)2 = [x + (y)]2 = x2 + 2x(y) + (y)2 (x y)2 = x2 2xy + y2 (x y)2 = x2 2xy + y2

Deductions : What happens when you add the two sides of Formulae (i) and (ii) (x + y)2 = x2 + 2xy + y2 + (x y)2 = x2 2xy + y2 (x + y)2 + (x y)2 = 2x2 + 2y2 or (x + y)2 + (x y)2 = 2(x2 + y2) ...(i) ...(ii)

On subtracting (ii) from (i), we get (x + y)2 (x y)2 = 4xy These formulae are very useful in finding the squares of binomials as well as of numbers. Let us take some examples. Example 5.1 : Find each of the following products : (i) (2a + 3b)2 (iii) (4x 1)2 (ii) (3x + 4y)2 (iv)

FH x + 1 I xK

Solution : (i) We know that (x + y)2 = x2 + 2xy + y2 Replacing x by 2a and y by 3b, we get (2a + 3b)2 = (2a)2 + 2(2a)(3b) + (3b)2 = 4a2 + 12ab + 9b2 (ii) Using Formula (i), we get (3x + 4y)2 = (3x)2 + 2(3x)(4y) + (4y)2 = 9x2 + 24xy + 16y2 (3x + 4y)2 = 9x2 + 24xy + 16y2. (4x 1)2 = (4x)2 2(4x).1 + 12 (iii) Using Formula (ii) above, we get

Special Products and Factorisation

89

= 16x2 8x + 1 (4x 1)2 = 16x2 8x + 1

(iv) Using Formula (i), we get

FH x + 1 I xK

+ 1 = x 2 + 2. x. 1 x x = x2 + 2 +

FH IK

1 x2

1 +2 x2 Example 5.2 : Calculate each of the following without actual multiplication :


2 = x +

FH x + 1 I xK

(a) 1012 Solution : (a) We can write 101 as 101 = 100 + 1

(b) 982

1012 = (100 + 1)2 = 1002 + 2 100 1 + 12 = 10000 + 200 + 1

(b) 982

1012 = 10201

Writing 98 = 90 + 8 and using Formula (i) will make the calculations a little lengthy, even though this method is correct. Let us write : 98 = 100 2 982 = (100 2)2 = 1002 2 100 2 + 22 = 10000 400 + 4 2 98 = 9604 Example 5.3 : Evaluate x2 +

1 1 , given that x x = 1 2 x

1 Solution : It is given that x x = 1 or or

FH x 1 I xK

= 12

2. x. 1 = 1 x2 + 1 2 x x 1 x2 + 2 2 = 1 x 1 x2 + 2 = 3 x

90

Mathematics

CHECK YOUR PROGRESS 5.1 1. Find each of the following products : (i) (5x + y)2 (iv) (ab c)2 (vii) (ii) (x 3)2 (iii) (ac + bd)2
2

(x)

FH x + 1IK 3 FG x + y IJ H y xK

FH x 1 I xK (viii) F H z 1IK
(v) 2

(vi) (2x 5y)2 (ix) (a2 + b2)2

2. Without actual multiplication calculate each of the following : (i) 972 (iv) 492 (ii) 2012 (v) 9992 (iii) 9982 (vi) 1022

3. Find each of the following :

1 (i) x2 + 1 , given that x + = 2 x x2


(ii) a 2 + (iii) x2 +

1 1 , given that a = 2 a a2

1 , 1 given that x = 3 2 x x (iv) x2 + y2, given that x + y = 5 and x y = 3


(v) xy given that x + y = 3 and x y = 1 (vi) x2 + y2 and xy, given that x + y = 10 and x y = 6.

5.5 THE SPECIAL PRODUCT (x + y) (x y) Let us find the product (x + y) (x y) (x + y) (x y) = x(x y) + y(x y) = x.x + x(y) + yx + y(y) = x2 xy + xy y2 = x2 y2 (x + y)(x y) = x2 y2

This is one of the most commonly used formula in algebra.

Special Products and Factorisation

91

Let us consider some examples. Examples 5.4 : Find each of the following products : (i) (3x + y)(3x y) (ii) (x2 y2)(x2 + y2)

Solution : (i) (3x + y)(3x y) = (3x)2 y2 = 9x2 y2 (3x + y)(3x y) = 9x2 y2 (ii) (x2 y2)(x2 + y2) (x2 y2)(x2 + y2) = (x2)2 (y2)2 = x4 y4 (x2 y2)(x2 + y2) = x4 y4 Example 5.5 : Evaluate 47 53 without actual multiplication. Solution : 47 53 = (50 3) (50 + 3) = 502 32 = 2500 9 = 2491 47 53 = 2491 The answer has been found without acutal multiplication. CHECK YOUR PROGRESS 5.2 1. Calculate each of the following : (i) (y + 2)(y 2) (iii) (a2 + 5)(a2 5) (v) 98 102 5.6 CUBES OF BINOMIALS Consider a binomial x + y. Let us find its cube. (x + y)3 = (x + y) (x + y)2 = (x + y) (x2 + 2xy + y2) = x(x2 + 2xy + y2) + y(x2 + 2xy + y2) = x.x2 + x.2xy + x.y2 + y.x2 + y.2xy + y.y2 = x3 + 2x2y + xy2 + x2y + 2xy2 + y3 (ii) (xy 1)(xy + 1) (iv) (x y2) (x + y2) (vi) 85 75

92

Mathematics

= x3 + 3x2y + 3xy2 + y3 (x + y)3 = x3 + 3x2y + 3xy2 + y3

Observe that there is another way of writing (x + y)3 : (x + y)3 = x3 + 3x2y + 3xy2 + y3 =x3 + 3xy(x + y) + y3 (x + y)3 = x3 + 3xy(x + y) + y3

Consider (x y)3 : There are two commonly used ways of finding this product. Let us study them. Method 1 : Direct Multiplication (x y)3 = (x y) (x y)2 = (x y) (x2 2xy + y2) = x(x2 2xy + y2) y(x2 2xy + y2) = x.x2 + x.(2xy) + x.y2 y.x2 y.(2xy) y.y2 = x3 2x2y + xy2 x2y + 2xy2 y3 = x3 3x2y + 3xy2 y3 (x y)3 = x3 3x2y + 3xy2 y3

Method 2 : Using (x + y)3 x y = x + (y) or (x y)3 = [x + (y)]3 (x y)3 = x3 3x2y + 3xy2 y3

Writing this formula in another way, we get (x y)3 = x3 3xy(x y) y3 Let us take some examples. Example 5.6 : Find each of the following products : (i) (3x + 2y)3 (ii) (x 4y)3 (iii) 1013 Solution. (i) (3x + 2y)3 = (3x)3 + 3(3x)2 2y + 3(3x)(2y)2 + (2y)3 = 27x3 + 3.9 x2.2y + 3.3x.4y2 + 8y3 = 27x3 + 54x2y + 36xy2 + 8y3 (3x + 2y)3 = 27x3 + 54x2y + 36xy2 + 8y3

Special Products and Factorisation

93

(ii) (iii) 1013 We can write

(x 4y)3 = x3 3x2(4y) + 3x(4y)2 (4y)3 = x3 12 x2y + 3x.16y2 64y3 (x 4y)3 = x3 12x2y + 48xy2 64 y3 101 = 100 + 1 1013 = (100 + 1)3 = 1003 + 3 1002 1 + 3 100 12 + 13 = 1000000 + 3 10000 + 3 100 + 1 = 1000000 + 30000 + 300 + 1 1013 = 1030301 CHECK YOUR PROGRESS 5.3

Calculate each of the following : (i) (2x + y)3 (iv) 993 (ii) (5x y)3 (v) 973 (iii) (xy z)3 (vi) 483

5.7 SOME OTHER SPECIAL PRODUCTS Besides the squares and cubes of binomials, there are some other special products which have useful applications. Let us now study some more special products. (x + y)(x2 xy + y2) Now, (x + y)(x2 xy + y2) = x(x2 xy + y2) + y(x2 xy + y2) = x.x2 + x(xy) + x.y2 + y.x2 + y(xy) + y.y2 = x3 x2y + xy2 + x2y xy2 + y3 = x3 + y3 (x + y)(x2 xy + y2) = x3 + y3

Similarly, on simplifying (x y)(x2 + xy + y2), you will get x3 y3. The reader is advised to multiply the above and check the result (x y)(x2 + xy + y2) = x3 y3 Consider the following product : (x + y + z) (x2 + y2 + z2 xy yz zx)

94

Mathematics

Now,

(x + y + z) (x2 + y2 + z2 xy yz zx) = x(x2 + y2 + z2 xy yz zx) + y(x2 + y2 + z2 xy yz zx) + z(x2 + y2 + z2 xy yz zx) = x3 + xy2 + xz2 x2y xyz zx2 + yx2 + y3 + yz2 xy2 yz2 xyz + zx2 + zy2 + z3 xyz yz2 xz2 = x3 + y3 + z3 3xyz

(x + y + z) (x2 + y2 + z2 xy yz zx) = x3 + y3 + z3 3xyz

Let us take some examples. Example 5.7 : Find each of the following products : (i) (2x + y) (4x2 2xy + y2) (ii) (x 2) (x2 + 2x + 4) (iii) (4x + y + z) (16x2 + y2 + z2 4xy yz 4zx) Solution : (i) (2x + y) (4x2 2xy + y2) = (2x + y) [(2x)2 (2x)y + y2] = (2x)3 + y3 = 8x3 + y3 (2x + y) (4x2 2xy + y2) = 8x3 + y3 = (x 2) (x2 + x.2 + 22) = x3 8 (x 2) (x2 + 2x + 4) = x3 8 = (4x + y + z) [(4x)2 + y2 + z2 (4x)y yz (4x) z)] = (4x)3 + y3 + z3 3(4x)yz = 64x3 + y3 + z3 12xyz (4x + y + z) (16x2 + y2 + z2 4xy yz 4zx) = 64x3 + y3 + z3 12xyz CHECK YOUR PROGRESS 5.4 1. Find each of the following products : (i) (2x + 3y)3 (ii) (4x 5)3 (iii) (xy 1)3 (iii) (4x + y + z) (16x2 + y2 + z2 4xy yz 4zx) (ii) (x 2) (x2 + 2x + 4)

Special Products and Factorisation

95

(iv) (xy +

z)3

I (v) F Hx 1 xK

(vi) (ax by)3 (viii) (y 3) (y2 + 3y + 9)

(vii) (2x + 1) (4x2 2x + 1) (ix) (1 + x) (1 x + x2)

(x) (2x + y + 3z) (4x2 + y2 + 9z2 2xy 6xz 3yz) 2. Calculate each of the following without actual multiplication : (i) 513 5.8 FACTORISATION You have studied some special products. Let us take (x + y) (x y) = x2 y2 You can also write it as x2 y2 = (x + y) (x y) In other words, the quadratic binomial x2 y2 has been written as a product of two linear binomials x + y and x y. Thus each one of the two x + y and x y is a factor of x2 y2. Note : If you divide x2 y2 by x + y, you get x y and vice versa. Definition The process of writing a polynomial as a product of two (or more) polynomials is called factorisation. Each polynomial in the product is called a factor of the given polynomial. (x + y) (x y) is the factorisation of x2 y2 and x + y and x y are the factors of x2 y2. (ii) 1023 (iii) 953

Thus factorisation is the reverse of the process of finding products. The polynomial obtained by multiplying two other polynomials can be factorised. Let us take the following : 1. Factorisation of difference of two squares As (x + y) (x y) = x2 y2 (x + y) and (x y) are factors of x2 y2 x2 y2 = (x + y) (x y) 2. Factorisation of a perfect square trinomial We know that (x + a)2 = x2 + 2ax + a2 or x2 + 2ax + a2 = (x + a)2 Thus (x + a) (x + a) is the required factorisation of x2 + 2ax + a2

96

Mathematics

(ii) Now, consider the expression (x a)2 (x a)2 = x2 2ax + a2 Thus, (x a) (x a) is the required factorization of x2 2ax + a2 Let us take some examples Example 5.8 : Factorise each the following polynomials : (i) x2 4 (iii) 9x2 + 6x + 1 Solution : (i) x2 4 = x2 22 This is of the form x2 y2 x2 4 = (x + 2) (x 2) (ii) x2 2x + 1 (iv) a2c2 b2

(ii) x2 2x + 1 = x2 2.x.1 + 12 i.e. x2 2x + 1 = (x 1) (x 1)

(iii) 9x2 + 6x + 1 = (3x)2 + 2.3x.1 + 12 = (3x + 1)2 i.e. 9x2 + 6x + 1 = (3x + 1) (3x + 1) (iv) a2c2 b2 = (ac)2 b2 This is of the form x2 y2 Hence a2c2 b2 = (ac + b) (ac b) CHECK YOUR PROGRESS 5.5 Factorise each of the following : (i) 4a2 9 (ii) 4a2 b2

(iii) a2c b2c (Take c common first) (iv) 9x2 25y2 (v) 25x2 + 40x + 16 (vii) x2 + 12x + 36 We can also evaluate some numerical expressions by using factorisation. Let us take some examples. (vi) x2 6x + 9

Special Products and Factorisation

97

Example 5.9 : Without actual multiplication, find the value of each of the following : (i) 1022 982 (ii) 512 492 (iii) 272 732

Solution : (i) 1022 982 = (102 + 98) (102 98) = 200 4 = 800 1022 982 = 800

(ii) 512 492 = (51 + 49) (51 49) = 100 2 = 200 512 492 = 200

(iii) 272 732 = (27 + 73) (27 73) = 100 (46) = 4600 272 732 = 4600

3. FACTORISATION INVOLVING CUBES Some of the cubic polynomials can be factorised using specials products. These are discussed below : (a) Factorisation of perfect cubes (i) Consider the special product (x + y)3 (x + y)3 = x3 + 3x2y + 3xy2 + y3 x3 + 3x2y + 3xy2 + y3 = (x + y)(x + y) (x + y) (ii) (x y)3 = x3 3x2y + 3xy2 y3 x3 3x2y + 3xy2 y3 = (x y)(x y) (x y) (b) Sum and difference of two cubes (i) We know that (x + y)(x2 + xy + y2) = x3 + y3 (ii) We know that (x y)(x2 + xy + y2) = x3 y3 x3 y3 = (x y)(x2 + xy + y2) is the factorisation of x3 y3 x3 + y3 = (x + y)(x2 xy + y2) is the factorisation of x3 + y3

98

Mathematics

Let us take some examples. Example 5.10 : Factorise each of the following polynomials : (i) x3 + 64y3 + 12x2y + 48xy2 (ii) 8x3 36x2y + 54xy2 27y3 (iii) x3 + 27 (iv) 8x3 1 (v) 125x3 64y3

Solution : (i) x3 + 64y3 + 12x2y + 48xy2 = x3 + 3x2(4y) + 3x(4y)2 + (4y)3 = (x + 4y)3 x3 + 64y3 + 12x2y + 48xy2 = (x + 4y)(x + 4y)(x + 4y) = (2x)3 3(2x)2(3y) + 3 2x(3y)2 (3y)3 = (2x 3y)3 8x3 36x2y + 54xy2 27y3 = (2x 3y)(2x 3y)(2x 3y) (iii) x3 + 27 = x3 + 33 = (x + 3)(x2 3x + 32) = (x + 3) (x2 3x + 9) (iv) x3 + 27 = (x + 3)(x2 3x + 9) 125x3 64y3 = (5x)3 (4y)3 = (5x 4y) [(5x)2 + 5x.4y + (4y)2] = (5x 4y) (25x2 + 20xy + 16y2) 125x3 64y3 = (5x 4y) (25x2 + 20xy + 16y2) Example 5.11 : Evaluate a3 + b3 given that a + b = 7 and a.b = 12 Solution : a3 + b3 = (a + b) (a2 ab + b2) = (a + b)(a2 + b2 + 2ab 2ab ab) = (a + b)[(a + b)2 3ab] a3 + b3 = 7(49 3 12) = 7(49 36) = 7 13 = 91 a3 + b3 = 91 CHECK YOUR PROGRESS 5.6 1. Factorise each of the following polynomials : (i) u2 v2 (ii) 25a2 1 (iii) x4 y4 (ii) 8x3 36x2y + 54xy2 27y3

Special Products and Factorisation

99

(iv) x3 8y3 (vii) x8 1

(v) a3 + 216 b3 (viii) a3 + 6a2 + 12a + 8

(vi) a3 343 (x) x2 + 22x + 121

(ix) x3 33x2 + 363x 1331 (xi) 9x2 30xy + 25y2 (xii) 81y2 36xy + 4x2

2. Evaluate each of the following without actual multiplication : (i) 852 152 (ii) 1012 992 (iii) 832 172 (iv) a3 b3 given that a b = 1, a.b = 4

1 1 given that a + = 2 3 a a 3 3 (vii) a b given that a b = 1, ab = 6


3 (v) a +

(vi) 63 53

5.9 FACTORISING A QUADRATIC POLYNOMIAL BY SPLITTING THE MIDDLE TERM Let us consider the product (x + a) (x + b) = x(x + b) + a(x + b) = x2 + (a + b)x + ab We can write (i) as x2 + ax + bx + ab and factorise by taking common factors from first two and last two terms separately x2 + ax + bx + ab = x(x + a) +b(x + a) = (x + a) (x + b) This gives us a method to factorise a quadratic polynomial. Let us take some examples and illustrate x2 + 5x + 6 = x2 + 3x + 2x + 6 = x(x + 3) + 2(x + 3) = (x + 3) (x + 2) What have we done ? We have splitted the middle term 5x into two factor 3x and 2x such that their product is equal to 6x2, i.e. the product of the constant and the quadratic term. We could have done in the following manner also : Comparing x2 + 5x + 6 with (i), we get a + b = 5 and ab = 6 Factors of 6 1, 6 2, 3 Sum of factors 7 5 ...(i)

The required pair of values of a and b are 2 and 3 respectively.

100

Mathematics

As x2 + (a + b)x + ab was factorised as (x + a)(x + b), therefore x2 + (2 + 3)x + 2 3 can be written as (x + 2) (x + 3) x2 + 5x + 6 = (x + 2)(x + 3) (ii) x2 5x + 6 (iv) x2 x 6 Example 5.12 : Factorise each of the following polynomials : (i) x2 + 3x + 2 (iii) x2 + x 6 Solution : (i) x2 + 3x + 2 We need two factors of 2 whose sum is 3 Factors of 2 2 and 1 = (x + 2)(x + 1) x2 + 3x + 2 = (x + 2)(x + 1) (ii) x2 5x + 6 Here, we need two factors of 6 whose sum is negative and product is positive. Both the numbers should be negative Factors of 6 1, 6 2, 3 The required numbers are 2 and 3 x2 5x + 6 = x2 + (2 3) x + (2)(3) = (x 2)(x 3) x2 5x + 6 = (x 2)(x 3) (iii) x2 + x 6 Here the product of the numbers is negative and the sum is positive One of the two factors is negative and the other is positive Factors of 6 1, 6 1, 6 2, 3 2, 3 Sum of the factors (1) + 6 = 5 1 + (6) = 5 (2) + 3 = 1 2 + (3) = 1 Right () or wrong () Hence, Sum of the factors (1) + (6) = 7 (2) + (3) = 5 Sum of the factors 2+1=3

x2 + 3x + 2 = x2 + (2 + 1) x + 2 1

The required pair of numbers is 2 and 3.

Special Products and Factorisation

101

Hence, (iv) x2 x 6

x2 + x 6 = x2 + (2 + 3) x + (2) 3 = (x 2)(x + 3) x2 + x 6 = (x 2) (x + 3)

Here the sum and product both are negative One number (smaller one) has to be positive and the other number has to be negative. Factors of 6 1, 6 2, 3 Sum of the factors 1 + (6) = 5 2 + (3) = 1

The required pair of numbers is 2 and 3. x2 x 6 = x2 + [2 + (3)]x + 2 (3) = (x + 2)(x 3) x2 x 6 = (x + 2)(x 3) CHECK YOUR PROGRESS 5.7

Factorise each of the following polynomials by splitting the middle term : (i) x2 + 7x + 6 (iv) x2 4x 5 (ii) x2 7x + 6 (v) x2 2x 8 (iii) x2 10x + 21 (vi) z2 2z 15

5.10 FACTORISATION OF QUADRATIC POLYNOMIALS OF THE TYPE ax2 + bx + c, WHERE a 0 OR a 1, BY SPLITTING THE MIDDLE TERM Consider the polynomial : 2x2 + 3x + 1. Here, the coefficient of x2 is 2 and not one. So, can we follow the method of splitting the middle term ? Yes, we can : but now we need to consider the factors of 2 1 (i.e. a c) not one (i.e., not just c). The factors of 2 1 are 2 and 1 only. 2x2 + 3x + 1 = 2x2 + (2 + 1)x + 1 because 2 + 1 = 3 = 2x2 + 2x + x + 1 = 2x(x + 1) + 1(x + 1) [By taking 2x common from the first two terms and 1 from the last two terms] = (2x + 1) (x + 1) 2x2 + 3x + 1 = (2x + 1)(x + 1) Let us take some more examples.

102

Mathematics

Example 5.13 : Factorise each of the following polynomials : (i) 3x2 + 5x + 2 (ii) 3x2 + 10x + 7 (iii) 2x2 11x + 14

Solution : (i) 3x2 + 5x + 2 The product of the coefficient of x2 and the constant term = 3 2 = 6 We need two factors of 6 whose sum is 5. Can you guess these ? Yes, they are 2 and 3 3x2 + 5x + 2 = 3x2 + (2 + 3)x + 2 = 3x2 + 2x + 3x + 2 = x(3x + 2) + 1 (3x + 2) = (3x + 2) (x + 1) 3x2 + 5x + 2 = (3x + 2) (x + 1)

Note that if you multiply 3x + 2 and x + 1, you get back 3x2 + 5x + 2. (ii) 3x2 + 10x + 7 The product = 3 7 = 21 Factors of 21 1, 21 3, 7 Sum of the factors 1 + 21 = 22 3 + 7 = 10 Right or wrong

3x2 + 10x + 21 = 3x2 + (3 + 7)x + 7 = 3x2 + 3x + 7x + 7 = 3x(x + 1) + 7(x + 1) = (3x + 7) (x + 1) 3x2 + 10x + 7 = (3x + 7) (x + 1)

(iii) 2x2 11x + 14 Product of 2 and 14 = 2 14 = 28. Also since the sum is negative and the product is positive, both the factors have to be negative. Factors of 28 1, 28 2, 14 4, 7 Sum of the factors (1) + (28) = 29 (2) + (14) = 16 (4) + (7) = 11 Right or wrong

2x2 11x + 14 = 2x2 (4 + 7)x + 14

Special Products and Factorisation

103

= 2x2 4x 7x + 14 = 2x(x 2) 7(x 2) = (2x 7) (x 2) 2x2 11x + 14 = (2x 7)(x 2) CHECK YOUR PROGRESS 5.8 Factorise each of the following polynomials by splitting the middle terms : 1. x2 + 9x + 20 3. 6x2 + 7x 10 5. 4x2 + 12x + 5 7. x2 + 4x 60 9. 9x2 12x + 4 2. x2 + x 20 4. x2 2x 8 6. x2 x 56 8. 3x2 2x 5 10. 15x2 14x + 3

5.11 APPLICATIONS OF FACTORISATON Factorisation helps us not only in expressing a given polynomial in terms of simpler polynomials, but also enables us to generate new polynomials using the given polynomials. 5.11.1 H.C.F. of Polynomials We already know the meaning of the term H.C.F. of any given numbers. It is the largest number which divides all the given numbers. For instance, the H.C.F. of 12 and 18 is 6 The H.C.F. of two or more polynomials is the polynomial of the highest degree which divides all the given polynomials completely. Let us recall how to find the H.C.F. of two numbers, say 24 and 36. We used the method of prime factorisation. 24 = 23 3 and 36 = 22 32 The first common factor is 2 and it occurs at least twice in the factorisation of the two numbers and so it must occur twice in the H.C.F. Similarly, the next common factor is 3 and it occurs at least once in both the numbers The H.C.F. of 24 and 36 = 22 3 = 12 Similarly, you can find the H.C.F. of two or more polynomials. Let us take some examples and illustrate.

104

Mathematics

Example 5.14 : Find the H.C.F. of 4x2y and x3y2 Solution : x occurs as a factor at least twice in the two monomials and y occurs at least once. The H.C.F. of 4x2y and x3y2 is x2y

Example 5.15 : Find the H.C.F. of (x 2)3(2x 3) and (x 2)2(2x 3)3 Solution : The first common factor is x 2 and it occurs at least twice in the given polynomials. It must occur two times in the H.C.F. Now 2x 3 is the next common factor and it occurs only once in the first polynomial. H.C.F. of the given polynomials = (x 2)2 (2x 3)

Let us take some more examples. Example 5.16 : Find the H.C.F. of each of the following pairs of polynomials : (i) (3x 1)4 (2x + 5)2 and (2x 1) (2x + 5) (3x 1)3 (ii) x2 4 and (x + 2)2 Solution : (i) (3x 1)4 (2x + 5)2 is the first polynomial. The second polynomial can be written as (3x 1)3 (2x + 5)(2x 1) The first common factor is 3x 1 and occurs at least three times. The next common factor is 2x + 5 and occurs at least once. H.C.F. of (3x 1)4 (2x + 5)2 and (3x 1)3 (2x + 5) (2x 1) = (3x 1)3(2x + 5) Note : (1) 2x 1 does not, occur in the first polynomial and so does not occur in the H.C.F. (2) Check that the H.C.F. divides both the polynomials completely. (ii) x2 4 and (x + 2)2 Now, x2 4 = x2 22 = (x +2) (x 2) The only common factor is x + 2 and it occurs at least once H.C.F. of x2 4 and (x + 2)2 is x + 2.

Special Products and Factorisation

105

CHECK YOUR PROGRESS 5.9 Find the H.C.F. of each of the following pairs of polynomials : (i) (x + 1)3 and (x + 1)2 (x 1) (iii) (x + 2)3 and x3 + 8 (ii) x2 + 4x + 4 and x + 2 (iv) (x + 1)2 (x + 5)3 and x2 + 10x + 25

(v) (2x 5)2 (x + 4)3 and (2x 5)3(x 4) 5.11.2 The L.C.M. of Polynomials Recall how we found the L.C.M. of two numbers, say 36 and 40. We write the prime factorisation of the numbers and find the product of all the different prime factors of the numbers, using each prime factor the maximum number of times it occurs in the factorisation of any of the numbers. 36 = 2 2 3 3 and 40 = 2 2 2 5

2 occurs three times, 3 occurs twice and 5 occurs once, at the most L.C.M. of 36 and 40 is 2 2 2 3 3 5 which is 360. Similarly, you can easily find the L.C.M. of polynomials. The L.C.M. of two or more polynomials is defined as the polynomial of the least degree which is a multiple of all the given polynomials. Let us take some examples to illustrate. Example 5.17 : Find the L.C.M. of each of the following pairs of polynomials : (i) (x 2)3(2x 3) and (x 2)2(2x 3)3 (ii) (3x 1)4(2x + 5)2 and (2x 1)(2x + 5)(3x 1)3 (iii) x2 4 and (x + 2)2 Solution. (i) (x 2)3 (2x 3) and (x 2)2 (2x 3)3 x 2 occurs 3 times at the most and 2x 3 also occurs 3 times at the most. L.C.M. of (x 2)3 (2x 3) and (x 2)2 (2x 3)3 = (x 2)3 (2x 3)3 (ii) (3x 1)4(2x + 5)2 and (2x 1)(2x + 5)(3x 1)3 The L.C.M. of the given polynomials = (3x 1)4(2x + 5)2(2x 1).

106

Mathematics

CHECK YOUR PROGRESS 5.10 Find the L.C.M. of each of the following pairs of polynomials : (i) (x + 1)3 and (x + 1)2 (x 1) (iii) (x + 2)3 and x3 + 8 (iv) (x + 1)2( x + 5)3 and x2 + 10x + 25 (v) (2x 5)2(x + 4)3 and (2x 5)3 (x 4) 5.12 THE RELATIONSHIP BETWEEN H.C.F. AND L.C.M. What is the H.C.F. of 6 and 4 ? What is their L.C.M.? You know very well that the H.C.F. of 6 and 4 is 2, and their L.C.M. is 12. Now, what is 2 12 ? Is it not the product 6 4 ? Yes! The product of the H.C.F. and the L.C.M. is the same as the product of the numbers. The same result is true even for polynomials. Let us verify this result with the help of some examples. Example 5.18 : Is the result given above true for each of the following pairs of polynomials ? (i) (x 2)3 (2x 3) and (x 2)2(2x 3)3 (ii) x2 1 and x3 1 (iii) (x 1)3 and (x + 1)2 Solution : (i) (x 2)3 (2x 3) and (x 2)2 (2x 3)3 The product of (x 2)3 (2x 3) and (x 2)2(2x 3)3 = (x 2)3 (2x 3) (x 2)2(2x 3)3 = (x 2)3 + 2 (3x 3)1 + 3 = (x 2)5(2x 3)4 The H.C.F. of the given polynomials is (x 2)2 (2x 3) and the L.C.M. of the given polynomials is (x 2)3(2x 3)3. Hence H.C.F. L.C.M. = (x 2)2(2x 3)(x 2)3(2x 3)3 = (x 2)5(2x 3)4 which is the same as the product of the given polynomials. (ii) x2 1 and x3 1 x2 1 = (x + 1)(x 1) (ii) x2 + 4x + 4 and x + 2

Special Products and Factorisation

107

and

x3 1 = (x 1) (x2 + x + 1) (x2 1)(x3 1) = (x + 1)(x 1) (x 1)(x2 + x + 1) = (x + 1)(x 1)2 (x2 + x + 1)

The H.C.F. of the given polynomials is x 1 and the L.C.M. of the given polynomials is (x + 1)(x 1)(x2 + x + 1) H.C.F. L.C.M. = (x 1) (x + 1) (x 1) (x2 + x + 1) = (x + 1)(x 1)2 (x2 + x + 1) which is the same as (x2 1)(x3 1). (iii) There is no factor common to (x 1)3 and (x + 1)2 H.C.F of (x 1)3 and (x + 1)2 is 1 and the L.C.M of (x 1)3 and (x + 1)2 is (x 1)3(x + 1)2. Hence H.C.F. L.C.M. = 1 (x 1)3 (x + 1)2 = (x 1)3(x + 1)2 = Product of the given polynomials. In the other words. Product of two given polynomials = Product of their H.C.F. and L.C.M. i.e., First polynomial Second polynomial = H.C.F. L.C.M. You can also use this method to find the L.C.M. of given polynomials. Example 5.19 : Find The L.C.M. of x 1 and x2 2x + 1. Solution : x2 2x + 1 = (x 1)2 and x 1 = x 1 H.C.F. is x 1. and

bx 1gdx L.C.M. =

2x + 1

x 1

= x2 2x + 1. Example 5.20 : Find the L.C.M. of x2 1 and x2 x 2. Solution : x2 1 = (x 1) (x + 1) and x2 x 2 = x2 2x + x 2 = x (x 2) + 1 (x 2) = (x + 1) (x 2) H.C.F. is x + 1.

dx 1idx L.C.M. =
2

2 x2 x 1 x +1 x 2 = x +1 x +1

i b gb g b g
= (x2 1)(x 2) = x3 2x2 x + 2.

108

Mathematics

Thus finding the L.C.M. of two polynomials is not difficult and can be done in either of the two given ways besides many others. Next given the H.C.F. and the L.C.M. of two polynomials and also given one of the polynomials, you can find the other. Example 5.21 : The H.C.F. of an unknown polynomial and x3 + 8 is x + 2 and their L.C.M. is (x + 2)3 (x2 2x + 4). Find the unknown polynomial. Solution : Let the polynomial be A. The H.C.F. of A and x3 + 8 is x + 2 and their L.C.M is (x + 2)3 (x2 2x + 4). H.C.F. L.C.M. = (x + 2)(x + 2)3(x2 2x + 4) = (x + 2)4 (x2 2x + 4) Product of the polynomials = A(x3 + 8) = A(x + 2) (x2 2x + 4) A(x + 2)(x2 2x + 4) = (x + 2)4(x2 2x + 4) Thus A=

bx + 2g dx 2x + 4i bx + 2gdx 2x + 4i
4 2 2

A = (x + 2)3.

Example 5.22 : The H.C.F. of two polynomials is x 2 and their L.C.M. is x4 + 2x3 8x 16. If one of the polynomials is x3 8, find the other polynomial. Solution : Let the other polynomial be A. Then, product of the polynomials = A(x3 8) H.C.F. L.C.M. = (x 2) (x4 + 2x3 8x 16) = x5 4x3 8x2 + 32 A(x3 8) = x5 4x3 8x2 + 32 A = (x5 4x3 8x2 + 32) (x3 8) = x2 4 Thus A = x2 4. Thus, the required polynomial is x2 4. CHECK YOUR PROGRESS 5.11 1. Find the H.C.F. and the L.C.M of each of the following pairs of polynomials : (i) (x + 2)2 and (x + 2)3 (ii) (x + 3)2 and x3 + 27

(iii) (x + 3)3 (x + 1)3 and (x + 1)2 (x + 3)4 (iv) x2 + 4x 60 and x2 + 5x 50

Special Products and Factorisation

109

(v) x2 2x 35 and (x 7)3 (vii) x3 + 1 and x3 1 (ix) 3x2 5x + 2 and x2 4

(vi) x2 x + 1 and x3 + 1. (viii) x2 x + 1 and (x + 1)2 (x) x3 y3 and x2 + xy + y2

2. The H.C.F. of two polynomials is 1 and their L.C.M. is (x 1)2(x + 1). If one of the polynomials is x2 2x + 1, find the other polynomial. 3. The H.C.F. of two polynomials is x + 1 and their L.C.M. is x6 1. If one of the polynomials is x3 + 1, find the other polynomial. 5.13 RATIONAL EXPRESSIONS An algebraic expression which is the quotient of two polynomials is called a rational expression.
x +1 ; x 1

For example,

x 2 3x + 5 ; x2 5

x3 ; 2 x + 2x 7

x 2 y2 ; 2x + 3y

2x 2 y 2 + 3xy ; 7x4 3y 2

x3 1 x2 1

p 2 2p and are all rational expressions in one or two variables. Are the polynomials, p3 x2 +1 and 3x2 + 4y rational expressions ?
x2 + 1 i.e., the denominator of 1 a polynomial can be written as 1, and so it is a rational expression. Yes, a polynomial is a rational expression because x2 + 1 = But a rational expression need not be a polynomial.

CHECK YOUR PROGRESS 5.12 1. Identify the rational expressions from the following : (i) 2x 3 4x 1 (ii)
3x 2 4x + 3 3x 3 + 11

2 3x 2 x + 5 (iii) x2 + 1

3x 2 y 2 (iv) x+y

2. Cite three examples each of rational expressions in (a) one variable, and (b) two variables.

110

Mathematics

5.14 REDUCTION OF RATIONAL EXPRESSIONS TO LOWEST FORM Sometimes the polynomials in the numerator and denominator of a rational expression have common factors, which can be cancelled out and the resulting rational expression is said to be in lowest terms. Let us take some examples to illustrate. Examples 5.23 : Reduce each of the following rational expressions to lowest form : x 2 + 2x 3 (i) 2 x + 4x 5

(ii)

dx 4ibx 1g . dx 3x + 2ibx + 2g
2 2

Solution. (i) x2 + 2x 3 = x2 + 3x x 3 = x (x + 3) 1(x + 3) = (x + 3)(x 1) and x2 + 4x 5 = x2 + 5x x 5 = x(x + 5) 1(x + 5) = (x + 5) (x 1)


x + 3 x 1 x+3 x 2 + 2x 3 = = x +5 x 1 x +5 x2 + 4x 5

b gb g b gb g

(ii) (x2 4) = (x + 2) (x 2) and


2

x2 3x + 2 = (x 1) (x 2)

dx 4ibx 1g = bx + 2gbx 2gbx 1g dx 3x + 2ibx + 2g bx 1gbx 2gbx + 2g


2

= 1. Example 5.24 : Reduce each of the following rational expressions into lowest form : (i)
x3 + 1 x 2 + 3x + 2

(ii)

b gd

x4 1 x 1 x2 + 1

Solution. (i) and

x3 + 1 = (x + 1) (x2 x + 1) x2 + 3x + 2 = (x + 1) (x + 2)
x3 + 1 = 2 x + 3x + 2

bx + 1gdx x + 1i bx + 1gbx + 2g
2

x2 x + 1 x+2

Special Products and Factorisation

111

(ii)

x4 1 = (x2 1) (x2 + 1) = (x 1) (x + 1) (x2 + 1)

bx 1gbx + 1gdx + 1i = bx 1gdx + 1i bx 1gdx + 1i


x4 1
2

= x + 1. CHECK YOUR PROGRESS 5.13 Reduce each of the following rational expressions into lowest form : (i) (iii) 3x 2 + 2x 5 6x 2 + x 15 x3 1 x4 + x2 + 1 (ii) (iv)
8x 3 + 1 4 x2 1

bx ygdx

x 8 y8
4

+ y4

(v)

bx + 4gdx + 5x + 6i dx + 7x + 12ibx + 2g
2 2

5.15 OPERATIONS ON RATIONAL EXPRESSIONS The four fundamental operations on rational expressions are performed exactly in the same way as in the case of rational numbers. Let us take some examples to illustrate. Example 5.25 : Add
2x + 1 x+2 and . x 1 x +1

Solution :

2x + 1 x + 1 + x 1 x + 2 2x + 1 x + 2 + = x 1 x +1 x 1 x +1
2 2

gb g b gb g b gb g d2x + 3x + 1i + dx + x 2i =
x2 1

3x 2 + 4x 1 = x2 1
2x + 1 x + 2 3x 2 + 4x 1 + = . x 1 x +1 x2 1

112

Mathematics

Example 5.26 : Subtract

x 1 3x 2 from . x +1 3x + 1

Solution :

3x 2 x + 1 x 1 3x + 1 3x 2 x 1 = 3x + 1 x + 1 3x + 1 x + 1

gb g b gb b gb g
i d
3x 2 + 4x + 1

d3x =

+ x 2 3x 2 2x 1

3x 2 + x 2 3x 2 + 2x + 1 = 3x 2 + 4x + 1 = 3x 1 + 4x + 1

3x 2

3x 2 x 1 3x 1 = . 3x + 1 x + 1 3x 2 + 4x + 1

Example 5.27 : Multiply

(i)

5x + 3 2x 1 with . 5x 1 x +1 2x + 1 x 1 with . x 1 x+3

(ii)

Solution : (i)

5x + 3 2x 1 5x + 3 2x 1 = 5x 1 x + 1 5x 1 x + 1

b b

gb g gb g

10x 2 5x + 6x 3 = 5x 2 + 5x x 1 = 10x 2 + x 3 5x 2 + 4 x 1

5x + 3 2x 1 10x 2 + x 3 = . 5x 1 x + 1 5x 2 + 4x 1 2x + 1 x 1 = x 1 x + 3

(ii)

b2x + 1gbx 1g bx 1gbx + 3g

2x + 1 x+3

2x + 1 x 1 2x + 1 = . x 1 x + 3 x+3

Special Products and Factorisation

113

Example 5.28 : Divide

(i)

2x + 1 3x + 4 by x 1 x 1 x 1 x2 + 1 by x+2 x 1

(ii)

Solution. (i)

2x + 1 3x + 4 2x + 1 x 1 = x 1 x 1 x 1 3x + 4

b2x + 1gbx 1g bx 1gb3x + 4g


2x + 1 . 3x + 4

2x + 1 3x + 4 2x + 1 = x 1 x 1 3x + 4

(ii)

x2 + 1 x 1 x2 + 1 x + 2 = x 1 x + 2 x 1 x 1

dx + 1ibx + 2g bx 1gbx 1g
2

x 3 + 2x 2 + x + 2 = x 2 2x + 1 x2 + 1 x 1 x 3 + 2x 2 + x + 2 = . x 1 x + 2 x 2 2x + 1 CHECK YOUR PROGRESS 5.14 1. Add the following rational expressions : (i) 2x + 3 x+2 and x+2 2x 3
2 2 (ii) 3x + 1 and 2x 3x + 1 x +1 x 1

2. Subtract the first rational expression from the second : (i)


2x + 3 2x 3 , x + 2 2x + 1

(ii)

2x 2 + 1 3x 2 2x + 1 , x 1 x+3

114

Mathematics

3. Multiply the rational expressions in each case :


7x + 2 x +1 and 2 2 7x 5x 2 2x + 3x + 1

(i)

(ii)

x3 + 1 x3 1 and x4 + 1 x4 1

17x + 3 3x 2 15x + 18 and (iii) x 2 6x + 9 2x 4

4. Divide x 2 11x 26 x 2 + 7x + 10 by x 2 4x 45 x 2 12x 13 6x 2 + x 1 4x 2 + 4x + 1 by . 4x 2 9 2x 2 7x 15

(i)

(ii)

LET US SUM UP
z z z z

(x + y)2 = x2 + 2xy + y2 (x y)2 = x2 2xy + y2 x2 y2 = (x + y) (x y) (x + y)3 = x3 + 3x2y + 3xy2 + y3 = x3 + 3xy(x + y) + y3

(x y)3 = x3 3x2y + 3xy2 y3 = x3 3xy (x y) y3

z z z z z

x3 + y3 = (x + y) (x2 xy + y2) x3 y3 = (x y) (x2 + xy + y2) x3 + y3 + z3 3xyz = (x + y + z) (x2 + y2 + z2 xy yz zx) x2 + (a + b)x + ab = (x + a) (x + b) HCF LCM = Product of Two Polynomials HCF of two polynomials =

Product of Two Polynomials LCM of the Polynomials

Special Products and Factorisation

115

LCM of two Polynomials =

Product of Two Polynomials HCF of the Polynomials

HCF LCM Unknown Polynomial = Given Polynomial A rational expression is an expression obtained by dividing one polynomial by the other. Reduction of rational expression into lowest forms means cancelling a common factor, if any, from the polynomials in the numerator and denominator. TERMINAL EXERCISE

z z

1. Find each of the following products : (i) (3x + 4)2 (iv) 1I FH x 2 K


2

(ii) (4x 3)2 (v)

(iii) (2ab + 3c)2

FH 2x + 5I K 3

2. Calculate each of the following without actual multiplication : (i) (105)2 (iii) (10.2)2
2 3. Find x +

(ii) 992 (iv) 952

1 if x2
1 (b) x x = 3

1 (a) x + x = 5 4. Evaluate each of the following : (i) (3x 1) (3x + 1) (iii) 97 103

(ii) (2x 3y) (2x + 3y) (iv) 1001 999

5. Find each of the following products : (i) (x + 2)3 (iii) (2ab 3cd)3 (ii) (2x 1)3 (iv)

FG 2x + 1 IJ H 2x K

6. Evaluate each of the following, without actual multiplication : (i) 493 (ii) 1053

116

Mathematics

7. Factorise each of the following : (i) 64x2 1 (iii) x6 + 1 (v) 25x2 90xy + 81y2 : 8. Factorise each of the following by splitting the middle term : (i) 2x2 + 17x + 8 (iii) 8x2 + 6x 5 (ii) 3x2 + 2x 8 (iv) 8x2 6x 5 (ii) a3 + 343 (iv) x6 1

9. Find the HCF and LCM of each of the following : (i) x2 8x + 7 and x2 10x + 21 (ii) (2x + 1)2 (4x 3)2 (x 2)3 and (2x + 1) (4x 3)4 (iii) 8(x 2)2 (4x 1)3 (3x + 1) and 12 (x 2)3 (4x 1) (3x + 1)4 10. The HCF of two polynomials x2 5x + 6 and x2 7x + 12 is x 3. Find the LCM of the polynomials. 11. The LCM of two polynomials (2x + 1)3 (3x 5)2 and (2x + 1)(3x 5)3 is (2x + 1)3 (3x 5)3. Find the HCF of the polynomials. 12. The HCF and LCM of two polynomials are (x2 1) and (x4 1) (x4 + x2 + 1) respectively. If one polynomial is x4 1, find the other polynomial. 13. Reduce each of the followng rational expressions to their lowest forms : (i) 6x 2 + 5x 6 12x 2 + 7x 10 (ii) 4x 2 25 2x 2 + 11x + 15

x6 + a 6 (iii) 2 x + a2 14. Perform the indicated operation : x+2 + x+3 x+3 x+2 2 (iii) x + 1 2x + 1 3x 3 x 1 (i) 15. Perform the indicated operation : (i)

x8 y8 (iv) 6 x y6

(ii)

2x 1 x 3 x+5 x+2

bx + 3gdx

8x + 15

x2 9

i 2x + 5
x+5

(ii)

3x 2 6x + 3
2

4x 2 + 12x + 5 6x 2 + x 35 (iii) 2x + 3 4x 2 + 8x + 3

(iv)

FG x Hx

b3x + 3g 2b x + 1g bx 5g
3

y 3 x 2 + xy + y2 x+y 2 3 + y3 2 xy x xy + y

IJ K

Special Products and Factorisation

117

ANSWERS Check Your Progress 5.1 1. (i) 25x2 + 10xy + y2 (ii) a2c2 + 2abcd + b2d2
2 (v) x +

(ii) x2 6x + 9 (iv) a2b2 2abc + c2 (vi) 4x2 20xy + 25y2 (viii) (x) (ii) 40401 (v) 998001 (ii) 6 (v) 2 z2 z +1 4
x 2 y2 + +2 y2 x 2

1 2 x2

(vii)

x 2 2x + +1 9 3

(ix) a4 + 2a2b2 + b4 2. (i) 9409 (iv) 2401 3. (i) 2 (iv) 17 Check Your Progress 5.2 1. (i) y2 4 (iv) x2 y4 Check Your Progress 5.3 (i) 8x3 + 12x2y + 6xy2 + y3 (iii) x3y3 3x2y2z + 3xyz2 z3 (v) 912673 Check Your Progress 5.4 (i) 8x3 + 36x2y + 54xy2 + 27y3 (iii) x3y3 3x2y2 + 3xy 1 (v) x3 3x + (vii) 8x3 + 1 (ix) 1 + x3 2. (i) 132651 (ii) 1061208

(iii) 996004 (vi) 10404 (iii) 11 (vi) 68 and 16.

(ii) x2y2 1 (v) 9996

(iii) a4 25 (iv) 6375

(ii) 125x3 75x2y + 15xy2 y3 (iv) 970299 (vi) 110592

(ii) 64x3 240x2 + 300x 125 (iv) x3y3 + 3x2y2z + 3xyz2 + z3 (vi) a3x3 3a2bx2y + 3ab2xy2 b3y3 (viii) y3 27 (x) 8x3 + y3 + 9z3 18xyz (iii) 857375

3 1 x x3

118

Mathematics

Check Your Progress 5.5 (i) (2a + 3)(2a 3) (iii) c(a + b)(a b) (v) (5x + 4)(5x + 4) (vii) (x + 6)(x + 6) Check Your Progress 5.6 1. (i) (u + v)(u v) (iii) (x2 + y2) (x + y) (x y) (v) (a + 6b) (a2 6ab + 36b2) (ii) (5a + 1)(5a 1) (iv) (x 2y)(x2 + 2yx + 4y2) (vi) (a 7) (a2 + 7a + 49) (ii) (2a + b) (2a b) (iv) (3x + 5y)(3x 5y) (vi) (x 3) (x 3)

(vii) (x4 + 1) (x2 + 1) (x + 1) (x 1) (viii)(a + 2)3 (x) (x + 11)2 (xii) (9y 2x)2 2. (i) 7000 (iii) 6600 (v) 2 (vii) 19 Check Your Progress 5.7 (i) (x + 6)(x + 1) (iii) (x 7)(x 3) (v) (x 4)(x + 2) Check Your Progress 5.8 1. (x + 5)(x + 4) 3. (6x 5)(x + 2) 5. (2x + 5)(2x + 1) 7. (x + 10)(x 6) 9. (3x 2)2 2. (x + 5) (x 4) 4. (x 4)(x + 2) 6. (x 8) (x + 7) 8. (3x 5) (x + 1) 10. (5x 3)(3x 1) (ii) (x 6) (x 1) (iv) (x 5)(x + 1) (vi) (z 5) (z + 3) (ii) 400 (iv) 13 (vi) 91 (ix) (x 11)3 (xi) (3x 5y)2

Special Products and Factorisation

119

Check Your Progress 5.9 (i) (x + 1)2 (iii) x + 2 (v) (2x 5)2 Check Your Progress 5.10 (i) (x + 1)3(x 1) (iii) (x + 2)3(x2 2x + 4) (v) (2x 5)3(x + 4)3(x 4) Check Your Progress 5.11 1. (i) (x + 2)2 ; (x + 2)3 (ii) x + 3 ; (x + 3)2(x2 3x + 9) (ii) x2 + 4x + 4 (iv) (x + 1)2(x + 5)3 (ii) x + 2 (iv) (x + 5)2

(iii) (x + 3)3(x + 1)2 ; (x + 3)4(x + 1)3 (iv) x + 10 ; (x2 + 4x 60)(x 5) (v) x 7 ; (x 7)3(x + 5) (vi) x2 x + 1 ; x3 + 1 (viii)1 ; (x2 x + 1)(x +1)2 (x) x2 + xy + y2 ; x3 y3 2. x + 1 Check Your Progress 5.12 1. (i), (ii)
2 2 2. (a) For example x + 1 , x 1 , 2 x etc. x +1 x +1 x 1

(vii) 1 ; x6 1 (ix) 1 ; (3x2 5x + 2) (x2 4)

3. x4 + x3 x 1

(b)

x2 + y2 , x+y

x + 2y x3 + y3 , 3x + 5y x y etc

Check Your Progress 5.13 (i) (iii) x 1 2x 3


2 (ii) 4 x 2 x + 1 2x 1

x2

x 1 x +1

(iv)

x 3 + x 2 y + xy 2 + y 3 1

(v) 1

120

Mathematics

Check Your Progress 5.14 1. (i) 2. (i)

b gb
b

5x 2 + 4 x 5 x + 2 2x 3

(ii)

5x 3 4 x 2 2 x x2 1

2 x 2 7 x 9 2x + 1 x + 2

gb g

3 2 (ii) x 11x + 2 x 4 x + 3 x 1

3. (i)

1 2x + 1 x 1

gb g
2

(ii)

b gb g dx x + 1idx + x + 1i dx + 1idx + 1i
2 2 4 2

(iii) 4. (i)

51x + 9 2x 6

bx + 1gbx 13g bx 9gbx + 5g

(ii)

b3x 1gb2x 3g bx 5gb2x + 1g

Terminal Exercise 1. (i) 9x2 + 24x + 16 (iii) 4a2b2 + 12abc + 9c2 (v) 4 x 2 + 20 x + 25 9 3 (ii) 9801 (iv) 9025 (b) 11 (ii) 4x2 9y2 (iv) 999999 (ii) 16x2 24x + 9 (iv) x2 x + 1 4

2. (i) 11025 (iii) 104.04 3. (a) 23 4. (i) 9x2 1 (iii) 9991 5. (i) x3 + 6x2 + 12x + 8 (ii) 8x3 12x2 + 6x 1

(iii) 8a3b3 36a2b2cd + 54abc2d2 27c3d3

3 1 (iv) 8x3 + 6x + 2 x + 3 8x
6. (i) 117649 7. (i) (8x + 1)(8x 1) (iii) (x2 + 1)(x4 x2 + 1) (ii) 1157625 (ii) (a + 7) (a2 7a + 49)

Special Products and Factorisation

121

(iv) (x + 1)(x 1)(x2 x + 1)(x2 + x 1) (v) (5x 9y)2 8. (i) (2x + 1)(x + 8) (iii) (4x + 5)(2x 1) 9. (i) x 7 ; (x2 8x + 7)(x 3) (2x + 1)2 (4x 3)4(x 2)3 24(x 2)3(4x 1)3 (3x + 1)4 (ii) (3x 4) (x + 2) (iv) (2x + 1)(4x 5)

(ii) (2x + 1)(4x 3)2 ;

(iii) 4(x 2)2(4x 1)(3x + 1) ; 10. (x 2)(x2 7x + 12) 11. (3x 5)2(2x + 1) 12. (x2 1)(x4 x2 + 1) 13. (i) 2x + 3 4x + 5 x4 x2a2 + a4

(ii)

2x 5 x+3

(iii)

(iv)

dx
x2

+ y2 x4 + y4
2

+ xy + y 2

id idx

xy + y 2

14. (i) (iii)

2 x 2 + 10x + 13 x+3 x+2

b gb g

(ii)

b gb g
27 x 1

x 2 + x + 13 x+5 x+2

15. (i)

b g b2x + 5gbx 5g
x+5
2

x 2 6x 2 3 x 1

(ii)

b g 2b x 5g

(iii)

b2x + 1g
3x 7

(iv) 1

122

Mathematics

6
Linear Equations
6.1 INTRODUCTION Consider the following statements (i) 6 + 5 = 11 (iii) 8 7 = 56 (v) 16 4 = 4 (ii) 7 + 6 = 18 (iv) 6 17 = 11 (vi) 18 7 = 116

Of the above statements, some are true, whereas the others are false. Can you say which of them are true and which are false ? Of course, your reply will be that (i), (iii), (iv) and (v) are true statements whereas (ii) and (vi) are false statements. We are able to call them true or false after finding their value. All the above statements (i) to (vi) are called numerical statements. Now consider the statements : (i) x + 10 = 22 (ii) x + y + 2 = 14 (iii) 2x y + 4 = 20 Statements (i), (ii) and (iii) can be true for some specific value of the variables involved. In (i), if x equals 12, it is true. For all other values of x, (i) is false. Similarly, (ii) is true for x = 10, y = 2. Can you see that (ii) is true for some other pairs of values of x and y. The same thing can be said for (iii) also. (i), (ii) and (iii) are all called equations, because they involve two statements involving variables and constants connected by a sign of equality. Further, (i) involves one variables x, whereas (ii) and (iii), involve two variables x and y. Thus, (i) is an equation in one variable, whereas and (ii) and (iii) are equations in two variables.

Linear Equations

123

In this lesson, we shall study about linear equations in one and two variables and discuss about methods of finding their solutions algebraic as well as graphic. We shall also solve system of two linear equations graphically and algebraically. We shall also solve word problems involving one or two variables. 6.2 OBJECTIVES After studying this lesson, the learner will be able to :
z z z z z z z z z z

identify a linear equation from a given collection of equations. cite examples of linear equations. write a linear equation in one variable and also give its solution. write a linear equation in two variables draw the graph of a linear equation in two variables. find the solution of a system of two linear equations graphically. check-the consistency or otherwise of systems of linear equations solve system of equations algebraically. translate a word problem into a linear equation in one or two variables. solve real life problems involving linear equations in one or two variables.

6.3 EXPECTED BACKGROUND KNOWLEDGE


z z z z z z

Algebraic expressions and polynomials in one or two variables. Four fundamental operations on algebraic expressions. Linear polynomials in one or two variables. Four fundamental operations on numbers. Knowledge of exponents. Plotting of points on a graph.

6.4 EQUATIONS IN ONE VARIABLE If one equation has only one variable, it is said to be an equation in one variable or one unknown. There are equations having more than one unknown. You will learn about them later. 6.4.1 Degree of an Equation Consider the following equations (i) 3x 4 = 8 (iii) x2 16 = 0 (ii) (x 4) 8 = 2(x + 3) (iv) x3 3x = x.

Each of these is an equation in one unknown, namely x.

124

Mathematics

You will observe that the greatest exponent of the variable in (i) and (ii) is one. In equation (iii), the greatest exponent of x is 2 and in equation (iv) it is 3. Equation (ii) can be simplified as follows : x 4 8 = 2x + 6 or or or x 2x 12 6 = 0 x 18 = 0 x + 18 = 0

The greatest exponent of the variable in an equation in one unknown is called the degree of the equation. Thus, the degree of equations (i) and (ii) is one whereas, the degree of equation (iii) is 2 and of the equation (iv) is 3. An equation in which the greatest exponent of the variable after simplification is one, is called a linear equation. Thus, the degree of a linear equation is one. Let us take some example to identify linear equations. Example 6.1 : Which of the following equations are linear equations ? (i) 5x 3 = 5 (ii) 8m 3 = 12 (ii) 4x + 4 = 4 (iv) 2(x + 3) 1 = x2

Solution : (i) , (ii) and (iii) are equation of degree one and hence linear equations whereas equation (iv) is of degree 2 and hence is not a linear equation. 6.5 GENERAL FORM OF A LINEAR EQUATION A linear equation in one unknown can be written as : ax + b = 0 where a and b are real numbers and a 0. 6.6 SOLUTION OF A LINEAR EQUATION The value (or values) of the variable (or variables) which make the equation a true statement is (are) called solution(s) of the equation. By solving an equation, we mean finding the solution (or solutions) of the equation. For example, x = 2 is a solution of the equation x + 3 = 5 because on substituting 2 for x we get,

Linear Equations

125

2 + 3 = 5 which is a true statement. Note. A solution is also called a root of the equation. Let us now solve some linear equations in one unknown. Example 6.2 : Solve each of the following equations : (i) x 12 = 7 (iii) 3(x + 5) = 24 Solution : (i) x 12 = 7 Adding 12 on both sides, we get, x 12 + 12 = 7 + 12 or x = 19 Thus, x = 19 is a solution of the given equation. Does it satisfy the given equation ? This can easily be checked. Check : Substituting 19 in place of x in the given equation, we get, 19 12 = 7 or

(ii)

y +1 = 7 6

(iv) 5(m 3) = 19 + 3(m 2).

7 = 7 which is a true statement. x = 19 is a solution of the given equation.

(ii)

y +1 = 7 6

or or or or

y =7 1 6 y =6 6

(Subtracting 1 from both sides)

y=6 6 y = 36

(Multiplying both sides by 6)

Check : Substituting 36 for y, we have


36 +1 = 7 6

or

6 + 1 = 7 which is true. y = 36 is the required solution.

126

Mathematics

(iii) 3(x + 5) = 24 or 3x + 15 = 24 or or or or 3x = 24 15 3x = 9 x=
9 3

(Subtracting 15 from both sides)

(Dividing both sides by 3)

x = 3.

(iv) 5(m 3) = 19 + 3(m 2) or or or or or or or

5m 15 = 19 + 3m 6 5m 15 = 13 + 3m 5m 15 3m = 13 2m 15 = 13 2m = 13 + 15 2m = 28 m = 28 2 m = 14

Thus, m = 14 is the required solution of the given equation. From the above examples, we see that A linear equation in one variable has only one solution. Similarly, you can solve the general equation ax + b = 0, (a 0). ax + b = 0 Subtracting b from both sides, we get ax = 0 b or ax = b Dividing both sides by a (a 0). we get, b x= a b Check : Substituting for x, we get a b a + b=0 a or b + b = 0 which is true for every value of b. b x = is the solution of the equation ax + b = 0. a

Linear Equations

127

CHECK YOUR PROGRESS 6.1 1. Which one of the following statements is a numerical statement ? (i) 65 + 1 = 70 (iii) x + 2 = 3 (ii) 7 2 + x = 3 (iv) x 16 = 8

2. Which of the following is an open statement ? (i) 7 + 7 = 14 (iii) 3 4 + 1 = 13 (ii) 6 x = 5 (iv) 8 3 + 1 = 25

3. Which of the following open statements is true for x = 1 ? (i) x 2 = 4 (iii) 3x = 3 (ii) x 5 = 6 (iv) 2x 7 = 0

4. Which of the following algebraic statements is false for p = 3 ? (i) p 7 = 4 (iii) 4p + 1 = 13 5. The degree of a linear equation is : (i) 1 (ii) 2 (iii) 3 (iv) 0. (ii) p + 3 > 0 (iv) 5p 2 = 5

6. Which of the following equations is of degree one ? (i) x2 1 = 0 (iii) 2x = 3 (ii) x3 1 = 0 (iv) x0 = 1

7. Which of the following numbers is the root of the equation 2(x + 3) = 18 ? (i) 21 (ii) 13 (iii) 12 (iv) 6

8. The root of 2m 1 = 17 is : (i) 18 (ii) 16 (iii) 9 (iv) 8

9. The degree of the equation 2z 4 = 3(z 4) is : (i) 16 (ii) 8 (iii) 1 (iv) 0

10. The value of p for which the equation 2p (4 p) = 5 p becomes true is : (i) 4.5 (ii) 3 (iii) 2.25 (iv) 0.5

11. Solve each of the following linear equations for x : (i) 2x + 4 = 14 (ii) 2x + 6 =x 3 (iii) 4 2x + x + 1 =1 3 2

128

Mathematics

6.7 WORD PROBLEMS In our day-to-day life, we come across many problems which can be solved using equations. To do so, follow the steps given below : 1. Represent the unknown by an alphabet say x,y,z,n,p,t etc.; 2. Translate the given statement into an equation 3. Solve the equation, and 4. Check the value found, by putting it in the original problem. Let us now take some examples. Example 6.3 : Which number when added to 6 gives 8 ? Solution : Let x be the given number 6 added to x mean x + 6 According to the question x + 6=8 This is a linear equation in x. Solving it for x, we get x = 2 i.e., 2 added to 6 gives us 8. Yes, this is true
The required number is 2.

Example 6.4 : The perimeter of a square is 64 cm. Find its side. Solution : Let the side of the square be x cm. Then, Perimeter = 4x
The equation becomes 4x = 64

or or

x = 64 4 x = 16

The side of the square is 16 cm.

Example 6.5 : Renu is 20 years younger to her mother. After 10 years, her mother will be twice as old as Renu will be then. How old is Renu now ? Solution : Let Renus present age be x years. Her mothers present age = (x + 20) years After 10 years,

Linear Equations

129

Renus age will be = (x + 10) years and her mothers age will be[(x + 20) + 10] years According to the problem, Renus mother will be 2 times as old as Renu after 10 years.

(x + 20) + 10 = 2(x + 10) x + 20 + 10 = 2x + 20 x + 30 = 2x + 20 x 2x = 20 30 x = 10

or or or or

i.e., Renu is presently 10 years old Check : Renus present age = 10 years Renu's mother's age = 30 years After 10 years, Renus age = 10 + 10 = 20 years After 10 years, Renus mothers age = 30 + 10 = 40 years and 40 is twice of 20 Thus, Renus present age is 10 years. Example 6.6 : Twice a certain number increased by 10 equals 32. Find the number. Solution : Let the number be x Then, twice the number = 2x
The equation is

2x + 10 = 32 or or or or 2x = 32 10 2x = 22 x = 22 2 x = 11

Thus, the required number is 11. CHECK YOUR PROGRESS 6.2 1. A man is 20 year older than his son. After 10 years his age becomes twice the age of his son. Find their present ages. 2. The denominator of a fraction is 2 more than its numerator. If 1 is added to each, the 2 fraction becomes . Find the fraction. 3

130

Mathematics

3. The sum of three consecutive natural numbers is 42. Find the numbers. 4. 4 added to 5 times a number equals 2 subtracted from 7 times the number. Find the number. 5. The angles of a triangle are such that sum of two angles equals the third and the ratio between the acute angles is 2 : 3. Find the angles of the triangle. 6.8 A LINEAR EQUATION IN TWO VARIABLES Recall that ax + b = 0 is the general form of a linear equation in one variable. A linear equation in two unknowns is an equation which after simplification contains two unknowns, each one of them in a separate term and having the exponent one. Given below are some linear equations in two unknowns : 1. 2x = 5y 7 2. 3.
3 3 x = 5 y 2 2 2 x + 3y = 6 2

4. x y =

5. 2x 1 = (y 3) + (2y x) A general linear equation in two unknowns is written as ax + by + c = 0 where a, b and c are real numbers with a 0 and b 0. 6.9 THE GRAPH OF A LINEAR EQUATION IN TWO UNKNOWNS An equation in two unknowns represents a relationship between the unknowns, and its graph is the diagrammatic form of this representation. (1) Representation of a Point in a plane In order to draw the graph of any equation, your must first know how to plot points in a plane (i.e. on a sheet of paper). This plane is called the coordinate plane or cartesian plane. You already know that a point on the number line represents a real number. Similarly, a point on a plane represents an ordered pair of numbers i.e., a pair of numbers taken in a fixed order. Let us now understand how to represent a point in a plane. For that we go through the following steps :

Linear Equations

131

Step 1 : Two perpendicular lines XOX and YOY are drawn intersecting each other at O. They are called the x-axis and y-axis respectively, and O is called the origin (See Fig. 6.1).

Fig. 6.1

Fig. 6.2

Step 2. These two coordinate axes divide the plane into four parts, each of which is called a quadrant (See Fig. 6.2) Step 3. The integers are marked along both the axes with O representing as zero on both the axes. (See Fig. 6.3).

Fig. 6.3

Fig. 6.4

Along the x-axis, positive integers are marked to the right of O, and negative integers to the left of O. Along the y-axis, positive integers are marked above O, and negative integers below O. Thus, the two axes can be taken as two perpendicular number lines. To represent on ordered pair say (2, 3), we proceed as follows :

132

Mathematics

Step 4. Mark a point A on mark 2 on x-axis and draw a line l through A parallel to the y-axis (See Fig. 6.4). Step 5. Mark a point B on mark 3 on y-axis and draw a line m through B parallel to the x-axis (See Fig. 6.5). l and m intersect each other at point P. P represents the point (2, 3) because P is at a distance of 2 units measured along the x-axis and 3 units measured along the y-axis. Let us take another example. Example 6.7 : Represent the point (3, 5) on the coordinate plane. Solution. Let us first draw the x-axis and the y-axis and mark points on them representing integers as in Fig. 6.6. The first number in the ordered pair is 3, and so we will take 3 units along the x-axis to the left of O. Similarly take 5 units along the y-axis above O and draw lines l and m parallel to the y-axis and the x-axis, respectively. The point A represents (3, 5) as in Fig. 6.7.
Fig. 6.5

Fig. 6.6

Fig. 6.7

Similarly, you can represent any ordered pair (x, y) by a point in the cartesian plane. Note that : 1. The first number in the ordered pair tells us the number of units to be taken along the x-axis and is thus called the x-coordinate or the abscissa of the point.

Linear Equations

133

2. The second number in the ordered pair tells us the number of units to be taken along the y-axis and is thus called the y-coordinate or the ordinate of the point. 3. If (x, y) is a point P in the cartesian plane, then P lies in : (i) the first quadrant if both x and y are positive real numbers; (ii) the second quadrant if x is negative and y is positive; (iii) the third quadrant if x and y both are negative; (iv) the fourth quadrant if x is positive and y is negative. What happens to P if y = 0 ? Obviously P is then x units along the x-axis and zero units along the y-axis and thus lies on the x-axis.
Any point on the x-axis has the coordinates (a, 0).

Similarly, a point on the y-axis will have the x-coordinate equal to zero, and will thus have the coordinates (0, b). Thus, O has the coordinates (0, 0). Therefore, given an ordered pair of numbers, you can easily find the point in the plane which corresponds to this ordered pair.

CHECK YOUR PROGRESS 6.3 Represent the following ordered pairs as points in the cartesian plane : (i) (5, 2), (3, 2), (3, 4), (2, 0) and (0, 5) (ii) (5, 2), (3, 2), (3, 4), (2, 0), (0, 5) and (5, 4) 6.10 FINDING ORDERED PAIR CORRESPONDING TO A POINT IN THE PLANE Given a point in a plane, can we associate an ordered pair of numbers with it ? Yes, we can find its coordinates with respect to a given pair of axes. Let us take an example to illustrate this. Example 6.8 : P is a point in a plane. Find its coordinates with respect to the coordinate axes XOX and YOY. Solution : Step 1 : Through P, draw a line l parallel to the y-axis and mark A as the point of

134

Mathematics

intersection of l with the x-axis (See Fig. 6.8). If OA = a units, the point A has the coordinates (a, 0). Step 2 : Through P draw a line m parallel to the x-axis and mark B as the point of intersection of m with the y-axis (See Fig. 6.9).

Fig. 6.8

Fig. 6.9

If B is b units from O, along the y-axis, then B has the coordinates (0, b).
The y-coordinate of P is b.

Hence the coordinates of P are (a, b), i.e., the ordered pair associated with P is (a, b). CHECK YOUR PROGRESS 6.4 Find the coordinates of the points P, Q, R and S in Fig. 6.10. 6.11. GRAPH OF A LINEAR EQUATION Let us draw the graphs of some linear equations. Remember that : (i) If a point lies on the graph of an equation, its coordinates satisfy the equation. (ii) If a point is not on the graph of an equation, its coordinates do not satisfy the equation. Here are some examples of graphs of linear equations.

Fig. 6.10

Linear Equations

135

Example 6.9 : Draw the graphs of the following equations. (i) x 2y = 0 (ii) 2x 5y 10 = 0.

Solution : In order to draw the graph of a linear equation, we plot at least three ordered pairs satisfying the equation. The steps followed are as given below. Equation (i) x 2y = 0 Step 1 : Transform the equation into the x-form or the y-form. Step 2 : Substitute three values of x to get the corresponding three values of y. y-form of the equation is : y = (i) x = 0 gives y =
0 =0 2 2 =1 2 6 =3 2 x 2

(ii) x = 2 gives y =

(iii) x = 6 gives y =

Three ordered pairs satisfying the equation are (0, 0), (2, 1) and (6, 3)

(0, 0), (2, 1) and (6, 3) are three solutions of x 2y = 0 Table x 9 0 0 2 1 6 3

Step 3 : Write the ordered pairs satisfying the given equation in the form of a table.

Step 4 : Plot the ordered pairs (0, 0) (2, 1) and (6, 3) in the same cartesian plane. (See Fig. 6.11) Step 5 : Join the point O, A and B and extend the line segment to get the line l (as Fig. 6.12). l is the, required graph of the equation x 2y = 0.

Fig. 6.11

6.12

136

Mathematics

Equation (ii) 2x 5y 10 = 0 or 2x = 5y + 10 or x =
5 y+5 2

Table x y 10 2 5 0

This is the x-form of the equation. Three of its ordered pairs are (10, 2), (5, 0) and (0, 2).

0 2

Plotting the ordered pairs (10, 2), (5, 0) and (0, 2), we get the three points P, Q and R. Join the points P, Q and R to get the line m (as shown in Fig. 6.13). m is the required line which is the graph of the linear equation 2x 5y 10 = 0

Fig. 6.13

Notes 1. In fact, we an draw the graph of a linear equation by plotting two ordered pairs only because two points determine a line uniquely. Plotting the third pair helps us to verify the correctness of the graph. 2. The graph of a linear equation in two unknowns is a straight line which is neither the x-axis, nor the y-axis and is not parallel to either of the axis. This is because for a linear equation in two unknowns, ax + by + c = 0, neither a nor b is zero. But what happens if out of a and b one of them is zero ? Let us take an example. Example 6.10 : Draw the graphs of each of the following equations : (i) x = 2 (ii) 2y + 5 = 0 Table x y 2 0 2 2 Solution : (i) The equation x = 2 can be written as x + 0.y = 2. Hence, some of the ordered pairs satisfying it are (2, 0), (2, 2), (2, 1), (2, 3), ... We plot three ordered pairs, say (2, 0), (2, 2) and (2, 1)

2 1

Linear Equations

137

We get three points A (2, 2), B (2, 0) and C (2, 1) on the cartesian plane. Join them to get the line l as in Fig. 6.14. Thus, l is the required graph of x = 2. Note that l is parallel to the y-axis. (ii) 2y + 5 = 0 This equation can be written as 0.x + 2y + 5 = 0 and the y-form of this equation is y= or
5 2

Table x 1 0 1
Fig. 6.14

y = 2.5

y 2.5 2.5 2.5

Any point whose y-coordinate is 2.5 is a solution of this equation. Three of the ordered pairs satisfying the equation are (1, 2.5), (0, 2.5) and (1, 2.5)

Plotting these ordered pairs, we get the three points M, N and R. Join them to get the line p (See Fig. 6.15) p is the required graph of the equation 2y + 5 = 0 This is a line parallel to the x-axis. Notes : 1. The graph of a linear equation in one unknown is either the x-axis or the y-axis, or is a line parallel to either of the axes. 2. The y-axis is the graph of the equation x = 0. 3. The x-axis is the graph of the equation y = 0. CHECK YOUR PROGRESS 6.5 1. Draw the graphs of x = 0 and of y = 0. 2. Draw the graphs of each of the following equations : (i) 2x 3y = 1 (iv) 2x + 3 = 0 (vii) x = 5 (ii) 3x + y = 4 (v) x + y = 0 (viii) 2x 3y = 8 (iii) y = 5 (vi) 3x + 3y = 6 (ix) x y = 0
Fig. 6.15

138

Mathematics

3. Which of the following ordered pairs make the equation 2x + y = 10 a true statement? (i) (0, 4) (ii) (0, 10) (iii) (8, 8) (iv) (3, 16) 4. State which of the following ordered pairs are not the solutions of the equation 4x = 3y + 8. (i) (7, 2) (ii) (5, 4) (iii) (8, 8) (iv) (0, 0).

6.12 THE SYSTEM OF LINEAR EQUATIONS IN TWO UNKNOWNS You know that ax + by + c = 0 (where a, b 0 and a, b, c are real numbers) is a linear equation in two unknowns (variables). A system of linear equation in two unknowns is given as : ax + by + c = 0 a'x + b'y + c' = 0 where a, b, c, a', b' and c' are all real numbers and a, b, a' and b' are non-zero. Let us now learn to solve a system of linear equations in two unknowns. 6.13. GRAPHICAL SOLUTION OF A SYSTEM OF LINEAR EQUATIONS In order to solve a system of linear equations in two unknowns graphically, we draw the graphs of the equations in the same cartesian plane and find the point(s) of intersection of their graphs. Let us take some examples. Example 6.11 : Solve the following system of linear equations graphically : x 2y = 7 x + y = 2 Solution : You have already learnt how to draw the graph of a linear equation in two unknowns. The graphs of these equations are straight lines. x 2y = 7 x = 7 + 2y Three of ordered pairs satisfying it are (1, 3), (7, 0) and (3, 2) Also, or x + y = 2 y = x 2 x y x y Table 1 x 2y = 7 1 3 7 0 3 2

Table 2 x + y = 2 0 2 2 0 1 3

Three of ordered pairs satisfying it are (0, 2), (2, 0) and (1, 3)

By plotting these ordered pairs we get a pair of straight lines as the graphs of the given equations (See Fig. 6.16).

Linear Equations

139

Fig. 6.16.

Now, how many ordered pairs are the solutions of both the equations ? In other words, how many points are common to both the lines l and m ? From the graph, it is clear that there is only one such point, namely, P, whose coordinates are (1, 3). Thus, x = 1 and y = 3 is the solution of the given system of equations. Example 6.12 : Solve graphically the following system of linear equations : 4x + 3y = 24 3y 2x = 6 Solution : 4x + 3y = 24 or y= 24 4 x 3 Table 1 4x + 3y = 24 Three of ordered pairs satisfying it are (6, 0), (0, 8) and (3, 4) x y 6 0 0 8 3 4

Plotting these ordered pairs, we get three points A (6, 0), P (3, 4) and B (0, 8) and on joining them, we get line l as the graph of 4x + 3y = 24. Also, or 3y 2x = 6 y= 6 + 2x 3 x y Table 2 3y 2x = 6 Three of ordered pairs satisfying it are (0, 2), (3, 0) and (3, 4). 0 2 3 0 3 4

140

Mathematics

Plotting these ordered pairs in the same cartesian plane we get three points M(0, 2), N(3, 0) and P (3, 4), and on joining them, we get the line m as the graph of 3y 2x = 6 (See Fig. 6.17).

Fig. 6.17

To get the common solution of both the equations, we look for the points that are common to both the lines l and m. The only point common to both the lines l and m is point P with coordinates (3, 4). In other words x = 3 and y = 4 is the only solution of the given system of linear equations Example 6.13. Draw the graph of : y 2y and find their common Solution. (i) y 2x = 3 4x = 10 solution, if any. 2x = 3

Table y 2x = 3 x 0 1 2 or y = 2x + 3 y 3 5 7 On plotting the ordered pairs (0, 3) (1, 5) and (2, 7), we get the straight line p as the graph of y 2x = 3 (ii) 2y 4x = 10 Table or y = 2x + 5 2y 4x = 10 x 0 1 2 y 5 7 9

Linear Equations

141

On plotting the ordered pairs (0, 5) (1, 7) and (2, 9), we get, the straight line q as the graph of 2y 4x = 10 in the same plane, (See Fig. 6.18).

Fig. 6.18

It is very clear from the graph that the straight lines p and q are parallel, and hence do not have any point common. Thus, there is no solution common to both the equations. Hence, the given system of equations has no solution. Example 6.14 : Draw the graphs of : y 2x = 3 2y 4x = 6 and obtain the common solutions, if any Table 1 y 2x = 3 Solution : or y 2x = 3 y = 2x + 3 x y 0 3 1 5 2 7

Three of its solutions are the ordered pairs (0, 3), (1, 5) and (2, 7) and on plotting these we get three points P, Q and R. On joining them, we get the line l as the graph of y 2x = 3. Similarly, 2y 4x = 6

142

Mathematics

or y = 2x + 3, and so the same ordered pairs satisfy the second equation also (See Fig. 6.19).

Fig. 6.19.

From Fig. 6.19, it is clear that the lines l and m coincide or are the same. This means that all the points of the line l are also on the line m and all the points of line m are also on the line l. Thus all points of the lines l and m are common. Thus, these two equations have an infinite number of common solutions. CHECK YOUR PROGRESS 6.6 1. Draw the graphs of each of the following systems of equations : (i) 2x y 3 = 0 4x 2y 10 = 0 (i) 2x y = 5 x + 3y = 6 (ii) x + y = 7 2x + 6y = 14 (ii) x + y = 8 2x y = 1

2. Solve graphically each of the following system of equations :

3. Find graphically the vertices of a triangle whose sides have the equations 2y x = 8, 5y x = 14 and y 2x = 1 respectively. (Hint : Draw the graphs of the three equations using the same cartesian plane and scale. 1 cm = 1 unit on both axes and find the points of intersection of two lines at a time).

Linear Equations

143

6.14 CONSISTENCY AND INCONSISTENCY OF A SYSTEM OF EQUATIONS You now know, how to solve a system of equations graphically. The system in Example 6.11 and 6.12 had a unique solution. In Example 6.13, the system of equations had no solution, whereas the system in Example 6.14 had infinite solutions. These three examples help you to conclude that : If the graphs of the pair of equations ax + by + c = 0 and a'x + b'y + c' = 0: (i) intersect each other, then the system has one and only one solution, i.e. the system has a unique solution and is said to be a consistent or compatible system. (ii) are parallel straight lines, then the system has no solution, and hence is an inconsistent system. (iii) are one and the same, i.e., are coincident straight lines, the system has infinite number of solutions and thus is a dependent system. This is also called a consistent system of equations. Notes : Any system of two linear equations in two unknowns has to be one of the types of systems described above. Solving the system graphically is not necessary if you want to determine whether the system of equations is consistent or not. There are other algebraic methods to do so. To understand some of these methods, let us study the examples solved in Section 6.14 of this lesson. Method I. Comparison of y-forms Let us compare the y-forms of the equation in Examples 6.12,6.13 and 6.14 and put the result in the form of a table. Equations 1. Example 6.12 4x + 3y = 24 3y 2x = 6 2. Example 6.13 y 2x = 3 2y 4x = 10 3. Example 6.14 y 2x = 3 2y 4x = 6 y= y= 24 4 x 3 2x + 6 3 No solution Inconsistent Unique solution Consistent y-forms Solutions Type of System

y = 2x + 3 y = 2x + 5 y = 2x + 3 y = 2x + 3

Infinite number of solutions

Dependent

144

Mathematics

From the above table you can easily conclude that if in the y-forms of the equations : 1. The coefficients of x are different (even if the constant terms are the same), the system of equations has a unique solution. 2. The coefficients of x are the same but the constant terms are different, the system of equations has no solution. 3. The coefficients of x and the constant terms are the same, i.e., the two y-forms are identical, the system has infinite number solutions. Method 2. The Ratio Method This is another method to determine whether a system of equations is consistent or not. A system of linear equations ax + by + c = 0 and a'x + b'y + c' = 0 is : (i) consistent if a b a b a = b c a b c

(ii) inconsistent if (iii) dependent if

a = b = c a b c

Let us take some examples and illustrate : Example 6.15 : In each of the following, find whether the system is consistent, inconsistent or dependent : (i) 5x + 2y = 16 (ii) 5x + 2y = 16 (iii) 5x + 2y = 15 7x 4y = 2
6 3x + y = 2 5 15 x + 3y = 24 2

Solution : (i) (a) Method 1 : Using y-forms The y-forms of the two equations are : 5 y = x+8 2 and y= 7 x 1 4 2

Since the coefficient of x are different in the two y-forms, the system of equations is consistent. (b) Method 2 : The Ratio Method Here a = 5, b = 2 and c = 16 and a = 7, b = 4 and c = 2

a = 5 b =1 c =8 , and a 7 b 2 c a b , the system of equations is consistent. a b

Since,

Linear Equations

145

(ii) Using y-forms of the two equations, we have 5 5 5 y = x + 8 and y = x + 2 2 3 Since the coefficient of x are the same and the constant terms are different, the system is inconsistent. (iii) The Ratio method Here a = 5, b = 2 and c = 16, a = b = 3 and c = 24

15 2

5 2 a = 15 2 = 3 a b 2 = b 3

and

16 2 = 24 3 a b = c =2 = a b c 3

Hence, the system of equations is dependent. CHECK YOUR PROGRESS 6.7 1. In each case, draw the graphs of the system of equations and state whether the system is consistent, inconsistent or dependent. (i) x+y=6 2 2x + 4y = 24 (ii) x = 5 2x y = 9 (iii) x + 3y = 3 2x 5y = 0

2. Find without drawing the graphs, whether each of the following systems of equations has : (a) a unique solution, or (b) infinite number of solutions or (c) no solution. (i) x + y = 1 4 = 4x 4y (ii) y = 2x 3 x=

y 3 + 2 2

y (iii) x 1 = 3 2
x+y=1

3. Using the Ratio Method find out whether each of the following system of equations is consistent, inconsistent or dependent : (i) 2x + 3y = 13 5x 2y = 4 (ii)

y2 = x+5 7 x + 10 = y 3 3

(iii) 3y = 7x + 21 5y + 2x = 23

146

Mathematics

4. Find the values of k for which the system of equations given below has infinite number of solutions : 3x + y = 0 x ky = 2 6.15 ALGEBRAIC METHODS OF SOLVING A SYSTEM OF EQUATIONS It is not always possible to find the exact solutions of a given system of equations graphically. Some algebraic methods are used to solve the system of equations and these are usually called elimination methods. Some of these methods are discussed below. 6.15.1 Elimination by Comparison Let us understand this method with the help of an example. Example 6.16 : Solve : 2y x = 5 4x 3y = 5 Solution : Method 1. Write the y-forms (or x-forms) of the given equations. Solution of the Example y-forms are : 2y x = 5 or y =
x 5 + 2 2 4x 5 3 3

4x 3y = 5 or y =
x 5 4x 5 + = 2 2 3 3

2. Equate the right hand side (R.H.S.) of the y-forms 3. Solve for equation for x or or or 4. Substance the value of x in one of the equations and get the value of y

3x + 15 = 8x 10 5x = 25 x=5 2y x = 5

or or or

2y 5 = 5 2y = 10 y=5

Thus, x = 5 and y =5 is the solution of the system of equations : 2y x =5 and 4x 3y = 5

Linear Equations

147

Check : Substitute the values 5 for x and 5 for y in both the equations and check whether the solution is correct or not. 2y x = 5 4x 3y = 5 gives gives 255=5 4535=5 True True

Note : It is not necessary to write the equations in y-form only, you can write them in the x-form also and solve them. Do it for Example 6.16 and check. CHECK YOUR PROGRESS 6.8 Solve each of the following pairs of equations by comparing the x-forms or y-forms. 1. x + 2y = 3 x 2y = 1 2. 2x + y = 1 3x y = 4 3. 2x + 3y = 3 3x + 2y = 2

6.15.2 Elimination by Addition or Subtraction Let us take an example to understand this method. Example 6.17. Solve : For x and y 3x 7 = y 4x 5y = 2 Solution : Method 1. Arrange the equations so that the like terms are in the same column. 2. Multiply the equation by suitable constants so that the coefficient of one of the unknowns has the same numerical co-efficient. 3. Eliminate the unknown whose coefficients have the same numerical value by : (a) adding if their signs are unlike (b) subtracting if their signs are like. Solution of the example (i) 3x 7 = y or 3x y = 7

(ii) 4x 5y = 2 or 4x 5y = 2 (i) 5 gives 15x 5y = 35

(ii) gives 4x 5y = 2 Eliminate y : 15x 5y = 35 Subtract : 4x 5y = 2 + 11x = 33 11x = 33

4. Find the value of the unknown left.

x=

or

33 11 x=3

148

Mathematics

5. Find the value of the other unknown by substituting the value of the unknown found in either of the two given equations.

Find y : (ii) is 4x 5y = 2 or or or or 4 3 5y = 2 12 5y = 2 5y = 10 y=2

Thus, x = 3 and y = 2 is the solution of the given system of equations. Check : Substituting the values of x and y in the two given equations, we get 3 3 7=2 or 97=2 True and 4 3 5 2=2 or 12 10 = 2 True x = 3 and y = 2 i.e. the ordered pair (3, 2) is the required solution. Here are some more solved examples. Examples 6.18 : Solve by addition or subtraction each of the following system of equations : (i) 10x + 4y = 20 13x 4y = 66 Solution : (i) 10x + 4y = 20 (ii) 13x 4y = 66 (ii) 3r 5s = 19 2r 4s = 16 ...(1) ...(2)

Here, the coefficient of y in (1) and (2) are numerically the same but opposite in sign. Hence, y can be eliminated by adding the two equations 10x + + 13x 23x or x = 2 4y = = 20 46 4y = 66

Substituting the value of x in (1), we get 10 (2) + 4y = 20 or or 4y = 40 y = 10

Thus, x = 2 and y = 10 is the solution required.


x = 2 and y = 10 or the ordered pair (2, 10) is the required solution.

(ii) 3r 5s = 19 2r 4s = 16 Multiplying (1) by 2 and (2) by 3, we get,

...(1) ...(2)

Here neither the coefficients of r nor of s are the same.

Linear Equations

149

6r 10s = 38 6r 12s = 48 Thus, subtracting (4) from (3), we get 6r 10s = 6r 12 s = or + 2s s = 5 = 10 38 48

...(3) ...(4)

Now, the coefficients of r are numerically the same and have the same sign.

Substituting the value of s in (2) we get 2r 4(5) = 16 or or or 2r + 20 = 16 2r = 4 r = 2

Thus, r = 2 and s = 5 is the required solution.

CHECK YOUR PROGRESS 6.9 Solve each of the following systems of equations and check your answers : 1. 3x + 2y = 11 2x + 3y = 4 2. x + y = 7 3x y = 11 3. 7x 2y = 1 3x + 4y = 15

6.15.3. Elimination by Substitution Let us take an example to understand this method. Example 6.19 : Solve for x and y : x 2y = 7 3x + y = 35 by elimination by substitution. Solution : The equation are : x 2y = 7 3x + y = 35 ...(1) ...(2)

150

Mathematics

Method 1. Express one unknown in terms of the other unknown form either of the equations.

Solution of the Example Expressing x in terms of y from (1), we get x 2y = 7 or x = 7 + 2y 3(7 + 2y) + y = 35

2. Substitute the value of the first unknown in the other equation.

Substituting for x in (2), we get

3. Solve the resulting equation for the second unknown.

Solving for y, we get 3(7 + 2y) + y = 35 or or or or 21 + 6y + y = 35 21 + 7y = 35 7y = 14 y=2 x 2 2 = 7 from (1) or or x4=7 x = 11.

4. Find the value of the first unknown by substituting the value of the second unknown found in either of the two given equations.

Solving for x we get

Thus, x = 11 and y = 2.
x = 11 and y = 2 is the solution of the given system of equations.

Example 6.20 : Solve by elimination by substitution the following system of equations : 3y = 2x 14 and x y = 10 Solution : The system of equations is 3y = 2x 14 and x y = 10 ...(1) ...(2)

Expressing y in terms of x from equation (1) we get y= 2 x 14 3 3

Substituting the value of y in terms of x in (2), we get x 2 x 14 3 3 or

FH

IK = 10

x 2 x + 14 = 10 3 3

Linear Equations

151

or or or

3x 2x + 14 = 30 x = 30 14 x = 16. 3y = 2 16 14

Substituting the value of x in (1), we get or or 3y = 18 y=6

Thus, x = 16 and y = 6 is the solution of the given system of equations. CHECK YOUR PROGRESS 6.10 Solve each of the following system of equations by substitution : 1. 5x + 3y = 17 x + 3y =1 6.16 EQUATIONS REDUCIBLE TO ax + by + c = 0 a'x + b'y + c' = 0 Consider the system of equations given below : 12 =1 u v 3+2 = 5 u v This is not a system of linear equations, but it can be reduced to such a system and then solved. Example 6.21. Reduce 12 =1 u v 3+2 = 5 u v to a system of linear equations and solve the same. Solution. Let 1 1 = x and =y u v x 2y = 1 3x + 2y = 5 ...(1) ...(2) 2.

y x + =1 3 2 3x+ y =1 5 2

The system of equations becomes :

152

Mathematics

This is a system of linear equations which can easily be solved by the methods you have learnt so far. Adding (1) and (2) we get, 4x = 4 or Now or

x = 1 1 2y = 1 2y = 2
1 = 1 u

...[Substituting for x in (1)] or or y = 1 u = 1

and or

y = 1
1 = 1 v

or

v = 1

The solution is u = 1, v = 1.

CHECK YOUR PROGRESS 6.11 1. Solve each of the following system of equations by comparison of x-forms or y-forms and check you answers. (i) 4x 3y = 8 x 2y = 3 (ii) 3y = 12 2x 3x 8y = 7

2. Solve each of the following system of equations by addition or subtraction and check your answers : (i) x + 2y = 24 xy= 4 (iii) 13x + 11y = 70 11x + 13y = 74 (ii) 7x + 6y = 71 5x 8y = 23 (iv) 13x 4y = 57 5x + 2y = 29

3. Solve each of the following system of equations by substitution and check your answers : (i) y = 3x 5 6x = 3y + 3 (iii) y = 2x 6 y= 0 (ii) x + y = a x 2y = b, a and b are constants (iv) p = 2q 1 q = 5 3p

Linear Equations

153

4. If 2x + y = 23 and 4x y = 19, find the value of A = 3x 2y and B = 5y + 2x. [Hint : Find x and y by solving the given equations and substitute their values in A and B] 5. Solve each of the following system of equations and check your answers. 6 (i) 4 p + q = 15 6p 8 = 14 q [Hint. substitute (iii) x y = 0.9 1 = y in (i)] q

y (ii) x + = 4 3 4
3x y = 23

11 2 x+y = 1

b g

[Hint. First simplify the second equation] 6. Find the solution for each of the following systems of equations and check your answers : (i) 3 3 =1 u v 1 + 1 =1 u v 6.17 WORD PROBLEMS You already know that there are many problems in daily life which can easily be solved by using the methods you have learnt in this lesson. For solving these problems, you will have to (i) first express the statements given in the problem algebraically, then (ii) find their common solution. Let us take some examples to understand these two steps. 6.17.1 Expressing the Statements Algebraically Example 6.22 : Express the following statements algebraically : 1. The perimeter of a rectangle is 52 cm. The breadth of the rectangle is 2 cm more than one third its length. Solution : This is the problem discussed in the beginning of the lesson. Let x denote the length and y denote the breadth of the rectangle (ii) 8 9 =1 u v 10 + 6 = 1 u v (iii) 2u + 3v = 6uv 5u 2v = 3uv

154

Mathematics

Statement (i) The perimeter is 52 cm (ii) The breadth is 2 cm more than one-third the length

Equation 2 (x + y) = 52 x y = 2+ 3

Thus, the system of equations representing the statements in the problem is : 2(x + y) = 52 y = 2+
x 3

or

x + y = 26 x + 3y = 6

2. The larger of two numbers is three times the smaller number. Their sum is 8 more than twice the smaller number. Solution. Let x represent the smaller number and y represent the larger number. Then : Statement (i) The larger number is 3 times the smaller number (ii) The sum is 8 more than twice the smaller number y = 3x x + y = 2x + 8 3. The sum of the digits of a two-digit number is 11. If the digits are reversed, the new number is 20 less than twice the original number. Solution; Let t represent the digit in the tens place and u represent the digit in the unit's place of the number. Then, the given number is 10 t + u. When the digits are reversed, the new number is 10u + t Statement (i) The sum of the digits is 11 (ii) The new number is 20 less than twice the given number
The given statements can be expressed as : u + t = 11

Equation y = 3x x + y = 8 + 2x

Thus, the system of equations representing the statement given is

Equation u + t = 11 10u + t = 2(10t + u) 20

and

10u + t = 2(10t + u) 20.

Linear Equations

155

CHECK YOUR PROGRESS 6.12 Express each of the following statements as a system of equations : (i) The perimeter of a rectangle is 72 cm. Three times its breadth exceeds twice the length by 3 cm. (ii) The sum of two numbers is 64 and the larger number is three times the smaller number. 6.17.2 Solving the Word Problems Let us now find solutions to word-problems by taking some examples. Examples 6.23 : Three years back (ago) Atuls age was four times Paruls age then. Five years hence (from now) Atuls age will be two times Paruls age then. Find their present ages. Solutions : Let the present age of Atul be x years and Paruls age be y years Then, the conditions in the problem can be expressed as follows : x 3 = 4(y 3) and or and From (1), we have x = 4y 9 Substituting in (2), we get 4y 9 2y = 5 or or y=7 x = 19. or 2y = 14 x + 5 = 2(y + 5) x 4y = 9 x 2y = 5 ...[Ages 3 years ago] ...[Ages 5 years from now] ...(1) ...(2)

Substituting y = 7 in (2), we get x 2 7 = 5 Hence, Atuls present age is 19 years and Paruls present age is 7 years. Example 6.24 : The length of a rectangle is 5 cm less than twice its breadth. If the perimeter is 110 cm., find the area of the rectangle. Solution : Let the length of the rectangle be x cm and the breadth be y cm. Then, the statements in the problems can be expressed as : x = 2y 5 and 2(x + y) = 110 ...(1) ...(2)

Substituting the value of x in terms of y from (1) in equation (2), we get 2(2y 5 + y) = 110 or 2 (3y 5) = 110

156

Mathematics

or

6y 10 = 110 y = 20

or

6y = 120

Substituting the value of y in (1), we get, x = 2 20 5 or x = 35 = 35 20 cm2 = 700 cm2 Check : Here, length = 35 cm and breadth = 20 cm 2 breadth 5 = 2 20 5 = 40 5 = 35 = length.
The first condition is verified.

Thus, the area of the rectangle = x.y cm2

Also, perimeter = 2(35 + 20) = 2 55 = 110 cm. Hence, the second condition is also verified. CHECK YOUR PROGRESS 6.13 1. Express each of the following statements as a system of equations (a) The sum of two numbers is 12 and their difference is 2. (b) Two numbers are in the ratio 4 : 7. If three times the larger number is added to two times the smaller, the sum is 59. (c) 2 pens and 5 note-books cost Rs 16 and 3 pens and 4 note books cost Rs 17. (d) The sum of the digits of a two-digit number is 10. The number obtained on reversing the digits is one more than twice the given number. (e) The perimeter of a rectangular plot of land is 32 m. If the length is increased by 2m and the breadth is reduced by 1m, the area of the plot remains unchanged (i.e., it is equal to the area of the plot given). 2. Solve each of the following problems by writing each of them as a system of equations : (a) The sum of two numbers exceeds three times the smaller by 3. The difference of the numbers is 5. Find the numbers. (b) A is five years older than B. Five years ago A; was twice as old as B was then. Find their present ages.

Linear Equations

157

(c) The perimeter of a rectangle is 20 cm. If the difference between the length and the breadth is 4 cm, find the length and breadth of the rectangle. (d) The sum of the weights of a father and his son is 105 kg. The weight of the son is one-sixth that of the father. Find their respective weights. (e) 400 people attended a school fete. Some people gave Rs 100 as a donation and some gave Rs 50. The total collection was Rs 34000. Find the number of people who gave Rs 100 each. LET US SUM UP
z z

An equation in one variable of degree one is called a linear equation in one variable. ax + b = 0, a 0 and a, b are real numbers is the general form of a linear equation in one variable. Solution of a linear equation is also called its root. To solve a word problem, it is translated to algebraic statement(s) and then solved. The equation ax + by + c = 0, a 0, b 0 and a, b, c are real numbers is called the general form of a linear equation in two variables. To draw the graph of a linear equation, we find three ordered pairs and plot them. The line joining the points is the graph of the linear equation. The algebraic methods of solving a system of linear equations are (i) elimination by substitution (ii) elimination by equating the co-efficients.

z z z

To solve a word problem, we translate it into linear equations and solve them. TERMINAL EXERCISE

1. Draw the graphs of each of the following system of equations and state whether they have (i) a unique solution (a) x + y = 2 xy=4 (d) x + 2y = 2 2x + 4y = 4 (ii) infinite number of solutions (iii) (b) 2x + 3y = 5 3x 2y = 1 (e) x + y = 0 2x 3y = 0 no solution

(c) x + 2y = 2 2x + 4y = 6

2. Without solving the system of equations, find whether the system is consistent, inconsistent or dependent : (a) 2x + 3y = 1 yx=2 (c) x y = 1 3y 3x = 6 (b) 3x + y = 4 x + 3y = 4 (d)
x + 3y = 2 2 x + 6y = 4

158

Mathematics

3. Solve each of the following system of equations : (a) 3x + y = 2 x 2y = 1 (c) 5xy =1 2 4x + y = 5 4x + y = 5 x + ky = 20 has (a) a unique solution, (b) infinite number of solutions. 5. Solve each of the following system of equations : a b (i) x + y = 2 a b =0 [a, b are non-zero constants, x, y 0] x y 1 (ii) 3x + y = 4 2x 1 = 1 , y 0 y (iii) 1+1 =4 u v (b) x 4y = 3 x+y=2 (d) x + y = 4 x 4y = 4

4. Find the values of k for which the system of equations

21=2 , u, v 0 u v 6. The sum of the digits of a two-digit number is 10. If 18 is added to the number, its digits are reversed. Find the number. 7. The sum of the present ages of A and B is 85 years. Five years ago, A was twice as old as B was then. Find their present ages. 8. Six chairs and two tables cost Rs 240. Also, the cost of a table is Rs 5 less than twice the cost of a chair. Find the total cost of 10 chairs and 3 tables. 9. If the numerator of a fraction is decreased by one, it becomes is increased by 5, the fraction becomes 2 , but if the denominator 3

1 . What is the fraction ? 2 5 times the breadth, find out 2

10. The perimeter of a rectangular is 48 cm. If its length is the length of the rectangle. Also find out its area.

Linear Equations

159

ANSWERS Check Your Progress 6.1 1. (i) 6. (iii) 11. (i) 5 2. (ii) 7. (iv) (ii) 6 3. (iii) 8. (iii) (iii) 5 4. (iv) 9. (iii) 5. (i) 10. (iii)

Check Your Progress 6.2 1. Father : 30 years, Son : 10 year 2. 3 5 3. 13, 14, 15 4. 3 5. 36, 54, 90

Check Your Progress 6.4 P(1, 1), Check Your Progress 6.5 3. (ii) and (iv) 4. (i) and (iv) Q(2, 1), R(2, 2) and S(2 1)

Check Your Progress 6.6 1. Graphs to be drawn. 2. (i) (3, 1) (ii) (3, 5) 3. (4, 2), (1, 3) and (2, 5)

Check Your Progress 6.7 1. (i) Dependent 2. (i) No Solution 3. (i) Consistent (iv) Inconsistent. Check Your Progress 6.8 1. x = 1, y = 1 Check Your Progress 6.9 1. x = 5, y = 2 Check your progress 6.10 1. x = 4, y = 1 2. x = 10 , y = 2 3 2. x = 9 ,y=5 2 3. x = 1, y = 3 2. x = 1, y = 1 3. x = 0, y = 1 (ii) Consistent (ii) Infinite number of solutions (ii) Consistent (iii) Consistent (iii) Unique solution. (iii) Consistent

160

Mathematics

Check Your Progress 6.11 1. (i) x = 5, y = 4 2 (i) x = 32 20 ,y= 3 3 (ii) x = 3, y = 2 (ii) x = 5, y = 6 (iv) x = 5, y = 2. (ii) x = (iv) p = 2a + b ab ,y= 3 3 9 8 ,q= 7 7

(iii) x = 2, y = 4 3. (i) x = 4, y = 7 (iii) x = 3, y = 0 4. A = 3, B = 59. 5. (i) p = 3, q = 2 (iii) x = 3.2, y = 2.3 6. (i) u = (iii) u =
3 ,v=3 2

(ii) x = 9, y = 4

(ii) u =

46 , v = 69 5

19 19 ,v= . 24 21

Check your Progress 6.12 (i) 2(x + y) = 72, 3y = 2x + 3, where x is length and y is breadth (ii) x + y = 64, x = 3y, where x is larger number. Check Your Progress 6.13 1. (a) x + y = 12 and x y = 2 (b) 7x 4y = 0 and 3y + 2x = 59. (c) 2x + 5y = 16 and 3x + 4y = 17. (d) x + y = 10 and 8x 19 y = 1, where x is the units digit and y the tens digit of the number. (e) x + y = 16 and 2y x 2 = 0, where x is length and y is breadth. 2. (a) Smaller number = 2, larger = 7. (b) As age = 15 years, Bs age = 10 years. (c) Length = 7 cm., Breadth = 3 cm. (d) Fathers weight = 90 kg., sons weight = 15 kg. (e) 280 people.

Linear Equations

161

Terminal Exercise 2. (a) Consistent 3. (a) x = (b) Consistent (c) Inconsistent (c) x = 12 17 ,y= 13 13 (d) Dependent

5 1 , y = (b) x = 1, y = 1 7 7

(d) x = 4, y = 0 1 4. (a) k 4 5. (i) x = a, y = b (iii) u = 6. 35 9. 7 9 1 1 ,v= 2 2 7. A : 55 years: B = 30 years 8. Rs 385 10. Length : 120 5760 cm, Area = cm2 7 49 1 (b) k = 4 (ii) x = 1, y = 1

162

Mathematics

7
Quadratic Equations
7.1 INTRODUCTION Many times in life we come across problems whose solution we find by hit and trial method with the help of available information. For example (i) Suppose we are to find two consecutive natural numbers whose product is 12, we can easily guess that the numbers are 3 and 4; similarly if the product is 56 again we can find the number as 7 and 8, but if the product is 552, it becomes difficult to answer the problem by a guess. (ii) Similarly suppose we are to find the dimensions of a rectangle with area 168 square metres and the length exceeds breadth by 2 m. It is again difficult to answer the problem by making a guess. The above type of problems can be solved by a systematic method. For example in the first case if the numbers are x and x + 1, we have to solve the equation x (x + 1) = 552 or x2 + x 552 = 0. Again in 2nd case, when we take breadth as x, the length will be x + 2 and we will have to solve the equation x (x + 2) = 168 or x2 + 2x 168 = 0. If we put different conditions on the length and breadth, we shall again get an equation of the type ax2 + bx + c = 0 but with different values of a, b and c. Such type of equation is called a quadratic equation, and in this lesson we shall deal with solutions of such equations. 7.2 OBJECTIVES After studying this lesson, the learner will be able to :
z z z z

identify a quadratic equation from a given collection of equations. write quadratic equations in standard form. solve a quadratic equation by (i) factorisation and (ii) using the quadratic formula. form a quadratic equation with given roots.

Quadratic Equations

163

z z

translate a word problem into a quadratic equation. solve word problems, using quadratic equations.

7.3 EXPECTED BACKGROUND KNOWLEDGE


z z z

identifying and solving Linear equations in one or two variables finding square root of natural numbers factorisation of polynomials.

7.4 QUADRATIC EQUATIONS Recall that p(x) = ax2 + bx + c is the general form of a quadratic polynomial where a, b and c are real number and a 0. In particular, consider a quadratic polynomial p(x) = 2x2 5x + 2. You know that for every real value of x, p(x) has a real value. For example p(x) = 1 for x = 1 and p(x) = 5 for x = 3. Can you find a value of x for which p(x) = 0 ? By hit and trial method you can find that for x = 2, p(x) = 0. Can you find some more values of x for which p(x) becomes zero ? After some more trials, you will find that for x = Thus, x = 2 and x = 2 and 1 are the numbers for which p(x) = 0. 2 1 , p(x) becomes zero. 2

1 are called the zeroes of p(x). 2

The values of the variables for which a polynomial p(x) vanishes are known as zeroes of the polynomial p(x). Quadratic Equation : Let p(x) = ax2 + bx + c, where a, b and c are real numbers and a 0 be a quadratic polynomial, then p(x) = 0 i.e. ax2 + bx + c = 0 is called a quadratic equation. Thus, An equation of the form ax2 + bx + c = 0, where a 0 and a, b and c are real numbers, is called a quadratic equation in x. For example, 3x2 5x + 7 = 0 is a quadratic equation in x and 2y2 + 3y 7 = 0 is a quadratic equation in y. 2 3 a 2 + 5 = 0 , 3 x2 5x = 0, x2 = 0 are some other examples of quadratic

equations. Example 7.1 : Determine which of the following are quadratic equations in x. (i) (x + 3) (x 2) = 5 (ii) 2x2 + 3x = 2x(x 7) (iii) 3x2 5x = 5(x2 x + 3) 1 (iv) x x = 0, x 0.

164

Mathematics

Solution : (i) (x + 3)(x 2) = 5 or or x2 + 3x 2x 6 = 5 x2 + x 11 = 0

which is of the form ax2 + bx + c = 0, a 0 Hence (x + 3) (x 2) = 5 is a quadratic equation in x. (ii) or or 2x2 + 3x = 2x (x 7) 2x2 + 3x = 2x2 14x 17x = 0

This is not of the form ax2 + bx + c = 0 where a 0. Hence, 2x2 + 3x = 2x(x 7) is not a quadratic equation. (iii) or or 3x2 5x = 5(x2 x + 3) 3x2 5x = 5x2 5x + 15 2x2 + 15 = 0

which is of the form ax2 + bx + c = 0, a 0 as 2x2 + 15 = 0 can be rewritten as 2x2 + 0.x + 15 = 0. Hence 3x2 5x = 5(x2 x + 3) is a quadratic equation in x. (iv) or or x 1 = 0, x 0 x

x2 1 = 0 x
x2 1 = 0 x 1 = 0, x 0 is a quadratic equation. x CHECK YOUR PROGRESS 7.1

which is of the form ax2 + bx + c = 0, a 0 Hence

Determine whether the following equations are quadratic equations : 1. x2 3x + 7 = 5 3. 3x2 5 x + 9 = 0 5. x 2 +


1 =0 x

2. x

3x 5 = 0

4. 2x 2 + 5x + 7 = 0 6. 3x
1 =0 3x 8. x(2x + 5) = x2 + 5x + 7

7. x2 + 3x + 1= x(x + 5)

Quadratic Equations

165

7.5 SOLUTION OF A QUADRATIC EQUATION If p(x) = ax2 + bx + c, a 0 and x = is such that p( ) = 0, then x = is called the solution of quadratic equation ax2 + bx + c = 0, a 0. In other words, (x ) is a factor of the polynomial ax2 + bx + c if is a solution of the equation ax2 + bx + c = 0, a 0. Example 7.2 : Determine whether x = 2x2 5x + 3 = 0. Solution : Putting x = 1 in 2x2 5x + 3 = 0, we get 2 1 3 and x = are solutions of the equation 2 2

1 L.H.S = 2 2 =

FH IK

5 1 +3 2

FH IK

1 5 +3 2 2 1 5 + 6 2

=1 which is not true as L.H.S R.H.S Hence x = 1 is not a solution of equation 2x2 5x + 3 = 0 2 3 in 2x2 5x +3 = 0, we get 2 3 L.H.S = 2 2 = =

Again, putting x =

FH IK

5 3 +3 2

FH IK

9 15 +3 2 2 9 15 + 6 2

= 0 = R.H.S Hence, x = 3 is the solution of the equation 2x2 5x + 3 = 0. 2

Note : L.H.S and R.H.S stands for Left Hand Side and Right Hand Side of the given equation respectively.

166

Mathematics

7.5.1 ROOTS OF THE QUADRATIC EQUATION If and are two zeroes of the quadratic polynomial p(x), we say that and are the roots of the corresponding quadratic equation p(x) = 0. For example 5 and 4 are the zeroes of the polynomial x2 x 20. Thus, 5 and 4 are the roots of the quadratic equation x2 x 20 = 0. x = 5 or x = 4 are also called the solutions of x2 x 20 = 0. The value (s) of the variable (x in this case) which satisfy an equation is (are) called the root(s) or solution(s) of the equation. We can find the solutions of a quadratic equation by the following methods : (A) FACTOR METHOD Let us now study how to solve quadratic equations by the factor method (whenever possible). Here is an example : Example 7.3 : Solve the following equations : (i) (x 5) (x + 3) = 0 (ii) x2 + 3x = 18. Solution : (i) (x 5) (x + 3) = 0 Either x 5 = 0 or or x + 3=0 x=5 x= 3

Thus the equation (x 5) (x + 3) = 0 has two roots, x = 5 and x = 3. (ii) x2 + 3x = 18 or or x2 + 3x 18 = 0 (x + 6) (x 3) = 0

Either x + 6 = 0 or x 3= 0 or x = 6 x=3

Hence, x = 6 and x = 3 are the solutions of the given quadratic equation. From the above example, we generalize the following steps to be followed in finding solution of a quadratic equation by factorisation

Quadratic Equations

167

(a) Write all the terms on one side by making R.H.S. zero. (b) Resolve into linear factors of the type (ax + b)(cx +d) of the terms on the L.H.S. (c) Equate each factor to zero. (d) Get the required solution. Remarks : If the product of two polynomials is zero, then either of them is zero or each of them is zero. i.e. or (ax + b) (cx + d) = 0 Either ax + b = 0 cx + d = 0.

(B) THE QUADRATIC FORMULA Sometimes, solving a quadratic equation by factor method is difficult or too lengthy. In such cases, we solve the equation by completing the squares. Let us solve some quadratic equations by completing the squares. Example 7.4 : Solve the following equations by the method of completing the squares (i) x2 4x 5 = 0 (ii) 3x2 4x 7 = 0 Solution. (i) x2 4x 5 = 0 The first two terms on the L.H.S. are the first two terms in the expansion of (x 2)2
x2 4x 5 = 0 can be rewritten as

x2 4x + 4 4 5 = 0 or or (x 2)2 9 = 0 (x 2)2 (3)2 = 0 (x 2 3) (x 2 + 3) = 0

(Adding and subtracting 4)

(By factorising the polynomial on the L.H.S.) or or or (x 5) (x + 1) = 0 Either x 5 = 0 x + 1= 0 x= 5 x = 1

Hence x = 5 and x = 1 are the two solutions of the given quadratic equation.

168

Mathematics

(ii) 3x2 4x 7 = 0 Multiplying the equation by 3 We get 9x2 12x 21 = 0 On completing the square (by adding and subtracting 4 in the L.H.S.), we get 9x2 12x + 4 4 21 = 0 or or or 9x2 12x + 4 25 = 0 (3x 2)2 (5)2 = 0 (3x 7) (3x + 3) = 0 (The co-efficient of x2)

or (3x 2 5) (3x 2 + 5) = 0 Either 3x 7 = 0 or 3x + 3 = 0 or or x= 7 3

x = 1. 7 and x = 1. 3

Hence the solutions of the given equation are x =

Now let us solve the standard quadratic equation ax2 + bx + c = 0, a 0 by the method of completing square(s) and find its roots. We have ax2 + bx + c = 0 , a 0 Multiplying both sides by 4a, we have 4a(ax2 + bx + c) = 4a 0 or or 4a2x2 + 4abx + 4ac = 0 [Adding b2 to both sides] (4a2x2 + 4abx + b2) = b2 4ac or or or or or (2ax + b)2 = b2 4ac (2ax + b)2 = or (4a2x2 + 4abx + 4ac) + b2 = 0 + b2

{e

b 2 4ac

j}

2ax + b = b 2 4ac 2ax = b b 2 4ac


2 x = b b 4ac 2a

This is known as quadratic formula and is the most convenient tool to solve a quadratic equation.

Quadratic Equations

169

It is clear from above that a quadratic equation of the form ax2 + bx + c = 0, a 0 can have two roots which are

b + b 2 4ac , b b 2 4ac 2a 2a
Here b2 4ac is a factor on which the nature of roots depend. We denote (b2 4ac) by D and it is called DISCRIMINANT. For a quadratic equation ax2 + bx + c = 0, a 0, if (i) D > 0, the equation will have two real and unequal roots (ii) If D = 0, the equation will have two equal roots both equal to (iii) D < 0, the equation will not have any real roots. Thus, a quadratic equation can have maximum two roots. Example 7.5 : Find the roots of quadratic equation 6x2 + x 15 = 0 using quadratic formula. Solution : We have 6x2 + x 15 = 0 Comparing with ax2 + bx + c = 0 we get a = 6, b = 1 and c = 15. Using the quadratic formula, the roots are b b 2 4ac 2a =
1 1 4 6 15

b . 2a

b gb g 2b6g

= 1 361 = 1 19 12 12 = 3 , 5 2 3 3 , 5 . 2 3

Thus, the two roots of the equation are

Example 7.6 : Solve the quadratic equation 4x2 + 4x + 1 = 0 using quadratic formula. Solution : Here a = 4, b =4 and c = 1 Using x = b b 2 4ac , we get 2a

170

Mathematics

x= =

4 16 4 4 1 8 4 0 = 1 8 2

b gb g

1 x = 2 is the solution of the quadratic equation. 7.6 TO FORM A QUADRATIC EQUATION WHEN ITS ROOTS ARE GIVEN Let a and b be the roots of a quadratic equation in x, then (x a) and (x b) are the factors of the quadratic equation
We have(x a) (x b) = 0

or or

x2 (a + b)x + ab = 0 x2 (sum of the roots)x + (product of roots) = 0

Example 7.7 : Form a quadratic equation whose roots are 3 and 5. Solution : Here sum of roots = (3 + 5) = 2 and product of roots = (3).5 = 15
The quadratic equation is

x2 (2)x + (15) = 0 i.e. x2 2x 15 = 0

Example 7.8 : Form a quadratic equation with two roots as 2 + 3 and 2 3 . Solution : Here, sum of roots is 2 + 3 + 2 3 = 4 and product of roots = 2 + 3 2 3 = 4 3 = 1.
The equation is x2 (4)x + 1 = 0

i d

id

or x2 4x + 1 = 0. Example 7.9 : If 3 is one root of the equation x2 ax + 6 = 0, find the value of a and also find the other root of the equation. Solution : Since 3 is a root of the equation x2 ax + 6 = 0
(3)2 a(3) + 6 = 0 a=5

Thus, the equation becomes x2 5x + 6 = 0

Quadratic Equations

171

Comparing with x2 (sum of the roots)x + Product of roots = 0 We have sum of roots = 5 Since one root is 3, other root is 5 3 = 2. CHECK YOUR PROGRESS 7.2 1. Determine whether the value x = 3 and x = 2x2 + 5x 3 = 0. 1 are solutions of the quadratic equation 2

2. Show that x = 2 is one of the roots of the equation 3x2 7x + 2 = 0. 3. Using factor method, solve the following equations : (i) x2 5x + 6 = 0 (iii) 4x2 + 8x 5 = 0 (i) x2 4x 5 = 0 (iii) 4x2 8x + 3 = 0 5. Solve for x : (i) 6x2 19x + 15 = 0 (iii) 3x2 10x + 3 = 0 (v) 21 + x = 2x2 (ii) x2 + 3x 5 = 0 (iv) 3x2 + x 2 = 0 (vi) 9x2 16 = 0. (ii) 6x2 + 7x 5 = 0 (iv) 2x2 + 5x 3 = 0 (ii) 4x2 12x 16 = 0 (iv) 2x2 x 6 = 0.

4. Solve the following equations by method of completing the squares :

6. Form the quadratic equation whose roots are given below : (i) 3 and 5 (iii) 3 + 2 and 3 2 (v) 6 and 5 (ii) 3 and 7 (iv) 3 and
1 3

(vi) 3 and 3
1 . Find the other root. 3

7. One root of the quadratic equation 3x2 10x + 3 = 0 is 8. If

1 is one root of the equation 3x2 + kx 3 = 0, find the value of k. Hence find the 3 other root.

7.7 WORD PROBLEMS In the following discussion, you will learn to apply the methods you have studied in this lesson to solve some problems of daily life.

172

Mathematics

The steps in solving them are the same as the ones you have learnt earlier. They are : (1) Translating the given problem into algebraic form. (2) Solving the equation obtained. (3) Interpreting (verifying) the solution obtained. Example 7.10 : Find two successive natural numbers whose squares have the sum 221. Solution : Let the two successive natural numbers be x and x + 1. As per problem, x2 + (x + 1)2 = 221 or or or or or or or or we get x = 10
Ist natural number = 10

x2 + x2 + 2x + 1 221 = 0 2x2 + 2x 220 = 0 x2 + x 110 = 0 x2 + 11x 10x 110 = 0 x(x + 11) 10 (x + 11) = 0 (x 10) (x + 11) = 0 Either x + 11 = 0 x 10 = 0 x = 11 x = 10

Rejecting x = 11, which is not a natural number.

2nd natural number = 11. Example 7.11 : The sum of two numbers is 15. If the sum of their reciprocal is 3/10, Find the two numbers. Solution : Let one number = x Other number = 15 x As per problem,
1 1 3 + = x 15 x 10

or or

3 15 x + x = 10 x 15 x

3 15 = 10 x 15 x

Quadratic Equations

173

or or or or or or or or

150 = 3x (15 x) 150 = 45x 3x2 50 = 15x x2 x2 15x + 50 = 0 x2 10x 5x + 50 = 0 x(x 10) 5(x 10) = 0 (x 10) (x 5) = 0 Either x 10 = 0 or x 5 = 0 x = 10 x = 5

When x = 5 Other number = 15 5 = 10 Or When x = 10 Other number = 15 10 = 5


The required numbers are 5, 10

Example 7.12 : The sum of a number and its reciprocal is Solution : Let the number = x 1 Reciprocal of x = x 1 17 As per problem , x + = x 4 or or or or or or or

17 . Find the number. 4

x 2 + 1 = 17 4 x
4x2 + 4 = 17x 4x2 17x + 4 = 0 4x2 16x x + 4 = 0 4x(x 4) 1(x 4) = 0 (4x 1) (x 4) = 0. Either 4x 1 = 0 x 4=0

174

Mathematics

x=

1 4

or 1 The required number is 4 or 4

x=4

Example 7.13 : A two-digit number is such that the product of the digits is 12. When 36 is added to this number the digits interchange their places. Determine the number. Solution : Let digit at tens place be x and digit at units place be y The number = 10x + y When digits are interchanged, the new number = 10y + x As per problem, product of digits = 12 i.e. Also, 10x + y + 36 = 10y + x or or or From (1) and (2) xy = 12 or or or or or (y 4)y = 12 y2 4y 12 = 0 y2 6y + 2y 12 = 0 y(y 6) + 2(y 6) = 0 (y + 2) (y 6) = 0 Either y + 2 = 0 or y 6 = 0 y = 2 or y=6 10x x = 10y y 36 9x = 9y 36 x=
9 y4 =y4 9

xy = 12

...(1)

b g

...(2)

Rejecting y = 2, we get y = 6 When y = 6, x = 2

Quadratic Equations

175

The required number = 10 2 + 6 = 26 Therefore, the required number is 26 Example 7.14 : Find two consecutive even positive integers whose squares have the sum 340. Solution : Let the two consecutive even positive integers be x and x + 2. As per problem, x2 + (x + 2)2 = 340 or or or or or

[As sum of their squares is 340]

2x2 + 4x 336 = 0 x2 + 2x 168 = 0 x2 + 14x 12x 168 = 0 x(x + 14) 12(x + 14) = 0 (x 12)(x + 14) = 0.

or Either x + 14 = 0 or x 12 = 0 x = 14 or x = 12 x = 12, x + 2 = 12 + 2 = 14

Since 14 is not a positive integer, x = 14 is not possible. Hence, the two consecutive even positive integers are 12, 14. Example 7.15 : The sides (in cm) of a right angled triangle are x 1, x and x + 1. Find x and hence the sides of the triangle. Solution : The sides (in cm) of a right angled triangle are x 1, x and x + 1
Q

x + 1 > x 1 and x + 1 > x

x + 1 is the hypotenuse (largest side) By Pythagoras Theorem, x2 + (x 1)2 = (x + 1)2 or or or or

x2 + x2 2x + 1 = x2 + 2x + 1 2x2 2x + 1 x2 2x 1 = 0 x2 4x = 0 x(x 4) = 0 x 4=0 x=4 Since x 0

or

176

Mathematics

x 1=4 1 = 3 x + 1=4 + 1 = 5

The sides of the triangle are 3 cm, 4 cm and 5 cm.

Example 7.16 : The sum of ages of Ashu and her mother is 45. The product of their ages is 126. Find their ages. Solution : Let Ashus age = x years
Her mothers age = (45 x) years

The product of their ages = x(45 x) = 45x x2 It is given that the product of their ages is 126.

45x x2 = 126 x2 45x + 126 = 0 x2 42x 3x + 126 = 0

or or

or x(x 42) 3(x 42) = 0 or

(x 42) (x 3) = 0 Either x 42 = 0 x = 42 x=3 or x3=0

or

We reject x = 42 because Ashus age cannot be greater than her mothers age.

x=3

Ashus age is 3 years and her mothers age = 45 3 = 42 years.

Example 7.17 : By increasing his average speed by 10 km/hr a motorist saves 36 minutes in travelling a distance of 120 km. Find out his actual speed. Solution. Let the actual speed of the motorist be x km/hr. He increases his vehicle's speed by 10 km/hr.

His new speed = (x + 10) km/hr speed = Distance Time

In the first case, time =

FH 120 I x K

hours

Quadratic Equations

177

120 In the second case, time = x +10 By the given condition we get,
3 120 120 = x x +10 5

FH

IK

hours

(36 min. =

3 hr) 5

or

120x + 1200 120x 3 = x x + 10 5

or or or

1200 3 x x + 10 = 5

6000 = 3x 2 + 30x x2 + 10x 2000 = 0

or x2 + 50x 40x 2000 = 0 or x(x + 50) 40(x + 50) = 0 or (x + 50) (x 40) = 0 Aliter

Either x + 50 = 0 x = 50

or or

x 40 = 0 x = 40

10 90 x = 10 8100 = 2 2 x = x = 80 2 100 2

or or

x = 40 or x = 50 x = 40

Rejecting x = 50 as speed cannot be negative.

The actual speed of the motorist is 40 km/hr.

CHECK YOUR PROGRESS 7.3 1. Find two consecutive natural numbers whose product is 240. 2. Two successive natural numbers are such that the sum of their squares is 145. Find the numbers.

178

Mathematics

3. The sum of a number and its reciprocal is

50 , Find the number . 7

4. A two digit number is such that the product of the digits is 12. When 9 is added to the number, the digits interchange their places. Determine the number. 5. Find two consecutive odd positive integers whose squares have the sum 290. 6. If the length of a rectangle exceeds the breadth by 4 cm and its area is 525 cm2, find its dimensions. 7. If one side of a right triangle exceeds the other by 7 cm and the hypotenuse is 13 cm. Find the sides. 8. The age of father is 10 times the age of his son. If the product of their ages is 160, find the age of father. LET US SUM UP
z

An equation of the form ax2 + bx + c = 0, a 0 and a, b and c are real numbers, is called a quadratic equation in x. Roots of the quadratic equation ax2 + bx + c = 0, a 0 are given by

b b 2 4ac 2a
z

b2 4ac is called discriminant of the quadratic equation. It is usually denoted by D. (i) If D > 0 then the quadratic equation have two real and unequal roots. (ii) If D = 0 then the quadratic equation have two equal roots. (iii) If D < 0 then the quadratic equation have no real roots. A quadratic equation whose roots are , is given by x2 ( + )x + = 0 The value (s) of the variable which satisfy an equation is (are) called a root(s) or solution(s) of the equation. TERMINAL EXERCISE

Solve the following equations by factorisation method : 1. (x 8) (x+ 4) = 13 2. 2x2 5x = 0 3. x2 8x + 15 = 0 4. 9x2 + 15 x 14 = 0 5. 6x2 + x 15 = 0

Quadratic Equations

179

6. x2 1 + 2 x + 2 = 0 7. ax2 + (a + b)x + b = 0 8. x2 11ax + 28a2 = 0 9. 3a2x2 + 2abx b2 = 0 10. 2x2 3x + 1 = 0 11. Form a quadratic equation whose root are : (a) 1 and 3 (b) 2 and 3 (c) 3 and 4. 12. Form a quadratic equation whose one root is 1 + 2 and the sum of its roots is 2. 13. The sum of the ages of a father and his son is 60 years and the product of their ages is 576. Find their ages. 14. The sum of a number and its reciprocal is 25 . Find the numbers. 12

15. Solve for x : (a) (x + 3)2 + 5(x + 3) + 4 = 0. (b) (x + 1)2 8 (x + 1) + 15 = 0.

180

Mathematics

ANSWERS Check Your Progress 7.1 1. Yes 5. No 2. Yes 6. Yes 3. No 7. No 4. Yes 8. Yes

Check Your Progress 7.2 1. x = 1 2 (ii) x = 1 , 5 , 2 3 5 1 (iii) , , 2 2 (iii) 3,1 2 2 (iv)


1 ,3 2

3. (i) x = 2, 3 4. (i) x = 5, 1 5. (i) x = 3,5 2 3

(ii) x = 4, 1

3 (iv) x = 2, 2

1 2 (iv) x = 1, (ii) x = 3 29 (iii) a = 3, 3 3 2 7 4 4 (v) x = 3, (vi) , . 2 3 3 2 (ii) x2 4x 21 = 0 (iii) x2 6x + 7 = 0 6. (i) x + 8x + 15 (iv) 3x2 10x + 3 = 0 (v) x2 11x + 30 = 0 (vi) x2 9 = 0. 7. 3 8. k = 8, the other root is 3.

Check Your progress 7.3 1. 15, 16 5. 11, 13 Terminal Exercise 1. x = 9, x = 5 2 7 ,x= 3 3 b 7. x = 1, x = a 1 10. x = , x = 1 2 4. x = 11. (a) x2 2x 3 = 0 12. x2 2x 1 = 0 13. Age of Father = 48 years, Age of Son = 12 years 14.
4 3 or 3 4

2. 8, 9 6. 21 25 cm

3. 7 or

1 7

4. 34 8. 40 years.

7. 5 cm, 12 cm

2. x = 0, x = 5. x =

5 2

3. x = 5; x = 3 6. x = 1; x =

3 5 ;x= 2 3

8. x = 4a, x = 7a

b b 9. x = , x = 3a a

(b) x2 x 6 = 0

(c) x2 x 12 = 0

15. (a) x = 4 or x = 7

(b) x = 2 or x = 4.

Number Patterns

181

8
Number Patterns
8.1 INTRODUCTION In our day-to-day life, we see patterns of geometric figures on clothes, pictures, posters etc. They make the learners motivated to form such new patterns. This becomes a topic of interest and knowledge to predict the next figure in a pattern. Consider the following patterns : (i)

(ii)

(iii)

(iv)

Fig. 8.1

Can you predict the next figures in (i), (ii), (iii) and (iv) ? A little careful study of the above patterns shows that the next figures in (i), (ii), (iii) and (iv) are

and

respectively.

Think about these and try to find the reasons for those.

182

Mathematics

Likewise number patterns are also faced by learners in their study. Number patterns play an important role in the field of Mathematics. Let us study the following number patterns : (i) 2, 4, 6, 8, 10, ... 1 1 (ii) 1, 1 , 2, 2 , 3, ... 2 2 (iii) 10, 7, 4, 1, 2, ... (iv) 2, 4, 8, 16, 32, ... (v) 4, (vi) 1, 1 1 1 , , , ... 2 16 128 1 1 1 , , , ... 2 3 4

(vii) 1, 11, 111, 1111, 11111, ... It is an interesting study to find whether some specific names have been given to some of the above number patterns and the methods of finding some next terms of the given patterns. In this lesson, you will study about number patterns called Arithmetic Progressions and Geometric Progressions. You will also study methods of finding general term and sum to n terms of an Arithmetic Progression. 8.2 OBJECTIVES After studying this lesson, the learner will be able to :
z

recognise number patterns and identify those which are Arithmetic or Geometric progressions. determine nth term of an Arithmetic progression (A.P) find the sum to n terms of an A.P.

z z

8.3 EXPECTED BACKGROUND KNOWLEDGE


z z z z

Idea of numbers Idea of the number line Knowledge of various number systems Four fundamental operations on numbers

8.4 RECOGNITION OF NUMBER PATTERNS Suppose you want to purchase a handkerchief whose cost is Rs 5. If you want to purchase two handkerchiefs, then you have to pay Rs 10.

Number Patterns

183

Therefore, if the number of handkerchiefs is increased by one successively, the respective cost (in Rs) would increase by 5 every time, i.e., the respective costs of one, two, three, ... handkerchiefs would be 5, 10, 15, ... Can you recognize the relationship between any consecutive numbers of the above pattern ? If you observe the pattern carefully, you will find that each successive number, other than the first number, is obtained by adding a constant number to the preceding number. Therefore, these numbers form a number pattern. Each number of the number pattern is called a term. Some examples of numbers patterns are given below : (a) 1, 2, 3, 4, ... (b) 4, 2, 0, 2, 4, ... (c) 3, 6, 9, 12, ... (d) 13, 9, 5, 1, 3, 7, ... 1 1 1 1 (e) 2 , 6 , 10 , 14 , ... 4 4 4 4 (f) 1, 2, 4, 8, 16, ... (g) 16, 4, 1, 1 , ... 4

(h) 5, 25, 125, 625, ... In number patterns (a), (c) and (e), each successive term, other than the first term, can be obtained by adding a constant number 1, 3 and 4 respectively to the preceding term. In the number patterns (b) and (d), each successive term, other than the first term, can be obtained by subtracting a constant number 2 and 4 respectively from the preceding term. But in the number patterns (f), (g) and (h), each term, other than the first term, can be obtained by multiplying the preceding term by a constant number 2, 8.5 ARITHMETIC PROGRESSION You are all aware that salaries of employees are often calculated on the basis of their basic salary, plus fixed increments (increases) for each year of service and all other usual allowances. Suppose a person begins to work for a firm in the scale of Rs 4500-125-7000. 1 and 5 respectively. 4

184

Mathematics

Then in successive years his basic salary (in Rs) will be 4500, 4625, 4750, 4875, 5000 and he will get usual allowances on the basic salary. The first term of this pattern is 4500 and each successive term, other than the first term, can be obtained by adding a constant number (here increament in Rs) 125 to the preceding number. We can say that the terms of the pattern progress arithmetically. Such a number pattern is called an Arithmetic Progression (abbreviated as A.P). The constant number is called common difference. A progression is said to be an Arithmetic Progression (abbreviated as A.P), if the difference of each term, except the first, from its preceding term is always the same. Probably the simplest arithmetic progression is of natural numbers : 1, 2, 3, 4, 5, 6, ... The multiplication tables are all familiar A.P's, 2, 4, 6, 8, 10, ... 3, 6, 9, 12, 15, ... 4, 8, 12, 16, 20, ... An arithmetic progression can start from any number, positive or negative for example : 5, 8, 11, 14, 17, ... 2 1 , 6 1 , 10 1 , 14 1 , ... 4 4 4 4 7, 2, 3, 8, ... Rational numbers can serve as the common difference, as in the A.P : 1 1 9, 11 , 14, 16 , 19, ... 2 2 and negative numbers, as in the A.P 10, 7, 4, 1, 2, 5, ... An arithmetic progression can be represented on the number line by a series of points placed the same distance apart. For example, the progression 13, 17, 21, 25, ... is shown in Fig. 8.2.

Fig. 8.2

Number Patterns

185

The successive terms of an A.P are usually denoted by t1, t2, t3, ... The common difference is usually denoted by d. Observe the following A.P : 3, 5, 7, 9, 11, ... It is clear from the above A.P that its first term is 3 and the common difference is 2 (usually denoted by d) You can rewrite the above A.P in the following manner :

In general, if the first term is denoted by a and the common difference by d, the standard form of an Arithmetic Progression would be a , a + d , a + 2d , a + 3d , ... It should be noted that, the common difference may be negative in which case the terms of the progression would decrease. For example : 15, 13 1 , 12, 10 1 , ... 2 2

The first term of this A.P is 15. 1 1 1 1 The common difference = 13 15 = 12 13 = 10 12 = 1 2 2 2 2 Let us take some examples to illustrate : Example 8.1 : Which of the following progressions are A.P. ? (a) 3, 6, 9, 12, ... (c) 13, 15, 18, 22, ... (b) 9, 14, 19, 24, ... (d) 21, 17, 13, 9, ...

Solution : (a) The given progression is 3, 6, 9, 12, ...

186

Mathematics

Here the first term is 3 and the common difference = 6 3 = 9 6 = 12 9 = 3


The given progression is an A.P.

(b) The given progression is 9, 14, 19, 24, ... Here the first term is 9 and the common difference is = 14 9 = 19 14 = 24 19 = 5
The given progression is an A.P.

(c) The given progression is 13, 15, 18, 22, ... Here the first term is 13. But its common difference is not the same as 15 13 18 15 22 18
The given progression is not an A.P.

(d) The given progression is 21, 17, 13, 9, ... Here the first term is 21 and the common difference is 17 21 = 13 17 = 9 13 = 4
The given progression is an A.P.

CHECK YOUR PROGRESSION 8.1 1. Which of the following progression are A.Ps ? (a) 6, 12, 18, 24, ... (c) 1, 3, 5, 7, ... (b) 7, 10, 12, 14, ... (d) 31, 29, 27, 25, ...

8.6 THE GENERAL TERM (OR nTH TERM) OF AN ARITHMETIC PROGRESSION If we need to find a particular term of an A.P, for example 12th term whose first term is 3 and common difference is 2, we can, of course, build up the A.P. as far as the required term by adding 2 successively to the first term, as 3, 5, 7, 9, ... But this method would become rather laborious if you had to find, say 102nd term of the progression. We can find an easier method if we look at the progression more closely. The first term (a) for an A.P is 3 and the common difference (d) = 2. The successive terms can be formed by adding 2 to the first term.

Number Patterns

187

1 2 3 4

st

term = 3 term = 3 + 2 term = (3 + 2) + 2 term = (3 + 2 + 2) + 2 ...

= 3 + 0.2 = 3 + 1.2 = 3 + 2.2 = 3 + 3.2 ...

= 3+( 1

1).2

nd

= 3 + ( 2 1).2 = 3 + ( 3 1).2 = 3 + ( 4 1).2 ...

rd

th

...

Without completing all the step-by-step calculations, we can find straight away that the 12th term by adding 2, eleven times to the first term. i.e., 12
th

term would be 3 + 11.2 = 3 + ( 12 1).2

This suggests us a formula for finding any term of the progression. ? i.e.,
th

term = 3 + (

1).2

In particular writing number 12 in each box we can find the 12th term : 12th term = 3 + (12 1).2 12th term = 2 + 11 2 We generally denote 12th term by t12 and write the above result as t12 = 3 + (12 1) 2 Thus the general term i.e. nth term can be written as : nth term = a + (n 1)d or tn = a + (n 1)d

where a and d denote the first term and the common difference respectively. We will get the same formula for the nth term if we use the standard form of A.P. We know that the standard form of an A.P. is a, a + d, a + 2d, a + 3d, ... In the similar manner, we can write

1st term 2nd term 3rd term ...

= = =

t t t

= a = a + 0.d = a+d = a + 2d ...

= a + ( 1 1)d

= a + ( 2 1)d

= a + ( 3 1)d ...

...

188

Mathematics

If you see the above pattern, the formula for the nth term or tn would be a + (n 1)d Hence, we can rewrite the standard form of an A.P as a, a + d, a + 2d, ....., tn 1, a + (n 1)d where tn1 denotes the (n 1)th term of the A.P. Example 8.2 : Identify the first term and the common difference of each of the following A.Ps : (i) 6, 10, 14, 18, ... (ii) 12, 9, 6, ... Solution : (i) The given A.P is 6, 10, 14, 18, .... Here, the first term (a) is 6 and the common difference (d) = 10 6 = 4 (ii) The given A.P. is 12, 9, 6, ... Here, the first term (a) is 12 and the common difference (d) = 9 12 = 3. Example 8.3 : Write the expressions for nth terms of the following A.P's and find the 100th term of each of the following : (i) 5, 8, 11, 14, ... (ii) 7, 11, 15, 19, ... Solution : (i) The given A.P is 5, 8, 11, 14, ... We know that the nth term of an A.P, whose first term is a and common difference is d, is given by tn = a + (n 1)d Here,

a = 5 and d = 8 5 = 3 tn = 5 + (n 1).3 = 5 + 3n 3 = 3n + 2

100th term = t100 = 3 100 + 2 = 302.

(ii) The given A.P is 7, 11, 15, 19, ... nth term of an A.P is given by tn Here,

=a + (n 1)d a = 7 and d = 11 (7) = 11 + 7 = 4 tn = 7 + (n 1).(4)

Number Patterns

189

=7 4n + 4 =3 4n

100th term = t100 = 3 4 100 = 403. tn = 2 + 3n,

Example 8.4 : If the nth term of a progression is given by show that it is an A.P. Solution : Given that nth term tn = 2 + 3n Putting n = 1, 2, 3, 4, ..., we get

1st term, t1 = 2 + 3 1 = 5 2nd term, t2 = 2 + 3 2 = 8 3rd term, t3 = 2 + 3 3 = 11 4th term, t4 = 2 + 3 4 = 14 ... ... ... ...

The progression is 5, 8, 11, 14, ... t1 = 5, t2 = 8, t3 = 11, t4 = 14, ... t2 t1 = 8 5 = 3 t3 t2 = 11 8 = 3 t4 t3 = 14 11 = 3

Here,

Since the common difference is always the same, the above progression is an Arithmetic Progression. Example 8.5 : Find the 21st term of an A.P whose first term is 3 and common difference is 5. Solution : Here, the first term, a = 3 and common difference d = 5

nth term = tn= a + (n 1)d = 3 + (n 1) 5 = 3 + 5n 5 = 8 + 5n

21st term = t21 = 8 + 5 21 = 8 + 105 = 97

The 21st term of the given progression is 97.

190

Mathematics

Example 8.6 : The 6th term of an A.P is zero and the 2nd term is 4. Find the first term and the common difference. Solution : Let the first term and common difference of the A.P be a and d respectively. Its nth term is given by tn = a + (n 1)d

2nd term = t2 = a + (2 1)d = a + d 6th term = t6 = a + (6 1)d = a + 5d

By the given condition a + 5d = 0 and a + d=4 ...(i) ...(ii)

Subtracting (ii) from (i), we have 4d = 4 Putting d = 1 in (i), we get a + 5(1) = 0 or a=5 or d = 1

The first term of the A.P. is 5 and the common difference is 1.

Example 8.7 : Which term of the A.P 1 , 1 , 3 , 1, 5 1 , ..... is 6 ? 4 4 2 4 4 Solution : The first term (a) = Common difference (d) = 1 Let the nth term be 6 4 or 1 + n 1 1 25 = 4 4 4 1 4

11= 1 2 4 4

b g

or 1 + (n 1) = 25 or n = 25

1 Hence, 25th term of the above A.P is 6 . 4

Number Patterns

191

Example 8.8 : The ages of three brothers are in A.P. The age of the youngest brother is onethird the age of the eldest. Find the ages of all, if the sum of their ages is 90 years. Solution : Let the ages (in years) of three brothers be a d , a , a + d By the given condition a d = or or or and or or
1 (a + d) 3

...(i)

3a 3d = a + d 2a = 4d a = 2d (a d) + a + (a + d) = 90 3a = 90 a = 30 30 = 2d ...(ii)

Putting a = 30 in (ii), we get or d = 15

Putting the values of a and d in (i), we get a d = 15 a = 30 and a + d = 45


The required age of the youngest brother is 15 years, that of 2nd brother is 30 years and of the eldest brother is 45 years.

Example 8.9 : The fourth term of an A.P is equal to three times its first term and 7th term exceeds twice the third term by 1. Find the first term and common difference. Solution : Let the first term and common difference of the A.P. be a and d respectively. t4 = a + 3d t7 = a + 6d and By the given condition a + 3d = 3a or and or or 2a 3d = 0 a + 6d = 2(a + 2d) + 1 a 2d + 1 = 0 a = 2d 1 ...(ii) ...(i) t3 = a + 2d

192

Mathematics

Putting the value of a in (i) we get, 2(2d 1) 3d = 0 or 4d 2 3d = 0 or d=2 2a = 6 or

Substituting d = 2 in (i) ,we get a=3

The required first term and common difference of the A.P. are 3 and 2 respectively.

Example 8.10 : Find three numbers in A.P whose sum is 21 and sum of their squares is 389. Solution : Let the three terms of the A.P be a d, a, a + d By the given condition (a d) + a + (a + d) = 21 or or and or or or or or or or 3a = 21 a = 7 (a d)2 + a2 + (a + d)2 = 389 a2 2ad + d2 + a2 + a2 + 2ad + d2 = 389 3a2 + 2d2 = 389 3 72 + 2d2 = 389 147 + 2d2 = 389 2d2 = 242 d2 = 121 d = 11 [using (i)] ...(i)

When a = 7, d = +11, the three numbers are 4, 7, 18 When a = 7, d = 11, the three numbers are 18, 7, 4. Example 8.11 : If m times the mth term of an A.P is equal to n times its nth term, prove that its (m + n)th term is zero. Solution : Let the first term and common difference of the A.P be a and d respectively

tm = a + (m 1)d tn = a + (n 1)d

By the given condition m tm = n tn

Number Patterns

193

or m{a + (m 1)d} = n{a + (n 1)d} or or or or m{a + (m 1)d} n{a + (n 1)d} = 0 (m n)a + {m (m 1) n (n 1)}d = 0 (m n)a + {(m2 n2) (m n)}d = 0 (m n)a + {(m + n) (m n) (m n)}d = 0

Dividing throughout by m n, we get or a + (m + n 1)d = 0 We know that

tm+n = a + (m + n 1)d tm+n = 0

Hence the result. CHECK YOUR PROGRESSION 8.2 1. Find the next two terms of the following A.P.s (a) 11, 19, 27, ... (b) 5 , 1, 3 , ... 4 4 1, 1, 3, 5 , ... 2 2 2 2

2. Write the formula for nth term, of the following A.P.s : (a) 6, 11, 16, 21, ... (b)

3. The nth term of a progression is given by the relation : (a) Tn = 10 n (c) Tn =


1 2n 3

(b) Tn = 7n + 2

Show that each of them is an A.P. 4. The 5th term of an A.P is 23 and 12th term is 37. Find the first term and common difference of the A.P. 5. Which term of the A.P : (a) 100, 95, 90, 85, ... is 20 ? (b) 4, 10, 16, 22, ... is 334 ? 6. The angles of a triangle are in A.P. If the least angle is one-third the largest angle, find the angles. 7. If the mth, nth and rth terms of an A.P. are x, y and z respectively. Prove that x(n r) + y(r m) + z (m n) = 0.

194

Mathematics

8.7 THE SUM OF THE FIRST n TERMS OF AN A.P We know that the standard form of an A.P is given by : a , a + d , a + 2d , ... , a + (n 2)d , a + (n 1)d tn1 tn where a and d are the first term and the common difference respectively, and tn1 and tn denote the (n 1)th and nth terms of the A.P. Let Sn denote the sum of first n terms of the A.P.
We can write

Sn = a + ( a + d) + (a + 2d) + ... + {a + (n 2)d}+ {a + (n 1)d} Writing the terms in reverse order, we get Sn = {a+ (n 1)d} + {a+ (n 2)d} + ... + (a + 2d) + (a + d) + a Adding corresponding terms of (1) and (2), we get

...(1)

...(2)

2Sn = {2a + (n 1)d} + {2a + (n 1)d} + {2a + (n 1)d} + ... + {2a + (n 1)d} = {2a + (n 1)d} {1 + 1 + 1 + ... + 1} = n {2a + (n 1)d} 2Sn = n{2a + (n 1)d} Sn = n {2a + (n 1)d} 2 ...(3)

which denotes the general formula for finding the sum of the first n terms of the A.P. (3) can also be rewritten as Sn = =
n a + a + n 1 d 2

m b gr
[Q nth term = tn = a + (n 1)d]

n a+t n 2

Sometimes nth term is named as the last term and is denoted by the letter l.

Sn =

n a+l 2

b g

...(4)

Example 8.12 : Find the sum of the first 10 terms of the following A.Ps : (a) 1, 6, 11, 16, ... (b) 193, 189, 185, ...

Number Patterns

195

Solution : (a) The given A.P. is : 1, 6, 11, 16, ... Here a = 1, d = 6 1 = 5 Number of terms = n = 10 We know that the sum of the first n terms of an A.P whose first term is a and common difference is d , is given by Sn =

n {2a + (n 1)d} 2 10 2 1 + 10 1 5 2

S10 =

gr

= 5(2 + 45) = 235

The required sum is 235 193, 189, 185, ... Here and a = 193, d = 189 (193) = 4 n = 10.

(b) The given A.P. is :

We know that, Sn = = n {2a + (n 1)d} 2 10 2 193 + 10 1 4 2 = 5(386 + 36) = 5 (350) = 1750 The required sum is 1750. Example 8.13 : Find the sum of each of the following A.P.s : (i) 2 + 4 + 6 + 8 + ... + 146 1 1 3 (ii) 5 + 4 + 3 + ... + ( 3) 4 2 4 Solution : (i) The given A.P. is : 2 + 4 + 6 + 8 + ... + 146 Here a = 2, d = (4 2) = 2 and the nth term = tn = 146

mb

g b

gr

196

Mathematics

We know that tn = a + (n 1)d = 146 or or or 2 + (n 1).2 = 146 2n = 146 n = 73

Number of terms in the above A.P. is 73

The required sum of the first 73 terms is given by S73 =

m b gr 73 2 + 72 2 l q = 2
73 2 + 73 1 .2 2 = 73 146 2 = 5329

(ii) The given A.P. is 5 1 + 4 1 + 3 3 + ... + (3) 4 2 4 Here Here nth term is 3. We know that tn = a + (n 1)d = 3 or or or or or 21 3 3 = 4 + n 1 4
21 3 n 1 = 3 4

1 1 21 1 3 a= 5 = , d = 4 5 = 4 2 4 4 4

Let us find out which term of this A.P. is 3

b g FH IK

b g

21 3n + 3 = 12 24 + 12 = 3n n = 12

The required sum of the first 12 terms is given by 12 21 S12 = 2 4 3

FH

IK

LMusing, S N

= n a + tn 2

qOQP

9 27 = 6 = . 4 2

Number Patterns

197

Example 8.14 : Find the sum of all multiples of 5 upto 1017. Solution : The multiples of 5 upto 1017 are : 5, 10, 15, 20, ..., 1015 We have to find the above sum i.e. We can write a + (n 1)d = 1015 or 5 + (n 1).5 = 1015 or or The required sum 5n = 1015 n = 203 203 5 + 1015 2 5 + 10 + 15 + 20 + ... + 1015

Here a = 5, d = 10 5 = 5 and tn = 1015

S203 = =

LMUsing, S N

= n a + tn 2

qOPQ

203 1020 2 = 103530 CHECK YOUR PROGRESSION 8.3 1. Find the following sums : (a) 7 + 12 + 17 + 22 + ... + 1002 (b) 25 + 28 + 31 + ... + 328 2. How many terms of the A.P. : (a) 1, 4, 7, 10, ... are needed to get the sum 715 ? (b) 10. 7, 4, 1, ... are needs to get the sum 104 ? 3. Find the sum of the first 10 odd natural numbers. 4. Sum of first 6 terms of an AP is 45. If the Ist term is 5, find its last term and common difference. 8.8 IDEA OF A GEOMETRIC PROGRESSION So far we have learnt that an Arithmetic Progression can be formed by repeatedly adding/ subtracting a constant number to/from a given first number (term). A progression can also

198

Mathematics

be formed by repeatedly multiplying or dividing its preceding term by a non-zero constant. If we consider that the starting prize in a game of Triple Your Money is 3, then repeated tripling (multiplication by 3) would form the progression of possible prizes as 3, 9, 27, 81, 243, ... ...(i)

If you observe each term of the above pattern/progression, you will find that there is a common ratio between two consecutive terms. In this case it is 3 : 1 as 9 = 27 = 81 = 243 = 3 3 9 27 81 1 It is usually denoted by r Such a progression is called a Geometric progression (abbreviated as G.P.) Let us take another example to consolidate : Consider that a person reads from the newspaper that the Government is stressing on more security for the railway passengers. Consider that he/she conveys this message to two persons and both of them pass on this message to two other persons each and so on. It is shown by following figure :

Step 0 : ...........................................................

Step 1 : ..................................

Step 2 : ..................

Step 3 : .........

Number Patterns

199

Generally rumours spreads in this manner. If you calculate the number of persons informed at each step, you will get the following progression. 1, 2, 4, 8, 16, ... This type of progression are called a Geometric Progression. A progression is said to be a Geometric Progression (abbreviated as G.P.), if the ratio of any two successive terms is always the same. You can verify that common ratio in (ii) is always the same as 2 = 4 = 8 = 16 = 2 1 2 4 8 ...(ii)

The ratio of any two successive terms of a Geometric Progression is called its common ratio. Now observe the G.Ps in (i) and (ii) 3, 9, 27, 81, ... and 1, 2, 4, 8, 16, ... Both can be rewritten as as follows :

In general if a be the first term and r be the common ratio, the standard form or general form of a Geometric progressions would be a , ar , ar2 , ar3 , ... The following progressions are all examples of G.Ps : (i) 3, 12, 48, 192, ... (ii) 5, 10, 20, 40, ...

200

Mathematics

(iii)

1, 1, 1 , 1 , ... 2 8 32 128

1 1, 1 (iv) 1, , , ... 4 16 64

(v) 4, 16, 64, 256, ... Example 8.15 : Identify G.Ps from the following progressions : (a) 1, 5, 25, 125, ... (b) 7, 14, 21, 42, ... 1 1 1 (c) 1, , , , ... 2 4 8 Solution : (a) The given progression is 1, 5, 25, 125, ... Here the first term is 1 and the common ratio =
The progression is a geometric progression.

5 = 25 = 125 = 5 1 5 25

(b) The given progression is 7, 14, 21, 42, ... Here the first term is 7 but the common ratio is not the same as 14 21 7 14

Since, the common ratio is not the same, this is not a geometric progression. (c) The given progression is 1, 1 , 1 , 1 , ... 2 4 8 1 Here the first term is 1 and the Common ratio is = . 2
The progression is a geometric progression.

Example 8.16 : Find the common ratio of the following G.P.s : (a) 11, 121, 1331, ... (b) 3, 15, 75, ... Solution : (a) The given progression is 11, 121, 1331, ... Here the first term = 11 Common ratio = 121 = 1331 = 11 11 121

Number Patterns

201

(b) The given progression is 3, 15, 75, ... Here the first term = 3 Common ratio =
+15 75 = 5 . = 3 +15

CHECK YOUR PROGRESSION 8.4 1. Identify Geometric Progressions from the following progressions : (a) 4, 20, 100, 500, ... (c) 11, 14, 17, 20, ... (b) 24, 12, 6, ... (d) 45, 15, 5, ...

2. Show that the following progressions are in G.Ps : (a) 16, 8, 4, 2, 1, ... 1 1 1 (b) 1, , , , ... 3 9 27 (c) 125, 25, 5, 1, ... 3. Find the common ratio of each of the progressions given in Question Number 2. LET US SUM UP
z

A progression is said to be an Arithmetic Progression (abbreviated as A.P.), if the difference of each term, except the first, from its preceding term is always the same. The successive terms of an A.P. are usually denoted by t1, t2, t3, ... The common difference in usually denoted by d. The standard form of an A.P is given by a , a + d , a + 2d , ..., tn1 , tn where a and d are the first term and common difference respectively.

z z z

The common difference may be negative also. In that case, the terms of an A.P will decrease. The general term (nth term) of an A.P is given by tn = a + (n 1)d.

202

Mathematics

The sum of the first n terms of an A.P whose first term is a and common difference is d is given by Sn = = n 2a + n 1 d 2

m b gr

n a+l , where l is the last term. 2

b g

A progression is said to be a Geometric Progression (abbreviation as G.P.) if the ratio of any two successive terms is always the same.

The ratio of any two successive terms of a G.P is called its common ratio and is usually denoted by r. The standard form of a Geometric Progression is given by a , ar , ar2 , ... where a and r are the first term and common ratio respectively. TERMINAL EXERCISE

1. Which of the following progressions are A.P.'s ? (a) 13 , 8 , 19 , ... 2 2 22 , 3 , 20 , ... 7 7 (b) 1, 1, 1, 1, ... 1 1 1 (d) 1, , , , ... 2 3 4

(c)

(e) 22, 52, 82, ... 2. Find the general term (or nth term) of the following A.Ps : (a) 1, 3, 5, ... (c) 3 , 2, 5 , ... 2 2 (b) (d) 15 , 2 , 13 , ... 7 7 5, 4, 1 , ... 3 3

3. Show that the following progressions are in A.P whose nth term is given by (a) n (c) 2n +1 3 (b) 3 5n n (d) + 1 2

4. Which term of the following A.P. : (a) 5, 8, 11, ... is 56 ?

Number Patterns

203

(b) 1, 6, 11, ... is 506 ? (c) 190, 186, 182, ... is 14 ? 5. How many terms are needed to make the sum of the A.P. 25, 28, 31, ... as 1625 ? 6. Find the sum of the following A.Ps for given number of terms : (a) 7 + 3 1 5 ...; 20 terms

(b) 15 + 13 + 11 + ...; 16 terms (c) 1 + 9 + 17 + ...; 20 terms

7. The sum of three consecutive terms of an A.P is 36 and their product is 1428. Find the terms. 8. If 5th and 9th terms of an A.P are 8 and 14 respectively, find the first term and the common difference. 9. The 14th term of an A.P is twice its 8th term. If its 6th term is 8, find the first term and common difference. 10. Find the sum of all even numbers upto 1001. 11. A man saved Rs 20,000 in 10 years. If the saving of each year is Rs 100 more than the saving in the preceding year, find his saving in the first year. 12. Show that the following progressions are in G.P. : (a) 1, 7, 49, ... 1 1 (b) 1, , , ... 4 16 (c) 16, 12, 9, ... (d) 1 , 1 , 1 , ... 125 25 5

204

Mathematics

ANSWERS Check Your Progress 8.1 1. (a), (c) and (d) are in A.P Check Your Progress 8.2 1. (a) 35, 43 2. (a) t n = 5n + 1 5. (a) 25th term Check Your Progress 8.3 1. (a) 100900 2. (a) 22 terms 4. 10,1 Check Your Progress 8.4 1. (a), (b) and (d) Terminal Exercise 1. (a) and (c) 2. (a) 1 2n 4. (a) 18th term 6. (a) 620 (b) 16 n 7 (c) 2+n 2 (d) 6 n 3 3. (a)
1 2

(b)

1, 1 2 4 3 2n 2 4. 15, 2 6. 30, 60, and 90

(b) tn =

(b) 56th term

(b) 18003 (b) 13 terms 3. 100

(b)

1 3

(c)

1 5

(b) 102th term (b) 0

(c) 45th term (c) 1540

5. 26

7. 7, 12 and 17 or 17, 12 and 7 8. 2, 3 2 9. 2, 2 10. 250500 11. Rs 1550

208

Mathematics

Module 2
Commercial Mathematics
It is a common saying by elders spend within your limits i.e., keep your expenditure less than your income. The latent meaning of this is to save something for difficult times. You must have seen birds and animals saving eatables for rainy season, in their nest or caves. Taking the lead from this, the students have been told about the importance and need of savings in this module. Many Indian mathematicians have worked on the topic of commercial Mathematics. Yodoksu (370 B.C.) worked on fractions and ratio and proportion. In the reigns of Ashoka and Chandragupta, there is a description of levying taxes. There is a description of many mathematicians working on practice and proportion (like Aryabhatt, Mahavira, Brahmgupta, Sridharacharya). In 900 A.D., Bakshali Manuscript was discovered which had a number of problems on Arithmetic. To keep your savings safe is another tough task. Banks and other financial institutions keep the money of their customers and on the expiry of the period pay extra money, called interest, in addition to the money deposited. This encourages citizens to save and keep the money safe. This is why calculation of interest on deposits in banks is included for teaching. The Government provides a number of facilities to the citizens. For that they levy certain taxes on citizens. Two of these taxes are sales tax and income tax to which the learners are introduced in this module. Financial transactions about buying and selling are generally done for profit. Due to greater supply of goods or sub-standard goods they are to be sold on loss. The learners are, therefore, introduced to percentage and profit and loss. Sometimes we have to buy articles on instalments because of non-availability of adequate funds. Due to this the students are taught to calculate interest when they buy articles on instalment plan. Sometimes when we are not able to return loaned money on time, the financer starts charging interest on interest also, which is called compound interest. Due to this the study of compound interest has been included in this module. The formulae of compound interest is also used in finding increase or decrease in prices of things. This is also taught under Appreciation and Depreciation of value.

Ratio and Proportion

209

9
Ratio and Proportion
9.1 INTRODUCTION You are aware of the concept of comparison of numbers in two different ways. One way is to see which number is larger and which is smaller. Another way is to see one number is how many times the other number. In the first case, we compare the numbers by difference and in the second we compare the numbers by division. In the second case, we talk of the ratio of two numbers. In this lesson we shall study to write a ratio in the simplest form, introduce the concept of proportion, direct and inverse proportion (variation) and use these concepts to solve real life problems, pertaining to time and distance, time and work, work and wages and partnership. 9.2 OBJECTIVES After studying this lesson, the learner will be able to :
z z z z

write a ratio in the simplest form determine whether given four numbers are in proportion illustrate the concept of direct and inverse proportion (variation) solve real life problems pertaining to time and distance, partnership, time and work, work and wages etc.

9.3 EXPECTED BACKGROUND KNOWLEDGE


z z z

Four Fundamental operations on numbers Comparison by division Knowledge of different units of measures and their conversions.

9.4 CONCEPT OF RATIO When we compare the two quantities by division, we say that we have formed a ratio of the two quantities.

210

Mathematics

Consider an example of a bag of wheat with weight 90 kg and a bag of rice with weight 40 kg. Here, we can say that the ratio of the weight of a bag of wheat to the weight of a bag of rice is 90 : 40 (read as 90 is to 40) The symbol : is used to denote a ratio. Thus, the ratio of any number a and b is written as a : b. This is also written as a . b

In the ratio 90 : 40, 90 and 40 are called terms of the ratio. The first term (90) is called the antecedent and the second term (40) is called the consequent. Remark : We cannot compare two quantities if they are not of the same kind i.e., we do not compare 5 boys and 6 cows or 10 kg and 500 m or 5 toys and 3 fruits. To compare two quantities as a ratio, it is necessary to express the two quantities in the same unit. For example, to compare 3 months and 37 days, we compare (3 30) days and 37 days. The ratio, thus, is 90 : 37. It may be noted that in the above examples ratio is not 3 : 37. Example 9.1 : In a class, there are 28 girls and 16 boys. Find (i) the ratio of the number of boys to that of girls (ii) the ratio of the number of girls to that of boys. Solution : (i) The required ratio is 16 : 28. (ii) The required ratio is 28 : 16. Remarks : From Example 1 given above, the ratio a : b is different from the ratio b : a. Hence, the order of two terms in a ratio is very important. Example 9.2 : A labourer earns Rs 3200 a month and spends Rs 2500. Find the ratio of his (i) expenditure to income (ii) savings to income (iii) savings to expenditure Solution : Here Income = Rs 3200; Expenditure = Rs 2500

Savings = Income Expenditure = Rs 3200 Rs 2500 = Rs 700

(i) Ratio of expenditure to income = 2500 : 3200 (ii) Ratio of savings to income = 700 : 3200 (iii) Ratio of savings to expenditure = 700 : 2500.

Ratio and Proportion

211

Example 9.3 : Find the ratio of (i) 5 days to 1 week (ii) 7 km to 750 m (iii) 250 ml to 1 l. Solution : (i) Since the given quantities are in different units, we convert 1 week into days, getting 7 days. Hence, the required ratio is 5 : 7. (ii) The two quantities are not in the same units. We, therefore first convert 7 km into m, getting 7000 m. Hence, the required ratio is 7000 : 750. (iii) The two quantities are not in same units, we therefore convert 1 litre into ml, getting 1000 ml. Hence, the required ratio is 250 : 1000. 9.4.1 Equal Ratios A ratio remains unchanged when both of its terms are multiplied or divided by the some nonzero number. Hence, 20 : 50 is equal to 10 : 25 or or 4 : 10 2:5

Note that in the ratio 2 : 5, the two terms 2 and 5 do not have a common factor other than 1. 9.4.2 Ratio in the Simplest Form A ratio a : b is said to be in the simplest form if its two terms do not have a common factor, other than 1, i.e. HCF of the two terms is 1. For example, the ratio 20 : 50 is not in the simplest form, because 10 is a common factor of its two terms, The simplest form of the ratio 20 : 50 is 2 : 5. Note : A ratio can be expressed in several ways, as seen above. The ratio 20 : 50 can be written as 10 : 25, or 4 : 10 or 2 : 5, etc. How to obtain the simplest form of a given ratio ? For a given ratio, say a : b, find the H.C.F. of its two terms a and b. Divide each of the terms by the H.C.F. obtained. The ratio formed of the two new numbers obtained is the simplest form of the given ratio.

212

Mathematics

Example 9.4 : Express each of the following ratios in the simplest form : (i) 24 : 30 (ii) 85 : 225 (ii) 150 : 400 (iv) 480 : 576

Solution : (i) The H.C.F. of 24 and 30 is 6.

24 : 30 =

24 30 : =4:5 6 6

(ii) The H.C.F. of 150 and 400 is 50.

150 : 400 =

150 400 : =3:8 50 50

Thus, 3 : 8 is the simplest form of the ratio 150 : 400. (iii) The H.C.F.of 85 and 225 is 5

85 : 225 =

85 225 : = 17 : 45 5 5

Thus, 17 : 45 is the simplest form of the ratio 85 : 225. (iv) The H.C.F.of 480 and 576 is 96

480 : 576 =

480 576 : =5:6 96 96

Thus, 5 : 6 is the simplest form of the ratio 480 : 576. Example 9.5 : Find the simplest form of the ratio of each of the following two quantities: (i) 65 km to 91 km (ii) 45 seconds to 2 minutes (iii) 6 hours to 1 day Solution : (i) The ratio of 65 km to 91 km is 65 : 91 The H.C.F. of 65 and 91 is 13.

65 : 91 =

65 : 91 =5:7 13 13

Thus, the simplest form of the ratio is 5 :7. (ii) The ratio of 45 seconds to 2 minutes is 45 : 120, because 2 minutes = 120 seconds. The H.C.F. of 45 and 120 is 15.

Ratio and Proportion

213

45 : 120 =

45 : 120 =3:8 15 15

Thus, the simplest form of the ratio is 3 :8. (iii) The ratio of 6 hours to 1 day is the same as the ratio for 6 hours to 24 hours (1 day = 24 hours), So, the ratio is 6 : 24 The H.C.F. of 6 and 24 is 6.

6 : 24 =

6 : 24 =1:4 6 6

Thus, the simplest form of the ratio is 1 :4. CHECK YOUR PROGRESS 9.1 1. Fill in the blanks to make each of the following a true statement : (i) In the ratio 3 : 7, the antecedent is (ii) The ratio 3 : 5 is and consequent is

from the ratio 5 : 3. . .

(iii) The simplest form of the ratio 15 : 20 is

(iv) The ratio of 2 months to 2 days, in the simplest form, is (v) The ratio 5 : 4 is of the ratio 100 : 80.

2. Express each of the following ratios in the simplest form : (i) 39 : 65 (iv) 120 : 144 (ii) 200 : 350 (v) 0.32 : 1.2 (iii) 172 : 528 (vi) 4860 : 8370

3. Find the ratio (in the simplest form) of : (i) 45 cm to 5 m (iii) 90 paise to Rs 3 (v) 1 hour to 15 seconds (ii) 200 g to 5 kg (iv) 35 minutes to 45 seconds (vi) Rs 38 to Rs 9.50.

4. In a year, Narmita earned Rs 84000 and paid Rs 1200 as income tax. Find the ratio (in the simplest form) of her (i) income to income tax (ii) income tax to income. 5. Total number of workers in a factory is 196. 49 of them are men and the rest are women. Find the ratio (in the simplest form) of : (i) the number of men to that of women (ii) the number of women to the total number of workers

214

Mathematics

6. A rectangular sheet of paper is 30 cm long and 21 cm wide. Find the ratio (in the simplest form) of its (i) width to length (ii) length to perimeter. 9.5 DIVISION OF A NUMBER IN THE GIVEN RATIO To divide a given number x in the given ratio l : m, we follow the procedure given below : Step 1 : Find the sum of the two terms of the ratio, i.e. l + m. Step 2 : Use the following formula to obtain the two parts of the given number x. First part = Second part = l x l+m m x l+m

Remark : The same procedure is followed even if the given number is to be divided in more than two parts in the given ratio. Example 9.6 : Divide 144 in two parts in the ratio 7 : 9. Solution : We have Sum of the two terms of the ratio = 7 + 9 = 16

First part = second part =

7 144 = 63 16 9 144 = 81 16

and

Thus, the two parts of 144 with given ratio are 63 and 81. Example 9.7 : Divide Rs 1050 among A, B, C in the ratio 3 : 5 : 7. Solution : We have Sum of the terms of the ratio = 3 + 5 + 7 = 15

3 1050I FH 15 K F5 I Bs share = Rs H 15 1050K As share = Rs Cs share = Rs 7 1050I FH 15 K

= Rs 210 = Rs 350 = Rs 490.

and

Ratio and Proportion

215

CHECK YOUR PROGRESS 9.2 1. (i) Divide 15 in the ratio 2 : 3. (ii) Divide 184 in the ratio 3 : 5 (iii) Divide 7780 in the ratio 7 : 8 : 5. 2. Divide : (i) Rs 140 in the ratio 2 : 5 (ii) Rs 154 in the ratio 3 : 4 (iii) 9 cm 8 mm in the ratio 2 : 5 (iv) 10 cm 5 mm in the ratio 1 : 4. 3. The ratio between two quantities is 2 : 7. If the second quantity is 9.8 kg, how much is the first ? 4. The angles of a triangle are in the ratio 3 : 5 : 7. Find the angles. [Hint : Sum of the three angles of a triangle is 180] 5. In a camp, the ratio of the number of teachers to the number of students is 3 : 50. If the total number of persons in the camp is 159, then find the number of teachers. 6. In a bank, the ratio of the number of clerks to the number of officers is 17 : 2. If the total number of persons working in the bank is 57, find the number of clerks in the bank. 7. The sides of a triangle are in the ratio 2 : 3 : 4. If the perimeter is 36 cm, then find its sides. 9.6. PROPORTION An equality of two ratios constitutes a proportion. Consider two ratios 8 : 14 and 20 : 35. On expressing these ratios in the simplest form, we find that 8 : 14 = 4 : 7 and Therefore, Thus, Similarly, 20 : 70 = 2 : 7 is a proportion. Thus, we define proportion as follows : Four numbers a, b, c, d (in order) are said to be in proportion, if the ratio of the first two is equal to ratio of the last two i.e. a : b = c : d. 20 : 35 = 4 : 7 8 : 14 = 20 : 35 8 : 14 = 20 : 35 is a proportion.

216

Mathematics

When four numbers a, b, c, d (in order) are in proportion, then we write a:b::c :d which is read as a is to be as c is to d or a to b as c to d. In a proportion a : b : : c : d, a, b, c and d are the first, second, third and fourth terms of the proportion. The first and fourth terms are called extremes, the second and third terms are called means. a : b=c : d a=c b d or ad = bc.

Note : In a proportion, the product of extremes is equal to the product of means. In other words a : b : : c : d if and only if ad = bc. Example 9.8 : Which of the following four numbers constitute a proportion ? (i) 3, 5, 15, 25 (ii) 3, 15, 25, 5 Solution : (i) Here, The product of extremes = 3 25 = 75 The product of means = 5 15 = 75 Since the two products are equal, the four numbers are in proportion. (ii) Here, The product of extremes = 3 5 = 15 The product of means = 15 25 = 375 Since the two products are not equal, the four numbers are not in proportion. Remark : From Example 9.8 above, we find that the order of four numbers for a proportion is important. Example 9.9 : Are, 5, 10, 30, 60 in proportion ? Solution : Ratio of first two terms is 5 : 10, which is equal to 1 : 2 Ratio of last two terms is 30 : 60, which is equal to 1 : 2

5 : 10 = 30 : 60

Hence, 5, 10, 30, 60 are in proportion. Aliter Here, Product of extremes = 5 60 = 300 Product of means = 10 30 = 300 Since the two products are equal, then given numbers are in proportion.

Ratio and Proportion

217

Example 9.10 : The first, second and fourth terms of a proportion are 5, 10 and 30 respectively. Find the third therm. Solution : Let the third term be x so that 5, 10, x, 30 are in proportion. 5 30 = 10 x x=
5 30 = 15 10

Thus, the third term is 15. Example 9.11 : The two extremes of a proportion are 12 and 28. If one of means is 24, then find the other mean. Solution : Let the other mean be x. We know that for a proportion, product of means = product of extremes 24 x = 12 28 x=
12 28 24

x = 14

Thus, the other mean is 14. Example 9.12 : Are 4, 8, 8, 16 in proportion ? Solution : Here, Product of means = 8 8 = 64 Product of extremes = 4 16 = 64 Since the two products are equal, the four numbers 4, 8, 8, 16 are in proportion. Remark : In Example 5 above, the means are the same. In this case, we say that 4, 8, 16 are in continued proportion. Thus, if a, b, b, c are in proportion, we say that a, b, c are in continued proportion. In such a case, b2 = ac and b is called the mean proportional of a and c. CHECK YOUR PROGRESS 9.3 1. Which of the following statements are true ? (i) 4 : 6 = 8 : 12 (iii) 16 : 24 : : 30 : 20 (ii) 4 : 28 = 7 : 1 (iv) 6.0 : 4.5 : : 4.8 : 3.6

218

Mathematics

2. Determine which of the following numbers are in proportion : (i) 18, 27, 12, 18 (iii) 2, 3, 14, 15 (ii) 12, 10, 16, 10 (iv) 40, 30, 60, 45

3. Find the value of x, if x : 6 : : 5 : 3. 4. If 18, x, x, 50 are in proportion, find the value of x. 5. If 3, x, 12 are in continued proportion, find x. 6. Set up all possible proportions from the numbers 15, 18, 35 and 42. 7. Find the mean proportional between 5 and 125. 9.7 DIRECT PROPORTION (VARIATION) If you buy 6 pens for Rs 12, you will have to pay Rs 16 for 8 pens and Rs 24 for 12 pens. Thus, you see that as the number of pens purchased increases, the price you have to pay also increases. Again, if you buy 4 pens, you will have to pay only Rs 8. Here you see that more the number of pens purchased, more money you have to pay; less the number of pens purchased, less money you have pay. To indicate the above situation, we say that the number of pens purchased and the price paid are in direct proportion. In general, two quantities a and b are said to be in direct proportion if increase/decrease in one quantity results in the corresponding increase/decrease in the other. We also say that a varies directly as b and write a = kb, where k is a constant, called the constant of variation or proportionality. 9.7.1 Inverse Proportion (Variation) If two persons, employed to construct a wall, take 24 days to complete it, then 4 persons employed to do the same job will take 12 days to complete it. Likewise, 6 persons will take only 8 days to complete the job. We may note that as the number of persons employed increases, the number of days required to complete the job decreases. Thus, we see that lesser the number of persons employed, more the number of days Or more the number of persons employed, lesser the number of days. We say that the number of persons employed to complete a job and the number of days taken by them are in inverse proportion (variation). In general, two quantities a and b are said to be in inverse proportion if increase/decrease in one quantity results in corresponding decrease/increase in the other quantity. We say that a varies inversely (indirectly) to b, if a = proportionality. k or ab = k, where k is a constant of b

Ratio and Proportion

219

9.8 APPLICATIONS OF DIRECT PROPORTION We will take some examples to illustrate the application of direct proportion (variation) in daily life situations. Example 9.13 : If 5 books cost Rs 70, how much will 8 books cost ? Solution : Let Rs x denote the cost of 8 books, we get the following : Quantity 5 8 5 : 8 = 70 : x
5 70 = 8 x

Cost (in rupees) 70 x

Since the cost varies directly as quantity. Equating the two ratios we get

x=

8 70 = 112 5

Thus, the cost of 8 books is Rs 112 Example 9.14 : A typist types 70 pages in 5 days. How many pages can he type in 7 days? Solution : Let y denote the number of pages that the typist would type in 7 days Since the number of pages vary directly as the number of days, we get the following : Number of pages 70 y Equating the two ratios, we get 5 : 7 = 70 : y 70 5 = y 7 y=
70 7 = 98 5

Number of days 5 7

Thus, the typist will type 98 pages in 7 days. Example 9.15 : If 15 men can build a 100 m long wall in some days; how many men should be employed to build 120 m long wall of the same height in the same time. Solution : Let n denote the number of persons to be employed to build a wall 120 m long. Since the number of men varies directly as the length of the wall, we get the following :

220

Mathematics

Number of men 15 n Equating the two ratios, we get 100 : 120 = 15 : n


100 15 = 120 n

Length of wall (in metres) 100 120

n=

15 120 = 18 100

Thus, 18 men will build 120 m long wall in the same time. Example 9.16 : A jeep travels 50 km in 1 hour. How much distance would it travel in 12 minutes ? Solution : Let d denote the distance (in km) travelled by the jeep in 12 minutes. Since the distance travelled and time taken vary directly we get the following : Time (in minutes) 60 12 Equating the two ratios we get 60 : 12 = 50 : d
60 50 = 12 d

Distance (in km) 50 d

d=

50 12 = 10. 60

Thus, the jeep would travel 10 km in 12 minutes. Example 9.17 : If a deposit of Rs 3000 carries an interest of Rs 900 in some time, what interest would a deposit of Rs 5000 earn in the same time ? Solution : Let I denote the interest that Rs 5000 would earn. Since the interest earned and the amount deposited vary directly, we get the following : Principal (in Rs) 3000 5000 Interest (in Rs) 900 I

Ratio and Proportion

221

Equating the two ratios, we get 3000 : 5000 = 900 : I


3000 900 = 5000 I

I=

5000 900 = 1500 3000

Thus, the deposit of Rs 5000 would earn an interest of Rs 1500 in the given time. 9.9 PARTNERSHIP When two or more individuals enter into business jointly we say that they enter into a partnership. These individuals are called partners. In partnership, all or some partners invest money in business for same or different periods of time. The profits or losses of a partnership business are shared among the partners in the ratio of their investments. The following examples illustrate the above concept. Example 9.18 : A and B start a business by investing Rs 40000 and Rs 60000 respectively. If at the end of the year they earn a profit of Rs 35000, find the share of each in the profit. Solution : Let A's share be Rs x. Then Bs share is Rs (35000 x). Since share of profit varies directly as the investment, equating the two ratios, we get 40000 : 60000 = x : (35000 x)
40000 x = 60000 35000 x 2 x = 3 35000 x

2(35000 x) = 3x 70000 2x = 3x 5x = 70000 x = 14000

Thus As share is Rs 14000; and that of B is Rs (35000 14000) i.e. Rs. 21000. CHECK YOUR PROGRESS 9.4 1. If 7 pens cost Rs 98, find the cost of 35 pens. 2. If the cost of 12 erasers is Rs 30, how many erasers can be bought for Rs 50 ?

222

Mathematics

3. If 5 litres of petrol cost Rs 140, how much will 8 litres of petrol cost ? 4. A carpenter prepares 18 chairs in 4 days. In how many days would he prepare 27 such chairs ? 5. A tailor stitches 10 trousers in 4 days. How many trousers can he stitch in 10 days ? 6. A woman can pack 270 bundles in 3 days. How many bundles can she pack in 7 days? 7. If 30 men can weave 40 metres of cloth in a day, how many meters of cloth can be woven by 240 men in a day ? 8. 4 taps of equal capacity can fill 6 tubs in a given time. How many tubs can be filled by 10 taps in the same time ? 9. If a family of 5 members consumes 50 kg of cereals in a month, what quantity of cereals would be consumed by family of 6 members in a month ? 10. A car travels 160 km in 4 hours. How long will it take to travel 400 km? 11. For every 50 students, 3 teachers are appointed. How many teachers, should be appointed, if a school has 1250 students ? 12. A man earns Rs 1750 per week. How much will he earn in 4 days ? 13. 10 men dig a trench in 9 days. In how many days would 15 men dig the same trench? 14. 20 farmers plough a field in 8 days. How many days will 16 farmers take to plough the field. 15. If the thickness of a pile of 20 cardboards is 50 mm, find the thickness of a pile of 30 similar cardboards ? 16. A deposit earns Rs 120 in 24 years. How much will it earn in 5 years ? 17. If the interest on Rs 3000 is Rs 150 for a certain period, what will be the interest on Rs 4500 for the same period, when the rate of interest remains the same ? 18. A and B start, a business with Rs 60000 and Rs 75000 respectively. At the end of the year, they earned a profit of Rs 40500. Find the share of each partner in the profit. 19. A and B start a business with Rs 70000 and Rs 80000 respectively. They earn Rs 50700 as the annual profit. If Rs 1500 per month out of the profit is paid as rent, find the share of each after paying the rent. [Hint. Subtract Rs (1500 12) i.e. Rs 18000 paid for the rent from the profit of Rs 50700. The balance of profit will be shared in the ratio 70000 : 80000] 20. Two partners together invested Rs 2.12 lakhs in a joint business. After a year, one gets Rs 35000 and the other gets Rs 18000 as profit. Find the sum invested by each.

Ratio and Proportion

223

9.10 INVERSE PROPORTION We will now take some examples to illustrate the application of inverse proportion (variation) in daily life situations. Example 9.19 : 4 girls can do a work 3 days. How many days will 6 girls take to do the some work ? Solution : Let x be the number of days in which 6 girls can do the work, Since the number of girls varies inversely as the number of days, we get the following : Girls 4 6 The ratios of like terms are 4 : 6 and 3 : x Equating ratio with the inverse of the other, we get 4 : 6=x : 3 4 x = 6 3 x= 43 =2 6 Days 3 x

Thus, 6 girls will complete the work in 2 days. Example 9.20 : A car takes 4 hours to cover some distance at an average speed of 55 km per hour. How much time will it take to cover the same distance at an average speed of 40 km per hour ? Solution : Let t (in hours) denote the time that the car will take at the speed of 40 km per hour, to cover the same distance. Since speed varies inversely as time, we get the following : Speed 55 40 The ratios of like terms are 55 : 40 and 4 : t. Equating the ratio with the inverse of the other, we get 55 : 40 = t : 4 or or
t 55 = 40 4

Time 4 t

55 4 = 5.5 40 Thus, travelling at a speed of 40 km per hour, the car will cover the same distance in 5.5 hours. t=

224

Mathematics

CHECK YOUR PROGRESS 9.5 1. If 3 men can build a wall in 15 days, how long will 5 men take to build the same wall? 2. 400 men have provision for 23 days. If 60 more men join them, how long will the provisions last ? 3. To cover some distance, a train, running at a speed of 80 km per hour, takes 12.5 hours. If it runs at a speed of 100 km per hour, how much time will it take to cover the same distance ? 4. A man deposited Rs 5000 in a bank for one year to get a fixed amount of interest. If the same interest is to be earned in 8 months at the same rate of interest, what sum head to be deposited ? 5. 6 taps of equal capacity can fill a reservoir in 15 minutes. How many taps can fill it in 10 minutes ? 6. A journey takes 21 minutes if I walk at 4.5 km per hour. How long will it take if I walk at 3.5 km per hour ? 7. 120 men had provisions for 200 days. After 5 days, 30 men left. How long will the remaining provision last ? LET US SUM UP
z z z

Ratio is a comparison of numbers or quantities of the same kind. In a ratio a : b, a is the first term (antecedent) and b the second term (consequent). The order terms in a ratio is important a : b is not the same as b : a

z z z z

A ratio is said to be in the simplest form if the H.C.F. of its two terms is 1. A statement indicating equality of two ratios is called a proportion. A proportion has four terms. In a proportion a : b : : c : d, a and d are called the extremes and b and c are called the means. Four numbers a, b, c, d are in proportion if and only if ad = bc. If a : b = b : c, then a, b, c are said to be in continued proportion. If a, b, c are in continued proportion, then b2 = ac and b is called mean proportion of a and c. Two quantities are said to be in direct proportion if the increase/decrease in one results in a corresponding increase/decrease in the other. Two quantities are said to be in inverse proportion if an increase/decrease in one results in a corresponding decrease/increases in the other.

z z z

Ratio and Proportion

225

TERMINAL EXERCISE 1. Write each of the following ratios in the simplest form : (i) 64 : 160 (i) 450 m, 5 km (iii) 16 mm, 3 cm 3. Divide (i) Rs 154 in the ratio 5 : 9 (ii) Rs 9292 in the ratio 1 : 3. 4. The ratio between two quantities is 2 : 5. If the second quantity is 9.5 kg, find the first quantity. 5. The angles of a triangle are in the ratio 3 : 7 : 8. Find the angles. 6. Determine the value of p if 20 : p : : 25 : 450. 7. Are 5, 9, 81, 45 in proportion ? 8. A dish for 250 persons uses 15 cups of sugar. How many cups of sugar are needed for 150 persons ? 9. If 12 boxes are required to hold 60 dozen apples, how many boxes will be required to hold 105 dozen ? 10. A 12 m length of an angle iron weighs 24.12 kg. What will a piece of length 8 m of the same weigh ? 11. A watch gains 42 seconds in 3 days 8 hours. In how much time will it gain 2 minutes 6 seconds ? 12. A mini bus uses 35 litres of diesel for a journey of 420 km. How many litres of diesel are needed for 600 km ? 13. A and B started a business by investing Rs 50000 and Rs 75000 respectively. Find the share of each out of a profit of Rs 15000. 14. A and B entered into a partnership investing Rs 35000 and Rs 42000 respectively. A is a working partner and gets Rs 300 per month for the same. Find the share of each in an annual profit of Rs 13500. 15. If the sales tax on a purchase worth Rs 600 is Rs 42, what will be the sales tax on the purchase worth Rs 1500 ? (ii) 216 : 384 (iii) 1.5 : 9 2. Find the ratio of the following in the simplest form : (ii) 50 minutes, 1 hour 5 minutes (iv) 5 kg 600 g, 6 kg 400 g.

226

Mathematics

ANSWERS Check Your Progress 9.1 1. (i) 3; 7 (iv) 30 : 1 2. (i) 3 : 5 (iv) 5 : 6 3. (i) 9 : 100 (iv) 140 : 3 4. (i) 70 : 1 5. (i) 1 : 3 6. (i) 7 : 10 Check Your Progress 9.2 1. (i) 6 and 9 2. (i) Rs 40 and Rs 100 (iv) 21 mm and 84 cm 3. 2.8 kg 6. 51 Check Your Progress 9.3 1. (i), (iv) 4. x = 30 2. (i), (iv) 5. x = 6 7. 25 3. x = 10 4. 36, 60, 84 7. 8 cm, 12 cm and 16 cm 5. 9 (ii) 69 and 115 (ii) Rs 66 and Rs 88 (iii) 2723, 3112 and 1945 (iii) 28 mm and 70 mm (ii) different (v) simplified form (ii) 4 : 7 (v) 4 : 15 (ii) 1 : 25 (v) 240 : 1 (ii) 1 : 70 (ii) 3 : 4 (ii) 5 : 17. (iii) 43 : 132 (vi) 18 : 31 (iii) 3 : 10 (vi) 4 : 1 (iii) 3 : 4

6. 15 : 18 : : 35 : 42 ; 18 : 15 : : 42 : 35 Check Your Progress 9.4 1. Rs 490 4. 6 days 7. 320 metres 10. 10 hours 13. 6 days 16. Rs 25 19. Rs 15260; Rs 17440 2. 20 5. 25 8. 15 11. 75 14. 10 days 17. Rs 225 20. Rs 140000; Rs 72000.

3. Rs 224 6. 630 9. 60 kg of cereals 12. Rs 1000 15. 75 mm 18. Rs 18000; Rs 22500

Ratio and Proportion

227

Check Your Progress 9.5 1. 9 days 4. Rs 7500 7. 260 days Terminal Exercises 1. (i) 2 : 5 2. (i) 9 : 100 (iv) 7 : 8 3. (i) Rs 55 and Rs 99 4. 3.8 kg 7. No 10. 16.08 kg 13. Rs 6000; Rs 9000 (ii) Rs 2323 and Rs 6969 5. 30, 70, 80 8. 9 11. 10 days 14. Rs 8100; Rs 5400 6. 360 9. 21 12. 50 litres 15. Rs 105 (ii) 9 : 16 (ii) 10 : 13 (iii) 1 : 6 (iii) 8 : 15 2. 20 days 5. 9 3. 10 hours 6. 27 minutes

228

Mathematics

10
Percentage and its Applications
10.1 INTRODUCTION In every day life, you come across situations in which the word percent is made use of very frequently. For example, you see a banner in the market. Sale upto 50 percent off You read news in the newspaper Votes turnout in the poll was over 65 percent Banks have lowered the rate of interest on fixed deposits from 7.5 percent to 6.5 percent There are many such situations in different walks of life where the concept of percentage finds its use. In this lesson we shall study percent as a fraction or a decimal and its application in solving problems of profit and loss, discount, sales tax, instalments etc. 10.2 OBJECTIVES After studying the lesson, the learner will be able to :
z z z z z

write a fraction and a decimal as a percent and vice-versa calculate specified percent of a given number or a quantity solve problems based on percentage solve problems on profit and loss calculate simple interest and amount when a given sum of money is interested for a specified time period on a given rate of interest. state the need for given discount define discount and discount series (successive discounts, no. more than three) find a single discount equivalent to a given discount series calculate the discount and the selling price of an article, given marked price of the article, and the rate of discount

z z z z

Percentage and its Applications

229

z z

solve inverse problems pertaining to discount calculate the sales tax on commission and the selling price of an article, given the marked price of the article and the rate of sales tax or commission solve inverse problems pertaining to sales tax solve inverse problems on commission determine the amount of each instalment when goods are purchased under investment plan (case of equal instalments only) determine the rate of interest when equal instalments are given

z z

10.3 EXPECTED BACKGROUND KNOWLEDGE


z

Four fundamental operations on whole numbers, fractions and decimals.

10.4 PERCENT You have learnt a lot about fractions. A fraction denotes part of a whole. For instances 3 out of 4 equal parts. Similarly, of 100 equal parts. A fraction whose denominator is 100 is read as a percent, for example percent. In Fig. 10.1, 33 out of 100 small squares are shaded. This means 33 of the larger square is shaded. 100 5 is read as five 100 3 means 4

7 17 means 7 out of ten equal parts and means 17 out 10 100

The word percent is abbreviated form of the Latin word per centum which means per hundred or hundredths. The symbol % is used for the term percent. A ratio whose second term is 100 is also called a percent.

Fig. 10.1

When we say Anita has secured 80% marks in mathematics. This means that she has secured 80 marks out of 100 or 40 marks out of 50. Similarly, when we say a man has spent 20% of his income on food, it means that out of every hundred rupees of his income, he has spent Rs 20 on food. Suppose, we wish to compare two fractions 3 4 and . 4 5

Since these are fractions having different denominators we need to convert them into fractions with common denominator.

230

Mathematics

3 3 5 = 15 4 4 4 = 16 = ; = 4 4 5 20 5 5 4 20 Since

16 > 15, 16 > 15 4 3 ; and so > 20 20 5 4

Here, we have converted each of the two fractions into fractions with least common denominator. As a convention, we convert if possible each fraction into a fraction with denominator 100. In the above example,
3 3 25 = 75 4 4 20 = 80 = ; = 4 25 100 5 5 20 100 4

Since 80 > 75. we get

4 3 > . 5 4

10.5 CONVERSION OF A DECIMAL INTO A PERCENT AND VICE VERSA Let us consider the following examples : 0.37 = 37 7 = 70 146 = 37%, 0.7 = = 70%; 1.46 = = 146% 100 10 100 100

Thus, to write a decimal as a percent, we move the decimal point two places to the right and put the % sign. 0.25 = 25% 0.1 = 10% Conversely, To write a percent as a decimal, we drop the % sign and insert or move the decimal point two places to the left. For example, 37% = 0.37 99% = 0.99 110% = 1.10 89% = 0.89 100% = 1.00 212% = 2.12 35% = 0.35 3% = 0.03 0.1% = 0.001 0.61 = 61% 0.07 = 7% 0.37 = 37% 1.4 = 140%

10.6 CONVERSION OF A PERCENT INTO A FRACTION AND VICE VERSA To write a percent as fraction, we drop the % sign and divide the number by 100. For example, 69% = 4.5% = 69 100 4.5 = 45 100 1000 13% = 170% = 13 100 170 100 3% = 216% = 3 100 216 100

Percentage and its Applications

231

In general, x% = Conversely,

x . 100

To write a fraction as a percent, we multiply the fraction by 100, simplify it and suffix the % sign. For example
1 1 = 4 100 % = 25% 4 2

FH IK 3 F3 I = H 2 100K % = 150%
3 3 = 5 100 % = 60% 5

FH

IK

CHECK YOUR PROGRESS 10.1 1. Write each of the following decimals as percent : (a) 0.56 (e) 0.97 (a) 75% (d) 2% (a) 35% (b) 0.03 (f) 0.8 (b) 14% (f) 25% (b) 40% (c) 0.75 (g) 0.04 (c) 3% (g) 400% (c) 70% (d) 0.02 (h) 1.4 (d) 115% (h) 350% (d) 85%

2. Write each of the following percents as decimal :

3. Write each of the following percents as fraction : 4. Convert each of the following fractions into percent : (a) 3 4 (b) 1 5 (c) 3 10 (d) 4 25

5. Aruna obtained 19 marks in a test of 25 marks. What was her percentage of marks ? 6. Gurmeet got half the answers correct. What percent of their answers were correct ? 7. A suit piece consists of cotton and rayon fibre in which cotton is 3 out of 8 parts. What is the percentage of cotton in the suit piece ? 8. Kavita read 84 pages of 100-page book. What percent of the book did she read ? 9. A class of a school had 45% girls. What percent of the class were boys ? 10. One-fourth of the shoes in a shop were on sale. What percent of the shoes were there on normal price ?

232

Mathematics

11. In the word PERCENTAGE, what percent of the letters are Es ? 12. If three fourths of students of a class wear glasses, what per cent of students of the class do not wear glasses ? 13. There are 20 eggs in a fridge and 6 of them are brown. What percent of eggs are brown ? 14. 60 candidates appeared in an examination and 45 of them passed. What percent of candidate passed ? 15. Mr. X spends Rs 310 out of Rs 500 and Mr. Y spent Rs 500 out of Rs 800 every week. Compute their spending as percentages and state who spends higher percentage. 16. In a class of 40 students, 10 secured first division, 15 secured second division, and 13 just qualified. What is the percentage of students that failed ? 10.7 CALCULATION OF PERCENT OF A QUANTITY To determine the percent of a number or quantity, we first change the percent to a fraction or a decimal and then multiply with the number. For example, or 45% of 90 = 0.45 90 = 40.50 45% of 90 =
45 90 = 40.50 100

60% of 120 = 0.60 120 = 72.00 18% of 215 = 0.18 215 = 38.70 135% of 80 = 1.35 80 = 108 Example 10.1 : A family spends 35% of its monthly budget of Rs 7500 on food. How much do they spend on food ? Solution : Expenditure on food = 35% of Rs 7500 = 0.35 Rs 7500 = Rs (0.35 7500) = Rs 2625.00 or Rs 2625. Example 10.2 : In a garden, there are 500 plants of which 35% are trees, 20% are shrubs and 25% are herbs. The rest are creepers. Find out the number of trees, shrubs, herbs and creepers. Solution : Number of trees = 35% of 500 = 0.35 500 = 175 Number of shrubs = 20% of 500 = 0.20 500 = 100 Number of herbs = 25% of 500 = 0.25 500 = 125 Since the remaining plants are creepers, Number of Creepers = 500 (175 + 100 + 125) = 500 400 = 100

Percentage and its Applications

233

Example 10.3 : 35% of students in a school are girls. If the total number of students is 1240, find the number of boys in the school. Solution : Number of girls in the school = 35% of 1240 = 0.35 1240 = 434
No. of boys in the school = 1240 434 = 806

Aliter Since 35% of the students in the school are girls, (100% 35%) i.e., 65% of the students in the school are boys.

Number of boys = 65% of 1240 = 0.65 1240 = 806

Example 10.4 : What percent of 240 is 96 ? Solution : Percent = 96 100 % 240

= 40% Example 10.5 : If 27% of a is 54, then find a. Solution : We have 27% of a = 54 27 100 a = 54 a= 54 100 = 200 27

Thus, the value of a is 200. Example 10.6 : 60 is reduced to 45. What is the reduction percent ? Solution : Let 45 is less than 60 by x%, then Reduction = 60 45 = 15 Reduction percent = 15 100 % = 25%. 60

Example 10.7 : If 80 is increased to 125, what is the increase percent ? Solution : Increase = 125 80 = 45 Increase percent = 45 100 % = 56.25%. 80

234

Mathematics

Example 10.8 : A voluntary organisation was collecting money for a relief camp. Their target was Rs 20000, but they exceeded their target by 45%. How much money did they collect ? Solution : The money collected = 20000 + 45% of 20000 = Rs 20000 + Rs (0.45 20000) = Rs 20000 + Rs (9000.00) = Rs 20000 + Rs 9000 = Rs 29000 Hence, they collected Rs 29000. Example 10.9 : 44% of the students of a class are girls. If the number of girls is 6 less than the number of boys, how many students are there in the class ? Solution : Given that 44% of the students are girls. So, (100 44)% i.e., 56% of the students are boys. Thus, there are 12% less girls than boys. By the given condition, 12 of the number of students = 6 100

Number of students =

6 100 = 50 12

Thus, there are 50 students in the class. Example 10.10 : Raman has to secure 40% marks for passing. He gets 178 marks and fails by 22 marks. Find the maximum marks. Solution : Since Ramesh secures 178 marks and fails by 22 marks, the pass marks are 178 + 22 = 200. Let x be maximum marks, then 40% of x = 200 or or 40 x = 200 100 x=

200 100 = 500 40 Thus, the maximum marks are 500. CHECK YOUR PROGRESS 10.2 1. Find : (a) 15% of 440 (c) 47% of Rs 1200 (b) 16% of 1250 (d) 39% of 1700 metres.

Percentage and its Applications

235

2. At a school, 40% of the students come on foot to the school. There are 600 students in the school. How many students come on foot to the school ? 3. There are 36 children in a class 25% of them are boys. How many are boys ? How many are girls ? 4. An alloy is a combination of zinc and copper with 30% zinc and 70% copper. If a piece of this alloy weighs 150 kg , how much zinc it contains ? 5. Naresh earns Rs 15400 per month. He keeps 50% for household expenses, 15% for his personal expenses, 25% for expenditure on his children and the rest he saves. What amount does he save per month ? 6. There are 32 boys in a class. On a particular day 12.5% of them were absent. How many boys were absent on that day ? 7. It takes me 45 minutes to go to school and I spend 80% of the time travelling by bus. How long does the bus journey last ? 8. During a general election, 70% of the population voted. If 70000 people cast their votes, what is the population of the town ? 9. The cost of a saree was Rs 450. Its cost has increased to Rs 495. By what percent did the cost increase ? 10. What percent of 160 is 64 ? 11. If 120 is reduced to 96, what is the percentage reduction ? 12. In an election, 25% voters did not cast their votes. A candidate secured 40% of the votes polled and was defeated by 900 votes. Find the total number of voters. 13. As income is 25% more than Bs and Bs income is 8% more than Cs. If As income is Rs 4050, then find the Cs income. 14. A reduction of 10% in the price of tea enables a dealer to purchase 25 kg more tea for Rs 22500. What is the reduced price per kg of tea ? Also, find the original price per kg. 15. A rise of 25% in the price of sugar compels a person to buy 1.5 kg of sugar less for Rs 240. Find the increased price as well as the original price per kg of the sugar. 16. A number is first increased by 10% and then decreased by 10%. What is the net increase or decrease percent ? 17. A man donated 5% of his monthly income to a charity and deposited 12% of the rest in a bank. If he has Rs 11704 with him now, what was his monthly income ? 10.8 APPLICATIONS OF PERCENTAGE We come across a number of situations in our day to day life wherein we use the concept of percent. In the following, we discuss applications of the concept of percentage in different fields.

236

Mathematics

10.8.1 PROFIT AND LOSS Let us recall the terms and formulae related to profit and loss that we have learnt earlier. Cost Price (C.P.) : The price at which an article is purchased, is called its cost price. Selling Price (S.P.) : The price at which an article is sold, is called its selling price. Profit (Gain) : When S.P.>C.P., then there is a profit. Profit = S.P. C.P. Loss : When C.P. > S.P, then there is a loss. Loss = C.P. S.P. Note : Gain or Loss is always calculated on C.P. Formulae and Formula (1) gives Profit = or or or S.P. C.P. = C. P. Profit % 100 C. P. Profit % 100 Profit% = ofit 100I FH Pr K% C. P. F Loss I Loss% = H C. P. 100K % ...(1) ...(2)

C. P. Profit % S.P. = C. P.+ 100 S.P. = Pr ofit % I C. P. FH 100 +100 K ...(3)

Similarly, Formula (2) gives Loss = or or C.P. S.P =


C. P. Loss% 100 C. P. Loss% 100 C. P. Loss% 100

S.P. = C. P.

or

S.P. =

FG 100 Loss% IJ C. P. H 100 K

....(4)

Let us now consider some examples to illustrate the applications of these formulae in solving problems related to profit and loss.

Percentage and its Applications

237

Examples 10.11 : A shopkeeper bought an almirah from a wholesale dealer for Rs 4500 and sold it for Rs 6000. Find his profit or loss percent. Solution : Here C.P. of the almirah = Rs 4500 S.P. of the almirah = Rs 6000 Since S.P. > C.P., there is a profit Profit = S.P. C.P. = Rs 6000 Rs 4500 = Rs 1500

Profit % =

ofit 100I % FH Pr K C. P. F 1500 100IJ % = G H 4500 K =


1 100 % i.e. 33 % . 3 3

Example 10.12 : A retailer buys a cooler for Rs 3800 and overhead expenses on it are Rs 50. If he sells the cooler for Rs 4400, determine his profit percent. Solution : Here, C.P. of the cooler = Rs (3800 + 50) = Rs 3850 S.P. of the cooler = Rs 4400 Since S.P. > C.P., there is a profit Profit = Rs 4400 Rs 3850 ofit 100I % FH Pr K C. P. F 550 I = H 3850 100K % = = Rs 550

Profit % =

100 % 2 i.e. 14 % . 7 7 Example 10.13 : By selling a scooter to a customer for Rs 22400 an auto-dealer makes a profit of 12%. Find the cost price of the scooter. Solution : Here, S.P. of the scooter = Rs 22400, Profit % = 12% Using formula (3), we have S.P. =

FG 100 + Pr ofit% IJ C. P. H 100 K

238

Mathematics

or

C.P. =

S. P.100 100 + Pr ofit % 22400 100 100 + 12 22400 100 112

= Rs

= Rs

= Rs 20000 Thus, the cost price of the scooter is Rs 20000. Example 10.14 : By selling a cycle for Rs 2024, a cycle merchant loses 12%. If he wishes to make a gain of 12%, what will be the selling price of the cycle ? Solution : First Part : S.P. = Rs 2024 and Loss % = 12% From formula (4), we have C.P. =
S. P.100 100 Loss% 2024 100 100 12 2024 100 = Rs 2300 88

= Rs

= Rs

Second Part : C.P. of the cycle = Rs 2300 and Gain (Profit)% = 12% Using formula (3), we have S.P. = = Pr ofit % I C. P. FH 100 +100 K 100 + 12 2300 100 112 2300 100

= Rs

= Rs 2576 Thus, the selling price of the cycle will be Rs 2576. Example 10.15 : If the cost price of 15 articles is the same as the selling price of 12 articles, find the gain or loss percent in the transaction. Solution : Let C.P. of an article be Rs x, Then C.P. of 15 articles = Rs 15x ...(i)

Percentage and its Applications

239

or or

S.P. of 12 articles = Rs 15x S.P. of 1 articles = Rs 15 xI FH 12 K ...(ii)

Since S.P. > C.P., there is a profit (gain) in the transaction Gain = Rs x xI FH 15 K 12 F 3x I F xI = Rs H 12 K or Rs H 4 K

or

Gain % =

FH Gain 100I K% C. P. F x 4 100IJ % = G Hx K F1 I = H 4 100K % = 25%

Thus, the gain in the transaction is 25%. Example 10.16 : By selling 45 oranges for Rs 160, a women loses 20%. How many oranges should she sell for Rs 112 to gain 20% on the whole ? Solution : First Part : S.P. of 45 oranges = Rs 160 Loss% = 20% So, by formula (4), we have C.P. of 45 oranges = S. P.100 100 Loss% 160 100 100 20 160 100 = Rs 200. 80

= Rs = Rs

Second Part : C.P. of 45 oranges = Rs 200 Gain% = 20%

S.P. of 45 oranges = =

100 + Gain% C.P. 100 100 + 20 Rs 200 100

240

Mathematics

= Rs

120 200 100

= Rs 240 Now, number of oranges for Rs 240 = 45 Number of oranges for Rs 112 =
45 112 = 21 240

Thus, the women should sell 21 oranges for Rs 112. CHECK YOUR PROGRESS 10.3 1. A shopkeeper buys an article for Rs 320 and sells it for Rs 240. Find his gain or loss percent. 2. A dealer buys a wrist watch for Rs 450 and spends Rs 30 on its to repairs. If he sells the same for Rs 600, find the profit per cent. 3. A dealer sold two machines at Rs 2400 each. On selling one machine he gained 20% and on selling the other he lost 20%. Find the dealers gain or loss percent. 4. A sells an article costing Rs 1000 to B and earns a profit of 6%. B, in turn sells it to C at a loss of 5%. At what price did C purchase the article ? 5. By selling 90 ball pens for Rs 160, a person loses 20%. How many ball pens should he sell for Rs 96, so as to have a gain of 20% ? 6. If the selling price of 20 articles is equal to the cost price of 23 articles, find the loss or gain percent. 7. A watch was sold at a profit of 12%. Had it been sold for Rs 33 more, the profit would have been 14%. Find the cost price of the watch. 8. By selling a book for Rs 258, a publisher gains 20%. For how much should he sell it to gain 30% ? 9. A vendor bought bananas at 6 for 5 rupees and sold them at 4 for 3 rupees. Find his gain or loss percent. 10.8.2 Simple Interest All the transactions that take place around us involve money. Sometimes, a person has to borrow some money as a loan from his friends, relatives, bank etc. He promises to return it after a specified time period. So, he has to give back not only the money he borrows but also some extra money to the lender for using his money. The money borrowed is called the principal, usually denoted by P. The extra money paid is called interest, usually denoted by I.

Percentage and its Applications

241

The sum of principal and the interest is called the amount usually denoted by A. A=P + I Interest is calculated on the principal Interest is mostly expressed as a rate percent per year (per annum). Interest depends on how much money (P) has been borrowed and the duration of the time (T) for which it is borrowed. Interest is calculated according to an agreement, which specifies a certain percent of the principal for each years use, called the rate of interest. I=P R T Interest as calculated above is called simple interest. Example 10.17 : A man borrowed Rs 50000 from a finance company for buying a motor bicycle for a period of 2 years. If the finance company charged simple interest at the rate of 15% per annum, how much interest was paid by the man to the finance company. Solution : Here, Principal (P) = Rs 50000 Time (T) = 2 years Rate (R%) = 15% = 0.15 We have I=P R T = Rs (50000 0.15 2) = Rs 15000 Hence, the man paid Rs 15000 as the interest to the finance company. Example 10.18 : A certain sum of money was deposited for 5 years. Simple interest at the rate of 12% was paid. Calculated the sum deposited if the simple interest received by the depositor is Rs 1200. Solution : Let the sum deposited be Rs P. Given that I = Rs 1200; T = 5 years; R = 12% We have I = P R T or P= I RT 1200 = Rs 2000 012 . 5

= Rs

Thus, the sum deposited was Rs 2000.

242

Mathematics

Example 10.19 : At what rate of simple interest will a sum of Rs 3000 become Rs 4920 at simple interest in 4 years ? Solution : Here, P = Rs 3000; A = Rs 4920; T = 4 years We have I = A P = Rs 4920 Rs 3000 = Rs 1920 Now, I=P R T R= = I 1920 = Rs PT 3000 4 16 = 16% 100

Thus, at the rate of 16%, Rs 3000 will become Rs 4920 in 4 years. Example 10.20 : In what time will Rs 8000 amount to Rs 12000, if simple interest is charged at the rate of 6% per annum ? Solution : Here, P = Rs 8000; A = Rs 12000; R = 6% = 0.06 We have Now, I=A P = Rs 12000 Rs 8000 = Rs 4000 I=P R T T= = I 4000 = PR 8000 0.06 1 100 = 8 years 3 12

= 8 years 4 months Thus, in 8 years 4 months, Rs 8000 will amount to Rs 12000 at the rate of interest 6% per annum. CHECK YOUR PROGRESS 10.4 1. Ramesh borrowed Rs 7000 from his friend at 8% per annum simple interest. He returned the money after 2 years. How much did he pay back altogether ? 2. Jaya deposited Rs 15600 in a bank. The bank pays interest at 8% per annum. Find the interest she will receive at the end of 3 years. 3. Subnam lent Rs 25000 to her friend. She gave Rs 10000 at 10% per annum and the remaining at 12% per annum. How much interest did she receive in 2 years ? 4. Nalini borrowed Rs 5000 from her friend at 8% per annum. She returned the money after 6 months. How much amount did she pay to her friend ?

Percentage and its Applications

243

5. Find the interest received by Anil if he deposits Rs 16000 for 8 months at the rate of 9% per annum. Also, find the amount. 6. Shalini deposited Rs 14500 in a finance company for 3 years and received Rs 4785 as simple interest. What was the rate of interest per annum ? 7. In how many years will Rs 8000 amount to Rs 16000, if simple interest is earned at the rate of 12% per annum ? 8. In how much time will simple interest be annum ? 9. In which case, is interest earned more : (a) Rs 5000 deposited for 5 years at 4% per annum (b) Rs 4000 deposited for 6 years at 5% per annum ? 10. At what rate of interest will simple interest be half the principal in 5 years ? 10.9 DISCOUNT You must have seen all around advertisements of the following types, especially during the festival seasons. SALE Discount upto 50% A discount is a reduction in the marked (or list) price. 25% discount means a reduction of 25% in the marked price of an article. For instance, if the marked price of an article is Rs 100, it is sold for Rs 75, i.e., Rs 25 less than the marked price. Note. Discount is always calculated on Marked Price. Marked Price (or list price) : The marked price (M.P) of an article is the price at which it is listed for sale. Discount : The discount is the reduction from the marked price of the article. Net Selling price : In case of discount sale, the price of the article obtained by subtracting discount from the list price is called the Net selling Price. Let us consider the following example to illustrate. Example 10.21 : A shirt with marked price Rs 165 is sold at a discount of 10%. Find its net selling price. Solution : Here, Marked Price (M.P.) of the shirt = Rs 165, Discount = 10%

1 th of the principal at the rate of 10% per 4

Net selling price = Marked price Discount

244

Mathematics

= Rs 165 10% of Rs 165 = Rs 165 Rs 10 165I FH 100 K

= Rs 165 Rs 16.50 = Rs 148.50 Thus, the net selling price of the shirt is Rs 148.50. Aliter Since the discount offered is 10% S.P. = M.P. 10% of M.P. = 90% of M.P. or S.P. = 90 Rs 165 100

= Rs 148.50 Example 10.22 : A pair of socks is marked at Rs 40 and is being offered at Rs 32. Find the discount per cent being offered. Solution : Here, M.P. = Rs 40; and S.P. = Rs 32 So, Discount = M.P. S.P. = Rs 40 Rs 32 = Rs 8 Discount % = = Discount 100 M. P. 8 100 40

= 20 Hence, the discount being offered is 20%. 10.9.1 Discount Series Sometimes a manufacturer, offers a discount of 10% on discounted price in addition to a previous discount of 20%, because suddenly he gets a supply of cloth at a very low price. He may allow another discount of 5% on the discounted price, to some of his customers for prompt payments. In other words, he allows a discount series. In a discount series, the first figure denotes the discount on the list price, the second denotes the discount on the discounted price and so on. If a shirt is marked for Rs 120 and a discount series 20%, 10% and 5% is offered, then computation for calculating net selling price is as under :

Percentage and its Applications

245

Marked price Rs 120 with a discount series 20%, 10% and 5%. 20 20% discount on Rs 120 = Rs 120 = Rs 24 100 Discounted price = Rs (120 24) = Rs 96

10 = Rs 9.60 10% discount on Rs 96 = Rs 96 100 Discounted price = Rs (96 9.60) = Rs 86.40 5 = Rs 4.32 100

5% discount on Rs 86.40 = Rs 86.40

Net selling price = Rs (86.40 4.32) = Rs 82.08. 10.9.2 Conversion of Discount Series to a Single Discount Instead of computing a series of discounts one by one, it is sometimes more convenient to reduce the series to a single discount. Let us take some examples to illustrate : Example 10.23 : Convert the discount series 20%, 10% and 5% to an equivalent single discount. Solution : Let the list price 20% discount on Rs 100 Discounted price 10% discount on Rs 80 Discounted price = Rs 100 = Rs 100 20 = Rs 20 100 = Rs (100 20) = Rs 80 10 = Rs 80 = Rs 8 100 = Rs (80 8) = Rs 72

5 = Rs 3.60 and 5% discount on Rs 72 = Rs 72 100 Discounted price = Rs (72 3.60) = Rs 68.40 Single discount on Rs 100 = Rs (100 68.40) = 31.60 or 31.6%

Example 10.24 : An old scooter is sold at three successive discounts of 10%, 5% and 2%. If the marked price of the scooter is Rs 18000, find the selling price of the scooter. Solution : Here, list price = Rs 18000 First discount of 10% = Rs 18000 10 = Rs 1800 100

Price after first discount = Rs (18000 1800) = Rs 16200

246

Mathematics

Second discount of 5% = Rs 16200 Price after second discount

5 = Rs 810 100

= Rs (16200 810) = Rs 15390 Third discount of 2% = Rs 15390 2 = Rs 307.80 100

Price after third discount = Rs (15390 307.80) = Rs 15082.20 Thus the net selling price of the scooter is Rs 15080.20 Aliter Net selling price of the scooter = (100 2)% of (100 5)% of (100 10)% of Rs 18000 98 95 90 = Rs 18000 100 100 100 = Rs 15082.20 Example 10.25 : Find the single discount equivalent to the discount series of 20%, 15% and 10%. Solution : Let the marked price = Rs 100 Price after the given discount series = (100 10)% of (100 15)% of (100 20)% of Rs 100 = 0.90 0.85 0.80 Rs 100 = Rs 51.20 Hence, the total discount = M.P. S.P. = Rs 100 Rs 51.20 = Rs 48.80 Hence, the equivalent single discount = Rs 48.80 on M.P. of Rs 100 = 48.8% Example 10.26 : A dealer buys a table listed at Rs 1500 and gets successive discounts of 20% and 10%. He spends Rs 20 on transportation and sells it at a profit of 10%. Find the selling price of the table. Solution : Here, list price of the table = Rs 1500 Price after a discount series of 20% and 10% = (100 10)% of (100 20)% of Rs 1500 = 90 80 Rs 1500 100 100

FH

IK

= Rs 1080

Percentage and its Applications

247

Since the dealer spends Rs 20 on transportation C.P. of the table = Rs 1080 + Rs 20 = Rs 1100 Profit = 10%

S.P. of the table = = =

Profit % I C. P. FH 100 +100 K 100 + 10 Rs 1100 100

110 Rs 1100 100 = Rs 1210 Thus, the selling price of the table is Rs 1210. CHECK YOUR PROGRESS 10.5 1. A coat is marked at Rs 1200. Find its selling price if a discount of 15% is offered. 2. A man pays Rs 2100 for a machine listed at Rs 2800. Find the rate of discount offered. 3. An article listed at Rs 2650 is sold at a discount of 10%. Due to festival season, the shopkeeper allows a further discount of 5%. Find the selling price of article. 4. Find a single discount equivalent to a discount series given in each of the following discount series : (a) 25%, 20% and 10% (b) 20%, 15% and 10% (c) 20%, 10% and 5%. 5. Which of the following discount series is better for a customer ? 20%, 10% and 5% OR 10%, 5% and 20%.

6. The list price of a table fan is Rs 840 and it is available to a retailer at 25% discount. For how much should the retailer sell it to earn a profit of 15% ? 7. The marked price of a TV set is Rs 25000. A discount series of 20%, 10%, 5% is allowed on it. How much money does one have to pay for the TV set ? 8. If a shopkeeper marks his goods 50% more than their cost price and allows a discount of 40%, find his gain or loss per cent. 9. The list price of a watch is Rs 320. After two successive discounts it is sold for Rs 244.80. If the first discount is 10%, what is the rate of second discount ? 10. A retailer buys shirts from a manufacturer at the rate of Rs 75 per shirt and marked them at Rs 100 each. He allows some discount and gets a profit of 30% on the cost price. What percentage discount does he allow to his customers ?

248

Mathematics

10.9.3 Sales Tax Government levies some taxes to have earning called revenue. One such tax which is levied on the sale of goods is called sales tax. The rates of sales tax are different for different commodities. Some essential commodities are exempted from sales tax. This tax is charged on the net selling price of commodities and its rate is expressed as a percentage. For example, if an article is sold for Rs 750 and the rate of sales tax is 8%, then Sales tax = Rs 750 8 = Rs 60 100 Price inclusive of sales tax = Rs (750 + 60) = Rs 810 The customer will have to pay Rs 810. Example 10.27 : The marked price of a pair of shoes is Rs 320. If the rate of sales tax is 4%, calculate the amount to be paid by a customer for the purchase of the shoes. Solution : Marked price of shoes = Rs 320 Rate of sales tax = 4% Sales tax = 4% of Rs 320 = 4 Rs 320 100

= Rs 12.80 Thus, the customer has to pay (Rs 320 + Rs 12.80) = Rs 332.80 for purchasing the shoes. Examples 10.28 : Anita purchased a shirt for Rs 594 including sales tax. If the rate of sales tax is 8%, find the list price of the shirt. Solution : Let the list price of the shirt be Rs P Then P + 8% of P = 594 or or or 108% of P = 594

108 P = 594 100


P= 594 = 550 108 .

Thus, the list price of the shirt is Rs 550. Example 10.29 : Hari Om bought a radio set for Rs 1870, after getting 15% discount on the list price and then 10% sales tax on the reduced price. Find the list price of the radio set. Solution : Let the list price or the radio set be Rs P. Thus, selling price of the radio after discount= Rs P 15% of Rs P = 85% of Rs P

Percentage and its Applications

249

= Rs

85 P 100
85 I FH 100 K P

Sales tax = 10% of Rs =

10 85 P 100 100 85 P 1000

= Rs

The net price to be paid for the purchase of the radio set is Rs 85 P + 85 PI FH 100 935 1000 K = Rs 1000 P

Equating it to Rs 1870, we get 935 P = 1870 1000 or P= 1870 1000 = 2000 935

Thus, the list price of the radio set is Rs 2000. Example 10.30 : The list price of a washing machine is Rs 9000. The dealer allows a discount of 5% on the cash payment. How much money will a customer pay to the dealer in cash, if the rate of sales tax is 10% ? Solution : Here, list price = Rs 9000 and discount = 5%
Cash price of the washing machine

= Rs 9000 5 9000 100 = Rs (9000 450) = Rs 8550

FH

IK

Sales tax = 10% of Rs 8550 = 10 Rs 8550 100

= Rs 855 Hence, the customer has to pay Rs 8550 + Rs 855 = Rs 9405 for the purchase of the washing machine. Example 10.31 : The list price of the air-conditioner is Rs 25630. The rate of sales tax is 10% The customer requests the dealer to allow a discount to such an extent that the price of the

250

Mathematics

air-conditioner amounts to Rs 25630 inclusive of sales tax. Find the discount in the price of the air-conditioner. Solution : Let Rs P be the net price exclusive of sales tax. 10 P 110 P = Then, price + sales tax = P + 100 100 This is given as Rs 25630

or

110 P = 25630 100 P = 23300 Discount allowed = Rs (25630 23300) = Rs 2330 CHECK YOUR PROGRESS 10.6

1. The marked price of a sewing machine is Rs 3500. If the sales tax on sewing machine is charged at the rate of 6%, find how much a customer has to pay for purchasing the machine. 2. Amita purchases a pair of socks whose list price is Rs 44. The shopkeeper charges sales tax at the rate of 5%. Find how much money Amita has to pay for purchasing the socks. 3. Mrs. Mohini purchased a saree for Rs 1100 including sales tax. If the list price of the saree is Rs 1000, find the rate of sales tax charged. 4. A refrigerator is available for Rs 13915 including sales tax. If the rate of sales tax is 10%, find the selling price of the refrigerator. 5. Radhika purchased a car with a marked price of Rs 2.1 lakhs at a discount of 5%. If the sales tax is charged at the rate of 12%, find the amount Radhika had to pay for purchasing the car. 6. Dayakant bought a set of cosmetic items for Rs 345 including 15% sales tax and a purse for Rs 110 including 10% sales tax. What percent is the sales tax charged on the whole transaction ? [Hint. C.P. of cosmetic items = Rs (345 1.15) ; and C.P. of purse = Rs (110 1.1)] 7. Kamal wants to buy a suitcase whose list price is Rs 504. The rate of sales tax is 5%. He requests the shopkeeper to reduce the list price to such an extent that he has to pay Rs 504 only. Calculate the discount given in the price of the suit-case. 10.10 COMMISSION Manufacturers of goods, farmers and owners of properties, frequently uses the services of a middle man to find a buyer in order to sell their goods or properties. The middle man is called an agent, who gets some money for the services rendered by him. This money paid is called commission. In general, commission is expressed in terms of percentage.

Percentage and its Applications

251

Example 10.32 : A book agent sold 140 books at Rs 20 each. His commission was 25%. How much money did he earn as commission ? Solution : Total price of books = Rs (20 140) = Rs 2800 Amount of commission = 25% of Rs 2800 = 25 Rs 2800 100

= Rs 700 Thus, the book agent earns Rs 700 as commission. Example 10.33 : A salesman earns Rs 300 as commission at the rate of 8%. For what amount did he sell the goods ? Solution : Let the amount be Rs x Then x 8 = 300 100 x= 300 100 8

= 3750 The sales man sold goods worth Rs 3750. Example 10.34 : A commission merchant charged Rs 427.50 for selling 1500 packets of salt, at 3% commission. At what price per bag did he sell the salt ? Solution : Let the selling price per packet be Rs x. Then amount of commission = 3% of Rs (1500 x) 3 = Rs 1500x 100 But this is given to be Rs 427.50. Thus, 45x = 427.50 or x= 427.50 45

FH

IK

= Rs 45x

or x = 9.5 Thus, the selling price per packet of salt was Rs 9.50. CHECK YOUR PROGRESS 10.7 1 1. A commission merchant sells 1200 tins of oil at Rs 270 a tin on commission of 2 %. 2 Find the amount of the commission. Also find the net proceeds.

252

Mathematics

2. An auctioneer sold a property worth Rs 17.8 lakhs. His commission was 1.25%. How much did the auctioneer earn as commission ? 3. A commission merchant charged Rs 4212 as commission at rate of 3% for selling rice at Rs 450 per bag. How many bags of rice did he sell ? 4. A commission merchant sells a certain amount of goods at a commission of 10% in the first two weeks of a month. His commission is then raised to 12%, and he sells an equal amount of goods in the remaining part of the month. If his total earning (commission) were Rs 8000, how much did he sell altogether ? 10.11 INSTALMENT BUYING With the cost of articles going up day by day it has become difficult for the common man to buy some articles like scooter, fridge, colour TV etc. which are needed by him and his family. Such articles are available on easy instalments. We shall study about instalment purchase scheme. 10.11.1 What is Instalment Buying ? Instalment purchase scheme, enables a person to buy costly goods like colour TV sets, refrigerator, video cameras, scooters etc. on convenient terms of payment. In this scheme, the customer does not make full payment of the cost of the article at the time of purchase, but makes a partial payment in the beginning and takes away the article for use. The remaining payment is made in easy monthly, quarterly or half yearly instalments, as per the agreement signed between the customer and the shopkeeper. Cash Price : It is the amount for which the article can be purchased on full payment i.e., the selling price of the article. Cash down Payment : It is the partial payment made by the customer at the time of signing the agreement and taking away the article for use. In fact, it is a part of the selling price. Instalments : It is the amount which is paid by the customer at regular intervals towards the remaining part of the selling price of the article. It may be noted that in the instalment plan only part payment of the total cost is paid by the customer at the time of purchase. The remaining part of the cost is paid on subsequent dates; and therefore the seller charges some extra amount for deferred payments. This extra amount is actually the interest charged on the amount of money which the customer owes to the seller at different times of instalments paid. In the following, we solve a few examples to illustrate the process. Example 10.35 : Bimla buys a sewing machine, which is available for Rs 2600 cash payment or under an instalment plan for Rs 1000 cash down payment and 3 monthly instalment of Rs 550 each. Find the rate of interest charged under the instalment plan. Solution : Cash payment price = Rs 2600 Cash down payment = Rs 1000

Percentage and its Applications

253

Balance to be paid in instalments = Rs 1600 Amount paid in 3 instalments = Rs 550 3 = Rs 1650 Interest charged in the instalment plan = Rs (1650 1600) = Rs 50 The buyer owes to the seller for the 1st month = Rs 1600 The buyer owes to the seller for the 2nd month = Rs (1600 550) = Rs 1050 The buyer owes to the seller for the 3rd month = Rs (1050 550) = Rs 500 Total = Rs 3150
She has to pay interest on Rs 3150 for 1 month.

But the interest she has to pay = Rs 50 If R% is the rate of interest p.a. then,

3150 R= =

1 R = 50 12 100

50 12 100 % 3150 400 % 21

1 = 19 % 21 1 Rate of interest paid by her in the instalment plan is 19 21 %. Example 10.36 : A computer is available for Rs 34000 cash or Rs 20000 cash down payment together with 5 equal monthly instalments. If the rate of interest charged under the instalment plan is 30% per annum, calculate the amount of each instalment. Solution : Cash price = Rs 34000

Cash down payment = Rs 20000 Balance to paid in 5 equal installments = Rs 14000 Let each instalment be = Rs P Interest charged under instalment plan = Rs (5P 14000) The buyer owes to the seller For the 1st month = Rs 14000 Fr the 2nd month = Rs (14000 P) For the 3rd month = Rs (14000 2P) For the 4th month = Rs (14000 3P) For the 5th month = Rs (14000 4P) Total = Rs (70000 10P)

254

Mathematics

Thus, he had to pay interest on Rs (70000 10P) for 1 month at the rate of 30% p.a.

(70000 10P)

1 30 = 5P 14000 12 100 1 = 5P 14000 40

or or or or or

(70000 10P)

70000 10P = 40(5P 14000) 70000 10P = 200P 560000 210 P = 560000 + 70000 P= 630000 210

= 3000

Amount of each instalment = Rs 3000. CHECK YOUR PROGRESS 10.8

1. A T.V. set is available for 21000 cash or for Rs 4000 cash down payment and 6 equal monthly instalments of Rs 3000 each. Calculate the rate of interest charged under the instalment plan. 2. Anil purchased a type writer priced at Rs 6800 cash payment under the instalment plan by making a cash down payment of Rs 2000 and 5 monthly instalments of Rs 1000 each. Find the rate of interest charged under the instalment plan. 3. A scooter is available for Rs 30000 cash or for Rs 15000 cash down payment and 4 equal 1 monthly instalments. If the rate of interest charged under the instalment plan is 33 %, 3 find the amount of each instalment. 4. A microwave oven is available for Rs 9600 cash or for 4000 cash down payment and 2 3 equal monthly instalment. If the rate of interest charged is 22 % per annum, find the 9 amount of each instalment. LET US SUM UP
z z z z

Percent means per hundred. Percents can be written as fractions as well as decimals and vice-versa. To write a percent as a fraction, we drop the % sign and divide the number by 100. To write a fraction as a percent, we multiply the fraction by 100, simplify it and suffix the % sign.

Percentage and its Applications

255

To determine the specific percent of a number or quantity, we change the percent to a fraction or a decimal and then multiply. When the selling price is more than the cost price of the goods, there is a profit (or gain) When the selling price is less than the cost price of the goods, there is a loss Profit (Gain) = S.P. C.P. Gain% = Further S.P. = Gain 100 C. P. 100 + Gain% C. P. 100 ; ; ; Loss = C.P. S.P. Loss = S.P. = Loss 100 C. P. 100 Loss% C. P. 100

z z

The simple interest (S.I.) on a principal (P) at the rate of R% for a time T years, is calculated, using the formula S.I. = P R T

z z z

Discount is a reduction in the list price of goods. Discount is always calculated on the marked price of the goods. (Marked price discount), gives the price, which a customer has to pay while buying an article. Two or more successive discounts are said to form a discount series. A discount series can be reduced to a single discount. Sales tax is charged on the sale price of goods. Commission is paid to an agent for his services in arranging the sale or purchase of goods from some one else. An Instalment plan enables a person to buy costlier goods. TERMINAL EXERCISE

z z z z

1. Write each of the following as a per cent : (a) 7 20 (b) 0.25 (c) 1.4 (d) 0.07

2. Write each of the following as a decimal : (a) 63% (b) 13% (c) 3% (d) 0.3%

3. Write each of the following as fraction : (a) 0.13% (b) 1.3% (c) 11.3% (d) 113%

256

Mathematics

4. Find each of the following : (a) 37% of 400 5. What percent of 700 is 294 ? 6. By what percent is 60 more than 45 ? 7. Find the number whose 15% is 270 ? 8. What number increased by 10% of itself is 352 ? 9. What number decreased by 7% of itself is 16.74 ? 10. Arun at the beginning of a year had a bank balance of its 17500 and at the end of the year he had a balance of Rs 21350. By what percent did his balance increase ? 11. A man loses 25% by selling a scooter for Rs 8400. For what amount did he buy the scooter ? 12. A commission merchant charged Rs 700 as commission for selling 100 bags of cotton at 5%. Find the price of one bag at which it was sold ? 13. Shalim deposited Rs 14000 in a bank for 2 years and received Rs 4200 as simple interest. At what rate per annum, was interest paid to him ? 14. Simple interest on a sum of money is 1 rd of the sum itself and the number of years is 3 thrice the rate percent. Find the rate of interest. (b) 3.5% of 800

15. Ahmad purchased a bicycle by making a cash down payment of Rs 400 and 3 monthly instalments of Rs 275 each. The bicycle was also available on cash payment of Rs 1200. Find the rate of interest per annum charged under the instalment plan. 16. Rita purchased a washing machine for Rs 4000 cash down payment and 4 equal monthly instalments. The washing machine was also available for Rs 15000 cash payment. If the rate of interest charged in the instalment plan is 18% p.a. Find the amount of each instalment.

Percentage and its Applications

257

ANSWERS Check Your Progress 10.1 1. (a) 56% (e) 97% 2. (a) 0.75 (e) 0.02 3. (a)
35 100 70 100

(b) 3% (f) 80% (b) 0.14 (f) 0.25 ; 35 : 100

(c) 75% (g) 4% (c) 0.03 (g) 4 (b)


40 100 85 100

(d) 2% (h) 140% (d) 1.15 (h) 3.5 ; 40 : 100

(c) 4.

; 70 : 100 (b) 20% 6. 50% 10. 75%` 14. 75%

(d)

; 85 : 100 (d) 16% 8. 84% 12. 25%

(a) 75%

(c) 30% 7. 37.5% 11. 30%

5. 76% 9. 55% 13. 30%

15. 62% ; 62.5 %; Y16. 5%.

Check Your Progress 10.2 1. (a) 66 (b) 200 (c) Rs 564 (d) 663 metres 5. Rs 1540; 9. 10% 13. Rs 3000 17. Rs 14000

2. 240 students 6. 4 boys 10. 40% 14. Rs 90 ; Rs100

3. 9 boys ; 27 girls 4. 45 kg 7. 36 minutes 11. 20% 15. Rs 40; Rs 32 8. 1 lakh 12. 6000 votes 16. 1% decrease

Check Your Progress 10.3 1. Loss 25% 5. 36 ball pens 9. 10% loss Check Your Progress 10.4 1. Rs 8120 5. Rs 960 ; Rs 16960 8. 2 years 6 months 2. Rs 3744 6. 11% 9. (b) 3. Rs 5600 7. 8 years 4 months 10. 10% 4. Rs 5200 2. Profit 25% 6. 15% gain 3. 4% loss 7. Rs 1650 4. Rs 1007 8. Rs 279.50

258

Mathematics

Check Your Progress 10.5 1. Rs 1020 4. (a) 46% 2. 25% (b) 38.8% 6. Rs 724.50 10. 2.5% 3. 2265.75 (c) 31.6% 7. Rs 17,100 8. Loss 10%

5. Both same 9. 15%

Check Your Progress 10.6 1. Rs 3710 5. Rs 2,23,440 Check Your Progress 10.7 1. Rs 8100 Check Your Progress 10.8 1 1. 21 % 19 Terminal Exercise 1. 2. 3. 4. (a) 35% (a) 0.63 (a) 13 10000 (b) 25% (b) 0.13 (b) 13 1000 (c) 140% (c) 0.03 (c) 113 1000 (d) 7% (d) 0.003 (d) 113 100 1 2. 17 % 7 3. Rs 4000 4. Rs 2000 2. Rs 22250 3. 312 bags 4. Rs 80,000 2. 46.20 3 6. 13 % 4 3. 10% 7. Rs 24 4. Rs 12650

(a) 148

(b) 28 8. 320 12. Rs 140 1 14. 3 % 3

5. 42% 9. 18

1 6. 33 % 3 10. 22%

7. 1800 11. Rs 11200 13. 15% 16. Rs 2850.86

1 15. 19 % 21

Compound Interest

259

11
Compound Interest
11.1 INTRODUCTION You have already studied about simple interest in your earlier classes. Recall that interest is the extra money paid by the user to the bank or money lender for the use of his money for some specified time period. If the money is retained by the user for the next time period also, the user will pay interest on the amount borrowed as well as on the interest accrued for the first time period. Thus, the user will have to pay more interest for the second period of time. This type of interest is called compound interest. In this lesson, you will learn to calculate compound interest. 11.2 OBJECTIVES After studying this lesson, the learner will be able to :
z z z z z

illustrate the concept of compound interest. calculate compound interest. find the difference between simple interest and compound interest. calculate amount of a sum invested for a given time. forecast the population of a town when rate of growth of population is given for a specific period of time. calculate the depreciated value of a machine, a vehicle, a building etc.

11.3 EXPECTED BACKGROUND KNOWLEDGE


z z

Knowledge of percentage Knowledge of simple interest.

11.4 THE CONCEPT OF COMPOUND INTEREST Recall that interest is the extra money paid by the user to the bank for the use of money. The money which has been borrowed is called the principal (P), the time interval for which the

260

Mathematics

money has been taken is called the time duration (T) and the extra money paid by the user to the bank upto the time the principal amount is returned is called the rate of interest (R). This is expressed as percentage of the principal. When the interest is calculated on the principal for the entire time period of loan, the interest is called simple interest and is given by S.I = PRT But if this interest is due (not paid) after the decided time period, then it becomes a part of the principal and added to the principal for the next time period. The interest is calculated for the next time period on the new principal. This way of calculating interest is called compound interest. The time period after which the interest is added to the principal for the next time period is called the conversion period. The conversion period may be one year, six month, three months or a month and the interest is said to be compounded annually, half yearly, quarterly or monthly respectively. Let us go through the following example : Suppose you borrow Rs 2000 from a bank at the rate of interest 10% per annum. 10 Interest for one year = Rs 2000 100 1 = Rs 200 If this interest is not paid at the end of one year, then it becomes a part of principal for the second year. Therefore, the principal for the second year becomes Rs (2000 + 200) = Rs 2200 Now interest for second year 10 = Rs 2200 100 1 = Rs 220
Amount payable at the end of two years

FH

IK

FH

IK

= Rs (2200 + 220) = Rs 2420. The total interest to be paid at the end of two years = Rs (200 + 220) = Rs 420. The interest charged in this way is called the compound interest. Thus for calculating the compound interest, the interest due after every year is added to the principal and then the interest is calculated for the next year on this new principal.

Compound Interest

261

11.5 FORMULA FOR COMPOUND INTEREST Let a sum P be borrowed for n years at the rate of r% per annum. We shall calculate the compound interest payable after n years. Here Principal = P Rate = r% Time = n years Pr r 1 Interest on P at r% for the first year = P = 100 100 Pr r Amount after one year = P + 100 = P 1 + 100 r Interest on P 1 + 100

FH

IK FH IK

FH

IK

r r at r% for the second year = P 1 + 100 100

r r r Amount after two years = P 1 + 100 + P 1 + 100 100

IK FH IK F r IF r I = P H 1 + 100 K H 1 + 100 K
r = P 1+ 100

FH

FH

IK

(In compound interest, amount after one year becomes principal for second year) r Similarly, amount after 3 years = P 1 + 100 r Thus, amount after n years = P 1 + 100

FH

IK

and so on.

FH

IK

If A represents amount and R represents A = P (1 + R)n Also Compound Interest = A P

r , then 100

= P(1 + R)n P = P[(1 + R)n 1] Note : (i) Simple interest and compound interest are equal for the first year. (ii) The time period is generally taken in years but this is not necessary. The interest can be compounded yearly, half-yearly (semi-annually), quarterly (after 3 months) or even monthly. The unit of time after which interest is compounded is called Conversion period.

262

Mathematics

Example 11.1 : Calculate the compound interest on Rs 10000 for 2 years at 8% per annum. Solution : Here P = Rs 10000, R = 8% and n=2 C.I. = P[(1 + R)n 1]

LMF I 1OP NH K Q 27 27 O = Rs 10000 L NM 25 25 1QP


= Rs 10000 1 + 8 100
2

= Rs 1664. Example 11.2 : Calculate the compound interest on Rs 10000 for 2 years at the rate of 8% per annum when the interest is compounded half yearly. Solution : Here P = Rs. 10000, R =

8 or 4% Half yearly and n = 4 Half years 2

A = P (1 + R)n 4 = 10000 1 + 100

FH

IK

= Rs 10000 26 26 26 26 25 25 25 25 = Rs 11698.58 C.I. = Rs (11698.58 10000) = Rs 1698.58

Example 11.3 : Calculate the compound interest on Rs 10000 for 1 year at the rate of 8% per annum when the interest is compounded quarterly. Solution : Here P = Rs 10000, R =

8 or 2% quarterly and n = 4 quarters 4

A = P (1 + R)n = Rs 10000 1 + 2 100

FH

IK

51 51 51 51 = Rs 10000 50 50 50 50 = Rs 10824.32

C.I. = Rs (10824.32 10000) = Rs 824.32

Example 11.4 : Calculate the compound interest on Rs 12000 for 1 years at the rate of 10% per annum when the interest is compounded annually. Solution : Here P = Rs 12000, R = 10% and n = 1 years

Compound Interest

263

We note that the interest is compounded annually and therefore at the end of one year, the amount will be given by A = P (1 + R)1 = Rs 12000 1 + 10 100

FH

IK

This A becomes the principal for the next 6 months or half year. The rate per half year will be R 10 = % = 5% 2 2
The amount say A', after 1 year will be given by

A' = A 1 + 5 100

IK 10 I F 1 + 5 I = Rs 12000 F H1 + 100 K H 100K


= Rs 12000 11 21 = Rs 13860 10 20

FH

C.I. = Rs (13860 12000) = Rs 1860

Example 11.5 : Find the sum of money which will amount to Rs 26010 in six months at the rate of 8% per annum, when the interest is compounded quarterly. Solution : Here P = ?, A = Rs 26010, R = n = 2 (two quarters)

FG 8 IJ % H 4K

or 2% quarterly and

2 26010 = P 1 + 100
2

FH IK 51I = PF H 50 K

P = Rs

26010 50 50 = Rs 25000 51 51

Hence the required sum = Rs 25000 Example 11.6 : The difference between simple interest and compound interest for a certain sum of money at the rate of 10% per annum for 1 years when the interest is compounded semi-annually, is Rs 183. Find the sum. Solution : Let the sum be Rs 100

S.I. = Rs

100 10 3 = Rs 15 100 2

264

Mathematics

C.I. = Rs 100 1 + 5 100 = Rs

R FH S T

IK 1U V W
3

1261 80

Difference = C.I. S.I. = Rs

FH 1261 15I K 80
100 80 183 61

= Rs

61 80

If the difference is Rs

61 , then sum = Rs 100 80

If the difference is Rs 183, then sum = Rs

= Rs 24000 Hence the required sum = Rs 24000 Example 11.7 : A sum of money becomes Rs 17640 in two years and Rs 18522 in 3 years at the same rate of interest when the interest is compounded annually. Find the sum and the rate of interest per annum. Solution : In compound interest, we know that A = P (1 + R)n Here and 17640 = P (1 + R)2 18522 = P(1 + R)3 ...(i) ... (ii)

Dividing (ii) by (i), we have 1 + R=

18522 17640 18522 1 18522 17640 = 17640 17640 882 = 1 5 or 17640 20 100

R= =

Rate = 5%

Substituting the value of R in (i), we have 5 17640 = P 1 + 100

FH

IK

P = Rs 17640 20 20 21 21 = Rs 16000

Hence the required sum is Rs 16000 and the rate of interest per annum is 5%.

Compound Interest

265

ALITER Interest for 3rd year = Rs (18522 17640) = Rs 882. Principal at the end of 2nd year = Rs 17640 If R is the rate % then R= 882 1 5 = or i.e. 5% 17640 1 20 100

Amount after 2 years is given by 5 17640 = P 1 + 100

FH

IK

where P is the principal

20 20 P = Rs 17640 21 21 = Rs 16000.

Example 11.8 : A sum of money is invested at compound interest for a year at 8% per annum when the interest is compounded half yearly. If the interest were compounded quarterly, it would have fetched Rs 5201 more than in the pervious case. Find the sum. Solution : Let the sum be Rs x Here, R = 8% or 4% half yearly and n = 2 half years 2

Compound interest in Ist case

LF 4I = Rs Mx H 1 + 100 K N
= Rs 51 x 625

OP Q

Compound interest in 2nd case

LF 2I O = Rs Mx H 1 + 100 K xP N Q 51 51 51 51 O = Rs x L NM50 50 50 50 1QP


4

= Rs
Difference = Rs

515201 x 6250000 515201 x 51 xO LM 6250000 625 P N Q

266

Mathematics

= Rs If the difference is Rs

5201 x 6250000

5201 x , the sum = Rs x 6250000


x 6250000 5201 5201 x

If the difference is Rs 5201, the sum = Rs

= Rs 6250000 Hence the required sum is Rs 6250000 Example 11.9 : A sum of Rs 15625 amounts to Rs 17576 at 8% p.a. compounded semi-annually. Find the time. Solution : Here P = Rs 15625, A = Rs 17576, R = Let the conversion periods be n half years Now,

8 % or 4% semi annually 2

A = P (1 + R)n 4 17576 = 15625 1 + 100

FH

IK

4 I FH1 + 100 K = 17576 15625 26 I FH 26 I = F K H 25 25 K


n n 3

n=3

1 Hence time = 3 Half years = 1 years. 2 Example 11.10 : Find the rate at which Rs 8000 amounts to Rs 9261 in 3 years, if the interest is compounded annually. Solution : Here P = Rs 8000, A = Rs 9261, n = 3 Let the rate of interest be R% Now A = P (1 + R)n 9261 = 8000 ( 1 + R)3 i.e. (1 + R)3 = =

21I FH 20 K

9261 8000
3

21 20 1 = 5 i.e. R= = 5% 20 100 Rate of interest is 5%. 1 + R=

Compound Interest

267

Example 11.11 : Find the rate at which Rs 4000 will give Rs 630.50 as compound interest in 9 months, interest being compounded quarterly. Solution : Here P = Rs 4000, A = Rs (4000 + 630.50) = Rs 4630.50 and n = 3 quarters Let the rate of interest = R Now

A = P(1 + R)n 4630.50 = 4000 (1 + R)3 (1 + R)3 = 4630.50 4000

i.e.

9261 = 21 = 8000 20

FH IK

1+R= R=

21 20 1 = 5 = 5% per quarter. 20 100

Rate of interest = 20% p.a.

CHECK YOUR PROGRESS 11.1 1. Calculate the compound interest on Rs 8000 for 3 years at 5% per annum when the interest is compounded annually. 2. Calculate the compound interest on Rs 62500 for 1 year at 8% per annum when the interest is compounded quarterly. 3. Calculate the compound interest on Rs 390625 for 2 years at 8% per annum when the interest is compounded half yearly. 4. Find the sum of money which will amount to Rs 27783 in 3 years at 5% per annum, the interest being compounded annually. 5. How much money will become Rs 194481 after 2 years at 10% per annum when the interest is compounded semi-annually ? 6. Find the difference between simple interest and compound interest for 2 years at 10% per annum, when the interest is compounded semi-annually, on Rs 8000. 7. The difference between simple interest and compound interest for a certain sum of money at 8% per annum for 1 years when the interest is compounded half-yearly is Rs 228. Find the sum.

268

Mathematics

8. A sum of money becomes Rs 18522 in three years and Rs 19448.10 in 4 years at the same rate of interest when the interest is compounded annually. Find the sum and the rate of interest per annum. 9. A sum of money is invested at compound interest for 2 years at 10% per annum when the interest is compounded yearly. If the interest were compounded half yearly, it would have earned Rs 881 more than in the previous case. Find the sum. 10. At what rate of interest per annum would the compound interest on Rs 12500 be Rs 9100 in 1 years, interest being compounded half-yearly ? 11.5 RATE OF GROWTH AND DEPRECIATION In our day to day life, we observe about growth of population, plants, viruses etc. and depreciation of articles like machinery, cars etc. after use. If the rate of growth/depreciation for period is denoted by r%, V0 is the value of article in the beginning and Vn is the value after n conversion periods, then Vn is given by : r Vn = V0 1 + 100 r Vn = V0 1 100

FH FH

IK IK

in case of growth and


n

in case of depreciation.

If the rate of growth/depreciation varies for each conversion period, then Vn is given by Vn = V0 1 +

Vn =

FG r IJ FG1 + r IJ FG1 + r IJ ..... (For growth) H 100K H 100K H 100K F r IJ FG1 r IJ FG1 r IJ ..... (For depreciation) V G1 H 100K H 100K H 100K
1 2 3 0 1 2 3

We solve some examples to illustrate the above concepts. Examples 11.12 : The population of a town is 281250. What will be its population after 3 years if the rate of growth of population is 4% per year ? Solution : Here V0 = 281250, r = 4% and n = 3

Vn = 281250 1 + 4 100

FH

IK

26 26 26 = 281250 25 25 25 = 316368 Hence the population of the town after 3 years = 316368.

Compound Interest

269

Example 11.13 : The cost of a car was Rs 35000 in January 2000. Its value depreciates at the rate of 10% each year. Find the value of the car in January 2003. Solution : Here V0 = Rs 350000, Vn= ? r = 10%, n = 3

10 Vn = Rs 350000 1 100

FH

IK

9 9 9 = Rs 350000 10 10 10 = Rs 255150 In January 2003, The cost of car will be Rs 255150 Example 11.14 : The cost of machinery is Rs 1360000 today. In the first year, the cost depreciates by 15% and subsequently, the price depreciates by 10% each year. By how much, the machinery has depreciated after 4 years. Solution : Here V0 = Rs 1360,000, n = 4, rate = 15%, first year & then 10% for next years.

Vt = Rs1360000 1 15 1 10 100 100 = Rs1360000 17 9 9 9 20 10 10 10 = Rs 842724

FH

IK FH

IK

Depreciation = Rs(1360000 842724) = Rs 517276

Example 11.15 : The application of manure increases the output of a crop by 10% in the first year, 5% in the second year and 4% in the third year. If the production of a crop in the year 2000 was 3.5 tons per hectare, find the production of crop per hectare in 2003. r Solution : We know that Vt = V0 1 + 100

FH

IK

Vt = 3.5 1 + 10 100

FH

5 I F1 + 4 I IK FH1 + 100 K H 100K

tons

11 21 26 = 3.5 tons 10 20 25 = 4.2042 tons So the production of crop in the year 2003 = 4.2042 tons/hectare.

270

Mathematics

Example 11.16 : The virus of a culture decreases at the rate of 4% per hour due to a medicine. If the virus count in the culture at 9.00 A.M. was 3.5 108, find the virus count at 11.00 A.M. on the same day. Solution : Here V0 = 3.5 108, R = 4% and n = 2

4 Vn = 3.5 108 1 100 = 3.2256 108

FH

IK

Hence virus count at 11.00 A.M. is 3.2256 108. CHECK YOUR PROGRESS 11.2 1. The population of a city is 2.7 108. If the rate of growth of population is 5% of what it was in the beginning of the year, find its population after 3 years. 2. The population of a town is 50000. In the first year, the rate of growth of the population was 5%. In the second year, because of some epidemics, the population decreased by 10%. In the third year, the population growth rate was noticed as 4%. Find the population of the town after 3 years. 3. The cost of a car was Rs 215000 in January 2001. If the rate of depreciation is 15% for the first year and 10% for the subsequent years, find its cost after 3 years. 4. A tree gains its height at the rate of 2% of what it was in the beginning of the month. It its height was 1.5 m in the beginning of January 2002, find its height at the end of the April of the same year (Give your answer correct to 2 decimal places). 5. The value of property increases at the rate of 20% of what it was in the beginning of the year. In how many years, the value of property will be almost double ? LET US SUM UP
z

If Principal = P, Rate = r%, Time = n conversion periods then Amount = Principal + Compound Interest = P (1 + R)n C.I. = P [(1 + R)n 1] Compound Interest is greater than simple interest except for the first conversion period. If V0 is the value of article in the beginning, Vn is the value after n conversion periods and r be the rate of growth/depreciation per period, then r Vn = V0 1 + 100 Vn
0

z z

FH IK r I = VF H1 100 K

in case of growth and


n

in case of depreciation.

Compound Interest

271

If the rate of growth/depreciation varies for each conversion period, then


r1 Vn = V0 1 + 100

and

Vn

FG IJ FG1 + r IJ FG1 + r IJ ... for growth H K H 100K H 100K F1 r IJ FG1 r IJ FG1 r IJ ... for depreciation. = V G H 100K H 100K H 100K
2 3 0 1 2 3

TERMINAL EXERCISE 1. Find the rate of interest per annum if Rs 31250 amounts to Rs 35152 in 1 years, interest being compounded half yearly. 2. A sum of Rs 1000 amounts to Rs 1331 after some time. If the rate of interest is 10% per annum compounded annually, find the time period. 3. Find the sum of money which will amount to Rs 26460 in six months at 20% per annum, when the interest is compounded quarterly. 4. Find the difference between simple and compound interests for 2 years at 10% per annum, when the interest is being compounded annually on Rs 2500. 5. The population of a town increases at the rate of 5% per annum, what will be the population of the town after two years, if the present population is 16000 ? 6. The price of a scooter depreciates at the rate of 20% per annum of its value at the beginning of each year. If the cost of scooter is 15625, what will be its value at the end of three years ? 7. The population of a town is 20000. It increases by 10% during first year. During second year, the population decreases by 10%. Find the population at the end of 2 years.

272

Mathematics

ANSWERS Check Your Progress 11.1 1. 1261 5. Rs 160000 9. Rs 1,60,000 2. Rs 5152.01 6. Rs 124.05 10. 40% p.a. 3. Rs 66351 7. Rs 46875 4. Rs 24000 8. Rs 16,000, 5%

Check Your Progress 11.2 1. 3.1256 108 5. 4 years Terminal Exercises 1. 8% 5. 17640 2. 3 years 6. Rs 8000 3. 24000 7. 19800 4. Rs 25 2. 49140 3. Rs 148027.50 4. 1.62 m

Banking

273

12
Banking
12.1 INTRODUCTION All of us need money to meet our day-to-day expenses. We earn money, spend money and also, save some money. We would like to keep the money in safe custody. Banks are the institutions where we keep the money in the form of deposits and those who need money can borrow as loans on payment of interest with certain conditions. Besides this, banks also help the people in various kinds of financial transactions. Some of the functions of a bank are : 1. Keeping money of the depositors and pay interest on deposits. 2. Giving loans to the borrowers on interest. 3. Buying and selling security bonds. 4. Receiving payments Telephone bills, Electricity bills, water bills, school fees etc. 5. Transferring money from one place to another. 6. Collection of taxes income tax, sales tax, house tax etc. 7. Issuing/Encashing travellers cheques in Local/Foreign currency. 8. Providing All Time Money (ATM) facility to the customers. 9. Issuing credit cards. 10. Exchange of Foreign currency etc. For the convenience of customers, the banks offer different types of accounts (deposits) some of which are : (i) Savings Bank Account (ii) Current Account (iii) Fixed deposit Account. (iv) Recurring deposit Account.

274

Mathematics

12.2 OBJECTIVES After studying this lesson, the learner will be able to :
z z z

state different types of accounts. calculate interest for a saving bank account, given the rate of interest. calculate interest/maturity value of fixed/term deposit given the rate of interest compounded yearly, semi-annually or quarterly (not more than four conversion periods).

12.3 EXPECTED BACKGROUND KNOWLEDGE (i) Knowledge of simple interest when principal (P), rate of interest (R) and time (T), for which money was invested are given i.e. S.I. = PRT. (ii) Compound interest when P, R, n are given i.e., C.I. = A P where A = P(1 + R)n, n is number of conversion periods. 12.4 TYPES OF ACCOUNTS You can open different types of accounts in a bank depending upon your needs. Some of these are : (a) Savings bank account (b) Fixed or term deposit account (c) Current account 12.4.1 Savings Bank Account This is the most popular account offered by the banks. This account encourages people to develop the habit of saving. Anybody can open this account with a minimum sum of Rs 1000 with cheque book facility. In this account, deposits and withdrawals can be made and the record is maintained by the bank in a Pass-Book given to the Account holder (i.e. the person who holds the account). The bank pays interest for the money that an account holder keeps in the account. The prevailing rate of interest is 4% per annum compounded half yearly. The rate of interest changes from time to time. On opening a savings bank account, a pass book is issued to the account holder. It contains date wise entries of deposits withdrawals and the interest earned. It bears savings bank account number also. The general format of a page of the savings bank passbook is as given below :

Banking

275

Date

Particulars

Amount withdrawn Rs P

Amount Deposited Rs P

Balance

Money can be withdrawn from the bank through a withdrawal slip (sample shown) or by a cheque (sample shown below).

Fi.g 12.1 Sample of Withdrawal slip

Fig. 12.2 Sample of Cheque

Note : In a savings bank account, (i) The bank pays interest for the month on the minimum closing balance from the 10th day of the month to the last day of the month. (ii) The interest is credited to the account every six months. (iii) The present rate of interest is 4% per annum compounded semi-annually. (iv) Usually not more than 25 withdrawals are allowed in a quarter from Savings Bank account.

276

Mathematics

12.4.2 Computation of Interest (i) Write down the minimum balance between the closing balance on 10th to the last day of the month. (ii) Add all the minimum balances for each month as per step (i) to obtain principal for one month. (iii) Calculate the simple interest on this sum for one month using the formula Interest = P R
1 12

where P is Principal, R is rate of interest per annum and time 1 month i.e. 1/12 year. (iv) If account is opened after the 10th day of month, no interest is payable for that month. (v) No interest is paid for the month in which the account is closed. Let us illustrate the above with some examples. Example 12.1 : Sidharth opened a savings bank account in State Bank of India on 5th March 2003 with a deposit of Rs 5000. He deposited Rs 1500 on 10th March 2003 and withdrew Rs 3000 on 29th March, 2003. Find the principal for which he will earn interest for March 2003. Solution : Balance on 5th March, 2003 = Rs 5000 Balance on 10th March, 2003 = Rs 6500 Balance on 29th March, 2003 = Rs 3500 Here minimum balance between 10th and 31st March, 2003 = Rs 3500. Therefore, the principal for which interest is earned for the month of March 2003 is Rs 3500. Example 12.2 : Anishas savings bank account pass-book has the following entries : Date 2002 Jan 6 Feb 11 April 5 April 27 May 3 June 15 By Cash By Cash By Cheque To Cheque By Cash To Cheque 18000.00 20000.00 20000.00 10000.00 5000.00 13000.00 20000.00 30000.00 35000.00 17000.00 30000.00 10000.00 Particulars Amount withdrawn Amount Deposited Balance Rs P Rs P Rs P

Find the sum on which Anisha will earn interest from January to June 2002.

Banking

277

Solution : The qualifying amount for the interest is the minimum balance between the 10th and the last day of the month. Principal for January = Rs 20000 20000 30000 17000 30000 10000

Principal for February = Rs Principal for March Principal for April Principal for May Principal for June Total = Rs = Rs = Rs = Rs

Rs 127000

Thus, the sum on which Anisha will earn interest for one month = Rs 127000. Example 12.3 : Rituss pass book has the following entries : Date 2002 Jan. 1 Jan. 9 Jan. 11 Feb. 10 April 3 June 5 June 25 B/F To Cash By Cheque To Cheque By Cash By Cheque By Cheque Rs 7000.00 Rs 10000.00 20000.00 5000.00 10000.00 5000.00 12000.00 5000.00 25000.00 15000.00 20000.00 30000.00 35000.00 Particulars Amount withdrawn Amount Deposited Balance Rs P Rs P Rs P

If the rate of interest is 4% per annum, find the interest earned by Ritu at the end of June on her savings bank account. Solution : Details of qualifying amount for the calculation of interest are as below: Month Jan. Feb. March April May June Total Rs Rs Rs Rs Rs Rs Rs Amount 5000.00 15000.00 15000.00 20000.00 20000.00 30000.00 105000.00

278

Mathematics

Here P = Rs 105000, R = 4% and T =

1 year. 12

105000 4 1 = Rs 350. 100 12 Interest earned by Ritu = Rs 350.

Interest = PRT = Rs

Example 12.4 : Ashok has a savings bank account in a bank. His passbook has the following entries: Date 2002 July 11 Aug 12 Sept 5 Sept 21 Nov 9 Dec 10 Dec 29 By cash By Cheque By Cheque To Cheque By Cheque By Cheque To cash 8000.00 26000.00 6000.00 4000.00 10000.00 8000.00 10000.00 6000.00 10000.00 20000.00 12000.00 20000.00 30000.00 4000.00 Particulars Amount withdrawn Amount Deposited Balance Rs P Rs P Rs P

The account is closed on 3rd January 2003. Find the amount received by Ashok, if the rate of interest is 4% per annum. Solution. Details of qualifying amount for the calculation of interest are : Month July August September October November December Total Rs Rs Rs Rs Rs Rs Rs Amount 6000.00 12000.00 12000.00 20000.00 4000.00 54000.00
1 year 12

Here P = Rs 54000, R = 4% and T =

Interest = PRT = Rs

LM 54000 4 1OP N 100 12 Q

= Rs 180.00

Amount received = Rs (4000 + 180) = Rs 4180.

Banking

279

Example 12.5 : The dates and respective balances in the pass book of Davids savings bank account are given below : Dates 2002 October 5 October 25 December 8 2003 January 11 February 5 February 9 February 11 March 15 March 29 Rs Rs Rs Rs Rs Rs 8000.00 6000.00 10000.00 7500.00 20000.00 10000.00 Rs Rs Rs 1500.00 3500.00 5000.00 Balances

Calculate the interest earned upto March 2003 if the rate of interest is 4% per annum. Solution : Details of qualifying amount for the calculation of interest are : Month Oct, 2002 Nov, 2002 Dec, 2002 Jan, 2003 Feb, 2003 March 2003 Total Here P = Rs 30000, R = 4% Interest = PRT = Rs Amount Rs Rs Rs Rs Rs Rs 1500 3500 5000 5000 7500 7500

Rs 30000 and T = 1/12 year

30000 4 1 = Rs 100 100 12

Interest earned by David = Rs 100.

280

Mathematics

Example 12.6 : Salim opened a savings bank account with a bank on 9th January 2002 with a cash deposit of Rs 10000. Subsequently he deposited Rs 2000 on the 8th day of every month. He withdrew Rs 3000 on 25th April and Rs 6000 on 28th June 2002. Write all entries of the passbook. If the bank pays interest at the rate of 4% per annum, calculate the interest upto the last day of 30th June 2002 and make the entry in the passbook alongwith the balance. Solution : The entries in the passbook are as below :
Date Particulars Amount withdrawn Rs 2002 9th Jan. 8th Feb. By cash By Cash 3000.00 6000.00 10000.00 2000.00 2000.00 2000.00 2000.00 2000.00 10000.00 12000.00 14000.00 16000.00 13000.00 15000.00 17000.00 11000.00 P Amount Deposited Rs P Balance Rs P

8th March By Cash 8th April By Cash

25th April To Cash 8th May 8th June By Cash By Cash

28th June To Cash

Details of qualifying amount for the calculation of interest are : Month Amount (in Rs) January February 10000 12000 March 14000 April 13000 May 15000 June 11000 Total 75000

Interest upto 30th June, 2002 = Rs

75000 1 4 = Rs 250 100 12

The interest entry is made on 172002 as below : 1.7.2002 By Interest 250.00 11250.00

CHECK YOUR PROGRESS 12.1 1. Krishna Murthy opened a savings bank account in State Bank of India on 7th July 2002. His pass book has the following entries :

Banking

281

Date 2002 7th July 11th July 25th August

Particulars

Amount withdrawn Amount Deposited Rs P Rs P

Balance Rs P

By cash By Cash By Cash

1100.00 400.00 1000.00 1500.00

1100.00 1500.00 2500.00 4000.00

10th September By Cheque

Calculate the principal for which he will earn interest for the months of July, August and September 2002, together. 2. The entries in the savings bank account pass book of Kamlesh are as under:
Date 2002 1st January 10th January 9th February 9th March 11th April 25th June B/F By Cash By Cheque By Cash By Cash By Cheque 5000.00 5000.00 5000.00 5000.00 5000.00 15000.00 20000.00 25000.00 30000.00 35000.00 40000.00 Particulars Amount withdrawn Amount Deposited Rs P Rs P Balance Rs P

Calculate the interest at the end of June 2002 at 4% per annum. 3. The entries in the passbook of a savings bank account holder who opened his account of 11th January 2002 are as follows :
Date 2002 11th January 11th Feb. 9th March 6th June 10th June 26th June By cash By Cheque By Cheque To Cheque By Cash To Cash 10000.00 16000.00 5000.00 5000.00 5000.00 15000.00 5000.00 10000.00 15000.00 5000.00 20000.00 4000.00 Particulars Amount withdrawn Amount Deposited Balance Rs P Rs P Rs P

282

Mathematics

If the rate of interest is 4% per annum, find the interest earned if the account is closed on : (i) 30th June, 2002 (ii) 3rd July, 2002. 4. Madhus Savings Bank Account passbook has the following entries :
Date 2002 July 1 July 9 Sept. 10 Sept. 14 December 5 December 10 December 23 B/F By Cheque To Cheque By Cash By Cash By Cash To Cheque 9000.00 9000.00 4000.00 4000.00 4500.00 1500.00 6000.00 10000.00 1000.00 5000.00 9500.00 11000.00 2000.00 Particulars Amount withdrawn Amount Deposited Rs P Rs P Balance Rs P

If the rate of interest is 6% per annum, calculate the interest entry on 1st January 2003 in the passbook alongwith the balance. 5. A page from the pass book of savings bank account is given below :
Date July 1, 2002 July 11, 2002 August 9, 2002 Particulars B/F By Cheque By Cheque Amount withdrawn Amount Deposited Rs P Rs P 15000.00 8000.00 10000.00 15000.00 Balance Rs P 2000.00 10000.00 20000.00 5000.00 20000.00

November 25, 2002 To Cheque December 19, 2000 By Cash

The account is closed on 2nd January 2003. Find the amount received if the rate of interest is 4% per annum. 6. Kavita opens a savings bank account with a bank on 8th January, 2002 with a cash deposit of Rs 10000. Subsequently she deposited Rs 6000 on the 6th day of every month. She withdraws Rs 4000 on 3rd April and Rs 12000 on 10th June 2002. If the bank pays interest at the rate of 5% per annum, payable at the end of June and December, write all the entries, including interest, which are made upto 1st July 2002.

Banking

283

12.4.3 Current Account In a saving bank account, the account holder is allowed to have a limited number of withdrawals in a half year. Big business concerns, companies, government organisations etc., have to do a number of transactions every day. For them banks offer a different type of account called current account. For this account, there is no limit on number of withdrawals or on the amount of withdrawals but the banks do not pay interest. Rather, sometimes, they charge some money as service charges. Here, the minimum balance for individuals account is Rs 5000 while for big concerns, it is Rs 10000. Depending upon the goodwill of the individual/company, banks allow the current account holder to get money over and above their deposits called overdraft. 12.4.4 Fixed Deposit Account Suppose you have some money which is not required for some time. The scheme suitable for depositing such money is the Fixed Deposit or Term deposit. Here the depositor agrees to keep the money with the bank for a fixed time. Obviously, the bank can use this money more freely than the money kept in the savings bank account. Hence the banks offer higher rates of interest on such deposits depending upon the period of deposits. The rate of interest per annum on term deposits is as below : (i) For 46 days and above but less than 179 days. (ii) For six months and above but less than 1 year. (iii) For one year and more. 5% 6% 6.5%

For senior citizens, and additional interest of 0.5% is given on deposits. The total amount receivable after the expiry of the time is called maturity value. Example 12.7 : Anju deposited Rs 2920 in a fixed deposit scheme in a bank for 60 days. If the bank pays interest at 6% per annum, find the amount she receives at the end of 60 days. Solution : Here P = Rs 2920, R = 6% per annum and T =
60 year 365

Interest = PRT = Rs = Rs 28.80

FG 2920 6 60IJ H 100 365 K

Amount Received by Anju = Rs (2920 + 28.80) = Rs 2948.80

Example 12.8 : Joginder makes a fixed deposit of Rs 31250 in a bank for 1 years. If the rate of interest is 8% per annum compounded half yearly, find the maturity value of the money deposited by him.

284

Mathematics

Solution : Note here that the interest for the first half year also forms a part of principal for the second half year and likewise for the third half years well. Thus, we shall make use of the formula for compound interest. Here P = Rs. 31250, R = and T = 1 8 = 4% per half year 2

1 year = 3 Half years 2 n = 3.

4 Amount = P(1 + R)n = Rs 31250 1 + 100 = Rs 35152

FH

IK

Maturity value of the deposit = Rs 35152 Example 12.9 : Amit makes a fixed deposit of Rs 31250 in a bank for 1 years. If the rate of interest is 8% per annum compounded yearly, find the maturity value of the money deposited by him. 1 Solution : Here P = Rs 31250, R = 8%, T = 1 years. 2 8 Amount after one year = Rs 31250 1 + 100 1 and amount after 1 year 2 8 A = Rs 31250 1 + 100 Hence, the maturity value = Rs 35100. Note : Note the difference in the maturity value in Example 8 and Example 9. In example 8, the interest is compounded half yearly while in Example 9, the interest is compounded yearly. Example 12.10 : Kapil makes a fixed deposit of Rs 20000 in a bank in a year. If the rate of interest 8% per annum compounded quarterly, find the maturity value of the money deposited by him. Solution : Here P = Rs. 20000, R = 2% per quarter, Time = 4 quarters

FH

IK
4 I IK FH1 + 100 K

FH

= Rs 31250 1.08 1.04 = Rs 35100

2 A = P(1 + R)n = Rs 20000 1 + 100 = Rs 21648.64

FH

IK

= Rs 20000 1.02 1.02 1.02 1.02 Hence the maturity value = Rs 21649 (approx.)

Banking

285

Example 12.11 : How much money should Nirmal deposit in a fixed deposit account in a bank so that she gets Rs 456976 after two years, the rate of interest being 8% per annum compounded half yearly. Solution : Here P = ?, A = Rs 456976, R = 4% per half year and T = 2 year = 4 half years i.e. n = 4 We know that A = P(1 + R)n

456976 = P 1 + 4 100 P = Rs

FH

IK

= P 26 25

FH IK

456976 25 25 25 25 26 26 26 26

= Rs 390625 Hence amount to be deposited = Rs 390625. CHECK YOUR PROGRESS 12.2 1. Find the amount at the end of 9 months if Rs 50000 is deposited and the interest 8% per annum is compounded quarterly. 2. Charu makes a fixed deposit of Rs 8000 in a bank for 1 years. It the rate of interest is 10% per annum and the interest is compounded half yearly, find the maturity value of the money deposited by her. 3. Pankaj deposits Rs 75000 in a fixed deposit account for 3 years. If the rate of interest is 10% per annum compounded annually, find the maturity value of the money deposited by him. 4. How much money should Shanta deposit in a fixed deposit account in a bank so as to enable her to receive a sum of Rs 9261 after 1 years, the rate of interest being 10% per annum compounded half yearly ? 5. How much money should Kamal deposit in a fixed deposit account in a bank so as to enable him to receive a sum of Rs 1061208 after nine months, the rate of interest being 8% per annum compounded quarterly ? LET US SUM UP
z

There are different types of accounts in a bank. Some of these are : (i) Saving bank account (ii) Fixed or term deposit account (iii) Current account

286

Mathematics

In a saving bank account : (i) the bank pays interest for the month on the minimum closing balance from the 10th day of the month to the last day of the month. (ii) the interest is credited to the account every six months.

Step for computing interest are : (i) Write down the minimum balance between the closing balance on 10th to the last day of the month. (ii) Add all the minimum balances for each month as per step (i) to obtain principal for one month. (iii) Calculate the simple interest on this sum for one month using the formula Interest = P R
1 12

where P is Principal, R is rate of interest per annum and time 1 month i.e. 1/12 year. (iv) If account is opened after the 10th day of month, no interest is payable for that month. (v) No interest is paid for the month in which the account is closed. TERMINAL EXERCISE 1. A page for the pass book of Mr. Dasss Saving bank Account in a particular year is given below : Date Particulars Amount withdrawn Amount Deposited Rs P Rs P By Cash By Cheque To Cheque By Cash By Cash 20000.00 15000.00 8000.00 2000.00 12000.00 Balance Rs P 15000.00 23000.00 3000.00 5000.00 17000.00

Jan. 9 Feb. 10 April 25 June 7 June 11

If the rate of interest is 4% per annum, find the interest earned by Mr. Dass at the end of June on his savings bank account.

Banking

287

2. Sujata has a savings bank account in a bank. Her passbook has the following entries :
Date 2002 July 1 July 9 August 10 October 19 November 2 Dec. 20 Dec. 27 B.F. By Cheque By Cash To Cheque By Cash To Cash By Cheque 6000.00 9200.00 2500.00 4000.00 9600.00 10600.00 2500.00 5000.00 9000.00 3000.00 12600.00 3400.00 14000.00 Particulars Amount withdrawn Rs P Amount Deposited Rs P Balance Rs P

The account is closed on 10th January, 2003. Find the amount received if the rate of interest is 4% per annum. 3. Vandana makes a fixed deposit of Rs 62500 in a bank for 1 years. If the rate of interest is 8% per annum compounded half yearly, find the maturity value of money deposited by her. 4. Smith makes a fixed deposit of Rs 10000 in a bank for a year. If the rate of interest is 8% per annum compounded quarterly, find the maturity value of the money deposited by her. 5. How much money should Sarla deposit in a fixed deposit account in a bank so that she gets Rs 194481 after two years, the rate of interest being 10% per annum compounded half-yearly ?

288

Mathematics

ANSWERS Check Your Progress 12.1 1. Rs 6600 2. Rs 583.33 3. (i) Rs 166.66 5. Rs 20240 Debit Rs P Credit Rs P Balance Rs P (ii) Rs 180

4. Interest Rs 165, Balance Rs 2165 6. Date 2002 Jan. 5 Feb. 6 March 6 April 3 April 6 May 6 June 6 June 10 July 1 By cash By Cash By Cash To Cash By Cash By Cash By Cash To Cash By interest Particulars

10000.00 6000.00 6000.00 6000.00 6000.00 6000.00 525.00

10000.00 16000.00 22000.00 18000.00 24000.00 30000.00 36000.00 24000.00 24525.00

4000.00 12000.00

Check Your Progress 12.2 1. Rs 53060.40 Terminal Exercise 1. Rs 240 2. Rs 14140 3. Rs 70304 4. Rs 10824.32 5. Rs 160000 2. Rs 9261 3. Rs 99825 4. Rs 8000 5. Rs 1000000

Lines and Angles

Module 3
Geometry
Geometry is a branch of Mathematics which deals with the study of different types of figures and their properties. Geometry means measurement of earth. As such geometry originated in ancient times when man started measuring land for making his home and boundaries for his fields. Egyptians and Babylonians discovered many formulae for finding the areas of different rectilinear figures and used them practically. The Indian Mathematicians also had contributed a lot towards development of knowledge of geometry as is evident from the civilizations of Harappa and Mohenjodaro. Sulbasutras used during vedic period, and the work of the great mathematician Baudhayan, who profounded and proved, what is now known as Pythagoras theorem are contributions to geometry worth mentioning from the glorius past of India. Euclid a Greek mathematician collected all available knowledge in geometry till his times (330 B.C.), arranged it in a systematic way and gave it a logical approach based on deductive reasoning. Since then efforts have been made to give it a perfect logical shape. To study geometry in its full and complete logical form, is a difficult task. As such we shall be studying geometry in a very informal way; defining terms with suitable examples, verifying and stating properties of figures and proving logically only a few important properties known as theorems. In this module on geometry, we shall study about lines, angles, triangles, quadrilaterals and circles along with their properties.

Mathematics

13
Lines and Angles
13.1 INTRODUCTION Observe the top of your desk or table. Now move your hand on the top of your table. It gives an idea of a plane. Its edges give an idea of a line, its corner that of a point and the edges meeting at a corner give an idea of an angle. 13.2 OBJECTIVES After studying this lesson, the learner will be able to :
z

illustrate the concepts of point, line, plane, angle, parallel lines, intersecting lines and pair of angles made by them. illustrate and verify properties of parallel lines prove that the sum of angles of a triangle is 180 explain the concept of locus and find the locus of a point under certain conditions.

z z z

13.3 EXPECTED BACKGROUND KNOWLEDGE We assume that the learner is familiar with the geometrical concepts and figures such as :
z z

point, line, plane, intersecting lines, rays and angles. parallel lines

13.4 POINT, LINE AND ANGLE In earlier classes, you have studied about a point, a line, a plane and an angle. Let us quickly recall these concepts. Point : If we press the tip of a pen or pencil on a piece of paper, we get a fine dot, which is called a point.

Fig. 13.1

Lines and Angles

A point is used to show the location and is represented by capital letters A, B, C etc. 13.4.1 Line Now mark two points A and B. Join them with the help of a ruler or a scale and extend it on both sides. This gives us a straight line or simply called a line.

Fig. 13.2

In geometry a line is extended infinitely on both sides and is marked with arrows to give this idea. A line is named using any two points on it, viz, AB or by a single small letter l, m etc. (See Fig. 13.3)

Fig. 13.3

The part of the line between two points A and B is called a line segment and will be named AB. Observe that a line segment is the shortest path between two points A and B. (See Fig. 13.4)

Fig. 13.4

13.4.2 Ray If we mark a point X and draw a line, starting from it extending infinitely in one direction only, then we get a ray XY.

Fig. 13.5

X is called the initial point of the ray XY. 13.4.3 Plane If we move our palm on the top of a table, we get an idea of a plane.

Fig.13.6

Mathematics

Similarly, floor of a room also gives the idea of part of a plane. Plane also extends infinitely lengthwise and breadthwise. Mark a point A on a sheet of paper. How many lines can you draw passing though this point ? As many as you wish.

Fig. 13.7

In fact we can draw an infinite number of lines through a point. Take another point B, at some distance from A. We can again draw an infinite number of lines passing through B.

Fig. 13.8

Out of these lines how many pass through both the points A and B ? Out of all the lines passing through A, only one passes through B. Thus, only one line passes through both the points A and B. We conclude that one and only one line can be drawn passing through two given points. Now we take three points in plane.

Fig. 13.9

We observe that a line may or may not pass through the three given points. If a line can pass through three or more points, then these points are said to be collinear. For example, points A, B and C in the Fig. 13.9 are collinear points. If a line can not be drawn passing through all three points (or more points) then they are said to be non-collinear. For example points P, Q and R in the Fig. 13.9. Since two points always lie on a line, we talk of collinear points only when their number is three or more.

Lines and Angles

Let us now take two distinct lines AB and CD in a plane.

Fig. 13.10

How many points can they have in common ? We observe that these lines can have. either (i) one point in common as in Fig. 13.10 (a) and (b). [In such a case they are called intersecting lines] or (ii) no points in common as in Fig. 13.10(c). In such a case they are called parallel lines. Now observe three (or more) distinct lines in plane.

Fig. 13.11

What are the possibilities ? (i) They may interest in more than one point as in Fig. 13.11(a) and 13.11(b). or or (ii) They may intersect in one point only as in Fig. 13.11(c). In such a case they are called concurrent lines. (iii) They may be non intersecting lines parallel to each other as in Fig. 13.11 (d).

13.4.4 Angle Mark a point O and draw two rays OA and OB starting from O. The figure we get is called an angle. Thus, an angle is a figure consisting of two rays starting from a common point.

Mathematics

Fig. 13.11

This angle may be named as angle AOB or angle BOA or simply angle O; and is written as AOB or BOA or O. [see Fig. 13.11] An angle is measured in degrees. If we take any point O and draw two rays starting from it in opposite directions then the measure of this angle is taken to be 180 degrees, written as 180.

Fig. 13.12

This measure divided into 180 equal parts is called one degree (1). Angle obtained by two opposite rays is called a straight angle. An angle of 90 is called a right angle, for example BOA or BOC.

Fig. 13.13

Two lines or rays making a right angle with each other are called perpendicular lines. In Fig. 13.13, we can say OA is perpendicular to OB or vice-versa. An angle less than 90 is called an acute angle. For example POQ is an acute angle in Fig. 13.14(a). An angle greater than 90 but less than 180 is called an obtuse angle. For example, XOY is an obtuse angle in Fig. 13.14(b).

Lines and Angles

(a) Fig. 13.14

(b)

13.5

PAIRS OF ANGLES

Fig. 13.15

Observe the two angles 1 and 2 in each of the figures in Fig. 13.15. Each pair has a common vertex O and a common side OA in between OB and OC. Such a pair of angles is called a pair of adjacent angles.

(a) Fig. 13.16

(b)

Observe the angles in each pair in Fig. 13.16[(a) and (b)]. They add up to make a total of 90. A pair of angles, whose sum is 90, is called a pair of complementary angles. Each angle is called the complement of the other.

Mathematics

(a) Fig. 13.17

(b)

Again observe the angles in each pair in Fig. 13.17[(a) and (b)]. These add up to make a total of 180. A pair of angles whose sum is 180, is called a pair of supplementary angles. Each angle is called the supplement of the other. Draw a line AB. From a point C on it draw a ray CD making two angles X and Y.

Fig. 13.18

If we measure X and Y and add, we will always find the sum to be 180, whatever be the position of the ray CD. We conclude If a ray stands on a line then the sum of the two adjacent angles so formed is 180. The pair of angles so formed as in Fig. 13.18 is called a linear pair of angles. Note that they also make a pair of supplementary angles. Draw two intersecting lines AB and CD, intersecting each other at O.

Fig. 13.19

Lines and Angles

AOC and DOB are angles opposite to each other. These make a pair of vertically opposite angles. Measure them. You will always find that AOC = DOB. AOD and BOC is another pair of vertically opposite angles. On measuring, you will again find AOD = BOC We conclude : If two lines intersect each other, the pairs of vertically opposite angles are equal. Activity for you. Attach two strips with a nail or a pin as shown in the figure.

Fig. 13.20

Rotate one of the strips, keeping the other in position and observe that the pairs of vertically opposite angles thus formed are always equal. A line which intersects two or more lines at distinct points is called a transversal. For example line l in Fig. 13.21 is a transversal.

Fig. 13.21

When a transversal intersects two lines, eight angles are formed.

Fig. 13.22

10

Mathematics

These angles in pairs are very important in the study of properties of parallel lines. Some of the useful pairs are as follows : (a) 1 and 5 is a pair of corresponding angles. 2 and 6, 3 and 7 and 4 and 8 are other pairs of corresponding angles. (b) 3 and 6 is a pair of alternate angles. 4 and 5 is another pair of alternate angles. (c) 3 and 5 is a pair of interior angles on the same side of the transversal. 4 and 6 is another pair of interior angles. In Fig. 13.22 above, lines m and n are not parallel; as such, there may not exist any relation between the angles of any of the above pairs. However, when lines are parallel, there are some very useful relations in these pairs, which we study in the following, When a transversal intersects two parallel lines, eight angles are formed, whatever be the position of parallel lines or the transversal

Fig. 13.23

If we measure the angles, we shall always find that 1 = 5, Also 2 = 6, 3 = 7 and 4 = 8

that is, angles in each pair of corresponding angles are equal. 3 = 6 and 4 = 5 3 + 5 = 180 and 4 + 6 = 180

that is, angles in each pair of alternate angle are equal. Also,

that is, sum of angles in each pair of interior angles is 180. Hence we conclude : When a transversal intersects two parallel lines, then (i) each pair of corresponding angles, are equal (ii) each pair of alternate angles are equal (iii) each pair of interior angles on the same side of the transversal are supplementary. You may also verify the truth of these results by drawing a pair of parallel lines (using parallel edges of your scale) and a transversal and measuring angles in each of these pairs.

Lines and Angles

11

Converse of each of these results is also true. To verify the truth of the first converse, we draw a line AB and mark two points C and D on it.

Fig. 13.24

At C and D, we construct two angles ACF and CDH equal to each other, say 50, as shown in Fig. 13.24. On producing EF and GH on either side, we shall find that they do not intersect each other, that is, they are parallel. In a similar way, we can verify the truth of the other two converses. Hence we conclude that When a transversal intersects two lines in such a way that (i) any pair of corresponding angles are equal or or (ii) any pair of alternate angles are equal (iii) any pair of interior angles on the same side of transversal are supplementary

then the two lines are parallel. Example 13.1 : Choose the correct answer out of the alternative options in the following multiple choice questions.

Fig. 13.25

12

Mathematics

(i) In Fig. 13.25, FOD and BOD are (a) supplementary angles (c) vertically opposite (a) complementary angles (c) a linear pair (iii) In Fig. 13.25, BOD is equal to (a) x (c) (90 x) (a) 36 (c) 108 (b) (90 + x) (d) (180 x) (b) 72 (d) 144 Ans. (d) Ans. (c) (b) complementary angles (d) a linear pair of angles (b) supplementary angles (d) adjacent angles Ans. (d) Ans. (b)

(ii) In Fig. 13.25, COE and BOE are

(iv) An angle is 4 times its supplement; the angle is

(v) What value of x will make AB a straight angle in Fig. 13.26

Fig. 13.26

(a) 30 (c) 50

(b) 40 (d) 60 Ans. (c)

Fig. 13.27

In the above figure, l is parallel to m and p is parallel to q. (vi) 3 and 5 form a pair of (a) Alternate angles (c) vertically opposite (b) interior angles (d) corresponding angles Ans. (d)

Lines and Angles

13

(vii) In Fig. 13.27, if 1 = 80 then 6 is equal to (a) 80 (c) 100 (b) 90 (d) 110 Ans. (c)

Fig. 13.28

(viii) In Fig. 13.28 OA bisects LOB, OC bisects MOB and AOC = 90. Show that points L, O and M are collinear. Solution : BOL = 2 BOA and BOM = 2BOC Adding (i) and (ii), BOL + BOM = 2BOA + 2BOC LOM = 2(BOA + BOC) = 2 90 = 180 = straight angle L, O and M are collinear. CHECK YOUR PROGRESS 13.1 1. Choose the correct answer out of the given alternative in the following multiple choice questions : ...(i) ...(ii)

Fig. 13.29

In Fig. 13.29, AB || CD and PQ intersects them at R and S respectively.

14

Mathematics

(i) ARS and BRS form (a) a pair of alternate angles (b) a linear pair (c) a pair of corresponding angles (d) a pair of vertically opposite angels (ii) ARS and RSD form a pair of (a) Alternate angles (c) Corresponding angles (iii) If PRB = 60 then QSC is (a) 120 (c) 30 (b) 60 (d) 90 (b) Vertically opposite angles (c) Interior angles

Fig. 13.30

(iv) AB and CD intersect at O. COB is equal to (a) 36 (c) 108 (b) 72 (d) 144

Fig. 13.31

2. In fig 13.31 above, AB is a straight line. Find x 3. In Fig 13.32 below, l is parallel to m. Find angles 1 to 7.

Fig. 13.32

Lines and Angles

15

13.6 TRIANGLE, ITS TYPES AND PROPERTIES Triangle is the simplest of all the closed figures formed in a plane by three line segments.

Fig. 13.33

It is a closed figure formed by three line segments having six elements, namely three angles (i) ABC or B (ii) ACB or C (iii) CAB or A and three sides : (iv) AB (v) BC (vi) CA It is named as ABC or BAC or CBA and read as triangle ABC or triangle BAC or triangle CBA. 13.6.1 Types of Triangles Triangles can be classified into different types in two ways. (a) On the basis of sides

(i)

(ii) Fig. 13.34

(iii)

(i) Equilateral triangle : A triangle in which all the three sides are equal is called an equilateral triangle. [ABC in Fig. 13.34(i)] (ii) Isosceles triangle : A triangle in which two sides are equal is called an isosceles triangle. [DEF in Fig. 13.34(ii)]. (iii) Scalene triangle : A triangle in which all sides are of different lengths, is called a scalene triangle [LMN in Fig. 13.34(iii)] (b) On the basis of angles :

(i)

(ii) Fig. 13.35

(iii)

16

Mathematics

(i) Obtuse angled triangle : A triangle in which one of the angles is an obtuse angle is called an obtuse angled triangle or simply obtuse triangle [PQR is Fig. 13.35(i)] (ii) Right angled triangle : A triangle in which one of the angles is a right angle is called a right angled triangle or right triangle. [UVW in Fig 13.35(ii)] (iii) Acute angled triangle : A triangle in which all the three angles are acute is called an acute angled triangle or acute triangle [XYZ in Fig. 13.35(iii)] Now we shall study some important properties of angles of a triangle. 13.6.2 Angle Sum Property of a Triangle We draw two triangles and measure their angles.

Fig. 13.36

In Fig. 13.36 (a), A = 80, B = 40 and C = 60 A + B + C = 80 + 40 + 60 = 180 P = 30, Q = 40, R = 110 P + Q + R = 30 + 40 + 110 = 180 In Fig. 13.36 (b),

What do you observe ? Sum of the angles of triangle in each case in 180. We will prove this result in a logical way naming it as a theorem. Theorem : The sum of the three angles of a triangle is 180.

Fig. 13.37

Given : A triangle ABC To prove : A + B + C = 180 Construction : Through A draw a line DE parallel to BC.

Lines and Angles

17

Proof : Since DE is parallel to BC and AB is a transversal. Similarly B = DAB C = EAC B + C = DAB + EAC A + B + C = A + DAB + EAC = 180 13.6.3 Exterior Angle of a Triangle Let us produce a side BC of the ABC to a point D. (Angles making a straight angle) (pair of alternate angles) (pair of alternate angles) ... (1)

Now adding A to both sides of (1)

Fig. 13.38

In Fig. 13.39, observe that there are six exterior angles of the ABC, namely 1, 2, 3, 4, 5 and 6.

Fig. 13.39

In Fig. 13.38, ACD so obtained is called an exterior angle of the ABC. Thus, The angle formed by a side of the triangle produced and another side of the triangle is called an exterior angle of the triangle. Corresponding to an exterior angle of a triangle, there are two interior opposite angles. Interior opposite angles are the angles of the triangle not forming a linear pair with the given exterior angle. For example A and B are the two interior opposite angles corresponding to the exterior angle ACD of ABC. We measure these angles.

18

Mathematics

A = 60 B = 50 and ACD = 110 We observe that ACD = A + B. This observation is true in general. Thus, we may conclude : An exterior angle of a triangle is equal to the sum of the two interior opposite angles. Examples 13.3 : Choose the correct answer out of the given alternatives in the following multiple choice questions : (i) Which of the following can be the angles of a triangle ? (a) 65, 45 and 80 (c) 60, 60 and 59 (b) 90, 30 and 61 (d) 60, 60 and 60. Ans. (d)

Fig. 13.40

(ii) In Fig. 13.40 A is equal to (a) 30 (c) 45 (b) 35 (d) 75 Ans. (c)

(iii) In a triangle, one angle is twice the other and the third angle is 60. Then the largest angle is (a) 60 (c) 100 Example 13.4 : (b) 80 (d) 120 Ans. (b)

Fig. 13.41

In Fig. 13.41, bisectors of PQR and PRQ intersect each other at O. Prove that QOR = 90 + 1 P. 2

Lines and Angles

19

Solution :

1 1 QOR = 180 2 PQR + 2 PRQ 1 = 180 PQR + PRQ 2

FH

IK

b 1 = 180 b180P g 2

1 1 = 180 90+ P = 90+ P . 2 2 CHECK YOUR PROGRESS 13.2 1. Choose the correct answer out of given alternatives in the following multiple choice questions: (i) A triangle can have (a) Two right angles (b) Two obtuse angles

(c) At the most two acute angles (d) All three acute angles (ii) In a right triangle, one exterior angle is 120, The smallest angle of the triangles is (a) 20 (c) 40 (iii) (b) 30 (d) 60

Fig. 13.42

In Fig. 13.42, CD is parallel to BA. ACB is equal to (a) 55 (c) 65 (b) 60 (d) 70

2. The angles of a triangle are in the ratio 2 : 3 : 5, find the three angles. 3. Prove that the sum of the four angles of a quadrilateral is 360. 4. In Fig. 13.43 ABCD is a trapezium such that AB || DC. Find D and C and verify that sum of the for angles is 360.

20

Mathematics

Fig. 13.43

5. Prove that if one angle of a triangle is equal to the sum of the other two angles, then it is a right triangle. 6. In Fig. 13.44, ABC is a triangle such that ABC = ACB. Find the angles of the triangle.

Fig. 13.44

13.7 LOCUS During the game of cricket, when a player hits the ball, it describes a path, before being caught or touching the ground.

Fig. 13.44

The path described is called Locus. A figure in geometry is a result of the path traced by a point (or a very small particle) moving under certain conditions. For example : (1) Given two parallel lines l and m, also a point P between them equidistant from both the lines .

Fig. 13.45

If the particle moves so that it is equidistant from both the lines, what will be its path?

Lines and Angles

21

Fig. 13.46

The path traced by P will be a line parallel to both the lines and exactly in the middle of them as in Fig. 13.46. (2) Given a fixed point O and a point P at a fixed distance d.

Fig. 13.47

If the point P moves in a plane so that it is always at a constant distance d from the fixed point O, what will be its path.

Fig. 13.48

The path of the moving point P will be a circle as shown in Fig. 13.48. (3) Place a small piece of chalk stick or a pebble on top of a table. Strike it hard with a pencil or a stick so that it leaves the table with a certain speed and observe its path after it leaves the table.

Fig. 13.49

22

Mathematics

The path traced by the pebble will be a curve (part of what is known as a parabola) as shown in Fig. 13.49. Thus, locus of a point moving under certain conditions is the path or the geometrical figure, every point of which satisfies the given condition(s). 13.7.1 Locus of a point equidistant from two given points. Let A and B the two given points.

Fig. 13.50

We have to find the locus of a point P such that PA = PB. Joint AB. Mark mid point of AB as M. Mark another point P using compasses such that PA = PB. Join PM and extend it on both sides. Using a pair of divider or a scale, it can easily be verified that every point on PM is equidistant from A and B. Also AMP = BMP = 90 That is, PM is the perpendicular bisector of AB.

Fig. 13.51

Thus, we may conclude the following : The locus of a point equidistant from two given points is the perpendicular bisector of the line segment joining the two points. Activity for you : Mark two points A and B on a sheet of paper and join them. Fold the paper along mid-point of AB so that A coincides with B. Make a crease along the line of fold. This crease is a straight line. This is the locus of the point equidistant from the given points A and B. It can be easily checked that every point on it is equidistant from A and B. 13.7.2 Locus of a point equidistant from two lines intersecting at O Let AB and CD be two given lines intersecting at O.

Lines and Angles

23

Fig. 13.52

We have to find the locus of a point P which is equidistant from both AB and CD. Draw bisectors of BOP and BOC.

Fig. 13.53

If we take any point P on any bisector l or m, we will find perpendicular distances PL and PM of P from the lines AB and CD are equal. that is, PL = PM

Thus, we may conclude : The locus of a point equidistant from two intersecting lines is the pair of lines, bisecting the angles formed by the given line. Activity for you : Draw two lines AB and CD intersecting at O, on a sheet of paper. Fold the paper through O so that AO falls on CO and OD falls on OB and mark the crease along the fold. Take a point P on this crease which is the bisector of BOD and check using a set square that PL = PM

Fig. 13.54

In a similar way find the other bisector by folding again and getting crease 2. Any point on this crease 2 is also equidistant from both the lines.

24

Mathematics

Example 13.5 : Find the locus of the centre of a circle passing through two given points. Solution : Let the two given points be A and B. We have to find the position or positions of centre O of a circle passing through A and B.

Fig. 3.55

Point O must be equidistant from both the points A and B. As we have already learnt, the locus of the point O will be the perpendicular bisector of AB.

Fig. 13.56

CHECK YOU PROGRESS 13.3 1. Find the locus of the centre of a circle passing through three given points A, B and C which are non-collinear. 2. There are two villages certain distance apart. A well is to be dug so that it is equidistant from the two villages such that its distance from each village is not more than the distance between the two villages. Representing the villages by points A and B and the well by point P, show in a diagram the locus of the point P. 3. Two straight roads AB and CD are intersecting at a point O. An observation post is to be constructed at a distance of 1 km from O and equidistant from the roads AB and CD. Show in a diagram the possible locations of the post. 4. Find the locus of a point which is always at a distance 5 cm from a given line AB.

Lines and Angles

25

LET US SUM UP
z

A line extends to infinity on both sides and a line segment is only a part of it between two points. Two distinct lines in a plane may either be intersecting or parallel. If three or more lines intersect in one point only then they are called cocurrent lines. If a line passes through three or more points, then they are called collinear points. Two rays starting from a common point form an angle. A pair of angles, whose sum is 90 is called a pair of complementary angles. A pair of angles whose sum is 180 is called a pair of supplementary angles. If a ray stands on a line then the sum of the two adjacent angles, so formed is 180. If two lines intersect each other the pairs of vertically opposite angles are equal. When a transversal intersects two parallel lines, then (i) corresponding angles in a pair are equal. (ii) alternate angles are equal. (iii) interior angles on the same side of the thransversal are supplementary.

z z z z z z z z z

z z

The sum of the angles of a triangle is 180. An exterior angle of a triangle is equal to the sum of the two interior opposite angles. TERMINAL EXERCISE

1. In Fig. 13.57, if x = 42, then determine (a) y (b) AOD

Fig. 13.57

2.

Fig. 13.58

In the above figure p, q and r are parallel lines intersected by a transversal l at A, B and C. Find 1 and 2.

26

Mathematics

3. The sum of two angles of a triangle is equal to its third angle. Find the third angle. What type of triangle is it ? 4.

Fig. 13.59

In Fig. 13.59 sides of ABC have been produced as shown. Find the angles of the triangle.

5.

Fig. 13.60

In Fig. 13.60, sides ABC of the triangle ABC have been produced as shown. Show that the sum of the exterior angles so formed is 360. 6.

Fig. 13.61

In Fig. 13.61 ABC is a triangle in which bisectors of B and C meet at O. Show that BOC = 125.

Lines and Angles

27

7.

Fig. 13.62

In Fig. 13.62 above, find the sum of the angles, A, F, C, D, B and E. 8.

Fig. 13.63

In Fig. 12.63 in ABC, AD is perpendicular to BC and AE is bisector of BAC. Find DAE. 9.

Fig. 13.64

In Fig.13.64 above, in PQR, PT is bisector of P and PR is produced to S show that PQR + PRS = 2PTR. 10. Prove that the sum of the (interior) angles of a pentagon is 540.

28

Mathematics

ANSWERS Check Your Progress 13.1 1. (i) (b) (ii) (a) 2. x = 17. 3. 1 = 3 = 4 = 6 = 110 and 2 = 5 = 7 = 70. (iii) (b) (iv) (c)

Check Your Progress 13.2 1. (i) (d) (ii) (b) (iii) (b) 2. 36, 54 and 90 4. D = 140 and C = 110 6. ABC = 45, ACB = 45 and A = 90. Check Your Progress 13.3 1. Only a point, which is the point of intersection of perpendicular bisectors of AB and BC. 2. Let the villages b A and B, then locus will be the line segment PQ, perpendicular bisector of AB such that AP = BP = QA = QB = AB

Fig. 13.65

3. Possible locations will be four points two points P and Q on the bisector of AOC and two points R and S on the bisector of BOC.

Fig. 13.66

4.Two on either side of AB and lines parallel to AB at a distance of 5 cm from AB. Terminal Exercise 1. (a) y = 27 (b) = 126 3. Third angle = 90, Right triangle 7. 360 2. 1 = 48 and 2 = 132 4. A = 35, B = 75 C = 70 8. 12.

Lines and Angles

29

Congruence of Triangles

29

14
Congruence of Triangles
14.1 INTRODUCTION You might have observed that leaves of different trees have different shapes, but leaves of the same tree have almost the same shape. Although they may differ in size. The geometrical figures which have same shape and same size are called congruent and the property is called congruence. In this lesson you will study congruence of two triangles, some relations between their sides and angles in details. 14.2 OBJECTIVES After studying this lesson, the learner will be able to :
z z z z z

verify and explain whether two given figures are congruent or not. state the criteria for congruency of two triangles and apply them in solving problems. prove that angles opposite to equal sides of a triangle are equal. prove that sides opposite to equal angles of a triangle are equal. prove that if two sides of triangle are unequal, then the longer side has the greater angle opposite to it. state and verify inequalities in a triangle. solve problems based on the above results.

z z

14.3 EXPECTED BACKGROUND KNOWLEDGE


z z z z z

Recognition of plane geometric figures Equality of lines and angles Types of angles Angle sum property of a triangle Paper cutting and folding.

30

Mathematics

14.4 CONCEPT OF CONGRUENCE In our daily life you observe various figures or objects. These figures or objects can be categorised in terms of its shape and size in the following manner. (i) Which have different shapes and sizes as shown in Fig. 14.1

Fig. 14.1

(ii) Which have same shapes but different sizes as shown in Fig. 14.2

Fig. 14.2

(iii) Which have same shape and same size as shown in Figures 14.3, 14.4 and 14.5. (a) two' one rupee coins

Fig. 14.3

(b) two postage stamps or post cards

Fig. 14.4

Congruence of Triangles

31

(c) two photo prints of same size from the same negative.

Fig. 14.5

We will deal with the figures which have same shapes and same sizes. Two figures, which have the same shape and same size are called congruent figures and this property is called congruence. 14.4.1 ACTIVITY FOR YOU Take a sheet of paper, fold it in the middle and keep a carbon (paper) between the two folds. Now draw a figure of a leaf or a flower or any object which you like, on the upper part of the sheet. You will get a carbon copy of it, on the sheet below. The figure you drew and its carbon copy are of the same shape and same size. Thus, these are congruent figures. Observe a butterfly folding its two wings. These appear to be one. 14.4.2 CRITERIA FOR CONGRUENCE OF SOME FIGURES Congruent figures when placed one over the other, exactly coincide with one another or cover each other. In other words, two figures will be congruent, if parts of one figure are equal to the corresponding parts of the other. For example : (1) Two line segments are congruent, when they are of equal length.

Fig. 14.6

(2) Two squares are congruent if their sides are equal.

Fig. 14.7

32

Mathematics

(3) Two circles are congruent, if their radii are equal, implying their circumferences are also equal.

Fig. 14.8

14.5 CONGRUENCE OF TRIANGLES Triangle is a basic rectilinear figure in geometry, having minimum number of sides. As such congruence of triangles plays a very important role in proving many useful results. Hence this needs a detailed study. Two triangles are congruent, if all the sides and all the angles of one are equal to the corresponding sides and angles of other. For example, in triangles PQR and XYZ in Fig. 14.9

Fig. 14.9

PQ = XY, PR = XZ, QR = YZ P = X, Q = Y and R = Z Thus we can say PQR is congruent to XYZ and we write PQR XYZ Relation of congruence between two triangles is always written with corresponding or matching parts in proper order. Here PQR XYZ

also means P corresponds to X, Q corresponds to Y and R corresponds to Z.

Congruence of Triangles

33

This congruence may also be written as QRP YZX which means, Q corresponds to Y, R corresponds to Z and P corresponds to X. It also means corresponding parts, (elements) are equal, namely QR = YZ, RP = ZX, QP = YX, Q = Y, R = Z and P = X. RPQ ZXY but NOT as Or NOT as PQR YZX. PQR ZXY. This congruence may also be written as

14.6 CRITERIA FOR CONGRUENCE OF TRIANGLES In order to prove, whether two triangles are congruent or not, we need to know that all the six parts of one triangle are equal to the corresponding six parts of the other triangle. We shall now learn that it is possible to prove the congruence of two triangles, even if we are able to know the equality of three of their corresponding parts. Consider a triangle ABC in Fig. 14.10

Fig. 14.10

Construct another triangle PQR such that QR = BC, Q = B and PQ = AB. (See Fig. 14.11).

Fig. 14.11

If we trace or cut out triangle ABC and place it over triangle PQR, we will observe that one covers the other exactly. Thus, we may say they are congruent. Alternatively, we can also measure the remaining parts, and observe that

34

Mathematics

AC = PR, A = P and C = R Showing that PQR ABC. It should be noted here that in constructing PQR congruent to ABC we used only two pairs of sides PQ = AB, QR = BC and the included angle between them Q = B. This means that equality of these three corresponding parts results in congruent triangles. Thus we have Criterion 1 : If any two sides and the included angle of one triangle are equal to the corresponding two sides and the included angle of the other triangle, the two triangles are congruent. This criterion is referred to as SAS (side angle side). Again, consider ABC in Fig. 14.12.

Fig. 14.12

Construct another PQR such that, QR = BC, Q = B and R = C. (See Fig. 14.13)

Fig. 14.13

By superimposition or by measuring the remaining corresponding parts, we observe that P = A, PQ = AB and PR = AC establishing that PQR ABC, which again means that equality of the three corresponding parts (two angles and the included side) of two triangles results in congruent triangles. We also know that the sum of the three angles of a triangle is 180, as such if two angles of one triangle are equal to the corresponding angles of another triangle, then the third angles will also be equal. Thus instead of included side we may have any pair of corresponding sides equal. Thus we have Criterion 2 : If any two angles and a side of one triangle are equal to corresponding angles and the side of the other triangle, then the two triangles are congruent. This criterion is referred to as ASA or AAS.

Congruence of Triangles

35

14.6.1 ACTIVITY FOR YOU In order to explore another criterion we again take a triangle ABC (See Fig. 14.14)

Fig. 14.14

Now take three thin sticks equal in lengths to sides AB, BC and CA of ABC. Place them in any order to form PQR or PQR near the ABC (Fig. 14.15).

Fig. 14.15

By measuring the corresponding angles, we find that, P = P = A, Q = Q = B and R = R = C, establishing that PQR PQR ABC which means that equality of the three corresponding sides of two triangles, results in congruent triangles. Thus we have Criterion 3 : If the three sides of one triangle are equal to the three corresponding sides of the other triangle, then the two triangles are congruent. This is referred to as SSS (side, side, side). Similarly, we can establish one more criterion which will be applicable for two right triangles only. Criterion 4 : If the hypotenuse and a side of one triangle are respectively equal to the hypotenuse and a side of the other triangle, then the two right triangles are congruent.

36

Mathematics

This criterion is referred to as R.H.S. (Right Hypotenuse side). Using these criteria we can easily prove, knowing three corresponding parts only, whether two triangles are congruent and establish the equality of remaining corresponding parts. Example 14.1 : In which of the following criteria, two given triangles are NOT congruent. (a) All corresponding sides are equal (b) All corresponding angles are equal (c) Two corresponding sides and their included angles are equal (d) All corresponding angles and any pair of corresponding sides are equal. Ans. (b) Example 14.2 : Two rectilinear figures are congruent if they have only (a) all corresponding sides equal (b) all corresponding angles equal (c) the same area (d) all corresponding angles and all corresponding sides equal. Ans. (d) Example 14.3 : In Fig 14.16, PX and QY are perpendicular to PQ and PX = QY. Show that AX = AY.

Fig. 14.16

Solution : In PAX and QAY XPA = YQA (Each is 90)

Congruence of Triangles

37

PAX = QAY and PX = QY PAX QAY AX = AY.

(Vertically opposite angles) (AAS)

Example 14.4 : In Fig. 14.17, ABC is a right triangle in which B = 90 and D is the mid point of AC. Prove that BD =
1 AC. 2

Fig. 14.17

Solution : Produce BD to E such that BD = DE

Fig. 14.18

In ADB and CDE, AD = CD DB = DE and Also ADB = CDE ADB CDE AB = EC DAB = DCE (D being mid point of AC) (By construction) (Vertically opposite angles) ...(i)

But they make a pair of alternate angles AB is parallel to EC ABC + ECB = 180 90 + ECB = 180 ECB = 180 90 = 90 (Pair of interior angles)

38

Mathematics

Now in ABC and ECB, AB = EC BC = CB and But ABC = ECB ABC ECB AC = EB BD =
1 EB 2 1 AC 2

(From (i) above) (Common) (Each 90)

BD =

CHECK YOUR PROGRESS 14.1 1. In ABC (Fig. 14.19) if B = C and AD BC, then ABD ACD by the criterion.

Fig. 14.19

(a) RHS (c) SAS

(b) ASA (d) SSS

2. In Fig. 14.20, ABC PQR. This congruence may also be written as

Fig. 14.20

(a) BAC RPQ (c) BAC RQP

(b) BAC QPR (d) BAC PRQ.

Congruence of Triangles

39

3. In order that two given triangles are congruent, along with equality of two corresponding angles, we must know the equality of (a) No corresponding side (b) Minimum one corresponding side (c) Minimum two corresponding sides (d) All the three corresponding sides 4. Two triangles are congruent,. if (a) All three corresponding angles are equal (b) Two angles and a side of one are equal to two angles and a side of the other. (c) Two angles and a side of one are equal to two angles and the corresponding side of the other (d) One angle and two sides of one are equal to one angle and two sides of the other. 5. In Fig.14.21, B = C and AB = AC. Prove that ABE ACD. Hence show that CD = BE.

Fig. 14.21

6. In Fig. 14.22, AB is parallel to CD. If O is the mid point of BC, show that it is also the mid point of AD.

Fig. 14.22

40

Mathematics

7. In ABC (Fig. 14.23), AD is BC, BE is AC and AD = BE. Prove that AE = BD.

Fig. 14.23

8. From Fig. 14.24, show that the triangles are congruent and make pairs of equal angles.

Fig. 14.24

14.7 ANGLES OPPOSITE TO EQUAL SIDES OF A TRIANGLE AND VICE VERSA Using the criteria for congruence of triangles, we shall now prove some important theorems. Theorem : The angles opposite to equal sides of a triangle are equal. Given : A triangle ABC in which AB = AC. To prove : B = C. Construction : Draw bisector of BAC meeting BC at D. Proof : In ABD and ACD, AB = AC BAD = CAD and AD = AD (Given) (By construction) (Common)
Fig. 14.25

Congruence of Triangles

41

Hence

ABD ACD B = C

(SAS)

The converse of the above theorem is also true. We prove it as a theorem. 14.7.1 The sides opposite to equal angles of a triangle are equal Given : A triangle ABC in which B = C To prove : AB = AC Construction : Draw bisector of BAC meeting BC at D. Proof : In ABD and ACD, B = C BAD = CAD and Hence Hence the theorem. Example 14.5 : Prove that the three angles of an equilateral triangle are equal. Solution : Given : An equilateral ABC To prove : Proof : Also From (i) and (ii), A = B = C Hence the result. Example 14.6 : ABC is an isosceles triangle in which AB = AC (Fig. 14.28). If BD AC and CE AB, prove that BD = CE. Solution : In BDC and CEB BDC = CEB (Each is 90)
Fig. 14.28

(Given) (By construction) (Common) (AAS)


Fig. 14.26

AD = AD ABD ACD AB = AC

A = B = C AB = AC C = B AC = BC B = A (Given)

Fig. 14.27

(Angles opposite equal sides) (Given)

...(i) ...(ii)

42

Mathematics

DCB = EBC and Hence BC = CB BDC CEB BD = CE

(Angles opposite equal sides of a ) (Common) (AAS)

This result can be stated in the following manner : Perpendiculars drawn to equal sides from opposite vertices (or altitudes) of an isosceles triangle are equal. The result can be extended to an equilateral triangle after which we can say that all the three altitudes of an equilateral triangle are equal. Example 14.7 : In ABC (Fig. 14.29), D and E are mid points of AC and AB respectively. If AB = AC, then prove that BD = CE. Solution : BE =
1 AB 2 1 AC 2

and

CD =

BE = CD BE = CD BC = CB

...(i) [By (i)] (Common) (Q AB = AC)


Fig. 14.29

In BEC and CDB,

and EBC = DCB BEC CDB CE = BD Hence

Example 14.8 : In ABC (Fig. 14.30) AB = AC and DAC = 124; find the angles of the triangle. Solution. BAC = 180 124 = 56 B = C (Angles opposite to equal sides of a triangle) Also Hence B + C = 124 B = C =
124 = 62 2
Fig. 14.30

A = 56, B = 62, and C = 62,

Congruence of Triangles

43

CHECK YOUR PROGRESS 14.2 1. In Fig. 14.31, PQ = PR and SQ = SR. Prove that PQS = PRS.

Fig. 14.31

2. Prove that ABC is a isosceles triangle, if the altitude AD bisects the base BC (Fig. 14.32).

Fig. 14.32

3. If the line l in Fig. 14.33 is parallel to the base BC of the isosceles ABC find the angles.

Fig. 14.33

4. ABC is an isosceles triangle such that AB = AC. Side BA is produced to a point D such that AB = AD. Prove that BCD is a right angle.

Fig. 14.34

44

Mathematics

5. In Fig. 14.35, D is the mid point of BC and perpendiculars DF and DE to sides AB and AC are equal in length. Prove that ABC is an isosceles triangle.

Fig. 14.35

6. In Fig. 14.36, PQ = PR. QS and RT are the angle bisector of Q = R respectively Prove that QS = RT.

Fig. 14.36

7. PQR and SQR are isosceles triangles on the same base QR (Fig. 14.37). Prove that PQS = PRS.

Fig. 14.37

8. In ABC, AB = AC (Fig. 14.38). P is any point in the interior of the triangle such that ABP = ACP. Prove that AP bisects BAC.

Fig. 14.38

Congruence of Triangles

45

14.8 INEQUALITIES IN A TRIANGLE We have learnt the relationship between sides and angles of a triangle when they are equal. We shall now study some relations among sides and angles of a triangle, when they are unequal.

Fig. 14.39

In Fig. 14.39, triangle ABC has side AB longer than the side AC. Measure B and C. You will find that these angles are not equal and C is greater than B. If you repeat this experiment, you will always find that this observation is true. This can be proved easily, as follows. 14.8.1 Theorem If two sides of a triangle are unequal, then the longer side has the greater angle opposite to it. Given. A triangle ABC in which AB > AC. To prove. ACB > ABC Construction. Mark a point D on the side AB such that AD = AC and join DC. Proof : In ADC, AD = AC But ACD = ADC ADC > ABC (Exterior angle is greater than opposite interior angles) Again ACB > ACD(Point D lies in the interior of the ACB). ACB > ABC
Fig. 14.40

(Angles opposite equal sides)

What can we say about the converse of this theorem. Let us examine. In ABC, (Fig. 14.41) compare C and B. It is clear that C is greater than B. Now compare sides AB and AC opposite to these angles by measuring them. We observe that AB is longer than AC. Again compare C and A and measure sides AB and BC opposite to these angles. We observe that C > A and AB > BC; i.e. side opposite to greater angle is longer.

Fig. 14.41

46

Mathematics

Comparing A and B, we observe a similar result. A > B and BC > AC; i.e. side opposite to greater angle is longer. You can also verify this property by drawing any type of triangle, a right triangle or an obtuse triangle. Measure any pair of angles in a triangle. Compare them and then compare the sides opposite to them by measurement. You will find the above result always true, which we state as a property. In a triangle, the greater angle has longer side opposite to it. Observe that in a triangle if one angle is right or an obtuse angle, then the side opposite to that angle is the longest. You have already learnt the relationship among the three angles of a triangle i.e., the sum of the three angles of a triangle is 180, we shall now study whether the three sides of a triangle are related in some way. Draw a triangle ABC.

Fig. 14.42

Measure its three sides AB, BC and CA. Now find the sum of different pairs AB + BC, BC + CA, and CA + AB separately and compare each sum of a pair with the third side, we observe that (i) AB + BC > CA (ii) BC + CA > AB and (ii) CA + AB > BC. Thus we conclude that Sum of any two sides of a triangle is greater than the third side. ACTIVITY FOR YOU Fix three nails P, Q and R on a wooden board or any surface.

Congruence of Triangles

47

Fig. 14.43

Take a piece of thread equal in length to QR and another piece of thread equal in length QP + PR. Compare the two lengths, you will find that the length corresponding to QP + PR > the length corresponding to QR confirming the above property. Example 14.9 : In which of the following four cases, is construction of a triangle possible from the given measurements : (a) 5 cm, 8 cm and 3 cm (b) 14 cm, 6 cm and 7 cm (c) 3.5 cm, 2.5 cm and 5.2 cm (d) 20 cm, 25 cm and 48 cm. Solution. In (a) 5 + 3 > 8, in (b) 6 + 7 > 14

in (c) 3.5 + 2.5 > 5.2, 3.5 + 5.2 > 2.5 and 2.5 + 5.2 > 3.5 and in (d) 20 + 25 > 48. Ans. (c) Example 14.10 : In Fig. 14.44, AD is a median of ABC. Prove that AB + AC > 2AD.

Fig. 14.44

Fig. 14.45

Solution : Produce AD to E such that AD = DE and join C to E. Consider ABD and ECD Here, BD = CD

48

Mathematics

ADB = EDC AD = ED Now in ACE, EC + AC > AE or AB + AC > 2AD (Q AD = ED AE = 2AD) ABD ECD AB = EC

CHECK YOUR PROGRESS 14.3 1. PQRS is a quadrilateral in which diagonals PR and QS intersect at O. Prove that PQ + QR + RS + SP > PR + QS 2. In a triangle ABC, AB = 5.7 cm, BC = 6.2 cm and CA = 4.8 cm. Name the greatest and the smallest angle. 3. In Fig. 14.46, if CBD > BCE then prove that AB > AC.

Fig. 14.46

4. In Fig. 14.47, D is any point on the base BC of a ABC. If AB > AC then prove that AB > AD.

Fig. 14.47

5. Prove that the sum of the three sides of triangle is greater than the sum of its three medians. (Use Example 14.10)

Congruence of Triangles

49

6. In Fig. 14.48, if AB = AD then prove that BC > CD. [Hint : ADB = ABD].

Fig. 14.48

7. In Fig. 14.49, AB is parallel to CD. If A > B then prove that BC > AD.

Fig. 14.49

LET US SUM UP
z z

Figures which have the same shape and same size are called congruent figures. Congruent figures, when placed one over the other completely cover each other. All parts of one figure are equal to the corresponding parts of the other figure. To prove that two triangles are congruent we need to know the equality of only three corresponding parts. These corresponding parts must satisfy one of the four criteria (i) SAS. (iii) SSS (ii) ASA or AAS (iv) RHS

z z z

Angles opposite to equal sides of a triangle are equal. Sides opposite to equal angles of a triangle are equal. If two sides of a triangle are unequal, then the longer side has the greater angle opposite to it. In a triangle, the greater angle has the longer side opposite to it. Sum of any two sides of a triangle is greater than the third side.

z z

50

Mathematics

TERMINAL EXERCISE 1. Two lines AB and CD bisect each other at O. Prove that CA = BD (Fig. 14.50) 2. In a ABC, if the median AD is perpendicular to the base BC then prove that the triangle is an isosceles triangle.

Fig. 14.50

Fig. 14.51

3. In Fig. 14.51, ABC and CDE are such that BC = CE and AB = DE. If B = 60, ACE = 30 and D = 90, then prove that the two triangles are congruent. 4. In Fig. 14.52, two sides AB and BC and the altitude AD of ABC are respectively equal to the sides PQ and QR and the altitudes PS. Prove that ABC PQR.

Fig. 14.52

5. In a right triangle, one of the acute angles is 30. Prove that the hypotenuse is twice the side opposite to the angle of 30. 6. Line segments AB and CD intersect each other at O such that O is the midpoint of AB. If AC is parallel to DB then prove that O is also the mid point of CD. 7. In Fig. 14.53, AB is the longest side and DC is the shortest side of a quadrilateral ABCD. Prove that C > A and D > B. [Hint : Join AC and BD].

Fig. 14.53

Congruence of Triangles

51

8. ABC is an isosceles triangle in which AB = AC and AD is the altitude from A to the base BC. Prove that BD = DC.

Fig. 14.54

9. Prove that the medians bisecting the equal sides of an isosceles triangle are also equal. [Hint : Show that DBC ECB)

Fig. 14.55

52

Mathematics

ANSWERS Check Your Progress 14.1 1. (a) 3. (b) 2 (b) 4. (c)

8. P = C Q = A and R = B. Check Your Progress 14.2 3. B = C = 65; A = 50 Check Your Progress 14.3 2. Greatest angle is A and smallest angle is B.

Congruence of Triangles

53

Concurrent Lines

53

15
Concurrent Lines
15.1 INTRODUCTION You have already learnt about concurrent lines, in the lesson on lines and angles. You have also studied about triangles and some special lines; i.e, medians, right bisectors of sides, angle bisectors and altitudes, which can be drawn in a triangle. In this lesson, we shall study the concurrency property of these lines, which are quite useful. 15.2 OBJECTIVES After studying this lesson, the learner will be able to :
z

distinguish between an angle bisector and perpendicular bisector of a side, an altitude and a median of a triangle. state and apply the concurrency property of angle bisectors, perpendicular bisectors of sides, altitudes and medians of a triangle.

15.3 EXPECTED BACKGROUND KNOWLEDGE Properties of intersecting lines, such as :


z

Two lines in a plane can either be parallel [See Fig. 15.1(a)] or intersecting [See Fig. 15.1(b) and (c)].

(a)

(b) Fig. 15.1

(c)

54

Mathematics

Three lines in a plane may : (i) be parallel to each other, i.e., intersect in no point [See Fig 15.2(a). or (ii) intersect each other in exactly one point [Fig. 15.2(b), or (iii) intersect each other in two points [Fig. 15.2(c), or (iv) intersect each other at the most in three points [Fig. 15.2(d)]

(a)

(b) Fig. 15.2

(c)

(d)

15.4 CONCURRENT LINES Three or more lines in a plane which intersect each other in exactly one point or which pass through the same point are called concurrent lines and the common point is called the point of concurrency (See Fig. 15.3).

(a)

(b) Fig. 15.3

(c)

15.4.1 ANGLE BISECTORS OF A TRIANGLE In triangle ABC, the line AD bisects A of the triangle. (See Fig. 15.4)

Fig.15.4

Concurrent Lines

55

A line which bisects an angle of a triangle is called an angle bisector of the triangle. How many angle bisectors can a triangle have ? Since a triangle has three angles, we can draw three angle bisectors in it. AD is one of the three angle bisectors of ABC. Let us draw second angle bisector BE of B (See Fig 15.5).

Fig. 15.5

Fig. 15.6

The two angle bisectors of the ABC intersect each other as I. Let us draw the third angle bisector CF of C (See Fig. 15.6). We observe that this angle bisector of the triangle also passes through I. In other words they are concurrent and the point of concurrency is I. We may take any type of triangle acute, right or obtuse triangle, and draw its angle bisectors, we will always find that the three angle bisectors of a triangle are concurrent. (see Fig. 15.7)

Fig. 15.7

Thus we conclude the following : Angle bisectors of a triangle pass through the same point, that is they are concurrent. The point of concurrency I is called the Incentre of the triangle. Can you reason out, why the name incentre for this point ? Recall that the locus of a point equidistant from two intersecting lines is the pair of angle bisectors of the angles formed by the lines. Since I is a point on bisector of BAC, it must be equidistant from them. Also I is a point on angle bisector of ABC, (See Fig. 15.8), it must

56

Mathematics

also be equidistant from them. Thus this point of concurrency I is at the same distance from the three sides of the triangle.

Fig. 15.8

Thus, we have IL = IM = IN (Fig. 15.8). Taking I as the centre and IL as the radius, we can draw a circle touching all the three sides, of the triangle, called Incircle of the triangle. I being the centre of the incircle is called the Incentre and IL the radius of the incircle is called the inradius of the triangle. Note : That incentre always lies in the interior of the triangle. 15.4.2 Perpendicular bisectors of the sides of a triangle ABC is a triangle, line DP bisects side BC at right angle. A line which bisects a side of a triangle at right angle is called the perpendicular bisector of the side. Since a triangle has three sides, so we can draw three perpendicular bisectors in a triangle. DP is one of the three perpendicular bisector of ABC (Fig. 15.9). We draw the second perpendicular bisector EQ, intersecting DP at O (Fig. 15.10). Now if we also draw the third perpendicular bisector FR, then we observe that it also passes through the point O (Fig. 15.11). In other words, we can say that the three perpendicular bisectors of the sides are concurrent at O.

Fig. 15.9

Fig. 15.10

Fig. 15.11

Concurrent Lines

57

We may repeat this experiment with any type of triangle, but we will always find that the three perpendicular bisectors of the sides of a triangle pass through the same point.

(a) Fig. 15.12

(b)

Thus we conclude that : The three perpendicular bisectors of the sides of a triangle pass through the same point, that is, they are concurrent. The point of concurrency O is called the circumcentre of the triangle Can you reason out; why the name circumcentre for this point ? Recall that the locus of a point equidistant from two given points is the perpendicular bisector of the line joining the two points. Since O lies on the perpendicular bisector of BC, so it must be equidistant from both the point B and C, so that BO = CO (Fig. 15.13).

Fig. 15.13

The point O also lies on the perpendicular bisector of AC, so it must be equidistant from both A and C, that is, AO = CO. Thus, we have AO = BO = CO.

58

Mathematics

If we take O as the centre and AO as the radius, we can draw a circle passing through the three vertices, A, B and C of the triangle, called circumcircle of the triangle. O being the centre of this circle is called the circumcentre and AO the radius of the circumcircle is called circumradius of the triangle. Note that the circumcentre will be 1. in the interior of the triangle for an acute triangle (Fig. 15.11) 2. on the hypotenuse for a right triangle [(Fig. 15.12(a)] 3. in the exterior of the triangle for an obtuse triangle [(Fig. 15.12(b)]. 15.4.3 ALTITUDES OF A TRIANGLE In ABC, the line AL is the perpendicular drawn from vertex A to the opposite side BC. (Fig. 15.14)

Fig. 15.14

Perpendicular drawn from a vertex of a triangle to the opposite side is called its altitude. How many altitudes can be drawn in a triangle ? There are three vertices in a triangle, so we can draw three of its altitudes. AL is one of these altitudes. Now we draw the second altitude BM, which intersects the first altitude at a point H (see Fig. 15.15). We also draw the third altitude CN and observe that it also passes through the point H (Fig. 15.16). This shows that the three altitudes of the triangle pass through the same point.

Fig. 1515

Fig. 15.16

Concurrent Lines

59

We may take any type of triangle and draw its three altitudes. We always find that the three altitudes of a triangle are concurrent.

Fig. 15.17

Fig. 15.18

Thus we conclude that : In a triangle, the three altitudes pass through the same point, that is, they are concurrent. The point of concurrency is called the Orthocentre of the triangle. Again observe that the orthocentre will be 1. in the interior of the triangle for an acute triangle (Fig. 15.16) 2. in the exterior of the triangle for an obtuse triangle (Fig. 15.17) 3. at the vertex containing the right angle for a right triangle (Fig. 15.18) 15.4.4 MEDIANS OF A TRIANGLE In ABC, AD joins the vertex A to the mid point D of the opposite side BC (Fig. 15.19)

(a) Fig. 15.19

(b)

60

Mathematics

A line joining a vertex to the mid point of the opposite side of a triangle is called its median. Clearly, three medians can be drawn in a triangle, AD is one of the medians. If we draw all the three medians in any triangle, we always find that the three medians pass through the same point [Fig. 15.20 (a), (b), (c)]

(a)

(b) Fig. 15.20

(c)

Here in each of the triangles ABC given above (Fig. 15.20) the three medians AD, BE and CF are concurrent at G. In each triangle we measure the parts into which G divides each median. On measurement, we observe that AG = 2GD, BG = 2GE and CG = 2GF that is, the point of concurrency G divides each of the medians in the ratio 2 : 1 Thus we conclude that : Medians of a triangle pass through the same point, which divides each of the medians in the ratio 2 : 1. The point of concurrency G is called the centroid of the triangle. ACTIVITY FOR YOU Cut out a triangle from a piece of cardboard. Draw its three medians and mark the centroid G of the triangle. Try to balance the triangle by placing the tip of a pointed stick or a needle of compasses below the point G or at G. If the position of G is correctly marked then the weight of the triangle will balance at G (Fig. 15.21).

Fig. 15.21

Concurrent Lines

61

Can you reason out, why the point of concurrency of the medians of a triangle is called its centroid. It is the point where the weight of the triangle is centred or it is the point through which the weight of the triangle acts. We consider some examples using these concepts. Example 15.1 : In an isosceles triangle, show that the bisector of the angle formed by the equal sides is also a perpendicular bisector, an altitude and a median of the triangle. Solution : In ABD and ACD AB = AC BAD = CAD AD = AD ABD ACD BD = CD (Given) [Q AD is bisector of A]

AD is also a median Also ADB = ADC = 90


Fig. 15.22

AD is perpendicular bisector of side BC

[Q ADB = 90 AD is an altitude also] Example 15.2 : In an equilateral triangle, show that the three angle bisectors are also the three perpendicular bisectors of sides, three altitudes and the three medians of the triangle. Solution : Since AB = AC AD, the angle bisector of A is also a perpendicular bisector of BC, an altitude and a median of the ABC (Refer Example 1 above) Similarly, since AB = BC and BC = AC BE and CF, angle bisectors of B and C respectively, are also perpendicular bisectors, altitudes and medians of the ABC.
Fig. 15.23

Example 15.3 : Find the circumradius of circumcircle and inradius of incircle of an equilateral triangle of side a. Solution : We draw perpendicular from the vertex A to the side BC. AD is also the angle bisector of A, perpendicular bisector of side BC and a median joining vertex to the midpoint of BC

62

Mathematics

Fig. 15.24

AD =

3 a as BC = a. 2 2 3 3 a = a 3 2 3

AG = circumradius in this case =

and

GD = inradius in this case =

1 3 3 a = a. 3 2 6

CHECK YOUR PROGRESS 15.1 1. In the given figure if BF = FC, BAE = CAE and ADE = GFC = 90 then name a median, an angle bisector, and altitude and a perpendicular bisector of the triangle.

Fig. 15.25

2. In an equilateral triangle show that the incentre, the circumcentre, the orthocentre and the centroid are the same point. 3. In an equilateral ABC (Fig. 15.26), G is the centroid of the triangle. If AG is 4.8 cm, find BE.

Concurrent Lines

63

Fig. 15.26

4. If H is the orthocentre of ABC, then show that A is the orthocentre of the HBC. 5. Choose the correct answers out of the given alternatives in the following questions : (i) In a plane, the point equidistant from vertices of a triangle is called its (a) centroid (c) circumcentre (b) incentre (d) orthocentre

(ii) In the plane of a triangle, the point equidistant from the sides of the triangle is called its (a) entroid (c) circumcentre LET US SUM UP
z

(b) incentre (d) orthocentre

Three or more lines in a plane which intersect each other in exactly one point are called concurrent lines. A line which bisects an angle of a triangle is called an angle bisector of the triangle. A line which bisects a side of a triangle at right angle is called a perpendicular bisector of the triangle. A line drawn perpendicular from a vertex of a triangle to its opposite side is called an altitude of the triangle. A line which joins a vertex of a triangle to the mid-point of the opposite side is called a median. In a triangle (i) Angle bisectors are concurrent and the point of concurrency is called incentre. (ii) Perpendicular bisectors of the sides are concurrent and the point of concurrency is called circumcentre. (iii) Altitudes are concurrent and the point of concurrency is called orthocentre. (iv) Medians are concurrent and the point of concurrency is called centroid, which divides each of the medians in the ratio 2 : 1.

z z

64

Mathematics

TERMINAL EXERCISE 1. In the given Fig. 15.27 D, E and F are the mid points of the sides of ABC. Show that BE + CF >
3 BC. 2

Fig. 15.27

2. ABC is an isosceles triangle such that AB = AC and D is the midpoint of BC. Show that the centroid, the incentre, the circumcentre and the orthocentre, all lie on AD.

Fig. 15.28

3. ABC is an isosceles triangle such that AB = AC = 17 cm and base BC = 16 cm. If G is the centroid of ABC, find AG. 4. ABC is an equilateral triangle of side 12 cm. If G be its centroid, find AG. ACTIVITIES FOR YOU : 1. Draw a triangle ABC and find its circumcentre. Also draw the circumcircle of the triangle. 2. Draw an equilateral triangle. Find its incentre and circumcentre. Draw its incircle and circumcircle. 3. Draw the circumcircle and the incircle for an equilateral triangle of side 5 cm.

Concurrent Lines

65

ANSWERS Check Your Progress 15.1 1. MedianAF, Angle bisector AE Altitude AD and perpendicular bisector GF 3. AD = 7.2 cm also BE = 7.2 cm 5. (i) (c) Terminal Exercise 3. AG = 10 cm 4. AG = 4 3 cm. (ii) (b)

66

Mathematics

16
Quadrilaterals
16.1 INTRODUCTION If you look around, you will find many objects bounded by four lines. A book, window door, some parts of window-grill, slice of bread, the floor of your room are all examples of a closed figure bounded by four line segments, such a figure is called a quadrilateral. The word quadrilateral has its origin from the two words quadric meaning four and lateral meaning sides. Thus, a quadrilateral is that geometrical figure which has four sides, enclosing a part of the plane. In this lesson, we shall study about terms and concepts related to quadrilateral with their properties. 16.2 OBJECTIVES After studying this lesson, the learner will be able to :
z

describe various types of quadrilaterals viz. trapeziums, parallelograms, rectangles, rhombuses and squares. verify properties of different types of quadrilaterals. verify that in a triangle the line segment joining the mid-points of any two sides is parallel to the third side and is half of it. verify that the line drawn through the mid-point of a side of a triangle parallel to another side bisects the third side. verify that if there are three or more parallel lines and the intercepts made by them on a transversal are equal, the corresponding intercepts on any other transversal are also equal. verify that a diagonal of a parallelogram divides it into two triangles of equal area. prove that parallelograms on equal (or same) bases and between the same parallels are equal in area. verify that triangles on the same or equal bases and between the same parallels are equal in area and its converse.

z z

z z

Quadrilaterals

67

16.3 EXPECTED BACKGROUND KNOWLEDGE


z z z z

Drawing line-segments and angles of given measure. Drawing circles/arcs of given radius. Drawing parallel and perpendicular lines. Four fundamental operations on numbers.

16.4 QUADRILATERAL Recall that if A, B, C and D are four points in a plane such that no three of them are collinear and the line segment AB, BC, CD and DA do not intersect except at their end points, then the closed figure made up of four line segments is called a quadrilateral with vertices A, B, C and D. A quadrilateral with vertices A, B, C and D is generally denoted by quad. ABCD. In Fig. 16.1 (i) and (ii), both the quadrilaterals can be named as quad. ABCD. In quadrilateral ABCD,

(i)
Fig. 16.1

(ii)

(i) AB and DC ; BC and AD are two pairs of opposite sides. (ii) A and C ; B and D are two pairs of opposite angles. (iii) AB and BC ; BC and CD are two pairs of consecutive sides or adjacent sides. Can you name the other pairs of consecutive sides ? (iv) A and B ; B and C are two pairs of consecutive angles or adjacent angles. Can you name the other pairs of consecutive angles ? (v) AC and BD are the two diagonals. In Fig. 16.2, angles denoted by 1, 2, 3 and 4 are the interior angles or the angles of the quad. ABCD. Angles denoted by 5, 6, 7 and 8 are the exterior angles of the quad. ABCD. Measure 1, 2, 3 and 4

68

Mathematics

(i)
Fig. 16.2

(ii)

What is the sum of these angles ? You will find that 1 + 2 + 3 + 4 = 360. i.e. sum of interior angles of a quadrilateral equals 360. Also what is the sum of exterior angles of the quadrilateral ABCD ? You will again find that 5 + 6 + 7 + 8 = 360 i.e., sum of exterior angels of a quadrilateral is also 360. 16.5 TYPES OF QUADRILATERALS You are familiar with quadrilaterals and their different shapes. You also know how to name them. However, we will now study different types of quadrilaterals in a systematic way. A family tree of quadrilaterals is given in Fig. 16.3 below : Quadrilateral Parallelogram Rectangle Square
Fig. 16.3

Trapezium

Kite

Rhombus

Let us describe them one by one. 16.5.1 Trapezium A quadrilateral which has one pair of opposite sides parallel is called a trapezium. In Fig. 16.4 [(i) and (ii)] ABCD and PQRS are trapeziums with AB || DC and PQ || SR respectively.

Quadrilaterals

69

(i)
Fig. 16.4

(ii)

16.5.2 Parallelogram A quadrilateral which has both pairs of opposite sides parallel, is called a parallelogram. In Fig. 16.5 [(i) and (ii)] ABCD and PQRS are parallelograms with AB||DC and AD||BC. These are denoted by ||gm ABCD and ||gm PQRS.

(i)
Fig. 16.5

(ii)

16.5.3 Rhombus A rhombus is a parallelogram in which any pair of adjacent sides is equal. In Fig. 16.6 ABCD is a rhombus.

Fig. 16.6

70

Mathematics

You may note that ABCD is a parallelogram with AB = BC = CD = DA i.e., each pair of adjacent sides is equal. 16.5.4 Rectangle A parallelogram one of whose angles is a right angle is called a rectangle. In Fig. 16.7, ABCD is a rectangle in which AB||DC, AD||BC and A = B = C = D = 90.

Fig. 16.7

16.5.5 Square A square is a rectangle, with a pair of adjacent sides equal. In other words, a parallelogram having all sides equal and each angle equal to a right angle is called a square.

Fig. 16.8

In Fig. 16.8, ABCD is a square in which AB||DC, AD||BC, and AB = BC = CD = DA and A = B = C = D = 90. Let us take some examples to illustrate.

Quadrilaterals

71

Example 16.1 : In Fig. 16.9, PQR is a triangle. S and T are two points on the sides PQ and PR such that ST||QR. Name the type of quadrilateral STRQ so formed.

Fig. 16.9

Solution : Quadrilateral STRQ is a trapezium, because ST||QR. Example 16.2 : The three angles of a quadrilateral are 100, 50 and 70. Find the measure of the fourth angle. Solution : We know that the sum of the angles of a quadrilateral is 360. Let the fourth angle be x. Then 100 + 50 + 70 + x = 360 220 + x = 360 x = 140 Hence, the measure of fourth angle is 140. CHECK YOUR PROGRESS 16.1 1. Name each of the following quadrilaterals.

(i)

(ii)

(iii)

(iv)
Fig. 16.10

(v)

(vi)

72

Mathematics

2. States which of the following statements are correct ? (i) Sum of interior angles of a quadrilateral is 360. (ii) All rectangles are squares. (iii) A rectangle is a parallelogram. (iv) A square is a rhombus. (v) A rhombus is a parallelogram. (vi) A square is a parallelogram. (vii) A parallelogram is a rhombus. (viii) A trapezium is a parallelogram. (ix) A trapezium is a rectangle. (x) A parallelogram is a trapezium. 3. In a quadrilateral, all its angles are equal. Find the measure of each angle. 4. The angles of a quadrilateral are in the ratio 5 : 7 : 7 : 11. Find the measure of each angle. 5. If a pair of opposite angles of a quadrilateral are supplementary, what can you say about the other pair of angles ? 16.6. PROPERTIES OF DIFFERENT TYPES OF QUADRILATERALS 16.6.1 Properties of a Parallelogram We have learnt that a parallelogram is a quadrilateral with both pairs of opposite sides parallel. Now let us establish some relationship between sides, angles and diagonals of a parallelogram. Draw a pair of parallel lines l and m as shown in Fig. 16.11. Draw another pair of parallel lines p and q such that they intersect l and m. You observe that a parallelogram ABCD is formed. Join AC and BD. They intersect each other at O.

(i)
Fig. 16.11

(ii)

Quadrilaterals

73

Now measure the sides AB, BC, CD and DA. What do you find ? You will find that AB = DC and BC = AD. Also measure ABC, BCD, CDA and DAB. What do you find ? You will find that DAB = DCB and ABC = CDA Again, Measure OA, OC, OB and OD. What do you find ? You will find that OA = OC and OB = OD

Draw another parallelogram and repeat the activity you will find that (i) The opposite sides of a parallelogram are equal. (ii) The opposite angles of a parallelogram are equal. (iii) The diagonals of a parallelogram bisect each other. The above said properties of a parallelogram can also be verified by Cardboard model which is as follows : Let us take a card board. Draw any parallelogram ABCD on it. Draw its diagonal AC as shown in Fig 16.12. Cut the parallelogram ABCD from the cardboard. Now cut this parallelogram along the diagonal AC. Thus, the parallelogram has been divided into two parts and each part is a triangle.

(i)
Fig. 16.12

(ii)

In other words, you get two triangles, ABC and ADC. Now place ADC on ABC in such a way that the vertex D falls on the vertex B and the side CD falls along the side AB. Where does the point C fall ? Where does the point A fall ? You will observe that ADC will coincide with ABC. In other words ABC ADC. Also AB = CD and BC = AD and B = D.

74

Mathematics

You may repeat this activity by taking some other parallelograms, you will always get the same results as verified earlier, thus, proving the above two properties of the parallelogram. Now you can prove the third property of the parallelogram, i.e., the diagonals of a parallelogram bisect each other. Again take a thin cardboard. Draw any parallelogram PQRS on it. Draw its diagonals PR and QS which intersect each other at O as shown in Fig. 16.13. Now cut the parallelogram PQRS.

Fig. 16.13

Also cut POQ and ROS. Now place ROS and POQ in such a way that the vertex R coincides with the vertex P and RO coincides with the side PO. Where does the point S fall ? Where does the side OS fall ? Is ROS POQ ? Yes, it is. So, what do you observe ? We find that RO = PO and OS = OQ You may also verify this property by taking another pair of triangles i.e. POS and ROQ. You will again arrive at the same result. You may also verify the following properties which are the converse of the properties of a parallelogram proved earlier. (i) A quadrilateral is a parallelogram if its opposite sides are equal. (ii) A quadrilateral is a parallelogram if its opposite angles are equal. (iii) A quadrilateral is a parallelogram if its diagonals bisect each other. 16.6.2 Properties of a Rhombus In the previous section we have defined a rhombus. We know that a rhombus is a parallelogram in which a pair of adjacent sides are equal. In Fig. 16.14, ABCD is a rhombus.

Quadrilaterals

75

Fig. 16.14

Thus, ABCD is a parallelogram with AB = BC. Since every rhombus is a parallelogram, therefore all the properties of a parallelogram are also true for rhombus, i.e. (i) Opposite sides are equal, i.e., AB = DC and AD = BC (ii) Opposite angles are equal, i.e., A = C and B = D (iii) Diagonals bisect each other i.e., AO = OC and DO = OB Since adjacent sides of a rhombus are equal and by the property of a parallelogram AB = BC = CD = DA Thus, all the sides of a rhombus are equal. Measure AOD and BOC. What is the measures of these angles ? You will find that each of them equals 90 Also AOB = COD and BOC = DOA AOB = COD = BOC = DOA = 90 Thus, the diagonals of a rhombus bisect each other at right angles. You may repeat this experiment by taking different rhombuses, you will find in each case, the diagonals of a rhombus bisect each other. Thus, we have the following properties of a rhombus. (i) All sides of a rhombus are equal (ii) The opposite angles of a rhombus are equal (iii) The diagonals of a rhombus bisect each other at right angles. (Each pair is a vertically opposite angles)

76

Mathematics

16.6.3 Properties of a Rectangle We know that a rectangle is a parallelogram one of whose angles is a right angle. Can you say whether a rectangle possesses all the properties of a parallelogram or not ? Yes it is. Let us study some more properties of a rectangle Draw a parallelogram ABCD in which B = 90. Join AC and BD as shown in the Fig. 16.15

Fig. 16.15

Measure AD, BCD and ADC, what do you find ? What are the measures of these angles ? The measure of each angle is 90. Thus, we can conclude that A = B = C = D = 90 i.e., each angle of a rectangle measures 90. Now measure the diagonals AC and BD. Also measure AO, OC, BO and DO. Do you find that AC = BD ? Yes, it is You will also find that AO = OC and BO = DO. Draw some more rectangles of different dimensions. Label them again by ABCD. Join AC and BD in each case. Let them intersect each other at O. Also measure AO, OC and BO, OD for each rectangle. In each case you will find that The diagonals of a rectangle are equal and they bisect each other. Thus, we have the following properties of a rectangle : (i) The opposite sides of a rectangle are equal (ii) Each angle of a rectangle is a right-angle. (iii) The diagonals of a rectangle are equal. (iv) The diagonals of a rectangle bisect each other. 16.6.4 Properties of a Square You know that a square is a rectangle, with a pair of adjacent sides equal. Now, can you conclude from definition of a square that a square is rectangle and possesses all the properties of a rectangle ? Yes it is, Let us now study some more properties of a square.

Quadrilaterals

77

Draw a square ABCD as shown in Fig. 16.16.

Fig. 16.16

Since ABCD is a rectangle, therefore we have (i) AB = DC, AD = BC (ii) A = B = C = D = 90 (iii) AC = BD and AO = OC, BO = OD But in a square we have AB = AD By property (i) we have AB = AD = CD = BC. Now measure AOB, BOC, COD and AOD. What do you find ? Does each angle measure 90 ? Yes Thus, AOB = BOC = COD = AOD = 90 Thus, we conclude that the diagonals AC and BD of a square bisect each other at right angles. You may also observe that since a square is a parallelogram also with AB = AD, therefore a square ABCD is a rhombus also. Thus, we have the following properties of a square (i) All the sides of a square are equal. (ii) Each of the angles measure 90. (iii) The diagonals of a square are equal. (iv) The diagonals of a square bisect each other at right angles. Let us study some examples to illustrate :

78

Mathematics

Example 16.3 : In Fig. 16.17, ABCD is a parallelogram. If A = 80, find the measures of the remaining angles. Solution : As ABCD is a parallelogram. It is given that A = 80
Q

A = C and B = D.

C = 80. AB || DC A + D = 180 D = (180 80) = 100 B = D = 100 C = 80, B = 100 and D = 100.
Fig. 16.17

Hence,

Example 16.4 : Two adjacent angles of a rhombus are in the ratio 4 : 5. Find the measure of all its angles. Solution : We know that the sum of two adjacent angles of a rhombus is 180. Let the two angels be 4x and 5x, then i.e. 4x + 5x = 180 9x = 180 x = 20 The two angles are 80 and 100 i.e. Also, A = 80 and B = 100
Fig. 16.18

Since A = C C = 80 B = D D = 100 Hence, the angles of the rhombus are 80, 100, 80 and 100.

Example 16.5 : One of the diagonals of a rhombus is equal to one of its sides. Find the angles of the rhombus. Solution : In rhombus, ABCD, AB = AD = BD ABD is an equilateral triangle. DAB = 1 = 2 = 60 ...(i) ...(ii)
Fig. 16.19

Similarly, BCD = 3 = 4 = 60

Quadrilaterals

79

Also from (i) and (ii) ABC = B = 1 + 3 = 60 + 60 = 120 ADC = D = 2 + 4 = 60 + 60 = 120 Hence, A = 60, B = 120, C = 60 and D = 120. Example 16.6 : The diagonals of a rhombus ABCD intersect at O. If ADC = 120 and OD = 6 cm, find (i) OAD (ii) side AB (iii) perimeter of the rhombus ABCD. Solution : Given that ADC = 120 i.e., ADO + ODC = 120 But i.e. Now, in DOA ADO + DOA + OAD = 180 From (i) and (ii), we have 60 + 90 + OAD = 180 OAD = 30 DAB = 60 ADO = ODC 2ADO = 120 ADO = 60 DOA = 90 ...(i) ...(ii) (AOD COD)
Fig. 16.20

Also, we know that the diagonals of a rhombus bisect each that at 90.

DAB is an equilateral triangle (ii) Now OD = 6 cm Since, OD + OB = BD 6 cm + 6 cm = BD BD = 12 cm AB = BD = AD = 12 cm AB = 12 m. = (4 12) cm = 48 cm Hence, the perimeter of the rhombus = 48 cm.

(iii) Now Perimeter = 4 side

80

Mathematics

CHECK YOUR PROGRESS 16.2 1. In a parallelogram ABCD, A = 62. Find the measures of the other angles. 2. The sum of the two opposite angles of a parallelogram is 150. Find all the angles of the parallelogram. 3. In a parallelogram ABCD, A = (2x + 10) and C = (3x 20). Find the value of x. 4. ABCD is a parallelogram in which DAB = 70 and CBD = 55. Find CDB and ADB. 5. ABCD is a rhombus in which ABC = 58. Find the measure of ACD. 6. In Fig. 16.21, the diagonals of a rectangle PQRS intersect each other at O. If ROQ = 40, find the measure of OPS. 7. AC is one diagonal of a square ABCD. Find the measure of CAB. 16.7 MID POINT THEOREM Draw any triangle ABC. Find the mid points of the side AB and AC. Mark them as D and E respectively. Join DE, as shown in Fig. 16.22. Measure BC and DE. What relation do you find between the length of BC and DE ? Of course, it is, DE =
1 BC 2
Fig. 16.22 Fig. 16.21

Again, measure ADE and ABC. Are these angles equal ?

Yes, they are equal, you know that these angles make a pair of corresponding angles. You know that when a pair of corresponding angles are equal, the lines are parallel. DE || BC

You may repeat this experiment with another two or three triangles and naming each of them as triangle ABC and the mid points as D and E of sides AB and AC. You will always find that DE = Thus, we conclude that
1 BC and DE || BC. 2

Quadrilaterals

81

In a triangle the line-segment joining the mid points of any two sides is parallel to the third side and is half of it. We can also verify the converse of the above stated result. Draw any PQR. Find the mid point of side RQ, and mark it as L. From L, draw a line LX || PQ, which intersects, PR at M. Measure PM and MR. Are they equal ? Yes, they are equal. You may repeat with different triangles and by naming each of them as PQR and taking each time L as the midpoint RQ and drawing a line LM || PQ, you will find in each case that RM = MP. Thus, we conclude that

Fig. 16.23

The line drawn through the mid point of one side of a triangle parallel to the another side bisects the third side. Let us consider some examples. Example 16.7 : In Fig. 16.24, D is the mid-point of the side AB of ABC and DE || BC. If AC = 8 cm, find AE. Solution : In ABC, DE || BC and D is the mid point of AB E is also the mid point of AC i.e. AE =
1 AC 2

FG 1 8IJ H2 K

cm

[Q AC = 8 cm]

Fig. 16.24

= 4 cm Hence AE = 4 cm. Example 16.8 : In Fig. 16.25, ABCD is a trapezium in which AD and BC are its non-parallel sides and E is the mid-point of AD. EF||AB. Show that F is the mid-point of BC. Solution : Since EG||AB and E is the mid-point of AD G is the mid point of DB In DBC, GF||DC and G is the mid-point of DB, F is the mid-point of BC.
Fig. 16.25

82

Mathematics

Example 16.9 : ABC is a triangle, in which P, Q and R are mid-points of the sides AB, BC and CA respectively. If AB = 8 cm, BC = 7 cm, and CA = 6 cm, find the sides of the triangle PQR. Solution : P is the mid-point of AB and R is the mid-point of AC PR||BC and PR = =
1 BC 2 1 7 2 1 AC 2 1 6 cm 2

[Q BC = 7 cm]

= 3.5 cm Similarly, PQ = =

[Q AC = 6 cm]

Fig. 16.25

= 3 cm and QR = =
1 AB 2 1 8 cm 2

[Q

AB = 8 cm]

= 4 cm. Hence, the sides of PQR are PQ = 3 cm, QR = 4 cm and PR = 3.5 cm. CHECK YOUR PROGRESS 16.3 1. In Fig. 16.27, ABC is an equilateral triangle D, E and F are the mid-points of the sides AB, BC and CA respectively. Prove that DEF is also an equilateral triangle.

Fig. 16.27

Quadrilaterals

83

2. In Fig. 16.28, D and E are the mid-points of the sides AB and AC respectively of a ABC. If BC = 10 cm; find DE.

Fig. 16.28

3. In Fig. 16.29, AD is a median of a ABC and E is the mid-point of AD. BE is produced to meet AC at F. DG||EF, meets AC at G. If AC = 9 cm, find AF.

Fig. 16.29

4. In Fig. 16.30, A and C divide the side PQ of PQR into three equal parts. AB||CD||QR. Prove that B and D also divide PR into three equal parts.

Fig. 16.30

5. In Fig. 16.31, ABC is a isosceles triangle in which AB = AC. M is the mid-point of AB and MN||BC. Show that AMN is also an isosceles.

84

Mathematics

Fig. 16.31

16.8 INTERCEPT THEOREM Recall that a line which intersects two or more lines is called a transversal. The segment cut off from the transversal by a pair of lines is called an intercept. Thus, in Fig. 16.32, XY is an intercept made by line l and m on transversal n.

Fig. 16.32

The intercepts made by parallel lines on a transversal have some special properties which we shall study here. Let l and m be two parallel lines and XY is an intercept made on the transversal n. If there are three parallel lines and they are intersected by a transversal, there will be two intercepts AB and BC as shown in Fig. 16.33 (ii).

(i)
Fig. 16.33

(ii)

Quadrilaterals

85

Now let us learn an important property of intercepts made on the transversals by the parallel lines. On a page of your note-book, draw any two transversals l and m intersecting the parallel lines p, q, r and s as shown in Fig. 16.34. These transversal make different intercepts. Measure the intercept AB, BC and CD. Are they equal ? Yes, they are equal.

Fig. 16.34

Also, measure LM, MN and NX. Do you find that they are also equal ? Yes, they are. Repeat this experiment by taking another set of two or more equidistant parallel lines and measure their intercepts as done earlier. You will find in each case that the intercepts made are equal. Thus, we conclude the following : If there are three or more parallel lines and the intercepts made by them on a transversal are equal, the corresponding intercepts made on any other transversal are also equal. Let us illustrate it by some examples : Example 16.10 : In Fig 16.35, p || q || r. The transversals l, m and n cut them at L, M, N; A, B, C and X, Y, Z respectively such that XY = YZ. Name the other pairs of equal intercepts. Solution. Given that XY = YZ and AB = BC LM = MN (Intercept theorem)

Thus, the other pairs of equal intercepts are AB = BC and LM = MN.


Fig. 16.35

86

Mathematics

Example 16.11. In Fig. 16.36, l || m || n and PQ = QR. If XZ = 20 cm, find YZ. Solution. We have PQ = QR By intercept theorem, XY = YZ Also XZ = XY + YZ = YZ + YZ 20 = 2YZ Hence, YZ = 10 cm. CHECK YOUR PROGRESS 16.4 1. In Fig 16.37, l, m and n are three equidistant parallel lines. AD, PQ and GH are three transversals. If BC = 2 cm and LM = 2.5 cm and AD ||PQ, find MS and MN. YZ = 10 cm
Fig. 16.36

Fig. 16.37

2. From Fig 16.38, when can you say that AB = BC and XY = YZ ?

Fig. 16.38

Quadrilaterals

87

3. In Fig. 16.39, LM = MZ = 3 cm, find XY, XP and BZ. Given that l || m || n and PQ = 3.2 cm, AB = 3.5 cm and YZ = 3.4 cm.

Fig. 16.39

16.9 THE DIAGONAL OF A PARALLELOGRAM AND DIVISION OF AREA Draw a parallelogram ABCD. Join its diagonal AC. Draw DP DC and QC DC. Consider the two triangles ADC and ACB in which the parallelogram ABCD has been divided by the diagonal AC. Because AB || DC, therefore PD = QC. Now, Area ADC = Area of ACB = As
1 DC PD 2 1 AB QC 2

...(i)

...(ii)
Fig. 16.40

AB = DC and PD = QC

Area (ADC) = Area (ACB)

Thus, we conclude the following : A diagonal of a parallelogram divides it into two triangles of equal area.

16.10 PARALLELOGRAMS AND TRIANGLES BETWEEN THE SAME PARALLELS Two parallelograms or triangles, having equal or same bases and having their other vertices on a line parallel to their bases, are said to be on the same or equal bases and between the same parallels. We will prove an important theorem on parallelogram and their area Theorm : Parallelograms on the same base (or equal bases) and between the same parallels are equal in area. Let us prove it logically.

88

Mathematics

Given : Parallelograms ABCD and PBCQ stand on the same base BC and between the same parallels BC and AQ. To prove : Area (ABCD) = Area (BCQP) Proof : Consider two triangles ABP and DCQ, we have and Now, AB = DC BP = CQ 1 = 2 ABP DCQ Area (ABP) = Area (DCQ) ...(i) ...(ii) ...(iii)
Fig. 16.41

(Opposite sides of a parallelogram) (Opposite sides of a parallelogram) (Corresponding Angles)

Area (||gm ABCD) = Area(ABP) + Area(Trap. BCDP) Area (||gm BCQP) = Area (CQD) + Area(Trap. BCDP)

From (i), (ii) and (iii), we get Area (||gm ABCD) = Area (||gm BCQP) Note : ||gm and Trap. stands for parallelogram and trapezium respectively. Result : Triangles, on the same base and between the same parallels,are equal in area Consider Fig. 16.41. Join the diagonals BQ and AC of the two parallelograms BCQP and ABCD respectively. We know that a diagonal of a ||gm divides it in two triangles of equal area. and Area (BCQ) = Area (PBQ) Area(ABC) = Area (CAD) Area (ABC) = Area (BCQ) Parallelogram on the same base (or equal bases) and between the same parallels are equal in area. Thus, we also conclude the following : Triangles on the same base (or equal bases) and between the same parallels are equal in area. 16.11 TRIANGLES ON THE SAME OR EQUAL BASES HAVING EQUAL AREAS HAVE THEIR CORRESPONDING ALTITUDES EQUAL Recall that the area of triangle =
1 (Base) Altitude 2

Quadrilaterals

89

Fig. 16.42

Here and

BC = QR Area (ABC) = Area (DBC) = Area (PQR)


1 BC DE 2 1 BC DE 2

[Given]

...(i)

Draw perpendiculars DE and PS from D and P to the line m meeting it in E and S respectively. Now Area (ABC) = Area (DBC) = and Area (PQR) =

...(ii)

1 QR PS 2 Also, BC = QR From (i), (ii) and (iii), we get 1 1 BC DE = QR PS 2 2

(Given)

...(iii)

or,

1 1 BC DE = BC PS 2 2 DE = PS

i.e., Altitudes of ABC, DBC and PQR are equal in length. Thus, we conclude the following : Triangles on the same or equal bases, having equal areas have their corresponding altitudes equal. Let us consider some examples : Example 16.12 : In Fig. 16.43, the area of parallelogram ABCD is 40 sq cm. If BC = 8m, find the altitude of parallelogram BCEF. Solution : or or, Area of ||gm BCEF = Area of ||gm ABCD = 40 sq cm 40 = BC Altitude of ||gm BCEF 40 = 8 Altitude of ||gm BCEF
40 cm or 5 cm. 8

...(i)

we know that Area (||gm BCEF) = EF Altitude

Altitude of ||gm BCEF =

Fig. 16.43

90

Mathematics

Example 16.13 : In Fig. 16.44, the area of ABC is given to be 18 cm2. If the altitude DL equals 4.5 cm, find the base of the BCD. Solution : Area (BCD) = Area (ABC) = 18cm2 Let the base of BCD be x cm Area of BCD = =
1 x DL 2
2

or

FG 1 x 4.5IJ cm H2 K F 9 xIJ 18 = G H4 K F18 4 IJ cm = 8 cm. x= G H 9K

Fig. 16.44

Example 16.14 : In Fig. 16.45, ABCD and ACED are two parallelograms. If area of ABC equals 12 cm2, and the length of CE and BC are equal, find the area of the trapezium ABED. Solution : Area(||gm ABCD) = Area(||gm ACED) The diagonal AC divides the ||gm ABCD into two triangles of equal area. Area (ABC) =
1 Area (||gm ABCD) 2

Fig. 16.45

Area (||gm ABCD)=Area (||gm ACED) = 2 12 cm2 = 24 cm2 Area of Trapezium ABED = Area (ABC) + Area (||gm ACED) = (12 + 24) cm2 = 36 cm2. CHECK YOUR PROGRESS 16.5 1. When are two parallelograms on the same base (or equal bases) of equal areas ? 2. The area of a triangle formed by joining the diagonal AC of a ||gm ABCD is 16 cm2. Find the area of the ||gm ABCD.

Quadrilaterals

91

3. The area of ACD in Fig 16.46 is 8 cm2. If EF = 4 cm, find the altitude of ||gm BCFE.

Fig. 16.46

LET US SUM UP
z z z z

A quadrilateral is a four sided closed figure, enclosing some area of the plane. The sum of the interior or exterior angles of a quadrilateral is each equal to 360. A quadrilateral is a trapezium if its one pair of opposite sides is parallel.

A quadrilateral is a parallelogram if both pair of sides are parallel. z In a parallelogram : (i) opposite sides and angles are equal. (ii) diagonals bisect each other. z A parallelogram is a rhombus if its adjacent sides are equal. z The diagonals of a rhombus bisect each other at right angle. z A parallelogram is a rectangle if its one angle is 90. z The diagonals of a rectangle are equal. z A rectangle is a square if its adjacent sides are equal. z The diagonals of a square intersect at right angles. z The diagonal of a parallelogram divides it into two triangles of equal area. z Parallelogram on the same base (or equal bases) and between the same parallels are equal in area. z The triangles on the same base (or equal bases) and between the same parallels are equal in area. z Triangles on equal bases having equal areas have their corresponding altitudes equal. TERMINAL EXERCISE 1. Which of the following are trapeziums ?

(i)
Fig. 16.47

(ii)

(iii)

92

Mathematics

2. In Fig. 16.48, PQ || FG || DE || BC. Name all the trapeziums in the figure.

Fig. 16.48

3. In Fig. 16.49, ABCD is a parallelogram with an area of 48 cm2. Find the area of (i) shaded region (ii) unshaded region.

Fig. 16.49

4. Fill in the blanks in each of the following to make them true statements : (i) A quadrilateral is a trapezium if ... (ii) A quadrilateral is a parallelogram if ... (iii) A rectangle is a square if ... (iv) The diagonals of a quadrilateral bisect each other at right angle. If none of the angles of the quadrilateral is a right angle, it is a ... (v) The sum of the exterior angles of a quadrilateral is ... 5. If the angles of a quadrilateral are (x 20), (x + 20), (x 15) and (x + 15), find x and the angles of the quadrilateral. 6. The sum of the opposite angles of a parallelograms is 180. What special type of a parallelogram is it ?

Quadrilaterals

93

7. The area of a ABD in Fig. 16.50 is 24 cm2. If DE = 6 cm, and AB || CD, BD || CE, AE || BC, find

Fig. 16.50

(i) Altitude of the parallelogram BCED. (ii) Area of the parallelogram BCED 8. In Fig. 16.51, the area of parallelogram ABCD is 40 cm2. If EF = 8 cm, find the altitude of DCE.

Fig. 16.51

94

Mathematics

ANSWERS Check Your Progress 16.1 1. 2. (i) Rectangle (v) Rhombus (i) true (v) True (ix) False 3. 90 (ii) trapezium (vi) Square (ii) False (vi) True (x) False 4. 60, 84 84 and 132 (iii) True (vii) False (iv) True (viii) False (iii) Rectangle (iv) Parallelogram

5. Other pair of opposite angles will also be supplementary Check Your Progress 16.2 1. B = 118, C = 62 and D = 118 2. A = 105, B = 75, C = 105 and D = 75 3. 30 5. ACD = 61 Check Your Progress 16.3 2. 5 cm Check Your Progress 16.4 1. MS = 2 cm and MN = 2.5 cm 2. l, m and n are three equidistant parallel lines 3. XY = 3.4 cm, XP = 3.2 cm and BZ = 3.5 cm Check Your Progress 16.5 1. When they are lying between the same parallel lines 2. 32 cm2 Terminal Exercise 1. (i) and (iii) 2. PFGQ, FDEG, DBCE, PDEQ, FBCG and PBCQ 3. (i) 4. 24 cm2 (ii) 24 cm2 (i) any one pair of opposite sides is parallel (ii) both pairs of opposite sides are parallel (iii) pair of adjacent sides is equal 6. It is a rectangle 7. (i) 8 cm (ii) 48 cm2 8. 5 cm (iv) rhombus (v) 360 5. x = 90, angles are 70, 110, 75 and 105 respectively. 3. 4 cm 3. 3 cm 6. OPS = 70 4. CDB = 55 and ADB = 55 7. CAB = 45

Similarity of Triangle

95

17
Similarity of Triangle
17.1 INTRODUCTION Looking around you will see many objects which are of the same shape but of same or different sizes. For examples, leaves of a tree have almost the same shape but same or different sizes. Similarly, photographs of different sizes developed from the same negative are of same shape but different sizes, the miniature model of a building and the building itself are of same shape but different sizes. All those objects which have the same shape but different sizes are called similar objects. Let us examine the similarity of plane figures : (i) Two line-segments of the same length are congruent but of different lengths are similar.

(ii) Two circles of the same radius are congurent but circles of different radii are similar.

(iii) Two equilateral triangles of different sides are similar.

96

Mathematics

(iv) Two squares of different sides are similar.

In this lesson, we shall study about the concept of similarity, especially similarity of triangles and the conditions thereof. We shall also study about various results related to them. 17.2 OBJECTIVES After studying this lesson, the learner will be able to :
z z z z z z

identify similar figures distinguish between congurent and similar plane figures cite the criteria for similarity of triangles viz. AAA, SSS and SAS. verify and use unstarred results given in the curriculum based on similarity experimentally prove the Baudhayan/Pythagoras Theorem apply these results in verifying experimentally (or proving logically) problems based on similar triangles.

17.3 EXPECTED BACKGROUND KNOWLEDGE Knowledge of


z z z z z

plane figures like triangles, quadrilaterals, circles, rectangles, squares, etc. criteria of congruency of triangles finding squares and square-roots of numbers ratio and proportion internal and external bisectors of angles of a triangle.

17.4 SIMILAR PLANE FIGURES

Fig. 17.2

Similarity of Triangle

97

In Fig. 17.2, the two pentagon seem to be of the same shape. We can see that A = A, B = B, C = C, D = D and E = E and AB = BC = CD = DE = EA . We say that the two pentagons are similar. Thus we say A ' B' B' C' C' D' D' E' E' A ' that Any two polygons, with corresponding angles equal and corresponding sides proportional, are similar Thus, two polygons are similar, if they satisfy the following two conditions : (i) Corresponding angles are equal (ii) The corresponding sides are proportional. Even if one of the conditions does not hold, the polygons are not similar as in the case of a rectangle and square given in Fig. 17.3. Here all the corresponding angles are equal but the corresponding sides are not proportional.

Fig. 17.3

17.5 SIMILARITY OF TRIANGLES Triangles are special type of polygons and therefore the conditions of similarity of polygons also hold for triangles. Thus, Two triangles are similar if (i) their corresponding angles are equal, and (ii) their corresponding sides are proportional

Fig. 17.4

We say that ABC is similar to DEF and denote it by writing ABC ~ DEF

98

Mathematics

The symbol ~ stands for the phrase is similar to If ABC ~ DEF, then by definition A = D, B = E, D = F and 17.5.1 AAA criterion for similarity We shall show that if either of the above two conditions is satisfied then the other automatically holds in the case of triangles. Let us perform the following experiment. Construct two s ABC and PQR in which P = A, Q = B and R = C as shown in Fig. 17.5. AB = BC = CA DE EF FD

Fig. 17.5

Measure the sides AB, BC and CA of ABC and also measure the sides PQ, QR and RP of PQR.

AB BC CA Now find the ratio PQ , QR and . RP


What do you find ? You will find that all the three ratios are equal and therefore the triangles are similar. Try this with different triangles with equal corresponding angles. You will find the same result. Thus, we can say that If in two triangles, the corresponding angles are equal the triangles are similar. This is called AAA similarity criterion. 17.5.2 SSS criterion for similarity. Let us now perform the following experiment : Draw a triangle ABC with AB = 3 cm, BC = 4.5 cm and CA = 3.5 cm

Similarity of Triangle

99

(i) Fig. 17.6

(ii)

Draw another PQR as shown in Fig. 17.6 (ii)

AB BC AC We can see that PQ = QR = PR


i.e., the sides of the two triangles are proportional Now measure A, B and C of ABC and P, Q and R of PQR. You will find that A = P, B = Q and C = R. Repeat the experiment with another two triangles having corresponding sides proportional, you will find that the corresponding angles are equal and so the triangle are similar. Thus, we can say that If the corresponding sides of two triangles are proportional the triangles are similar. 17.5.3 SAS Criterian for Similarity Let us conduct the following experiment. Take a line AB = 3 cm and at A construct an angle of 60. Cut off AC = 4.5 cm. Join BC

Fig. 17.7

Now take PQ = 6 cm. At P, draw an angle of 60 and cut off PR = 9 cm. Measure B, C, Q and R. We shall find that B = Q and C = R Thus, ABC ~ PQR

100

Mathematics

Thus, we conclude that If one angle of a triangle is equal to one angle of the other triangle and the sides containing these angles are proportional, the triangles are similar. Thus, we have three important criteria for the similarity of triangles. They are given below: (i) If in two triangles, the corresponding angles are equal, the triangles are similar. (ii) If the corresponding sides of two triangles are proportional, the triangles are similar. (iii) If one angle of a triangle is equal to one angle of the other triangle and the sides containing these angles are proportional, the triangle are similar. Example 17.1 : In Fig. 17.8 are given two triangles ABC and PQR

Fig. 17.8

Is ABC ~ PQR ? Solution : We are given that A = P and B = Q We also know that A + B + C = P + Q + R = 180 Thus, according to first criterion of similarity ABC ~ PQR. Example 17.2 :

Fig. 17.9

In Fig. 17.9, ABC ~ PQR. If AC = 4.8 cm, AB = 4 cm and PQ = 9 cm, find PR.

Similarity of Triangle

101

Solution : It is given that ABC ~ PQR Let i.e.,

AB AC PQ = PR
PR = x cm 4 4.8 = 9 x 4x = 9 4.8 x = 10.8 PR = 10.8 cm. CHECK YOUR PROGRESS 17.1

Find the values of x and y if ABC ~ PQR

(i)

Fig. 17.10

(ii)

Fig. 17.11

(iii)

Fig. 17.12

102

Mathematics

17.6 BASIC PROPORTIONALITY THEOREM We state below the Basic Proportionality Theorem : If a line is drawn parallel to one side of a triangle, the other two sides of the triangle are divided proportionally. Thus, in Fig. 17.13, DE || BC, According to the above result AD = AE DB EC We can easily verify this by measuring AD, DB, AE and EC. You will find that AD = AE DB EC We state the converse of the above result as follows : If a line divides any two sides of a triangle in the same ratio, the line is parallel to third side of the triangle. Thus, in Fig. 17.13, if DE divides sides AB and AC of ABC such that DE || BC. We can verify this by measuring ADE and ABC and finding that ADE = ABC These being alternate angles, the lines DE and BC are parallel. We can verify the above two results by taking different triangles. Let us solve some examples based on these. Example 17.3 : In Fig. 17.14, DE || BC. If AD = 3 cm, DB = 5 cm and AE = 6 cm, find AC. Solution : DE || BC (Given). Let EC = x AD AE = DB EC 3 6 = 5 x 3x = 30 x = 10 EC = 10 cm AC = AE + EC = 16 cm.
Fig. 17.14 Fig. 17.13

AD = AE , then DB EC

Similarity of Triangle

103

5 Example 17.4 : In Fig. 17.15, AD = 4 cm, DB = 5 cm, AE = 4.5 cm and EC = 5 cm. Is 8 DE || BC ? Given reasons for your answer. Solution : We are given that AD = 4 cm and DB = 5 cm. Similarly, AD 4 = DB 5 4.5 AE 9 8 =4 = 45 = EC 2 45 5 8 DE || BC. CHECK YOUR PROGRESS 17.2 1. In Fig. 17.16 (i), (ii) and (iii), PQ || BC. Find the value of x in each case.

Fig. 17.15

According to converse of Basic Proportionality Theorem

Fig. 17.16

2. In Fig. 17.17 [(i), (ii) and (iii)], find whether DE is parallel to BC or not ? Give reasons for your answer.

Fig. 17.17

17.7 BISECTOR OF AN ANGLE OF A TRIANGLE We now state an important result as given below : The internal bisector of an angle of a triangle divides the opposite side in the ratio of sides containing the angle

104

Mathematics

Thus, according to the above result, if AD is the internal bisector of A of ABC, then BD AB = DC AC We can easily verify this by measuring BD, DC, AB and AC and finding the ratios. We will find that BD AB = DC AC
Fig. 17.18

Repeating the same activity with other triangles, we may verify the result. Let us solve some examples to illustrate this. Example 17.5 : The sides AB and AC of a triangle are 6 cm and 8 cm. The bisector AD of A intersects the opposite side BC in D such that BD = 4.5 cm. Find the length of segment CD. Solution : According to the above result, we have BD AB = DC AC (Q AD is the internal bisector of A of ABC) or 4.5 6 = x 8 6x = 4.5 8 x=6 i.e., the length of line-segment CD = 6 cm. Example 17.6 : The sides of a triangle are 28 cm, 36 cm and 48 cm. Find the lengths of the line-segments into which the smallest side is divided by the bisector of the angle opposite to it. Solution : The smallest side is of length 28 cm and the sides forming the angle. A opposite to it are 36 cm and 48 cm. Let the angle bisector AD meet BC in D. BD 36 = 3 = DC 48 4 4BD = 3DC or BD = 36 DC = 3 DC 48 4
Fig. 17.19

BC = BD + DC = 28 cm DC + 3 DC = 28 4
Fig. 17.20

Similarity of Triangle

105

4 DC = 28 7

FH

IK

cm = 16

BD = 12 cm and DC = 16 cm CHECK YOUR PROGRESS 17.3

1. In Fig. 17.21, AD is the bisector of A, meeting BC in D. If AB = 4.5 cm, BD = 3 cm, DC = 5 cm, find x.

Fig. 17.21

2. In Fig. 17.22, PS is the internal bisector of P of PQR. The dimensions of some of the sides are given in Fig. 17.22. Find x.

Fig. 17.22

3. In Fig. 17.23, RS is the internal bisector of R of PQR. For the given dimensions, express p, the length of QS in terms of x, y and z.

Fig. 17.23

106

Mathematics

17.8 SOME MORE IMPORTANT RESULTS Let us study another important result on similarity in connection with a right triangle and the perpendicular from the vertex of the right angle to the opposite side. We state the result below and try to verify the same. If a perpendicular is drawn from the vertex of the right angle of a right triangle to the hypotenuse, the triangles on each side of the perpendicular are similar to each other and to the triangle. Let us try of verify this by an activity. Draw a ABC, right angled at A. Draw AD to the hypotenuse BC, meeting it in D. Let As Therefore Similarly, DBA = , As ADB = 90, BAD = 90 BAC = 90 and BAD = 90 , DAC = DCA = 90
Fig. 17.24

ADB and CDA are similar, as it has all the corresponding angles equal. ADB ~ CDA ~ CAB

Also, the angles of BAC are , 90 and 90 Another important result is about relation between sides and areas of similar triangles. It states that The ratio of the areas of similar triangles is equal to the ratio of the squares on their corresponding sides Let us verify this result by the following activity. Draw two triangles ABC and PQR which are similar i.e., their sides are proportional.

Fig. 17.25

Similarity of Triangle

107

Draw AD BC and PS QR Measure the lengths of AD and PS. Find the product AD BC and PS QR You will find that AD BC = BC2 and PS QR = QR2 Now AD BC = 2. Area of ABC PS QR = 2. Area of PQR As BC 2 Area of ABC AD BC = = Area of PQR PS QR QR 2 ...(i)

BC AB = AC QR = PQ PR
BC 2 AB2 = AC 2 Area of ABC = = Area of PQR QR 2 PQ 2 PR 2

The activity may be repeated by taking different pairs of similar triangles. Let us illustrate these results with the help of examples. Example 17.7 : Find the ratio of the area of two similar triangles if one pair of their corresponding sides are 2.5 cm and 5.0 cm. Solution : Let the two triangles be ABC and PQR Let BC = 2.5 cm and QR = 5.0 2.5 Area ABC 1 BC 2 = = = 2 2 4 Area PQR QR 5.0 Example 17.8 : In a ABC, PQ || BC and intersects AB and AC at P and Q respectively. If AP = 2 , find the ratio of areas of APQ and ABC. BP 3 Solution : In Fig. 17.26, PQ || BC AQ 2 AP = QC = 3 BP BP 3 = AP 2
Fig. 17.26

b b

g g

b g b g

1 + BP = 1 + 3 = 5 AP 2 2

108

Mathematics

AB 5 = AP 2

AP 2 = AB 5
2 2

Area APQ AP 2 = = Area ABC AB2

b b

g g

FG AP IJ = FG 2 IJ H ABK H 5 K

4 . 25

CHECK YOUR PROGRESS 17.4 1. In Fig. 17.27, ABC is a right triangle with A = 90 and C = 30. Show that DAB ~ DCA ~ ACB.

Fig. 17.27

2. Find the ratio of the areas of two similar triangles if the corresponding sides are of lengths 3 cm and 5 cm. 3. In Fig. 17.28, ABC is a triangle in which DE || BC. If AB = 6 cm and AD = 2 cm, find the ratio of the area of ADE and trapezium DBCE.

Fig. 17.28

4. P, Q and R are the mid-points of the sides AB, BC and CA of the ABC respectively. Show that the area of PQR is one-fourth the area of ABC. 5. In two similar triangles ABC and PQR, if the corresponding altitudes AD and PS are in the ratio of 4 : 9, find the ratio of the areas of ABC and PQR.

LMHint : Use AB = AD = BC = CA OP PQ PS QR PR Q N
6. If the ratio of the areas of two similar triangles is 16 : 25, find the ratio of their corresponding sides.

Similarity of Triangle

109

17.9 BAUDHAYAN/PYTHAGORAS THEOREM We know prove an important theorem, called Baudhayan/Phythagorus Theorem using the concept of similarity. Theorem : In a right triangle, the square on the hypotenuse is equal to the sum of the squares on the other two sides. Given. A right triangle ABC, in which B = 90. To Prove : AC2 = AB2 + BC2 Construction. From B, draw BD AC (See Fig. 17.29) Proof : BD AC and From (i), we get From (ii), we get ADB ~ ABC BDC ~ ABC AB AD = AC AB AB2 = AC.AD BC DC = AC BC BC2 = AC.DC AB2 + BC2 = AC (AD + DC) = AC. AC = AC2 The theorem is known after the name of famous Greek Mathematician Pythagoras. This was originally stated by the Indian Mathematician. Baudhayan about 200 years before Pythagoras. 17.9.1 Converse of Pythagoras Theorem The conserve of the above theorem states : In a triangle, if the square on one side is equal to sum of the squares on the other two sides, the angle opposite to first side is a right angle. This result can be verified by the following activity. Draw a triangle ABC with side 3 cm, 4 cm and 5 cm. i.e., AB = 3 cm, BC = 4 cm and AC = 5 cm. You can see that AB2 + BC2 = (3)2 + (4)2 = 9 + 16 = 25
Fig. 17.30

...(i) ...(ii)

...(A)

Fig. 17.29

...(B)

Adding (A) and (B), we get

110

Mathematics

AC2 = (5)2 = 25 AB2 + BC2 = AC2 The triangle in Fig. 17.30 satisfies the condition of the above result. Measure ABC, you will find that ABC = 90. Construct triangles of sides 5 cm, 12 cm and 13 cm, and of sides 7 cm, 24 cm, 25 cm. You will again find that the angles opposite to side of length 13 cm and 25 cm are 90 each. Let us solve some examples using above results. Example 17.9 : In a right triangle, the sides containing the right angle are of length 5 cm and 12 cm. Find the length of the hypotenuse. Solution : Let ABC be the right triangle, right angled at B AB = 5 cm, BC = 12 cm = (12)2 + (5)2 = 144 + 125 = 169 AC = 13 i.e., the length of the hypotenuse is 13 cm. Example 17.10 : Find the length of diagonal of a rectangle the lengths of whose sides are 3 cm and 4 cm. Solution : In Fig. 17.31, is a rectangle ABCD. Join the diagonal BD. Now DCB is a right triangle. BD2 = BC2 + CD2 = 42 + 32 = 16 + 9 = 25 BD = 5 i.e., the length of diagonal of rectangle ABCD is 5 cm. Example 17.11 : In an equilateral triangle, verify that three times the square on one side is equal to four times the square on its altitude. Solution : The altitude AD BC and Let and Let BD = CD AB = BC = CA = 2a BD = CD = a AD = x
Fig. 17.32 Fig. 17.31

Also, AC2 = BC2 + AB2

Similarity of Triangle

111

x2 = (2a)2 (a)2 = 3a2 3.(Side)2 = 3. (2a)2 = 12 a2

4.(Altitude) = 4. 3a2 = 12a2 Hence the result. Example 17.12 : ABC is a right triangle, right angled at C. If CD, the length of perpendicular from C on AB is p, BC = a, AC = b and AB = c, show that (i) pc = ab

1 1 1 (ii) p 2 = 2 + 2 a b
Solution : (i) CD AB ABC ~ ACD (ii) or a c = p b pc = ab. AB2 = AC2 + BC2 c2 = b2 + a2
Fig. 17.33

F ab I H pK
or

= b2 + a2

1 1 a 2 + b2 1 + = = 2 a b2 a 2 b2 p2

CHECK YOUR PROGRESS 17.5 1. The sides of certain triangles are given below. Determine which of them are right triangles : [AB = c, BC = a, CA = b] (i) a = 4 cm, b = 5 cm, c = 3 cm (ii) a = 1.6 cm, b = 3.8 cm, c = 4 cm (iii) a = 9 cm, b = 16 cm, c = 18 cm (iv) a = 7 cm, b = 24 cm, c = 25 cm 2. Two poles of height 6 m and 11 m, stand on a plane ground. If the distance between their feet is 12 m, find the distance between their tops.

112

Mathematics

3. Find the length of the diagonal of a square of side 10 cm. 4. In Fig. 17.34, C is acute and AD BC. Show that AB2 = AC2 + BC2 2BC. DC

Fig. 17.34

5. L and M are the mid-points of the sides AB and AC of ABC, right angled at B. Show that 4LC2 = AB2 + 4BC2 6. P and Q are points on the sides CA and CB respectively of ABC, right angled at C. Prove that AQ2 + BP2 = AB2 + PQ2 7. PQR is an isosceles right triangle with Q = 90. Prove that PR2 = 2PQ2. 8. A ladder is placed against a wall such that its top reaches upto a height of 4 m of the wall. If the foot of the ladder is 3 m away from the wall, find the length of the ladder. LET US SUM UP
z z

Objects which have the same shape but different sizes are called similar objects. Any two polygons, with corresponding angles equal and corresponding sides proportional, are similar. Two triangles are said to be similar, if (a) their corresponding angles are equal and (b) their corresponding sides are proportional

Criteria of similarity AAA criterion SSS criterion SAS criterion

If a line is drawn parallel to one-side of a triangle, it divides the other two sides in the same ratio and its converse.

Similarity of Triangle

113

The internal bisector of an angle of a triangle divides the opposite side in the ratio of sides containing the angle. If a perpendicular is drawn from the vertex of the right angle of a right angled triangle to the hypotenuse, the triangles so formed are similar to each other and to the given triangle. The ratio of the areas of two similar triangles is equal to the ratio of squares of their corresponding sides. In a right triangle, the square on the hypotenuse is equal to sum of the squares on the remaining two sides (Baudhyan) Pythagoras Theorem In a triangle, if the square on one side is equal to the sum of the squares on the remaining two sides, then the angle opposite to the first side is a right angle converse of (Baudhayan) Pythagoras Theorem. TERMINAL EXERCISE

1. Write the criteria for the similarity of two polygons. 2. Enumerate different criteria for the similarity of the two triangles. 3. In which of the following cases, s ABC and PQR are similar (i) A = 40, B = 60, C = 80, P = 40, Q = 60 and R = 80 (ii) A = 50, B = 70, C = 60, P = 50, Q = 60 and R = 70 (iii) AB = 2.5 cm, BC = 4.5 cm, CA = 3.5 cm PQ = 5.0 cm, QR = 9.0 cm, RP = 7.0 cm (iv) AB = 3 cm, BC = 4 cm, CA = 5.0 cm PQ = 4.5 cm, QR = 7.5 cm, RP = 6.0 cm. 4. In Fig. 17.35, AD = 3 cm, AE = 4.5 cm, DB = 4.0 cm, find CE, given that DE || BC.

Fig. 17.35

Fig. 17.36

5. In Fig. 15.36, DE || AC. From the dimension given in the figure, find the value of x.

114

Mathematics

6. In Fig. 17.37 is shown a ABC in which AD = 5 cm, DB = 3 cm, AE = 2.50 cm and EC = 1.5 cm. Is DE || BC ? Give reasons for your answer.

Fig. 17.37

Fig. 17.38

7. In Fig. 17.38, AD is the internal bisector of A of ABC. From the given dimension, find x. 8. The perimeter of two similar s ABC and DEF are 12 cm and 18 cm. Find the ratio of the area of ABC to that of DEF. 9. The altitudes AD and PS of two similar s ABC and PQR are of length 2.5 cm and 3.5 cm. Find the ratio of area of ABC to that of PQR. 10. Which of the following are right triangles ? (i) AB = 5 cm, BC = 12 cm, CA = 13 cm (ii) AB = 8 cm, BC = 6 cm, CA = 10 cm (iii) AB = 10 cm, BC = 5 cm, CA = 6 cm (iv) AB = 25 cm, BC = 24 cm, CA = 7 cm (v) AB = a2 + b2, BC = 2ab, CA = a2 b2 11. Find the area of an equilateral triangle of side 2a. 12. Two poles of height 12 m and 17 m, stand on a plane ground and the distance between their feet is 12 m. Find the distance between their tops. 13. In Fig. 17.39, show that AB2 = AC2 + BC2 + 2BC.CD 14. A ladder is placed against a wall and its top reaches a point at a height of 8 m from the ground. If the distance between the wall and foot of the ladder is 6 m, find the length of the ladder. 15. In an equilateral triangle, show that three times the square of a side equals four times the square on medians.
Fig. 17.39

Similarity of Triangle

115

ANSWERS Check Your Progress 17.1 1. (i) x = 4.5, y = 3.5 (ii) x = 70, y = 50 (iii) x = 2 cm, y = 7 cm

Check Your Progress 17.2 1. 2. (i) 6 (i) No (ii) 6 (ii) Yes (iii) 10 cm (iii) Yes

Check Your Progress 17.3 1. 7.5 cm 3.


yz x

2. 4 cm

(x = 1 is not possible)

Check Your Progress 17.4 2. 9 : 25 6. 4 : 5 Check Your Progress 17.5 1. (i) Yes (iv) Yes 2. 13 m Terminal Exercise 3. (i) and (iii) 6. Yes : 8. 4 : 9 10. (i), (ii), (iv) and (v) 11.
AD AE = DB EC

3. 1 : 8

5. 16 : 81

(ii) No

(iii) No

3. 10 2 cm

8. 5 m

4. 6 cm 7. 4.5 cm 9. 25 : 49

5. 4.5 cm

3a 2

12. 13 m

14. 10 m

116

Mathematics

18
Circles
18.1 INTRODUCTION You are already familiar with geometrical figures such as a line segment, an angle, a triangle, a quadrilateral and a circle. Examples, of a circle are a wheel, a bangle, alphabet O, etc. In this lesson we shall study in some details about the circle and related concepts. 18.2 OBJECTIVES After studying this lesson, the learner will be able to :
z z z z z z

define a circle give examples of various terms related to a circle illustrate congruent circles and concentric circles identify and illustrate terms connected with circles like chord, arc, sector, segment, etc verify experimentally results based on arcs and chords of a circle use the results in solving problems

18.3 EXPECTED BACKGROUND KNOWLEDGE


z z z z z z z

Line segment and its length Angle and its measure Parallel and perpendicular lines Closed figures such as triangles, quadrilaterals polygons, etc. Perimeter of a closed figure Region bounded by a closed figure Congruence of closed figures

18.4 CIRCLE AND RELATED TERMS 18.4.1 Circle : A circle is a collection of all points in a plane which are at a constant distance from a fixed point in the same plane.

Circles

117

Radius : A line segment joining the centre of the circle to a point on the circle is called its radius. In Fig. 18.1, there is a circle with centre O and one of its radius is OA. OB is another radius of the same circle. Activity for you : Measure the length OA and OB and observe that they are equal. Thus All radii (plural of radius) of a circle are equal The length of the radius of a circle is generally denoted by the letter r. It is customary to write radius instead of the length of the radius. A closed geometric figure in the plane divides the plane into three parts namely, the inner part of the figure, the figure and the outer part. In Figure 18.2 the shaded portion is the inner part of the circle, the boundary is the circle and the unshaded portion is the outer part of the circle. Activity for you (a) Take a point Q in the inner part of the circle (See Figure 18.3). Measure OQ and find that OQ < r. The inner part of the circle is called the interior of the circle. (b) Now take a point P in the outer part of the circle (Figure 18.3). Measure OP and find that OP > r. The outer part of the circle is called the exterior of the circle. 18.4.2 Chord A line segment joining any two points of a circle is called a chord. In Figure 18.4 AB and PQ and CD are three chords of a circle with centre O and radius r. The chord PQ passes through the centre O of the circle. Such a chord is called a diameter of the circle. Diameter is usually denoted by d. A chord passing though the centre of circle is called its diameter. Activity for you : Measure the length d of PQ, the radius r and find that d is the same as 2r. Thus we have d = 2r i.e. the diameter of a circle = twice the radius of the circle.
Fig. 18.4 Fig. 18.3 Fig. 18.2

Fig. 18.1

118

Mathematics

Measure the length PQ, AB and CD and find that PQ > AB and PQ > CD, we may conclude Diameter is the longest chord of a circle. 18.4.3 Arc A part of a circle is called an arc. In Figure 18.5(a) ABC is an arc and is denoted by arc ABC . or

(a) Fig. 18.5

(b)

18.4.4 Semicircle A diameter of a circle divides a circle into two equal arcs, each known as a semicircle. In Figure 18.5(b), PQ is a diameter and 18.4.5 Sector The region bounded by an arc of a circle and two radii at its end points is called a sector. In Figure 18.6, the shaded portion is a sector formed by the arc PRQ and the unshaded portion is a sector formed by the arc PTQ. 18.4.6 Segment A chord divides the interior of a circle into two parts, each called a segment. In Figure 18.7, the shaded region PAQP and the unshaded region PBQP are both segments of the circle. PAQP is called a minor segment and PBQP is called a major segment. 18.4.7 Circumference
Fig. 18.7

is a semicircle and so is

Fig. 18.6

Choose a point P on a circle. If this point moves along the circle once and comes back to its original position then the distance covered by P is called the circumference of the circle

Circles

119

Fig. 18.8

Activity for you : Take a wheel and mark a point P on the wheel where it touches the ground. Rotate the wheel along a line till the point P comes back on the ground. Measure the distance between the Ist and last position of P along the line. This distance is equal to the circumference of the circle. Thus, The length of the boundary of a circle is the circumference of the circle. Activity for you Consider different circles and measure their circumference(s) and diameters. Observe that in each case the ratio of the circumference to diameter turns out to be the same. The ratio of the circumference of a circle to its diameter is always a constant. This constant is universally denoted by Greek letter . Therefore, radius. 22 An approximate value of is . AryabhataI (476 AD), a famous Indian Mathematician 7 gave a more accurate value of which is 3.1416. In fact this number is an irrational number. 18.5. MEASUREMENT OF AN ARC OF A CIRCLE Consider an arc PAQ of a circle (Fig 18.9). To measure its length we put a thread along PAQ and then measure the length of the thread with the help of a scale. Similarly, you may measure the length of the arc PBQ. 18.5.1 Minor arc An arc of a circle whose length is less than that or a semicircle of the same circle is called a minor arc. PAQ a minor arc (See Fig. 18.9) c = c = , where c is the circumference of the circle, d its diameter and r is its d 2r

Fig. 18.9

120

Mathematics

18.5.2 Major arc An arc of a circle whose length is greater than that of a semicircle of the same circle is called a major arc. In Figure 18.9, arc PBQ is a major arc. 18.6 CONCENTRIC CIRCLES Circles having the same centre but different radii are called concentric circles (See Fig. 18.10). 18.7 CONGRUENT CIRCLES OR ARCS
Fig. 18.10

Two circles (or arcs) are said to be congruent if we can superpose (place) one over the other such that they cover each other completely. 18.8 SOME IMPORTANT RULES Activity for you : (i) Draw two circles with centre O1 and O2 and radius r and s respectively (See Fig. 18.11)

(i) Fig. 18.11

(ii)

(ii) Supeimpose the circle (i) on the circle (ii) so that O1 coincides with O2 (iii) We observe that circle (i) will cover circle (ii) if and only if r = s Two circles are congurent if and only if they have equal radii. In Figure 18.12 if arc PAQ = arc RBS then POQ = ROS and conversely if POQ = ROS then arc PAQ = arc RBS. Two arcs of a circle are congurent if and only if the angles subtended by them at the centre are equal. In Figure 18.13, if arc PAQ = arc RBS then PQ = RS and conversely if PQ = RS then
Fig. 18.12

Circles

121

arc PAQ = arc RBS. Two arcs of a circle are congurent if and only if their corresponding chords are equal. Activity for you : (i) Draw a circle with centre O (ii) Draw equal chords PQ and RS (See Fig. 18.14) (iii) Join OP, OQ, OR and OS (iv) Measure POQ and ROS we observe that POQ = ROS Conversely if POQ = ROS then PQ = RS
Fig. 18.14 Fig. 18.13

Equal chords of a circle subtend equal angles at the centre and conversely if the angles subtended by the chords at the centre of a circle are equal, then the chords are equal. Note : The above results also hold good in case of congruent circles. We take some examples using the above properties : Example 18.1 : In Fig. 18.15, chord PQ = chord RS. Show that chord PR = chord QS. Solution : The arcs corresponding to equal chords PQ and RS are equal. Add to each arc, the arc QR, yielding arc PQR = arc QRS chord PR = chord QS Example 18.2 : In Figure 18.16 arc AB = arc BC, AOB = 30 and AOD = 70. Find COD. Solution : Since arc AB = arc BC Since AOB = BOC (Equal arcs subtend equal angles at the centre) BOC = 30 COD = COB + BOA + AOD = 30 + 30 + 70 = 130.
Fig. 18.16 Fig. 18.15

122

Mathematics

Activity for you : (i) Draw a circle with centre O (See Fig. 18.17). (ii) Draw a chord PQ. (iii) Draw ON from O on the chord PQ. (iv) Measure PN and NQ You will observe that . PN = NQ. The perpendicular drawn from the centre of a circle to a chord bisects the chord. Activity for you : (i) Draw a circle with centre O (See Fig. 18.18). (ii) Draw a chord PQ. (iii) Find the mid point M of PQ. (iv) Join O and M. (v) Measure OMP or OMQ with set square or protractor We observe that OMP = OMQ = 90.
Fig. 18.18 Fig. 18.17

The line joining the centre of a circle to the mid point of a chord is perpendicular to the chord. Activity for you : Take three non collinear points A, B and C. Join AB and BC. Draw perpendicular bisectors MN and RS of AB and BC respectively. Since A, B, C are not collinear, MN is not parallel to RS. They will intersect only at one point O. Join OA, OB and OC and measure them. We observe that OA = OB = OC
Fig. 18.19

Now taking O as the centre and OA as radius draw a circle which passes through A, B and C. Repeat the above procedure with another three non-collinear points and observe that there is only one circle passing through there given non-collinear points. This gives us a method to draw a circle passing through three non-collinear points. There is one and only one circle passing through three non-collinear points. Note. It is important to note that a circle can not be drawn to pass through three collinear points.

Circles

123

Activity for you : (i) Draw a circle with centre O [Fig. 18.20(a)]. (ii) Draw two equal chords AB and PQ of the circle. (iii) Draw OM AB and ON PQ. (iv) Measure OM and ON and observe that they are equal. Equal chords of a circle are equidistant from the centre. In Fig. 18.20(b) OM = ON. Measure and observe that AB = PQ. Thus, Chords that are equidistant from the centre of a circle are equal. The above results hold good in case of congurent circles also. We now take a few examples using these properties of circles. Examples 18.3 : In Figure 18.21, O is the centre of the circle and ON PQ. If PQ = 8 cm and ON = 3 cm, find OP. Solution : ON PQ (given) and since perpendicular drawn from the centre of a circle to a chord bisects the chord. PN = NQ = 4 cm In a right triangle OPN, OP2 = PN2 + ON2 OP2 = 42 + 32 = 25 OP = 5 cm. Examples 18.4 : In Figure 18.22, OD is perpendicular to the chord AB of a circle whose centre is O and BC is a diameter. Prove that CA = 2OD. Solution : Since OD AB (Given) D is the mid point of AB
Fig. 18.22 Fig. 18.20b Fig. 18.20a

Fig. 18.21

(Perpendicular through the centre bisects the chord) Also O is the mid point of CB (Since CB is a diameter) Now in ABC, O and D are mid points of the two sides BC and BA of the triangle ABC. Since the line segment joining the mid points of any two sides of a triangle is parallel and half of the third side. i.e. OD = 1 CA 2 CA = 2OD.

124

Mathematics

Example.18.5 : A regular hexagon is inscribed in a circle. What angle does each side of the hexagon subtend at the centre ? Solution : A regular hexagon has six sides which are equal. Therefore each side subtends the same angle at the centre. Let us suppose that a side of the hexagon subtends an angle x at the centre. Then, we have 6x = 360 x = 60
Fig. 18.23

Hence, each side of the hexagon subtends an angle of 60 at the centre. Example 18.6 : In Fig. 18.24, two parallel chords PQ and AB of a circle are of lengths 7 cm and 13 cm respectively. If the distance between PQ and AB is 3 cm, find the radius of the circle. Solution : Let O be the centre of the circle. Draw perpendicular bisector OL of PQ which also bisects AB at M. Join OQ and OB (Fig. 18.24). Let OM = x cm and radius of the circle be r cm Then OB2 = OM2 + MB2 and OQ2 = OL2 + LQ2 13 r2 = x 2 + 2 r2
Fig. 18.24

FH IK

...(i)
2

and

2 7 = x+3 + 2

b g FH IK
2

...(ii)

Therefore from (i) and (ii) , x 2 + 13 2

FH IK = bx + 3g + FH 7 I 2K
2

6x = x= r2 = r=

169 9 49 4 4 7 2

FH 7 I + FH 13 I 2K 2K
2

49 + 169 = 218 4 4 4

218 2 218 cm. 2

Hence the radius of the circle is

Circles

125

CHECK YOUR PROGRESS 18.1 In questions 1 to 5, fill in the blanks to make each of the statements true. 1. In Figure 18.25, (i) AB is a ... of the circle. (ii) Minor arc corresponding to AB is ... . 2. A ... is the longest chord of a circle.
Fig. 18.25

3. The ratio of the circumference to the diameter of a circle is always ... . 4. The value of as 3.1416 was given by great Indian Mathematician ... . 5. Circles having the same centre are called ... circles. 6. Diameter of a circle is 30 cm. If the length of a chord is 20 cm, find the distance of the chord from the centre. 7. Find the circumference of a circle whose radius is (i) 7 cm (ii) 11 cm.

FH Take = 22 I 7K

8. In the Figure 18.26, RS is a diameter which bisects the chords PQ and AB at the points M and N respectively. Is PQ || AB ? Give reasons.

Fig. 18.26

Fig. 18.27

9. In the Figures 18.27, a line l intersects the two concentric circles with centre O at points A, B, C and D. Is AB = CD ? Give reasons. LET US SUM UP
z z

The circumference of a circle of radius r is equal to 2 r . Two arcs of a circle are congurent if and only if either the angles subtended by them at the centre are equal or their corresponding chords are equal. Equal chords of a circle subtend equal angles at the centre and vice versa.

126

Mathematics

z z

Perpendicular drawn from the centre of a circle to a chord bisects the chord. The line joining the centre of a circle to the mid point of a chord is perpendicular to the chord. There is one and only one circle passing through three non-collinear points. Equal chords of a circle are equidistant from the centre and the converse. TERMINAL EXERCISE

z z

1. If the length of a chord of a circle is 16 cm and the distance of the chord from the centre is 6 cm, find the radius of the circle. 2. Two circles with centre O and O' (See Fig. 18.28) are congurent. Find the length of the arc CD.

Fig. 18.28

3. A regular pentagon is inscribed in a circle. Find the angle which each side of the pentagon subtend at the centre. 4. In Figure 18.29, AB = 8 cm and CD = 6 cm are two parallel chords of a circle with centre O. Find the distance between the chords.

Fig. 18.29

5. In Figure 18.30, arc PQ = arc QR. POQ = 15 and SOR = 110. Find SOP.

Fig. 18.30

Circles

127

6. In Figure 18.31, AB and CD are two equal chords of a circle with centre O. Is chord BD = chord CA ? Give reasons.

Fig. 18.31

7. If AB and CD are two equal chords of a circle with centre O (Fig. 18.32) and OM AB , ONCD . Is OM = ON ? Give reasons.

Fig. 18.32

8. In Figure 18.33, AB = 14 cm and CD = 6 cm are two parallel chords of a circle with centre O. Find the distance between the chords AB and CD.

Fig. 18.33

9. In Figure 18.34, AB and CD are two chords of a circle with centre O, intersecting at a point P inside the circle.

Fig. 18.34

OMCD , ONAB and OPM = OPN

Is (i) OM = ON, (ii) AB = CD ? Give reasons.

128

Mathematics

10. C1 and C2 are concentric circles with centre O (See Fig 18.35), l is a line intersecting C1 at points P and Q and C2 at points A and B respectively. If ON l, is PA = BQ ? Give reasons.

Fig. 18.35

Circles

129

ANSWERS Check Your Progress 18.1 1. (i) Chord (ii) APB 2. Diameter 5. Concentric 8. Yes Terminal Exercise 18.2 1. 10 cm 4. 4 3 = 1 cm. 2. 2a cm 5. 80 3. 72 6. 5 5 cm. 9. Yes 3. Constant 4. AryabhataI

7. (i) 44 cm (ii) 69.14 cm.

6. Yes (Equal arcs have corresponding equal chords of a circle) 7. Yes (equal chords are equidistant from the centre of the circle) 8. 10 2 cm 9. (i) Yes (ii) Yes (OMP ONP)

10. Yes (N is the middle point of chords PQ and AB).

130

Mathematics

19
Angles in a Circle and Cyclic Quadrilateral
19.1 INTRODUCTION You must have measured the angles between two straight lines, let us now study the angles made by arcs and chords in a circle and a cyclic quadrilateral. 19.2 OBJECTIVES After studying this lesson, the learner will be able to :
z z z z z z

prove that angles in the same segment of a circle are equal cite examples of concyclic points define cyclic quadrilaterals prove that sum of the opposite angles of a cyclic quadrilateral is 180 use properties of a cyclic quadrilateral solve problems based on Theorems (proved) and solve other numerical problems based on verified properties.

19.3 EXPECTED BACKGROUND KNOWLEDGE


z z z

Angles of a triangle Arc, chord and circumference of a circle Quadrilateral and its types

19.4 ANGLES IN A CIRCLE Central Angle. The angle made at the centre of a circle by the radii at the end points of an arc (or a chord) is called the central angle or angle subtended by an arc (or chord) at the centre. In Figure 19.1, POQ is the central angle made by arc PRQ.
Fig.19.1

The length of an arc is closely associated with the central angle subtended by the arc. Let us define the degree measure of an arc in terms of the central angle.

Angles in a Circle and Cyclic Quadrilateral

131

The degree measure of a minor arc of a circle is the measure of its corresponding central angle. In Figure 19.2, Degree measure of PQR = x The degree measure of a semicircle in 180 and that of a major arc is 360 minus the degree measure of the corresponding minor arc. Relationship between length of an arc and its degree measure. Length of an arc = circumference If the degree measure of an arc is 40 40 = 2 r then length of the arc PQR = 2 r. 360 9 Inscribed angle : The angle subtended by an arc (or chord) on any point on the remaining part of the circle is called an inscribed angle. In Figure 19.3, PAQ is the angle inscribed by arc PRQ at point A of the remaining part of the circle or by the chord PQ at the point A. 19.5. SOME IMPORTANT PROPERTIES ACTIVITY FOR YOU : Draw a circle with centre O. Let PAQ be an arc and B any point on the circle. Measure the central angle POQ and an inscribed angle PBQ by the arc at remaining part of the circle. We observe that POQ = 2PBQ Repeat this activity taking different circles and different arcs. We observe that The angle subtended at the centre of a circle by an arc is double the angle subtended by it on any point on the remaining part of the circle. Let O be the centre of a circle. Consider a semicircle PAQ and its inscribed angle PBQ 2 PBQ = POQ
Fig.19.5 Fig.19.4 Fig.19.2

degree measure of the arc 360

Fig.19.3

132

Mathematics

(Since the angle subtended by an arc at the centre is double the angle subtended by it at any point on the remaining part of the circle) But POQ = 180 2PBQ = 180 PBQ = 90 Thus, we conclude the following : Angle in a semicircle is a right angle. Theorem : Angles in the same segment of a circle are equal. Given : A circle with centre O and the angles PRQ and PSQ in the same segment formed by the chord PQ (or arc PAQ) To prove : PRQ = PSQ Construction : Join OP and OQ. Proof : As the angle subtended by an arc at the centre is double the angle subtended by it at any point on the remaining part of the circle, therefore we have POQ = 2 PRQ and POQ = 2PSQ 2PRQ = 2PSQ PRQ = PSQ We take some examples using the above results Example 19.1 : In Figure 19.7, O is the centre of the circle and AOC = 120. Find ABC. Solution : It is obvious that x is the central angle subtended by the arc APC and ABC is the inscribed angle. But Solution : x = 2ABC x = 360 120 2ABC = 240 ABC = 120 POQ = 2PAQ = 70 ...(i)
Fig.19.7

(Since PQ is a diameter of the circle)

...(i) ...(ii)
Fig.19.6

From (i) and (ii), we get

Example 19.2 : In Figure 19.8 O is the centre of the circle and PAQ = 35. Find OPQ. (Angle at the centre is double the angle on the remaining part of the circle)

Angles in a Circle and Cyclic Quadrilateral

133

Since

OP = OQ

(Radii of the same circle) ...(ii)

OPQ = OQP (Angles opposite to equal sides are equal) But OPQ + OQP + POQ = 180 2OPQ = 180 70 = 110 OPQ = 55.

Fig.19.8

Example 19.3 : In Figure 19.9, O is the centre of the circle and AD bisects BAC. Find BCD. Solution : Since BC is a diameter BAC = 90 (Angle in the semicircle is a right angle) As AD bisects BAC But BAD = 45 BCD = BAD BCD = 45.
Fig.19.9

(Angles in the same segment of a circle are equal) Example 19.4 : In Figure 19.10, O is the centre of the circle, POQ = 70 and PSOQ . Find MQS. Solution : 2PSQ = POQ = 70 (Angle subtended at the centre of a circle is twice the angle subtended by it on the remaining part of the circle) PSQ = 35 Fig.19.10 Since MSQ + SMQ + MQS = 180 (Sum of the angles of a triangle) 35 + 90 + MQS = 180 MQS = 180 125 = 55. CHECK YOUR PROGRESS 19.1 1. In Figure 19.11, ADB is an arc of a circle with centre O, if ACB = 35, find AOB.

Fig. 19.11

134

Mathematics

2. In Figure 19.12, AOB is a diameter of a circle with centre O. Is APB = AQB = 90? Give reasons.

Fig. 19.12

3. In Figure 19.13, PQR is an arc of a circle with centre O. If PTR = 35, find PSR.

Fig. 19.13

4. In Figure 19.14, O is the centre of a circle and AOB = 60. Find ADB.

Fig. 19.14

19.6 CONCYCLIC POINTS Definition : Points which lie on a circle are called concyclic points. Let us now find certain conditions under which points are concyclic. If you take a point P, you can draw not only one but many circles passing through it as in Fig. 19.15.

Fig.19.15

Now take two points P and Q on a sheet of a paper. You can draw as many circles as you wish, passing through the points. (Fig. 19.16).

Angles in a Circle and Cyclic Quadrilateral

135

Fig. 19.16

Let us now take three points P, Q and R which do not lie on the same straight line. In this case you can draw only one circle passing through these three non-colinear points (Figure 19.17).

Fig. 19.17

Further let us now take four points P, Q, R, and S which do not lie on the same line. You will see that it is not always possible to draw a circle passing through four non-collinear points. In Fig 19.18 (a) and (b) points are noncyclic but concyclic in Fig 19.18(c).

(a)

(b) Fig. 19.18

(b)

Note. If the points P, Q and R are collinear then it is not possible to draw a circle passing through them. Thus we conclude 1. Given one or two points there are infinitely many circles passing through them. 2. Three non-collinear points are always concyclic and there is only one circle passing through all of them. 3. Three collinear points are not concyclic (or noncyclic). 4. Four non-collinear points may or may not be concyclic.

136

Mathematics

19.6.1 CYCLIC QUADRILATERAL A quadrilateral is said to be a cyclic quadrilateral if there is a circle passing through all its four vertices. For example, Fig. 19.19 shows a cyclic quadrilateral PQRS. Theorem. Sum of the opposite angles of a cyclic quadrilateral is 180. Given : A cyclic quadrilateral ABCD To prove : BAD + BCD = ABC + ADC = 180 Construction : Draw AC and DB Proof : and ACB = ADB BAC = BDC [Angles in the same segment] ACB + BAC = ADB + BDC = ADC
Fig.19.20 Fig.19.19

Adding ABC on both the sides, we get ACB + BAC + ABC = ADC + ABC But ACB + BAC + ABC = 180 ADC + ABC = 180 BAD + BCD = 360 (ADC + ABC) = 180.

[Sum of the angles of a triangle]

Hence proved. Converse of this theorem is also true. If a pair of opposite angles of a quadrilateral is supplementary, then the quadrilateral is cyclic. Verification : Draw a quadrilateral PQRS Since in quadrilateral PQRS, P + R = 180 and S + Q = 180
Fig.19.21

Therefore draw a circle passing through the point P, Q and R and observe that it also passes through the point S. So we conclude that quadrilateral PQRS is a cyclic quadrilateral. We solve some examples using the above results. Example 19.5 : ABCD is a cyclic parallelogram. Show that it is a rectangle. Solution : A + C = 180 (ABCD is a cyclic quadrilateral) Since A = C [Opposite angles of a parallelogram]
Fig.19.22

Angles in a Circle and Cyclic Quadrilateral

137

or

A + A = 180 2A = 180 A = 90

Thus ABCD is a rectangle. Example 19.6 : A pair of opposite sides of a cyclic quadrilateral is equal. Prove that its diagonals are also equal (See Figure 19.23). Solution : Let ABCD be a cyclic quadrilateral and AB = CD. arc AB = arc CD (Corresponding arcs)

Adding arc AD to both the sides; arc AB + arc AD = arc CD + arc AD arc BAD = arc CDA Chord BD = Chord CA BD = CA
Fig.19.23

Example 19.7 : In Figure 19.24, PQRS is a cyclic quadrilateral whose diagonals intersect at A. If SQR = 80 and QPR = 30, find SRQ. Solution : Given Since or But SQR = 80 SQR = SPR [Angles in the same segment] SPR = 80 SPQ = SPR + RPQ = 80 + 30. SPQ = 110. (Sum of the opposite angles of a cyclic quadrilateral is 180) SPQ + SRQ = 180
Fig.19.24

SRQ = 180 SPQ = 180 110 = 70

Example 19.8 : PQRS is a cyclic quadrilateral. If Q = R = 65, find P and S. Solution : P + R = 180 Similarly, P = 180 R = 180 65 P = 115 S = 180 Q = 180 65 S = 115.
Fig.19.25

Q + S = 180

138

Mathematics

CHECK YOUR PROGRESS 19.2 1. In Figure 19.26, AB and CD are two equal chords of a circle with centre O. If AOB = 55, find COD.

Fig. 19.26

2. In Figure 19.27, PQRS is a cyclic quadrilateral, and the side PS is extended to the point A. If PQR = 80, find ASR.

Fig. 19.27

3. In Figure 19.28, ABCD is a cyclic quadrilateral whose diagonals intersect at O. If ACB = 50 and ABC = 110, find BDC.

Fig. 19.28

4. In Figure 19.29, ABCD is a quadrilateral. If A = BCE, is the quadrilateral a cyclic quadrilateral ? Give reasons.

Fig. 19.29

Angles in a Circle and Cyclic Quadrilateral

139

LET US SUM UP
z

The angle subtended by an arc (or chord) at the centre of a circle is called central angle and an angle subtended by it at any point on the remaining part of the circle is called inscribed angle. Points lying on the same circle are called concyclic points. The angle subtended by an arc at the centre of a circle is double the angle subtended by it at any point on the remaining part of the circle. Angle in a semicircle is a right angle. Angles in the same segment of a circle are equal. Sum of the opposite angles of a cyclic quadrilateral is 180. If a pair of opposite angles of a quadrilateral is supplementary, then the quadrilateral is cyclic. TERMINAL EXERCISES

z z

z z z z

1. A square PQRS is inscribed in a circle with centre O. What angle does each side subtend at the centre O ? 2. In Figure 19.30, C1 and C2 are two circles with centre O1 and O2 and intersect each other at points A and B. If O1O2 intersect AB at M then show that (i) O1AO2 O1BO2 (ii) M is the mid point of AB (iii) ABO1O 2

Fig. 19.30

(Hint. From (i) conclude that 1 = 2 and then prove that AO1M BO1M (by SAS rule)).

140

Mathematics

3. Two circle intersect in A and B. AC and AD are the diameters of the circles. Prove that C, B and D are collinear.

Fig. 19.31

[Hint. Join CB, BD and AB, Since ABC = 90 and ABD = 90] 4. In Figure 19.32, AB is a chord of a circle with centre O. If ACB = 40, find OAB.

Fig. 19.32

5. In Figure 19.33, O is the centre of a circle and PQR = 115. Find POR.

Fig. 19.33

6. In Figure 19.34, O is the centre of a circle, AOB = 80 and PQB = 70. Find PBQ.

Fig. 19.34

Angles in a Circle and Cyclic Quadrilateral

141

ANSWERS Check Your Progress 19.1 1. 70 2. Yes 3. 35 4. 30

Check Your Progress 19.2 1. 55 Terminal Exercise 1. 90 4. 50 5. 130 6. 70. 2. 80 3. 20 4. Yes

Secants, Tangents and Properties

143

20
Secants, Tangents and Properties
8.1 INTRODUCTION Look at a moving cycle. You will observe that at any instant of time, the wheels of the moving cycle touch the road at a very limited area, more correctly a point. If you roll a coin on a smooth surface, say a table or floor, you will find that at any instant of time, only one point of the coin comes in contact with the surface it is rolled upon. What do you observe from the above situations ?

Fig. 20.1

If you consider a wheel or a coin as a circle and the touching surface (road or table) as a line, the above illustrations show that a line touches a circle. In this lesson, we shall study about the possible contacts that a line and a circle have and try to study their properties.

144

Mathematics

20.2 OBJECTIVES After studying this lesson, the learner will be able to :
z z z

define a secant and a tangent to the circle differentiate between a secant and a tangent verify and use important results (given in the curriculum) related to tangents and secants to circles.

20.3 EXPECTED BACKGROUND KNOWLEDGE


z z z z z

Measurement of angles and line segments Drawing circles of given radii Drawing lines perpendicular and parallel to given lines Knowledge of previous results about lines and angles, congruence and circles. Knowledge of Pythagoras Theorem.

20.4 SECANTS AND TANGENTSAN INTRODUCTION You have read about lines and circles in your earlier lessons. Recall that a circle is the locus of a point in a plane which moves in such a way that its distance from a fixed point in the plane always remains constant. The fixed point is called the centre of the circle and the constant distance is called the radius of the circle. You also know that a line is a collection of points, extending indefinitely to both sides, whereas a line segment is a portion of a line bounded by two points.

Fig. 20.2

Now consider the case when a line and a circle co-exist in the same plane. There can be three distinct possibilities as shown in Fig. 20.2. You can see that in Fig. 20.2(i), the line XY does not intersect the circle, with centre O. In other words, we say that the line XY and the circle have no common point. In Fig. 20.2 (ii), the line XY intersects the circle in two distinct points A and B, and in Fig. 20.2 (iii), the line XY intersects the circle in only one point and is said to touch the circle at the point P.

Secants, Tangents and Properties

145

Thus, we can say that in case of intersection of a line and a circle, the following three possibilities are there : (i) The line does not intersect the circle at all, i.e., the line lies in the exterior of the circle. (ii) The line intersects the circle at two distinct points. In that case, a part of the line lies in the interior of the circle, the two points of intersection lie on the circle and the remaining portion lies in the exterior of the circle. (iii) The line touches the circle in exactly one point. We therefore define the following : Tangent A line which touches a circle at exactly one point is called a tangent line and the point where it touches the circle is called the point of contact. Thus, in Fig. 20.2 (iii), XY is a tangent to the circle at P, which is called the point of contact. Secant A line which intersects the circle in two distinct points is called a secant line (usually referred to as a secant). In Fig. 20.2 (ii), XY is a secant line to the circle and A and B are called the points of intersection of the line XY and the circle with centre O. 20.5 TANGENT AS A LIMITING CASE OF A SECANT Consider the secant XY of the circle with centre O, intersecting the circle in the points A and B. Imagine that one point A, which lies on the circle, of the secant XY is fixed and the secant rotates about A, intersecting the circle at B', B'', B''', B'''' as shown in Fig. 20.3 and ultimately attains the position of the line XAY, when it becomes tangent to the circle at A. Thus, we say that A tangent is the limiting position of a secant when the two points of intersection coincide.
Fig. 20.3

20.6 TANGENT AND RADIUS THROUGH THE POINT OF CONTACT Let XY be a tangent to the circle, with centre O, at the point P. Join OP. Take points Q, R, S and T on the tangent XY and join OQ, OR, OS and OT. As Q, R, S and T are points in the exterior of the circle and P is on the circle.

146

Mathematics

OP is less than each of OQ, OR, OS and OT.

From our, previous study of Geometry, we know that of all the segments that can be drawn from a point (not on the line) to the line, the perpendicular segment is the shortest : As OP is the shortest distance from O to the line XY

Fig. 20.4

OP XY

Thus, we can state that A radius, through the point of contact of tangent to a circle, is perpendicular to the tangent at that point. The above result can also be verified by measuring angles OPX and OPY and finding each of them equal to 90. 20.7 TANGENTS FROM A POINT OUTSIDE THE CIRCLE Take any point P in the exterior of the circle with centre O. Draw lines through P. Some of these are shown as PT, PB, PA, PC, PD and PT' in Fig.20.5. How many of these touch the circle ? Only two. Repeat the activity with another point and a circle. You will again find the same result. Thus, we can say that From an external point, two tangents can be drawn to a circle.
Fig. 20.5

If the point P lies on the circle, can there still be two tangents to the circle from that point ? You can see that only one tangent can be drawn to the circle in that case. What about the case when P lies in the interior of the circle ? Note that any line through P in that case will intersect the circle in two points and hence no tangents can be drawn from an interior point to the circle. (A) Now, measure the lengths of PT and PT. You will find that PT = PT (B) Let us see this logically also. Consider two s OPT and OPT OTP = OT'P OT = OT OP = OP (Each being a right angle) (Radii of the same circle) (Common) ...(i)

Secants, Tangents and Properties

147

OPT OPT PT = PT ...(ii)

From (A) and (B), we can say that The lengths of two tangents from an external point are equal. Also, in Fig. 20.6 as OPT OPT OPT = OPT
Fig. 20.6

Thus, we can say that The tangents drawn from an external point to a circle are equally inclined to the line joining the point to the centre of the circle. Let us now take some examples to illustrate. Example 20.1 : In Fig. 20.7, OP = 5 cm and radius of the circle is 3 cm. Find the length of the tangent PT from P to the circle, with centre O. Solution : OTP = 90, Let PT = x

In right triangle OTP, we have OP2 = OT2 + PT2 or or 52 = 32 + x2 x2 = 25 9 = 16 x=4


Fig. 20.7

i.e. the length of tangent PT = 4 cm Example 20.2 : In Fig. 20.8, tangents PT and PT are drawn from a point P at a distance of 25 cm from the centre of the circle whose radius is 7 cm. Find the lengths of PT and PT. Solution : Here OP = 25 cm and OT = 7 cm We also know that OTP = 90 PT2 = OP2 OT2 = 625 49 = 576 = (24)2 PT = 24 cm PT = PT PT = 24 cm We also know that
Fig. 20.8

148

Mathematics

Example 20.3 : In Fig. 20.9, A, B and C are three exterior points of the circle with centre O. The tangents AP, BQ and CR are of lengths 3 cm, 4 cm and 3.5 cm. Find the perimeter of ABC. Solution : We know that the length of two tangents from an external point to a circle are equal AP = AR, BP = BQ, CQ = CR and AP = AR = 3 cm, BP = BQ = 4 cm CR = CQ = 3.5 cm AB = AP + PB; = (3 + 4) cm = 7 cm BC = BQ + QC; = (4 + 3.5) cm = 7.5 cm CA = AR + CR = (3 + 3.5) cm = 6.5 cm Perimeter of ABC = (7 + 7.5 + 6.5) cm = 21 cm
Fig. 20.9

Example 20.4 : In Fig. 20.10, AOB = 50. Find ABO and OBT Solution : We know that OA XY OAB = 90 ABO = 180 (OAB + AOB) = 180 (90 + 50) = 40 We know that OBA = OBT OBT = 40 ABO = OBT = 40 CHECK YOUR PROGRESS 20.1 1. Fill in the blanks with suitable words : (i) A tangent is to the radius through the point of contact. . coincide.
Fig. 20.10

(ii) The lengths of tangents from an external point to a circle are (iii) A tangent is the limiting position of a secant when the two

Secants, Tangents and Properties

149

(iv) From an external point

tangents can be drawn to a circle. tangent(s) can be drawn to the

(v) From a point in the interior of the circle, circle.

2. In Fig. 20.11, POY = 40, Find the OYP and OYT. 3. In Fig. 20.12, the incircle of PQR is drawn. If PX = 2.5 cm, RZ = 3.5 cm and perimeter of PQR = 18 cm , find the length of QY.

Fig. 20.11

Fig. 2012

4. Write an experiment to show that the lengths of tangents from an external point to a circle are equal. 20.8 INTERSECTING CHORDS INSIDE AND OUTSIDE A CIRCLE. You have read various results about chords in the previous lesson. We will now verify some results regarding chords intersecting inside a circle or outside a circle, when produced. Let us perform the following activity : Draw a circle with centre O and any radius. Draw two chords AB and CD intersecting at P inside the circle. Measure the lengths of the line-segments PD, PC, PA and PB. Find the products PA PB and PC PD. You will find that they are equal. Repeat the above activity with another two circles after drawing chords intersecting inside. You will again find that PA PB = PC PD Let us now consider the case of chords intersecting outside the circle. Let us perform the following activity : Draw a circle of any radius and centre O. Draw two chords BA and DC intersecting each other outside the circle at P. Measure the lengths of line segments PA, PB, PC and PD. Find the products PA PB and PC PD.

Fig. 20.13

150

Mathematics

You will see that the product PA PB is equal to the product PC PD, i.e., PA PB = PC PD Repeat this activity with two other circles with chords intersecting outside the circle. You will again find that PA PB = PC PD. Thus, we can say that If two chords AB and CD of a circle intersect at a point P (inside or outside the circle), then PA PB = PC PD. 20.8 INTERSECTING SECANTS AND TANGENTS OF A CIRCLE To see if there is some relation between the intersecting secant and tangent outside a circle, we conduct the following activity: Draw a circle of any radius with centre O. From an external point P, draw a secant PAB and a tangent PT to the circle. Measure the length of the line-segments PA, PB and PT. Find the products PA PB and PT PT or PT2. What do you find ? You will find that PA PB = PT2
Fig. 20.15 Fig. 20.14

Repeat the above activity with two other circles. You will again find the same result. Thus, we can say If PAB is a secant to a circle intersecting the circle at A and B, and PT is a tangent to the circle at T, then PA PB = PT2 Let us illustrate these with the help of examples : Examples 20.5 : In Fig. 20.16, AB and CD are two chords of a circle intersecting at a point P inside the circle. If PA = 3 cm, PB = 2 cm, PC = 1.5 cm, find the length of PD. Solution : It is given that PA = 3 cm, PB = 2 cm and PC = 1.5 cm Let we know that PD = x PA PB = PC PD 3 2 = (1.5) x
Fig. 20.16

Secants, Tangents and Properties

151

x=

3 2 3 2 = =4 15 . 1.5

Length of the line-segment PD = 4 cm. Example 20.6 : In Fig. 20.17, PAB is a secant to the circle from a point P outside the circle. PAB passes through the centre of the circle and PT is a tangent. If PT = 8 cm and OP = 10 cm, find the radius of the circle, using PA PB = PT2. Solution : Let x be the radius of the circle It is given that and OP = 10 cm PA = PO OA = (10 x) cm PB = OP + OB = (10 + x) cm PT = 8 cm We know that PA PB = PT2 (10 x) (10 + x) = 82 Fig. 20.17 or 100 x2 = 64 or x2 = 36 or x = 6 i.e., radius of the circle is 6 cm. Example 20.7 : In Fig. 20.18, BA and DC are two chords of a circle intersecting each other at a point P outside the circle. If PA = 4 cm, PB = 10 cm, CD = 3 cm, find PC. Solution : We are given that PA = 4 cm, PB = 10 cm, CD = 3 cm, Let PC = x We know that or, or PA PB = PC PD 4 10 = (x + 3) x x2 + 3x 40 = 0 (x + 8) (x 5) = 0 x=5 PC = 5 cm
Fig. 20.18

CHECK YOUR PROGRESS 20.2 1. In Fig. 20.19, if PA = 3 cm, PB = 6 cm, PD = 4 cm, find the length of PC. 2. If in Fig. 20.19, PA = 4 cm, PB = x + 3, PD = 3 cm and PC = x + 5, find the value of x.

Fig. 20.19

Fig. 20.20

Fig. 20.21

152

Mathematics

3. In Fig. 20.20, if PA = 4 cm, PB = 10 cm, PC = 5 cm, find PD 4. In Fig. 20.20, if PC = 4 cm, PD = (x + 5) cm, PA = 5 cm and PB = (x + 2) cm, find x. 5. In Fig. 20.21, PT = 2 7 cm, OP = 8 cm, find the radius of the circle, if O is the centre of the circle. 20.9 ANGLES MADE BY A TANGENT AND A CHORD. Let there be a circle with centre O and let XY be a tangent to the circle at point P. Draw a chord PQ of the circle through the point P as shown in the Fig. 20.22. Mark a point R on the major arc and let S be a point on the minor arc . The segment formed by the major arc and chord PQ is said to be the alternate segment of QPY and the segment and chord PQ is said to be the formed by the minor alternate segment to QPX. Let us see if there is some relationship between angles in the alternate segments and the angle between tangents and chord. Join QR and PR. Measure PRQ and QPY (See Fig. 20.22) What do you find ? You will see that PRQ = QPY Repeat this activity with another circle and same or different radius. You will again find that QPY = PRQ. Now measure QPX and QSP. You will again find that these angles are equal. Thus, we can state that The angles formed in the alternate segments by a chord through the point of contact of a tangent to a circle is equal to the angle between the chord and the tangent. This result is more commonly called as Angle in the Alternate Segment. Let us now check the converse of the above result. Draw a circle, with centre O, and draw a chord PQ and let it form PRQ in alternate segment as shown in Fig. 20.23. At P, draw QPY = QRP. Extend the line segment PY to both sides to form line XY. Join OP and measure OPY.
Fig. 20.22

Fig. 20.23

What do you observe ? You will find that OPY = 90 showing thereby that XY is a tangent to the circle.

Secants, Tangents and Properties

153

Repeat this activity by taking different circles and you find the same result. Thus, we can state that If a line makes with a chord angles which are equal respectively to the angles formed by the chord in alternate segments, then the line is a tangent to the circle. Let us now take some examples to illustrate. Example 20.8 : In Fig. 20.24, XY is tangent to a circle with centre O. If AOB is a diameter and PAB = 40, find APX and BPY. Solution : By the Alternate-segment theorem, we know that BPY = BAP Again, BPY = 40 APB = 90
Fig. 20.24

[Q BAP = 40 (Given)] [Angle in a semi-circle] (Angles on a line)

And, BPY + APB + APX = 180 APX = 180 (BPY + APB) = 180 (40 + 90) = 50. Example 20.9 : In Fig. 20.25, ABC is an isosceles triangle with AB = AC and XY is a tangent to the circumcircle of ABC. Show that XY is parallel to base BC. Solution : In ABC, AB = AC 1 = 2 Again XY is tangent to the circle at A. 3 = 2 1 = 3 But these are alternate angles XY || BC

Fig. 20.25

(Angle in the Alternate segment)

CHECK YOUR PROGRESS 20.3 1. Explain with the help of a diagram, the angle formed by a chord in the alternate segment of a circle. 2. In Fig. 20.26, XY is a tangent to the circle with centre O at a point P. If OQP = 40, find the value of a and b. 3. In Fig. 20.27, PT is a tangent to the circle from an external point P. Chord AB of the circle, when produced meets TP in P. TA and TB are joined and TM is the angle bisector of ATB.

154

Mathematics

Fig. 20.26 If PAB = 30 and ATB = 60, show that PM = PT. LET US SUM UP
z z z z z z

Fig. 20.27

A line which intersects the circle in two points is called a secant of the circle. A line which touches the circle at a point is called a tangent to the circle. A tangent is the limiting position of a secant when the two points of intersection coincide. A tangent to a circle is perpendicular to the radius through the point of contact. From an external point, two tangents can be drawn to a circle, which are of equal length. If two chords AB and CD of a circle intersect at a point P (inside or outside the circle), then PA PB = PC PD

If PAB is a secant to a circle intersecting the circle at A and B, and PT is a tangent to the circle at T, then PA PB = PT2

The angles formed in the alternate segments by a chord through the point of contact of a tangent to a circle are equal to the angles between the chord and the tangent. If a line makes with a chord angles which are respectively equal to the angles formed by the chord in alternate segments, then the line is a tangent to the circle. TERMINAL EXERCISE

1. Differentiate between a secant and a tangent to a circle with the help of a figure. 2. Show that a tangent is a line perpendicular to the radius through the point of contact, with the help of an activity.

Secants, Tangents and Properties

155

3. In Fig. 20.28, if AC = BC and AB is a diameter of the circle, find x, y and z.

Fig. 20.28

4. In Fig. 20.29, OT = 7 cm and OP = 25 cm, find the length of PT. If PT is another tangent to the circle, find PT and POT.
Fig. 20.29

5. In Fig. 20.30, the perimeter of ABC equals 27 cm. If PA = 4 cm, QB = 5 cm, find the length of QC. 6. In Fig. 20.30, if BAC = 70, find BOC. [Hint : OBC + OCB = 1 (ABC + ACB)] 2

Fig. 20.30

7. In Fig. 20.31, AB and CD are two chords of a circle intersecting at the interior point P of a circle. If PA = (x + 3) cm, PB = (x 3) cm, PD = 3 cm and PC = 5
1 cm, find x. 3
Fig. 20.31

8. In Fig. 20.32, chords BA and DC of the circle, with centre O, intersect at a point P outside the circle. If PA = 4 cm and PB = 9 cm, PC = x and PD = 4x, find the value of x.

Fig. 20.32

156

Mathematics

9. In Fig. 20.33, PAB is a secant and PT is a tangent to the circle from an external point. If PT = x cm, PA = 4 cm and AB = 5 cm, find x.

Fig. 20.33

10. In Fig. 20.34, O is the centre of the circle and PBQ = 40. Find (i) QPY (ii) POQ (iii) OPQ
Fig. 20.34

Secants, Tangents and Properties

157

ANSWERS Check Your Progress 20.1 1. (i) Perpendicular 2. 50, 50 3. 3 cm Check Your Progress 20.2 1. 4.5 cm 3. 8 cm 5. 6 cm Check Your Progress 20.3 2. a = b = 50 Terminal Exercise 3. x = y = z = 45 4. PT = 24 cm; PT = 24 cm, POT = 60 5. QC = 4.5 7. x = 5 9. x = 6 10. (i) 40 (ii) 80 (iii) 50. 6. BOC = 125 8. x = 3 2. 3 cm 4. 10 cm (ii) equal (iii) points of intersection (iv) two (v) no

Constructions

157

21
Constructions
21.1 INTRODUCTION One of the aims of the studying Geometry is to acquire the skill of drawing figures accurately. You have learnt how to construct geometrical figures namely triangles, squares and circles with the help of ruler and compasses. You have constructed angles of 30, 60, 90, 120 and 45. You have also drawn perpendicular bisector of a line segment and bisector of an angle. In this lesson we will extend our learning to construct some other important geometrical figures. 21.2 OBJECTIVES After studying this lesson, the learner will be able to :
z z

divide a given line segment internally in a given ratio. Construct a triangle from the given data (i) SSS (ii) SAS (iii) ASA (iv) RHS (v) perimeter and base angles (vi) base, sum/difference of the other two sides and one base angle.

(vii) two sides and a median corresponding to one of these sides.


z

Construct rectilinear figures such as parallelograms, rectangles, squares, rhombuses and trapeziums. Construct a quadrilateral from the given data (i) four sides and a diagonal (ii) three sides and both diagonals (iii) two adjacent sides and three angles

158

Mathematics

(iv) three sides and two included angles (v) four sides and an angle.
z z

Construct a triangle equal in area to a given quadrilateral. Construct tangents to a circle from a point (i) outside it (ii) on it using the centre of the circle Construct circumcircle of a triangle Construct incircle of a triangle.

z z

21.3 EXPECTED BACKGROUND KNOWLEDGE We assume that the learner already knows how to use a pair of compasses and ruler to construct
z z z z z

angles of 30, 45, 60, 90, 105, 120. the right bisector of a line segment bisector of a given angle. parallelograms, rhombuses, rectangles, and squares a circle

21.4 DIVISION OF A LINE SEGMENT IN THE GIVEN RATIO INTERNALLY Construction 1 : To divide a line segment internally in a given ratio. Given a line segment AB. You are required to divide it internally in the ratio 2 : 3. We go through the following steps. Step 1 : Draw a ray AC making an acute angle with AB. Step 2 : Starting with A, mark off 5 points C1, C2, C3, C4 and C5 at equal distances from the point A. Step 3 : Join C5 and B. Step 4 : Through C2 (i.e. the second point), draw C2D parallel to C5B meeting AB in D.

Fig. 21.1

Then D is the required point which divides AB internally in the ratio 2 : 3 as shown in Fig. 21.1

Constructions

159

CHECK YOUR PROGRESS 21.1 1. Draw a line segment 7 cm long. Divide it internally in the ratio 3 : 4. Measure each part. Also write the steps of construction. 2. Draw a line segment PQ = 8 cm. Find the point R on it such that PR = [Hint : Divide the line segment PQ internally in the ratio 3 : 1]. 21.5 CONSTRUCTION OF TRIANGLES Construction 2 : To construct a triangle when three sides are given (SSS) Suppose you are required to construct ABC in which AB = 6 cm, AC = 4.8 cm and BC = 5 cm. We go through the following steps : Step 1 : Draw AB = 6 cm. Step 2 : With A as centre and radius 4.8 cm, draw an arc. Step 3 : With B as centre and radius 5 cm draw another arc intersecting the arc of Step 2 at C. Step 4 : Join AC and BC. Then ABC is the required triangle. [Note : You may take BC or AC as the base] Construction 3 : To construct a triangle, when two sides and the included angle is given (SAS) Suppose you are required to construct a triangle PQR in which PQ = 5.6 cm, QR = 4.5 cm and PQR = 60 For constructing the triangle, we go through the following steps. Step 1 : Draw PQ = 5.6 cm Step 2 : At Q, construct an angle PQX = 60 Step 3 : With Q as centre and radius 4.5 cm draw an arc cutting QX at R. Step 4 : Join PR Then PQR is the required triangle. [Note : You may take QR = 4.5 cm as the base instead of PQ] Construction 4. To construct a triangle when two angles and the included side are given (ASA). Let us construct a ABC in which B = 60, C = 45 and BC = 4.7 cm.
Fig. 21.3 Fig. 21.2

3 PQ . 4

160

Mathematics

To construct the triangle we go through the following steps : Step 1 : Draw BC = 4.7 cm. Step 2 : At B, construct CBQ = 60 Step 3 : At C, construct BCR = 45 meeting BQ at A. Then ABC is the required triangle. Note : To construct a triangle when two angles and any side (other than the included side) are given, we find the third angle (using angle sum property of the triangle) and then use above method for constructing the triangle.
Fig. 21.4

Construction 5 : To construct a right triangle, when its hypotenuse and a side are given. Let us construct a right triangle ABC, right angled at B, side BC = 3 cm and hypotenuse AC = 5 cm. To construct the triangle, we go through the following steps. Step 1 : Draw BC = 3 cm Step 2 : At B, construct CBP = 90. Step 3 : With C as centre and radius 5 cm draw an arc cutting BP in A. Step 4 : Join AC ABC is the required triangle. Construction 6 : To construct a triangle when its perimeter and two base angles are given. Suppose we have to construct a triangle whose perimeter is 9.5 cm and base angle are 60 and 45. To construct triangle, we go through the following steps. Step 1 : Draw XY= 9.5 cm
Fig. 21.5

Fig. 21.6

Constructions

161

Step 2 : At X, construct YXP = 30 [Which is 1/2 60] Step 3 : At Y, construct XYQ = 22 [Which is 1/2 45] Let XP and YQ intersect at A Step 4 : Draw right bisector of XA intersecting XY at B. Step 5 : Draw right bisector of YA intersecting XY at C. Step 6 : Join AB and AC. ABC is the required triangle. Construction 7 : To construct a triangle when sum of two sides, third side and one of the angles on the third side are given. Suppose you are required to construct a triangle ABC. When AB + AC = 8.2 cm, BC = 3.6 cm and B = 45. To construct the triangle, we go through the following steps : Step 1 : Draw BC = 3.6 cm. Step 2 : At B, construct CBK = 45.

Fig. 21.7

Step 3 : From BK, cut off BP = 8.2 cm. Step 4 : Join CP. Step 5 : Draw right bisector of CP intersecting BP at A. Step 6 : Join AC ABC is required triangle.

162

Mathematics

Construction 8 : To construct a triangle when difference of two sides, the third side and one of the angles on the third side are given. Suppose we have to construct a ABC, in which BC = 4 cm B = 60, AB AC = 1.2 cm. To construct the triangle we go through the following steps : Step 1 : Draw BC = 4 cm. Step 2 : Construct CBP = 60. Step 3 : From BP cut off BK = 1.2 cm. Step 4 : Join CK Step 5 : Draw right bisector of CK meeting BP produced at A. Step 6 : Join AC ABC is the required triangle.
Fig. 21.8

Construction 9 : To construct a triangle when its two sides and a median corresponding to one of these sides, are given. Suppose you have to construct a ABC in which AB = 6 cm, BC = 4 cm and median CD = 3.5 cm. We go through the following steps. Step 1 : Draw AB= 6 cm. Step 2 : Draw right bisector of AB meeting AB in D. Step 3 : With D as centre and radius 3.5 cm draw an arc. Step 4 : With B as centre and radius 4 cm draw another arc intersecting the arc of Step 3 in C. Step 5 : Join AC and BC. Then ABC is the required triangle. CHECK YOUR PROGRESS 21.2 1. Construct a DEF, given that DE = 5.1 cm, EF = 4 cm and DF = 5.6 cm. Write the steps of construction as well. Note : You are also required to write the steps of construction in each of the remaining problems. 2. Construct a PQR, given that PR = 6.5 cm, P = 120 and PQ = 5.2 cm.
Fig. 21.9

Constructions

163

3. Construct a ABC given that BC = 5.5 cm, B = 75 and C = 45. 4. Construct a right triangle in which one side is 3 cm and hypotenuse is 7.5 cm. 5. Construct a right angled isosceles triangle in which one of equal side is 4.8 cm. 6. Construct a ABC given that AB + BC + AC = 10 cm, B = 60, C = 30. 7. Construct a ABC in which AB = 5 cm, A = 60, BC + AC = 9.8 cm. 8. Construct a LMN, when M = 30, MN = 5 cm and LM LN = 1.5 cm. 9. Construct a triangle PQR in which PQ = 5 cm, QR = 4.2 cm and median RS = 3.8 cm. 21.6 CONSTRUCTION OF RECTILINEAR FIGURES You are advised to draw rough sketch for the given data in each of the following constructions. You will observe, that it helps you to visualise/understand the steps of construction. Construction 10 : To construct a parallelogram when two adjacent sides and the included angle are given. Suppose that you have to construct a parallelogram in which the adjacent sides are 4 cm and 3 cm and included angle is 60. To construct the required parallelogram we go through the following steps : Step 1 : Draw AB = 4 cm Step 2 : At A, construct BAK = 60. Step 3 : From AK cut off AD = 3 cm. Step 4 : With B and D as centres and radii equal to 3 cm and 4 cm respectively draw two arcs cutting each other at C. Step 5. Join CD and BC. Then ABCD is the required parallelogram. Construction 11 : To construct a rectangle when one of its diagonal and a side are given. Suppose that you have to construct a rectangle ABCD in which AB = 4 cm and AC = 5.0 cm. Recall that in a rectangle, each angle is 90 and opposite sides are equal. To construct the rectangle we go through the following steps. Step 1 : Draw AB = 4 cm.
Fig. 21.10

164

Mathematics

Step 2 : At B, draw ABK = 90. Step 3 : With A as centre and radius 5 cm, draw an arc cutting BK at C. Step 4 : With C as centre and radius 4 cm, draw an arc. Step 5 : With A as centre and radius = BC, draw an arc cutting the arc drawn in Step 4 at D. Step 6 : Join DC and AD ABCD is the required rectangle. Construction 12 : To construct a square when its side is given. Suppose you have to construct a square PQRS in which PQ = 4.4 cm. We have to follow the following steps to construct the square : Step 1 : Draw PQ = 4.4 cm. Step 2 : Construct PQT = 90 at Q. Step 3 : From QT cut off QR = 4.4 cm. Step 4 : From P and R, draw two arcs of radii 4.4 cm each to cut each other at S. Step 5 : Join PS and RS. PQRS is the required square.
Fig. 21.12 Fig. 21.11

Construction 13 : To construct a parallelogram when two diagonals and the angle between them is given. Suppose that the lengths of two diagonals are 8 cm and 6 cm and the angle between them is 60. Recall that diagonals of a parallelogram bisect each other. To construct the parallelogram, we go through the following steps : Step 1 : Draw AC = 8 cm. Step 2 : Draw right bisector of AC meeting it at O. Step 3 : Construct COP = 60 and produce PO to Q. Step 4 : Cut off OB = OD = 3 cm (1/2 6, length of second diagonal) from OP and OQ. Step 5 : Join AB, BC, AD and CD. ABCD is the required parallelogram.
Fig. 21.13

Constructions

165

Construction 14 : To construct a rhombus when one diagonal and side are given. Suppose, you have to construct a rhombus, when one of its diagonal is 5.5 cm and the side is 3.3 cm. To construct the rhombus, we go through the following steps : Step 1 : Draw AC = 5.5 cm. Step 2 : With A as centre and radius 3.3 cm, draw two arcs one above AC and the other below AC. Step 3 : With C as centre and radius 3.3 cm draw two arcs one above AC and the other below AC intersecting the arcs of Step 2 in B and D respectively. Step 4 : Join AB, BC, CD and AD. ABCD is the required rhombus.
Fig. 21.13

Construction 15 : To construct a trapezium in which one of parallel sides, two non-parallel sides and the distance between parallel sides are given. Suppose you have to draw a trapezium in which one of parallel sides is 6 cm, two non-parallel sides are of length 4 cm and 5 cm and distance between parallel sides is 3 cm. To construct the trapezium we go through the following steps : Step 1 : Draw AB = 6 cm. Step 2 : At A, draw AP AB. Step 3 : From AP cut off AK = 3 cm. Step 4 : At K, draw KL AK. Step 5 : With A and B as centres and radii 4 cm and 5 cm respectively draw two arcs cutting KL at D and C respectively. Step 6 : Join AD and BC. Then ABCD is the required trapezium. CHECK YOUR PROGRESS 21.3 1. Construct a parallelogram if the lengths of its adjacent sides are 5.5 cm and 4 cm and the included angle is 75. Write steps of construction as well. Note : You are also required to write the steps of construction in each of the following problems.
Fig. 21.15

166

Mathematics

2. Construct a parallelogram if its sides are 4 cm and 6 cm and one of its diagonal is 7 cm. 3. Construct a parallelogram if its diagonals are 8 cm and 5 cm and the angle between them is 45. 4. Construct a rectangle whose sides are 4.2 cm and 3.4 cm. 5. Construct a square whose side measures 5.1 cm. 6. Construct a rhombus whose side is 5 cm and one diagonal is 8 cm. 7. Construct a rhombus whose diagonals measure 5 cm and 4 cm. 8. Construct a trapezium one of whose parallel side is 7 cm, non-parallel sides are 4.2 cm and 5.3 cm and the distance between the parallel sides is 3.5 cm. 21.7 CONSTRUCTION OF QUADRILATERALS You are advised to draw rough sketch from the given data in each of the following constructions. You will observe that it helps you to visualize/understand the steps of construction. Construction 16 : To construct a quadrilateral when four sides and one diagonal are given. Suppose you have to construct a quadrilateral ABCD in which AB = 3 cm, BC = 3.5 cm, CD = 4.1 cm, AD = 3.8 cm and diagonal BD = 5 cm. We go through the following steps : Step 1 : Draw AB = 3 cm. Step 2 : With A as centre and radius 3.8 cm draw an arc. Step 3 : With B as centre and radius 5 cm draw another arc intersecting the arc of Step 2 in D. Step 4 : Join AD and BD.
Fig. 21.16

Step 5 : With B and D as centres and radii 3.5 cm and 4.1 cm respectively, draw two arcs intersecting each other at C. Step 6 : Join BC and DC. ABCD is the required quadrilateral. Construction 17 : To construct a quadrilateral when three sides and both diagonals are given. Suppose you are required to construct a quadrilateral ABCD when AB = 3.6 cm, BC = 2.7 cm, AD = 3cm, AC = 4.8 cm and BD = 5 cm. To construct the required quadrilateral, we go through the following steps. Step 1 : Draw AB = 3.6 cm.

Constructions

167

Step 2 : With A and B as centres and radii 3 cm and 5 cm respectively draw two arcs intersecting each other at D. Step 3 : Join AD. Step 4 : With B and A as centres and radii 2.7 cm and 4.8 cm draw two arcs intersecting each other at C. Step 5. Join DC and BC. ABCD is the required quadrilateral. Construction 18. To construct a quadrilateral when two adjacent sides and three angles are given. Suppose you have to construct a quadrilateral ABCD in which AB = 5 cm, BC = 4.2 cm, B = 60, C = 90 and A = 75. Step 1 : Draw AB = 5 cm Step 2 : Construct BAP = 75. Step 3 : Construct ABQ = 60 Step 4 : Cut off BC = 4.2 cm from BQ Step 5 : At C, construct BCR = 90 cutting AP at D. ABCD is the required quadrilateral. Construction 19 : To construct a quadrilateral when three sides and two included angles are given. Suppose you have to construct a quadrilateral PQRS in which PQ = 3 cm, QR = 4 cm, RS = 6 cm, Q = 120 and R = 90. To construct the quadrilateral, we go through the following steps Step 1 : Draw QR = 4 cm. Step 2 : At Q and R, construct RQK = 120 and construct QRL = 90 Step 3 : From QK, cut off QP = 3 cm
Fig. 21.19 Fig. 21.18 Fig. 21.17

168

Mathematics

Step 4 : From RL, cut off RS = 6 cm Step 5 : Join PS. Then PQRS is the required quadrilateral. Construction 20 : To construct a quadrilateral when four sides and an angle are given. Suppose that you have to construct a quadrilateral ABCD in which AB = 5.5 cm, BC = 3.5 cm, CD = 4 cm, AD = 5 cm, and A = 45. To construct the quadrilateral ABCD, we go through the following steps. Step 1 : Draw AB = 5.5 cm Step 2 : At A, construct BAK = 45 Step 3 : Cut off AD = 5 cm from AK. Step 4 : With B and D as centres and radii 3.5 cm and 4 cm respectively, draw two arcs cutting each other at C. Step 5 : Join BC and DC ABCD is the required quadrilateral. CHECK YOUR PROGRESS 21.4 1. Construct a quadrilateral ABCD in which AB = 4 cm, BC = 5.2 cm, CD = 5.8 cm, DA = 6.5 cm and AC = 7.8 cm. Also write the steps of construction. Note : You are also required to write the steps of construction in each of the following problems. 2. Construct a quadrilateral ABCD in which AB = BC = CD = 4 cm, AC = 6 cm and BD = 7 cm. 3. Construct a quadrilateral ABCD in which AB = 6 cm, BC = 6.5 cm, A = 45, B = 120 and C = 90. 4. Construct a quadrilateral PQRS in which PQ = QR = 5 cm, RS = 6 cm, Q = 120 and R = 60 5. Construct a quadrilateral ABCD in which AB = 4 cm, BC =3.6 cm, CD = 5 cm, AD = 5.2 cm and B = 45. 6. Construct a quadrilateral PQRS in which PQ = 5 cm, RS = 6 cm, PS = 6.2 cm, PR = 7 cm and the diagonal PR makes an angle of 30 with PQ. 21.8 CONSTRUCTION OF A TRIANGLE EQUAL IN AREA TO A GIVEN QUADRILATERAL Construction 21 : To construct a triangle equal in area to a given quadrilateral. Suppose quad. ABCD is given
Fig. 21.20

Constructions

169

We have to construct a triangle equal in area to the quadrilateral ABCD. For that we go through the following steps : Step 1 : Join AC. Step 2 : Through D, draw a line segment DE || AC intersecting BC produced at E.

Fig. 21.21

Step 3 : Join AE. Then ABE is the required triangle. Construction 22 : To construct a quadrilateral and to construct a triangle equal in area to this quadrilateral. Suppose you have to construct a quadrilateral ABCD, in which AB = 3 cm, BC = 4.2 cm, CD = 3.6 cm, DA = 4.5 cm and B = 135 and then to construct a triangle equal in area to this quadrilateral. We go through the following steps : Step 1 : Draw AB = 3 cm. Step 2 : Construct ABK = 135 and cut off BC = 4.2 cm from BK. Step 3 : With C and A as centres and radii 3.6 cm and 4.5 cm respectively draw two arcs intersecting each other at D. Step 4 : Join AD and CD. ABCD is the quadrilateral. Step 5 : Join DB. Step 6 : Through C draw CL || DB meeting AB produced in E. Step 7 : Join DE Then DAE is the required triangle.
Fig. 21.22

170

Mathematics

CHECK YOUR PROGRESS 21.5 1. Draw a quadrilateral PQRS and construct a triangle equal in area to this quadrilateral. Also write the steps of construction. Note : You are required to write the steps of construction in each of the following problems. 2. Construct a quadrilateral ABCD in which, AB = 4 cm, BC = 3.5 cm, CD = 4.2 cm, DA = 3 cm and AC = 6 cm. Construct a triangle equal in area to this quadrilateral. 3. Construct a rectangle ABCD in which AB = 5 cm and BC = 3.5 cm. Construct a triangle, equal in area to the rectangle on AB as base. 21.9 CONSTRUCTION OF TANGENTS TO A CIRCLE Construction 23 : To draw a tangent to a given circle at a given point on it using the centre of the circle. Suppose C be the given circle with centre O and a point P on it. You have to draw a tangent to the circle. We go through the following steps. Step 1 : Join OP. Step 2 : At P, draw PT OP. Step 3 : Produce TP to Q. Then TPQ is the required tangent.
Fig. 21.23

Construction 24 : To draw tangents to a circle from a given point outside it. Suppose C be the given circle and a point A outside it. You have to draw tangents to the circle from the point A. For that, we go through the following steps : Step 1 : Join OA. Step 2 : Draw the right bisector of OA. Let R be mid point of OA. Step 3 : With R as centre and radius equal to RO, draw a circle intersecting the given circle at P and Q. Step 4 : Join AP and AQ. Then AP and AQ are the required tangents. CHECK YOUR PROGRESS 21.6 1. Draw a circle of 3 cm radius. Take a point A on the circle. At A, draw a tangent to the circle by using the centre of the circle. Also write steps of construction. 2. Draw a circle of radius 2.5 cm. From a point P outside the circle, draw two tangents PQ and PR to the circle. Verify that lengths of PQ and PR are equal. Also write steps of construction.
Fig. 21.24

Constructions

171

21.10. CONSTRUCTION OF CIRCUMCIRCLE AND INCIRCLE OF A TRIANGLE. Construction 25 : To construct circumcircle of a triangle. Suppose a ABC is given. You have to draw a circumcircle of this triangle. To construct it, we go through the following steps. Step 1 : Draw the given ABC. Step 2 : Draw right bisectors of any two sides ray BC and AC which meet each other at O. Step 3 : Join O with any one vertex say B. Step 4 : With O as centre and radius OB, draw a circle. This is the required circumcircle. Construction 26 : Construct a triangle with sides 4 cm, 5 cm and 6 cm. Draw a circumcircle of this triangle. We go through the following steps. Step 1 : BC = 5 cm. Step 2 : With B and C as centres and radii 4 and 6 cm respectively draw two arcs intersecting each other at A. Step 3 : Join AB and AC. ABC is the required triangle. Step 4 : Draw right bisectors of AB and BC which meet each other at O. Step 5 : Join OB. Step 6 : With O as centre and radius OB, draw a circle. This is the required circumcircle. Construction 27 : To construct incircle of a triangle. Suppose you have to construct a triangle ABC with AB = 4 cm, BC = 3.5 cm and B = 60 and draw its incircle. We go through the following steps Step 1 : Draw BC = 3.5 cm Step 2 : Draw CBM = 60 Step 3 : From BM, cut off BA = 4 cm
Fig. 21.27 Fig. 21.25

Fig. 21.26

172

Mathematics

Step 4 : Join AC ABC is the required triangle. Step 5 : Draw bisectors of B and C meeting each other at I. Step 6 : From I, draw IK perpendicular to BC meeting BC in D. Step 7 : With I as centre, and radius = ID, draw a circle. This is the required incircle. CHECK YOUR PROGRESS 21.7 1. Construct a ABC with AB = 5 cm, BC = 4.5 cm, B = 75 and draw its circumcircle. Also write steps of construction. Note : You are required to write the steps of construction in each of the following problems. 2. Construct an isosceles ABC with base BC = 4 cm and one of the equal sides AB = 3 cm and draw its circumcircle. 3. Construct a triangle with base 4 cm and base angles 60 and 75 and draw its incircle. 4. Construct an equilateral triangle of side 5 cm and draw its incircle. TERMINAL EXERCISE 1. Draw a line segment PQ = 8 cm long. Divide it internally in the ratio 3 : 5. Also write the steps of construction. Note : You are also required to write the steps of construction in each of the following problems. 2. Draw a line segment AB = 6 cm. Find a point C on AB such that AC : CB = 3 : 2. Measure AC and CB. 3. Construct a triangle with perimeter 14 cm and base angle 60 and 90. 4. Construct a right angled triangle whose hypotenuse is 8 cm and one if its other two sides is 5.5 cm. 5. Construct a ABC in which BC = 3.5 cm, AB + AC = 8 cm and B = 60. 6. Construct a ABC in which AB = 4 cm, A = 45 and AC BC = 1 cm. 7. Construct a parallelogram having its diagonals as 5 cm and 6 cm and angle between them is 45. 8. Construct a rectangle with sides 8 cm and 6 cm. Measure the length of its two diagonals. 9. Construct a square with one side 4.2 cm. Measure its diagonals.

Constructions

173

10. Construct a square if its diagonal is 7 cm. 11. Construct a rhombus when the diagonals are 9 cm and 7 cm. 12. Construct a trapezium with one of parallel sides as 6 cm, two non-parallel sides as 4 cm and 4.5 cm and distance between parallel sides as 2.5 cm. 13. Construct a trapezium ABCD in which AB = 8 cm, BC = 4.5 cm, CD = 4 cm, B = 60 and AB || CD. [Hint : C = 180 60 = 120] 14. Draw a quadrilateral ABCD in which AB = 4 cm, BC = 5 cm, CD = 4.5 cm, DA = 5 cm and AC = 7 cm. Construct a triangle equal in area to this quadrilateral. 15. Draw a circle of diameter 6 cm. From a point P outside the circle, draw two tangents to the circle. 16. Construct triangle with sides 4 cm, 3 cm and 5.5 cm and draw it circumcircle. 17. Construct a right angled triangle with base 4 cm and hypotenuse 6 cm and draw its incircle.

174

Mathematics

22
Co-ordinate Geometry
22.1 INTRODUCTION The problem of locating a village or a road on a large map can involve a good deal of searching. But the task can be made easier by dividing it into squares of managable size. Each square is identified by a combination of a letter and a number, or of two numbers, one of which refers to a vertical division of the map into columns, and the other to a horizontal division into rows.

4 3 2

C (i)

3 (ii) Fig. 22.1

Co-ordinate Geometry

175

In the above Fig. 22.1(i), we can identify the shaded square on the map by the coding, (B,2)or (4, 2) [See Fig. 22.1(ii))]. The pair of numbers used for coding is called ordered pair. If we know the coding of a particular city, roughly we can indicate its location inside the shaded square on the map. But still we do not know its precise location. The method of finding the position of a point in a plane very precisely was introduced by the French Mathematician and Philosopher, Rene Descates (15961650). In this, a point in the plane is represented by an ordered pair of numbers, called the Cartesian co-ordinates of a point. In this lesson, we will learn more about cartesian co-ordinates of a point, distance between two points in a plane, section formula and co-ordinates of the centroid of a triangle. 22.2 OBJECTIVES After studying this lesson, the learner will be able to :
z z z

fix the position of different points in a plane. find the distance between two different points whose co-ordinates are given. find the co-ordinates of a point, which divides the line segment joining two points, in a given ratio internally. find the co-ordinates of the mid-point of the join of two points. find the co-ordinates of the centroid of a triangle with given vertices. solve problems based on the above concepts.

z z z

22.3 EXPECTED BACKGROUND KNOWLEDGE


z z z

Idea of number line Fundamental operations on numbers. Properties of a right triangle.

22.4 CO-ORDINATE SYSTEM Recall that you have learnt to draw the graph of a linear equation in two variable in Lesson 6. The position of a point in a plane is fixed w.r.t to its distances from two axes of a reference, which are usually drawn by the two graduated number lines XOX and YOY, at right angles to each other at O (See Fig. 22.2). The horizontal number line XOX is called x-axis and the vertical number line YOY is called y-axis. The point O, where both axes intersect each other called the origin. The two axes together are called rectangular coordinate system. It may be noted that, the positive direction of x-axis is taken to the right of the origin O, OX and the negative direction is taken to the left of the origin O, i.e., the side OX. Similarly, the portion of y-axis above the origin O, i.e., the side OY is taken as positive and the portion below the origin O, i.e., the side OY is taken as negative.

176

Mathematics

Fig. 22.2

22.5 CO-ORDINATES OF A POINT The position of a point is given by two numbers, called co-ordinates which refer to the distances of the point from these two axes. By convention the first number, the x-co-ordinate (or abscissa), always indicates the distance from the y-axis and the second number, the y-coordinate (or ordinate) indicates the distance from the x-axis.

Fig. 22.3

In the above Fig 22.3, the co-ordinates of the points A and B are (3, 2) and (2, 4) respectively. You can say that the distance of the point A(3, 2) from the y-axis is 3 units and from the x-axis is 2 two units. It is customary to write the co-ordinates of a point as an ordered pair i.e., (x co-ordinate, y co-ordinate).

Co-ordinate Geometry

177

It is clear from the point A(3, 2) that its x co-ordinate is 3 and the y-co-ordinate is 2. Similarly x co-ordinate and y co-ordinate of the point B(2, 4) are 2 and 4 respectively. In general, co-ordinates of a point P(x, y) imply that distance of P from the y-axis is x units and its distance from the x-axis is y units. You may note that the co-ordinates of the origin O are (0, 0). The y co-ordinate of every point on the x-axis is 0 and the x co-ordinate of every point on the y-axis is 0. In general, co-ordinates of any point on the x-axis to the right of the origin is (a, 0) and that to left of the origin is (a, 0), where 'a' is a non-zero positive number. Similarly, y co-ordinates of any point on the y-axis above and below the x-axis would be (0, b) and (0, b) respectively where 'b' is a non-zero positive number. You may also note that the position of points (x, y) and (y, x) in the rectangular co-ordinate system is not the same. For example position of points (3, 4) and (4, 3) are shown in Fig 22.4.

Fig. 22.4

Example 22.1 : Write down x and y co-ordinate for each of the following points : (a) (1, 1) (b) (3, 2) (c) (7, 5) (b) x co-ordinate is 3 y co-ordinate is 2. (d) x co-ordinate is 2 y co-ordinate is 6. (d) (2, 6)

Solution : (a) x co-ordinate is 1 y co-ordinate is 1 (c) x co-ordinate is 7 y co-ordinate is 5.

178

Mathematics

Example 22.2 : Write down distances from y and x axes respectively for each of the following points : (a) A(3, 4) (b) B(5, 1) (c) C(3, 3) (d) D(8, 9)

Solution : (a) The distance of the point A from the y-axis is 3 units and from the x-axis is 4 units. (b) The distance of the point B from the y-axis is 5 units to the left of the origin and from the x-axis is 1 unit. (c) The distance of the point C from the y-axis is 3 units to the left of the origin and from the x-axis is also 3 units below the origin. (d) The distance of the point D from the y-axis is 8 units to the right of the origin and from the x-axis is 9 units below the origin. 22.6 QUADRANTS The two axes XOX and YOY divide the plane into four parts called quadrants.

Fig. 22.5

The four quadrants (See Fig. 22.5) are named as follows : XOY : I Quadrant ; YOX : II Quadrant YOX : IV Quadrant

X'OY' : III Quadrant ;

We have discussed in Section 22.4 that (i) along x-axis, the positive direction is taken to the right of the origin and negative direction to its left. (ii) along y-axis, portion above the x-axis is taken as positive and portion below the x-axis is taken as negative (See Fig. 22.6).

Co-ordinate Geometry

179

Fig. 22.6

Fig. 22.7

Therefore, co-ordinates of all points in the first quadrant are of the type (+, +) (See Fig. 22.7) Any point in the II quadrant has x co-ordinate negative and y co-ordinate positive [(, +)]. Similarly, in III quadrant, a point has both x and y co-ordinates negative [(, )] and in IV quadrant, a point has x co-ordinate positive and y co-ordinate negative [(+, )]. For example : (a) P(5, 6) lies in the first quadrant as both x and y co-ordinates are positive. (b) Q(3, 4) lies in the second quadrant as its x co-ordiante is negative and y co-ordinate is positive. (c) R (2, 3) lies in the third quadrant as its both x and y co-ordinates are negative. (d) S(4, 1) lies in the fourth quadrant as its x co-ordinate is positive and y coordinate is negative.

CHECK YOUR PROGRESS 22.1 1. Write down x and y co-ordinates for each of the following points : (a) (3, 3) (b) (6, 5) (c) (1, 3) (d) (4, 2)

2. Write down distances of each of the following points from the y and x axis respectively. (a) A(2, 4) (b) B(2, 4) (c) C(2, 4) (d) D(2, 4)

3. Group each of the following points quadrantwise : A(3, 2), F (6, 1), B (2, 3), G (4, 5), C(7, 6), H(11, 3), D(1, 1), P(3, 12), E(9, 9), Q(13, 6),

180

Mathematics

22.7 PLOTTING OF A POINT WHOSE CO-ORDINATES ARE GIVEN The point can be plotted by measuring its distances from the axes. Thus, any point (h, k) can be plotted as follows : (i) Measure OM equal to h along the x-axis (See Fig. 22.8). (ii) Measure MP perpendicular to OM and equal to k. Follow the rule of sign in both cases. For example points (3, 5) and (4, 6) would be plotted as shown in Fig 22.9.
Fig. 22.8

Fig. 22.9

22.8 DISTANCE BETWEEN TWO POINTS The distance between any two points P(x1, y1) and Q(x2, y2) in the plane is the length of the line segment PQ. From P, Q draw PL and QM perpendiculars on the x-axis and PR perpendicular on QM. Then,

OL = x1, OM = x2, PL = y1 and QM = y2 PR = LM = OM OL = x2 x1


Fig. 22.10

Co-ordinate Geometry

181

QR = QM RM = QM PL = y2 y1 Since PQR is a right angled triangle

PQ2 = PR2 + QR2 = (x2 x1)2 + (y2 y1)2 (By the Pythagoras Theorem)

PQ =

bx

x1

g + by
2

y1

Distance between two points =

bdifference of abscissaeg + bdifference of ordinatesg


2

Corollary : The distance of the point (x1, y1) from the origin (0, 0) is

bx 0g + by 0g
1 2 1

x12 + y12

Let us consider some examples to illustrate. Example 22.3 : Find the distance between each of the following points : (a) P(6, 8) and Q(9, 12) (b) A(6, 1) and B(6, 11) Solution : (a) Here the points are P(6, 8) and Q(9, 12) By using distance formula, we have PQ = = Hence,

b9 6g + mb12 8gr
2

152 + 202 =

225 + 400 =

625 = 25

PQ = 25 units.

(b) Here the points are A(6, 1) and B(6, 11) By using distance formula, we have AB = = Hence,

m6 b6gr + m11 b1gr


2

02 + 12 2 = 12

AB = 12 units.

Example 22.4 : The distance between two points (0, 0) and (x, 3) is 5. Find x. Solution : By using distance formula, we have the distance between (0, 0) and (x, 3) is

bx 0g + b3 0g
2

182

Mathematics

It is given that

bx 0g + b3 0g
2

=5

or

x 2 + 32 = 5

squaring both sides x2 + 9 = 25 or or Hence, x2 = 16 x= 4 x = + 4 or 4.

Example 22.5 : Show that the points (1, 1), (3, 0) and (1, 2) are collinear. Solution : Let P(1, 1), Q(3, 0) and R(1, 2) be the given points

Now,

b3 1g + b0 1g = 4 + 1 = 5 units QR = b 1 3g + b2 0g = 16 + 4 = 2 5 units RP = b 1 1g + b2 1g = 4 + 1 = 5 units PQ + RP = d 5 + 5 i units = 2 5 units = QR


PQ =
2 2 2 2

P, Q and R are collinear points.

Example 22.6 : Find the radius of the circle whose centre is at (0, 0) and which passes through the point (6, 8) Solution : Let A(0, 0) and B(6, 8) be the given points. Now, radius of the cirlce is same as the distance of the line segment AB.

AB = =

b6 0g + b8 0g
2

36 + 64 =

100
Fig. 22.11

= 10 Hence radius of the cirlce is 10 units. CHECK YOUR PROGRESS 22.2 1. Find the distance between each of the following pair of points : (a) (3, 2) and (11, 8) (c) (3, 4) and (2, 5) (b) (1, 0) and (0, 3) (d) (2, 11) and (9, 3)

Co-ordinate Geometry

183

2. Find the radius of the cirlce whose centre is at (2, 0) and which passes through the point (7,12) 3. Show that the points (5, 6), (1, 2) and (2, 1) are collinear. 22.9 SECTION FORMULA To find the co-ordinates of a point, which divides the line segment joining two points, in a given ratio internally. Let A(x1, y1) and B(x2, y2) be the two given points and P(x, y) be a point on AB which divides it in the given ratio m : n. We have to find the co-ordinates of P. Draw the perpendiculars AL, PM, BN on OX, and, AK, PT on PM and BN respectively. Then, from similar triangles APK and PBT, we have
Fig. 22.12

AP AK = KP = PB PT TB Now, AK = LM = OM OL = x x1 PT = MN = ON OM = x2 x

...(i)

KP = MP MK = MP LA = y y1, TB = NB NT = NB MP = y2 y
From (i), we have

m x x1 y y1 = = n x 2 x y2 y

The first two relations give


m x x1 = n x2 x

or or or

mx2 mx = nx nx1 x(m + n) = mx2 + nx1 x=


mx 2 + nx1 m+ n AP KP = , we get PB TB

Similarly, from the relation

y y1 m = y y which gives on simplification, n 2

184

Mathematics

y=

my 2 + ny1 m+n mx 2 + nx1 my 2 + ny1 , and y = m+n m+n

x=

...(i)

Hence co-ordinates of a point which divides the line segment joining the points (x1, y1) and (x2, y2) in the ratio m : n internally are :

FG mx + nx H m+n
2

my 2 + ny1 m+n

IJ K

22.9.1 Mid-Point Formula The co-ordinates of the mid-point of the line segment joining two points (x1, y1) and (x2, y2) can be obtained by taking m = n in the section formula above. Putting m = n in (1) above, we have x=
nx 2 + nx1 x 2 + x1 = n+n 2 ny 2 + ny1 y 2 + y1 = n+n 2

and

y=

The co-ordinates of the mid-point joining two points (x1, y1) and (x2, y2) are :

FG x H

+ x1 , 2

y 2 + y1 2

IJ K

Let us take some examples to illustrate : Example 22.7 : Find the co-ordinates of a point which divides the line segment joining each of the following points in the given ratio : (a) (2, 3) and (7, 8) in the ratio 2 : 3 internally (b) (1, 4) and (0, 3) in the ratio 1 : 4 internally. Solution : (a) Let A(2, 3) and B(7, 8) be the given points. Let P(x, y) divide AB in the ratio 2 : 3 internally. Using section formula, we have x=
2 7 + 3 2 20 = =4 5 2+3

Co-ordinate Geometry

185

and

y=

2 8 + 3 3 25 = =5 5 2+3

P(4, 5) divides AB in the ratio 2 : 3 internally.

(b) Let A(1, 4) and B(0, 3) be the given points. Let P(x, y) divide AB in the ratio 1 : 4 internally Using section formula, we have x= and
4 13 P , 5 5

1 0 + 4 1 4 = 1+ 4 5 1 3 + 4 4 13 = 1+ 4 5

b g

y=

b g

FG H

IJ K

divides AB in the ratio 1 : 4 internally.

Example 22.8 : Find the mid-point of the line-segment joining two points (3, 4) and (5, 12). Solution : Let A(3, 4) and B(5, 12) be the given points. Let C(x, y) be the mid-point of AB. Using mid-point formula, we have, x=
3+5 =4 2 4 + 12 =8 2

and

y=

C(4, 8) are the co-ordinates of the mid-point of the line segment joining two points (3, 4) and (5, 12).

Example 22.9 : The co-ordinates of the mid-point of a line segment are (2, 3). If co-ordinates of one of the end points of the line segment are (6, 5), find the co-ordiants of the other end point. Solution : Let other the end point be A(x, y) It is given that C(2, 3) is the mid point
We can write,
Fig. 22.13

2= or or

x+6 2

and or or

3=

y+5 2

4=x + 6 x = 2

6=y+5 y=1

A(2, 1) be the co-ordinates of the other end point.

186

Mathematics

22.10 CENTROID OF A TRIANGLE To find the co-ordinates of the centroid of a triangle whose vertices are given. Definition : The centroid of a triangle is the point of concurrency of its medians and divides each median in the ratio of 2 : 1. Let A(x1, y1), B(x2, y2) and C(x3, y3) be the vertices of the triangle ABC. Let AD be the median bisecting its base. Then, using mid-point formula, we have D=

FG x H

+ x3 y2 + y3 , 2 2

IJ K

Fig. 22.14

Now, the point G on AD, which divides it internally in the ratio 2 : 1, is the centroid. If (x, y) are the co-ordinates of G, then 2 x= x2 + x3 + 1 x1 x + x + x 2 3 2 = 1 2 +1 3 y2 + y3 + 1 y1 y + y + y 2 3 2 = 1 2 +1 3

2 y=

Hence, the co-ordinates of the centroid are given by

FG x H

+ x 2 + x 3 y1 + y 2 + y 3 , 3 3

IJ K

Example 22.10 : The co-ordinates of the vertices of a triangle are (3, 1), (10, 7) and (5, 3). Find the co-ordinates of its centroid. Solution : Let A(3, 1), B(10, 7) and C(5, 3) be the vertices of a triangle. Let G(x, y) be its centroid then, x=
3 + 10 + 5 =6 3

Co-ordinate Geometry

187

and

y=

1 + 7 + 3 =3 3

Hence G(6, 3) are the co-ordinates of the centroid. CHECK YOUR PROGRESS 22.3 1. Find the co-ordinates of the point which divides internally the line segment joining the points : (a) (1, 2) and (4, 7) in the ratio 1 : 2 (b) (3, 2) and (5, 4) in the ratio 1 : 1 2. Find the mid-point of the line joining : (a) (0, 0) and (8, 5) (b) (7, 0) and (0, 10) 3. Find the centroid of the triangle whose vertices are (5, 1), (3, 2) and (1, 8). LET US SUM UP
z

If (2, 3) are the co-ordinates of a point, then x co-ordinate (or abscissa) is 2 and the y co-ordinate (or ordinate) is 3. In any co-ordinates (x, y), x indicates the distance from the y-axis and 'y' indicates the distance from the x-axis. The co-ordinates of the origin are (0, 0) The y co-ordinate of every point on the x-axis is 0 and the x co-ordinate of every point on the y-axis is 0. The two axes XOX and YOY divides the plane into four parts called quadrants. The distance of the line segment joining two points (x1, y1) and (x2, y2) is given by :

z z

z z

bx
z z

x1

g + by
2

y1

The distance of the point (x1, y1) from the origin (0, 0) is

x12 + y12

The co-ordinates of a point, which divides the line segment joining two points (x1,y1) and (x2, y2) in a ratio m : n internally are given by :

FG mx + nx H m+ n
2

my 2 + ny1 m+ n

IJ K

188

Mathematics

The co-ordinates of the mid-point of the line segment joining two points (x1, y1) and (x2, y2) are given by :

FG x H
z

+ x1 y 2 + y1 , 2 2

IJ K
IJ K

The co-ordinates of the centroid of a triangle whose vertices are (x1, y1), (x2, y2) and (x3, y3) are given by :

FG x + x + x H 3
1 2

y1 + y 2 + y 3 3

TERMINAL EXERCISE 1. In Fig 22.15, AB = AC. Find x.

Fig. 22.15

2. The length of the line segment joining two points (2, 3) and (4, x) is x.

13 units. Find

3. Find the lengths of the sides of the triangle whose vertices are A(3, 4), B(2, 1) and C(4, 6). 4. Prove that the points (2, 2), (2, 1) and (5, 2) are the vertices of a right angled triangle. 5. Find the co-ordinates of a point which divides the join of (2, 1) and (3, 4) in the ratio of 2 : 3 internally. 6. Find the centre of a circle, if the end points of a diameter are P(5, 7) and Q(3, 11). 7. Find the centroid of the triangle whose vertices are P(2, 4), Q(7, 3) and R (4, 5).

Co-ordinate Geometry

189

ANSWERS Check Your Progress 22.1 1. (a) 3; 3 (b) 6; 5 (c) 1; 3 (d) 4; 2

2. (a) 2 units; 4 units (b) 2 units to the left of the origin; 4 units above the x-axis (c) 2 units to the left of the origin; 4 units below the origin (d) 2 units; 4 units below the origin 3. Quadrant I : B(2, 3), D(1, 1) and P(3, 12) Quadrant II : A(3, 2), F(6, 1) and Q(13, 6) Quadrant III : E(9, 9) and G(4, 5) Quadrant IV : C(7, 6) and H (11, 3) Check Your Progress 22.2 1. (a) 10 units 2. 13 units. Check Your Progress 22.3 1. (a) (2, 1) (b) (1, 1) 5 2. (a) 4 , 2 (b)

10 units

(c)

106 units

(d)

185 units

FH

IK

(b)

FH 7 , 5I K 2

3.

FH 1 , 5I 3 3K
2. 0 or 6

Terminal Exercise 1. 3 units 3. AB = 5. (0, 1)

26 units, BC =

29 units and CA =
7. (3, 2)

101 units

6. (1, 2)

Area of Plane Figures

193

Module 4
Mensuration
With the advancement of civilization on this planet earth, the human beings were faced with the following types of things in their immediate environment : part of the earth on which they live (plane), fruit and vegetables trees, birds and animals, stone crests and other solids or plane objects. The desire to know more about them, to have a measure of sizes and capacity and to acquire them was a natural phenomenon. With this came into being the science of measurement of land and knowledge about rectilinear and solid figures, which was given the name of mensuration. Old civilization like Greek, Egyptians, Arabian and especially Indian contributed a lot to the development of this science. In the old Indian scriptures, there are rules laid down to perform special types of yajnas to fulfil specific desires in specially built altars (vedis) These vedis were in the shape of squares, rectangles, and circles made of cuboidal, cubical and spherical bricks. The whole lot of Sulba Sutras and Srauta Sutras contain the knowledge of methods of construction of such vedis and the properties of the used shapes. Baudhayan Sulba Apastamba Sulba and Katyayan Sulba which date back to 8th century B.C. are famous among all these sutras. After that even the Indian mathematician like Aryabhatt (476 AD), Brahmgupta (598 AD), Bhaskaracharya (1114 AD) and Varahmihir (1120 AD) devoted much of their works in providing rules for finding perimeters and areas of plane figures and volumes of solids. One of the oldest text Surya Sidhant deals with the shapes (mostly spherical) and orbits (mainly elliptical) of heavenly bodies. In this text, we shall study about the perimeter and area of plane figures and surface area, total surface area and volumes of solids.

194

Mathematics

23
Area of Plane Figures
13.1 INTRODUCTION In our day-to-day life, we have to estimate the amount of wood required for a rectangular table top or the area of cloth required for table cover, or to find the area of paths, parks, plots and floor etc. In this lesson, we shall study the methods of finding perimeters and areas of plane figures, especially square, rectangle, triangle, special quadrilaterals and circle. We will also study methods to find areas of rectangular or circular paths. 23.2 OBJECTIVES After studying this lesson the learner will be able to :
z z z

identify rectilinear and non-rectilinear plane figures. explain the meaning of perimeter and area of closed figures. find the perimeter and area of rectilinear figures, like a square, a rectangle, a triangle, a quadrilateral, a trapezium, a parallelogram and a rhombus. find the area of a triangle using Heros formula. find the area of rectilinear paths of different types in a rectangle (or a rectangular enclosure). find the circumference and area of a circle. find area and perimeter of a sector. find the area of circular paths inside or outside a circle (or a circular enclosure). Solve problems from day-to-day life situations based on the above concepts

z z

z z z z

23.3 EXPECTED BACKGROUND KNOWLEDGE


z z

Measurement of line-segments Knowledge and conversion of units of measurements

Area of Plane Figures

195

z z z

Four fundamental operations on numbers Drawing plane geometric figures Properties of special quadrilaterals

23.4 PERIMETER OF PLANE FIGURES The distance covered to walk along the boundary of a plane figure is called its perimeter. In connection with the circle, this distance covered is called circumference of the circle. The perimeter is measured in terms of linear units. 23.5 AREA OF PLANE FIGURES The measure of the planar region enclosed by a plane figure is called its area. This is measured in terms of square units. 23.5.1 A Unit Square A square of side 1 cm is called a unit cm square and its area is taken to be 1 cm2 and its perimeter is taken to be 4 cm.

Fig. 23.1

Similarly a square of side 1 m is called a unit meter square and its area is taken to be 1 m2 and its perimeter as 4 m. In general, area of a square of side a unit is a2 sq. unit and its perimeter is 4a unit. Example 23.1 : Find the area of a square whose perimeter is 80 meters. Solution : Perimeter of square = 4a = 80 meters

a=

80 or 20 metres 4

Area of square = a2 sq. m

= 20 20 sq. m or 400 sq m.

196

Mathematics

Example 23.2. The area of a square park is 625 sq m. Find the length of the wire required to fence around the park. Solution. Area of square park (a2) = 625 sq. m

Side of square (a) =

625 m =

25 25 m or 25 m

Now,

length of wire = Perimeter of square = 4a = 4 25 m = 100 m

i.e. length of the wire required to fence the park is 100 m. Example 23.3. Find the area of a square whose diagonal is 15 m. Solution. Diagonal of the square ABCD = AC = =

AB2 + BC 2

a 2 + a 2 or

2a m

2 a = 15 or a =

15 m 2
225 or 112.5 sq m. 2

Area of the square ABCD = a2 sq m = 23.6. RECTILINEAR FIGURES

Fig. 23.2

The figures whose boundaries are formed by line-segments are called rectilinear figures. For example square, rectangle, triangle, trapezium, quadrilateral, parallelogram and rhombus are rectilinear figures while a circle and a sector are non-rectilinear figures. 23.7 PERIMETER AND AREA OF A RECTANGLE Take 4 centimetre squares and join them in a row as shown in Fig. 23.3(i). Add another row of four centimetre square over the and above the first row to form the rectangle ABCD as shown in Fig 23.3(ii).

(i) Fig. 23.3

(ii)

Area of Plane Figures

197

Rectangle ABCD is formed by 8 small unit squares. Therefore the planar region enclosed by it is 8 cm2. You can see that 4 2 = 8. If we take another rectangle PQRS with base formed by 5 unit squares and has four such rows (See Fig. 23.4), we see that it contains 20 or (5 4) unit cm squares.

Its area is 20 cm2 (5 4) cm2 20 cm2


Fig. 23.4

i.e. or

From here we can generalise that the area of a rectangle of length l cm and breadth b cm can be written as (l b) cm2 or lb cm2. In case of Fig 23.3(ii), the distance covered to go around the rectangle ABCD is (4 + 2 + 4 + 2) cm or 12 cm. Similarly, the distance covered to go around the rectangle PQRS in Fig. 23.4 is (5 + 4 + 5 + 4) cm or 18 cm. Do you observe that in each case the distance covered is obtained by adding 2 length and 2 breadth? i.e. Perimeter of a Rectangle = 2(l + b) A square is a special rectangle in which length and breadth are equal. Thus, the area of a square = (side)2 square units rectangle = (length breadth) square units And, the perimeter of a square = 4 side linear units rectangle = 2 (length + breadth) linear units Example 23.4 : The length and breadth of a rectangular field are 23.7 m and 14.5 m respecively. Find : (i) the area of the field (ii) the length of wire required to put a fence around the boundary of the field. Solution : (i) Area of the field = Length breadth

198

Mathematics

= (23.7 14.5) sq m = 343.65 sq m (ii) Length of wire = Perimeter of the field = 2(length + breadth) = 2(23.7 + 14.5) m = 2(38.2) or 76.4 m Example 23.5 : A rectangular plot measures 400 m 121 m. Find the side of a square plot having area equal to the rectangular plot. Solution : Area of rectangular plot = (400 121) sq m = (20)2 112 sq m

Area of square plot = (20)2 112 sq m Side of square plot =


202 112 m

= 220 m. 23.8. AREA OF A PARALLELOGRAM We know from our study of geometry that parallelograms on the same base and between the same parallels are equal in area.

Area (||gm ABCD) = Area Rect. PBCQ = (BC PB) sq units.


Fig. 23.5

Area of a Parallelogram = Base Altitude square units

Example 23.6 : Find the area of a parallelogram whose base and altitude are 12 cm and 8 cm respectively. Solution : We know that Area of ||gm ABCD = Base altitude = (12 8) cm2 = 96 cm2. 23.9. AREA OF A TRIANGLE We know that a diagonal of a parallelogram divides it into two triangles of equal area

Area of DBC = =

1 Area of ||gm ABCD 2 1 Base Altitude 2 1 Base Altitude square units 2

Area of a triangle =

Fig. 23.6

Area of Plane Figures

199

Example 23.7 : The base of a triangular field is three times its altitude. If the cost of cultivating the field at the rate of Rs 15 per square metre is Rs 20250, find the base and the altitude of the field. Solution : Area of the field = 20250 sq m 15

= 1350 sq m Let the altitude of the triangular field be x m


Base = 3x m Area of the field = 1 3 .3x.x = x 2 sq m 2 2

As given in the question, 3 x2 = 1350 2 or or x2 = 1350 2 = 900 3

x = 30

Thus, the base of the field is 90 m and its altitude is 30 m. 23.10 AREA OF A TRAPEZIUM In Fig. 23.7, ABCD is a trapezium in which AB || DC.
Area of trapezium ABCD

= Area of ABD + Area BCD = = 1 1 .a.h + .b.h 2 2 1 (a + b).h 2

Fig. 23.7

Thus, the area of a trapezium = 1 (sum of the parallel sides) Distance between them square units 2

Example 23.8 : Find the area of a trapezium lengths of whose parallel sides are 20 cm and 12 cm and the distance between them is 5 cm. Solution : We know that Area of Trapezium = 1 (Sum of the Parallel sides) Distance between them 2

200

Mathematics

= =

1 [20 + 12] 5 cm2 2

1 32 5 cm2 2 = 80 cm2. 23.11 AREA OF A QUADRILATERAL Area of quadrilateral ABCD = Area ABC + Area ACD (See Fig. 23.8) = = 1 AC. h + 1 AC. h 1 2 2 2 1 AC h + h 1 2 , 2
Fig. 23.8

where h1 and h2 are lengths of perpendiculars from vertices B and D on diagonal AC

Example 23.9 : Find the area of a field in the shape of a quadrilateral, one of whose diagonals is 40 m long and the perpendiculars from the other two vertices on this diagonal are 32 m and 18 m. Solution : Area of quadrilateral field ABCD = Area of ABC + Area of ACD 1 1 = 2 40 32 + 2 40 18 sq m = (640 + 360) sq m = 1000 sq m. 23.12 AREA OF RHOMBUS
Fig. 23.9

LM N

OP Q

In Fig. 23.10, ABCD is a rhombus whose diagonal AC and BD bisect each other at right angles. Thus : Area of rhombus ABCD = Area ABD + Area BCD = = 1 BD. AP + 1 BD. PC 2 2 1 1 BD (AP + PC) = (BD) (AC) 2 2

Thus, the area of rhombus = 1 product of its diagonals square units. 2


Fig. 23.10

Area of Plane Figures

201

Example 23.10. The diagonals of a rhombus are of length 12 cm and 8 cm. Find the area of the rhombus. Solution. We know that Area of a Rhombus = = 1 Product of Diagonals 2 1 (12 8) cm2 2

= 48 cm2 Example 23.11. Find the area of a rhombus, one of whose diagonals is 12 cm and the side is 10 cm. Solution : In Fig.23.11, let ABCD be the given rhombus with AB = BC = CD = AD = 10 cm and the diagonal BD = 12 cm. AC and BD bisect (at right angles) at O (See Fig. 23.11)

OB = OD =6 cm OA =
102 62 cm = 8 cm

Diagonal AC = 16 cm 1 (product of diagonals) 2 1 12 16 cm2 2


Fig. 23.11

Area of rhombus = =

= 96 cm2 23.13 AREA OF A TRIANGLE USING HEROS FORMULA The area of triangle ABC whose sides are given by a, b and c is given by
=

s s a s b s c

b gb gb g

where s =

a+b+c 2

This formula is known as Heros formula after the name of Greek Mathematician Heron of Alexendria. The formula was also obtained by the Indian Mathematicians Brahmagupta and Aryabhata. Example 23.12 : The sides of a triangular field are 165 m, 143 m and 154 m. Find the area of the field. Solution : Here we use the Heros formula for finding the area of the field

202

Mathematics

s s a s b s c

b gb gb g
a+b+c 2

where a, b, c are sides and s =

s=
=

165 + 143 + 154 = 462 or 231 m 2 2


231 231 165 231 143 231 154 sq m

= =

b gb 231b66gb88gb77g

gb

sq m

11 7 3 3 2 11 2 2 2 11 7 11 sq m

= 11 11 7 3 2 2 = 10164 sq m. Thus area of the triangular field = 10164 sq m Example 23.13 : Find the area of a field in the shape of a trapezium whose parallel sides are of length 11 m and 25 m and the non-parallel sides are of length 15 m and 13 m. Solution : Let ABCD be a field in the shape of a trapezium with AB || CD in which AB = 25 m, CD = 11 m, AD = 13 m and BC = 15 m [Fig. 23.12] Draw CE || DA and let CF be perpendicular to AB Now, Area of BCE = because s =
21 21 14 21 15 21 13

gb

gb

13 + 14 + 15 or 21 m 2 =

21 7 6 8 or 84 sq m
1 14 .CF = 7 CF 2
Fig. 23.12

Also,

area of BCE =

b g

7 CF = 84 or CF = 12 m

Area of trapezoidal field ABCD

= =

1 (sum of parallel sides) distance between them 2 1 (11 + 25) 12 sq m 2

= 216 sq m 23.14 AREA OF RECTANGULAR PATHS In Fig. 23.13, ABCD is a rectangular park of dimensions a b and let there be a path of uniform width c all around the park as shown

Area of Plane Figures

203

Area of path = Area of Rectangle PQRS Area of rectangle ABCD = (PQ QR) (AB BC) = [(a + 2c). (b + 2c)] (a b) = ab + 2bc + 2ac + 4c2 ab = 4c2 + 2c(a + b)
Fig. 23.13

In Fig. 23.14, ABCD is a rectangular park of dimensions a b, in which PQRS and EFGH are two perpendicular paths of width c, parallel to the sides of the park.

Fig. 23.14

Area of paths = Area of path EFGH + Area of PQRS Area of path QLMN = a.c + b.c c.c = (a + b c)c Let us take some examples to illustrate : Example 23.14 : A rectangular hall 15 m long and 12 m broad is surrounded by a verandah 2 m wide. Find the area of verandah. Solution : Dimension of rectangular enclosure PQRS are (15 + 2 2) m and (12 + 2 2)m or 19 m and 16 m From Fig. 23.15, we see that
Fig. 23.15

Area of Verandah = Area of rectangle PQRS Area of rectangle ABCD = (19 16 15 12) sq m = (304 180) sq m or 124 sq m

204

Mathematics

Example 23.15 : A rectangular piece of land measures 100 m by 80 m. From the centre of each side a path 5 m wide runs across up to the opposite side. Find the area of the paths. Solution : Area of paths = Area of rectangle ABCD + Area of rectangle PQRS Area of square QLMN

Fig. 23.16

= [(100 5) +(80 5) (5 5)] sq m = [500 + 400 25] sq m = 875 sq m CHECK YOUR PROGRESS 23.1 1. The area of a square field is 225 square metres. Find its perimeter. 2. What will be the diagonal of a square whose perimeter is 60 m? 3. The length and breadth of a rectangular field are 22.5 m and 12.5 m. Find (i) the area of the field, (ii) the length of the wire required to put a fence along the boundary of the field. 4. The sides of a rectangular field of area 726 sq metres are in the ratio 3 : 2. Find the length of its diagonal correct up to one decimal place. 5. Find the area of a triangular field whose sides are 50 m, 78 m and 112 m. Also, find the length of the perpendicular from the opposite vertex to the side measuring 112 m. 6. Find the area of a field in the form of a quadrilateral ABCD in which AB = 165 m, BC = 143 m, CD = 85 m, AD = 85 m and the diagonal AC = 154 m. 7. Find the area of a parallelogram with base and altitude of length 20 cm and 12 cm respectively. 8. The parallel sides of a trapezium are 20 metres and 16 metres long respectively and the distance between them is 12 m. Find its area. 9. The perimeter of a rhombus is 146 cm and one of its diagonals is 48 cm, find the other diagonal and the area of the rhombus. 10. Find the area of a quadrilateral one of whose diagonals is 30 metres long and the lengths of perpendiculars from the other two vertices to this diagonal are 10 m and 14 m respectively.

Area of Plane Figures

205

11. A rectangular courtyard 120 m long and 90 m broad has a path of uniform width 5 m on the inside running round it. Find the area of the path. 12. A rectangular lawn 80 metres by 60 metres has two roads, each 2 metres wide running in the middle of it, one parallel to the length and the other parallel to the breadth. Find the area of the roads. 23.15 CIRCUMFERENCE AND AREA OF A CIRCLE Recall that a circle is the locus of a point which moves in a plane in such a way that its distance from a fixed point (in the same plane) always remains constant. The fixed point (O) is called the centre of the circle and the constant distance is called the radius of the circle. Any straight line drawn through the centre of circle, whose end point lie on the circle is called a diameter, whose length is equal to twice the radius of the circle. We state without any logical proof that the ratio of the circumference of a circle to its diameter is always constant which is also equal to the ratio of the area enclosed by a circle to the square of its radius (r).

A C = 2 = constant. 2r r 1 The value of this constant is nearly 3 and is usually 7 denoted by the Greek letter (pie), which is an irrational number but its value, correct to four places of decimals is 3.1416, to avoid lengthy calculations the value of 22 is often taken to be . 7 C = or C = 2 r 2r
i.e. and
A = r2

Fig. 23.17

or

A = r2 .

Thus, circumference of a circle = 2r linear units and, the area of the circle = r2 square units. Example 23.16 : Find the circumference and area of a circle of radius 3.5 cm [Use = 22/7] Solution : Circumference of the circle = 2 r 22 35 . cm = 2 7 = 22 cm Also, area of the circle = r 2
22 3.5 3.5 cm2 7 = 38.5 cm2

206

Mathematics

Example 23.17 : The radius of a wheel is 42 cm. How many revolutions will it make in going 26.4 km ? Solution : Distance travelled in one revolution = Circumference of the wheel 22 42 = 2 r = 2 cm 7 = 264 cm
No. of revolutions required to travel 26.4 km

26.4 1000 100 264

= 10000 Example 23.18 : The difference between the circumference and the radius of a circle is 74 metres. Find the diameter of the circle. Solution : We have 2 r r = 74 or or or or r (2 1) = 74 r 44 1 = 74 7

FH

IK I rF H 37 7 K = 74
r= 74 7 or 14 m 37

The diameter of the circle = 2(14) m = 28 m.

23.16 PERIMETER AND AREA OF SECTOR OF A CIRCLE Two radii OA and OB enclose a portion of the circular region making central angle . The region is called a sector of the circle. In Fig. 23.18, sector AOB is the sector with central angle . Let l be the length of arc AB. The, AB = circumference 360 [Corresponding arcs subtend proportional central angles] or l = or l = 2 r . 2 r 360 360
Fig. 23.18

Area of Plane Figures

207

Perimeter of the sector = OA + OB + AB = 2r + 2 r 360

Also area of sector is given by


Area of sector AOB = 360 r 2

or

Area of sector AOB = r 2 . 360

23.17. AREA OF CIRCULAR PATHS If we have a circular field of radius r, surrounded by a path of uniform width (d) so that r + d = R (say) then, the area of the circular path = Area of the outer circle Area of the inner circle = ( R2 r2) sq unit = (R2 r2) sq unit Let us take some examples based on the above formula :
Fig. 23.19

Example 23.19 : A circular road runs round a circular garden. If the difference between the circumference of outer circle and inner circle is 66 meters, find the width of the road. Solution : Let the radius of inner circle be r and that of outer circle be R Width of the road = (R r) = d (say) Now, we have 2 R 2 r = 66 or 2 (R r) = 66

d = (R r) = 66 7 1 m = 10.5 m. 22 2
Fig. 23.20

Thus width of the road is 10.5 m 3 is 75 metres. Find 7 (i) the area of path. (ii) the cost of gravelling the path at Rs 7 per square meter. Solution : Here i.e. 3 2 r = 75 7 2 22 r = 528 7 7

Example 23.20 : A path of width 2 meters runs around a circular plot whose circumference

Fig. 23.21

208

Mathematics

or

r=

528 1 7 = 12 7 2 22

i.e., the radius of the plot is 12 m. (i) 22 14 2 12 2 Area of path = (R2 r2) = 7 = 22 52 2 1144 m = sq m 7 7

(ii)

Cost = Rs

FG 1144 7IJ H 7 K

= Rs 1144. Example 23.21. Find the area of sector of a circle whose radius is 6 cm when (a) the angle at the centre is 35 (b) when the length of arc is 22 cm Solution. (a) Area of sector = r 2 . 360 = 22 6 6 35 cm2 7 360

= 11 sq cm (b) Here length of arc l = 22 cm

2 r = 22 cm 360 Area of sector = r 2 . 360 = = 1 r.2 r 2 360 1 r. l 1 = 6 22 sq cm = 66 sq cm 2 2 CHECK YOUR PROGRESS 23.2 1. Find the areas and circumference of the circles with radius (i) 1.4 m (b) 4.9 m. 2. The difference between the circumference and diameter of a circle is 210 m. Find the radius of the circle.

Area of Plane Figures

209

3. The difference between the area of a circle and the square of its radius is 105 sq m. Find the circumference of circle. 4. The driving wheel of a locomotive engine, 1.4 m in radius, makes 70 revolutions in one minute. Find the speed of the train in km/hr. 5. A circular park of radius 21 m has a road 7 m wide on its outside all around. Find the area of the road. 6. Find the areas of sectors of a circle with radius 3.5 m having central angle (i) 60 (ii) 90.

7. The length of the minute hand of a clock is 7 cm. Find the area covered by the minute 22 hand in 6 minutes. Use = 7 . LET US SUM UP
z z z

LM N

OP Q

Area of a square of side a = a2 sq units and perimeter = 4a units Perimeter of rectangle = 2(length + breadth) units Area of rectangle = (length breadth) sq units Area of a triangle = 1 base height sq units 2 =
s s a s b s c sq units

b gb gb g

where a, b anc c are sides of the triangle and s=


z

a+b+c units 2

Area of a parallelogram = Base Altitude sq units Area of a trapezium = Area of a rhombus = 1 (sum of parallel sides) Distance between them sq units 2 1 (Product of the diagonals) sq units 2

Area of rectangular path = Area of outer rectangular enclosure Area of inner rectangular enclosure Area of perpendicular paths in the middle of a rectangular field = Area of path parallel to length + Area of path parallel to breadth Area of common square.

210

Mathematics

z z z

Circumference of a circle = 2 r units Area of a circle = r2 sq units Area of circular path = (R2 r2), where R > r sq units Area of a sector of a circle = r 2 . 360 Perimeter of a sector = 2 r + 2 r 360

LM N

OP units Q

TERMINAL EXERCISE 1. The side of a square park is 37.5 m. Find its area. 2. The perimeter of a square is 480 m. What is its area ? 3. How long will a man take to walk round the boundary of a square field of area 40000 sq meters at the rate of 4 km an hour ? 4. The length of a room is three times its breadth. If its breadth is 4.5 m, find the area of its floor. 5. The perimeter of a rectangle is 980 meters; and the length is to breadth is 5 : 2. Find its area. 6. Find the area of each of the following parallelograms : (i) side 20 m and the corresponding altitude 12 m. (ii) one side is 13m, second side is 14 m and the diagonal is 15 m. 7. The area of a rectangular field is 27000 sq meters and the ratio between its length and breadth is 6 : 5. Find the cost of wire required to go four times round the field at the rate of Rs 7 per 10 metres of length of wire. 8. Find the area of the following trapezia : Lengths of parallel sides (i) 30 m and 20 m (ii) 15.5 m and 10.5 m (iii) 17 m and 40 m (iv) 40 m and 22 m Perpendicular distance between them 15 m 7.5 m 14.6 m 12 m

9. Find the area of a piece of land in the shape of a quadrilateral one of whose diagonals is 20 m and lengths of perpendiculars from the other two vertices on the diagonal are of length 12 m and 18 m respectively.

Area of Plane Figures

211

10. Find the area of a field in the form of a trapezium whose parallel sides measure 48 m and 160 m and the non-parallel sides measure 50 m and 78 m respectively. 11. Find the area of a quadrilateral ABCD in which AB = 85 meters, BC = 143 metres, CD = 165 metres, DA = 85 metres and DB = 154 metres. 12. Find the areas of the following triangles whose sides are : (i) 25 m, 60 m, 65 m (ii) 60 m, 111 m, 153 m 13. The sides of a triangle are 51 m, 52 m, and 53 m. Find the perpendicular from the opposite side on the side of length 52 m. Also find the areas of the two triangles into which the original triangle is divided. 14. Find the area of a rhombus, one of whose diagonals measures 8 m and the side is 5 m. 15. The difference between the two parallel sides of a trapezium is 8 metres, the perpendicular between them is 24 metres and the area of the trapezium is 312 sq metres. Find the lengths of the two parallel sides. 16. A rectangular piece of land measures 200 m by 150 m. From the centre of each side a path of 10 m wide runs across up to the opposite side. Find the area of paths. 17. A rectangular plot of grass measures 65 m by 40 m. It has a path of uniform width 8 m all around inside it. Find the cost of spreading red sand stone on the path at the rate of Rs 5.25 per sq m. 18. A rectangular plot of land measuring 30 m by 20 m has two paths each 2 m wide on both the sides (inside and outside) of the boundary. Find the total area of the two paths. 19. A path 3 metres wide runs around a circular park whose radius is 9 meters. Find the area of the path. 20. The difference between the circumference and diameter of a circle is 30 m. Find the radius of the circle. 21. From the circular piece of cardboard with radius 1.47 m, a sector with central angle 60 has been removed. Find the area of the portion removed. 22. A circular plot with a radius of 15 m has a road 2 m wide running all around inside it. Find the area of the road.

212

Mathematics

ANSWERS Check Your Progress 23.1 1. 60 m 4. 39.7 m 7. 240 sq m 10. 360 sq m 2. 15 2 m 5. 1680 sq m, 30 m 8. 216 sq m 11. 2000 sq m 3. 281.25 sq m, 70 m 6. 12936 sq m 9. 55 m, 1320 sq m 12. 276 sq m

Check Your Progress 23.2 1. (i) 6.16 sq m, 8.8 m 2. 49 m 5. 1078 sq m 7. 15.4 cm. Terminal Exercise 1. 1406.25 sq m 4. 60.75 sq m 7. Rs 1848 8. (i) 375 sq m (ii) 97.5 sq m (iii) 416.1 sq m (iv) 372 sq m 9. 300 sq m 10. 3120 sq m 11. 12936 sq m 13. 45 m, 540 sq m, 630 sq m 16. 3400 sq m 19. 198 sq m 22. 176 sq m 2. 14400 sq m 5. 49000 sq m 3. 12 min 6. (i) 240 sq m (ii) 168 sq m (ii) 75.46 sq m, 30.8 m 3. 44 m 6. (i) 6.41 sq m 4. 36.96 km/hr (ii) 9.6 sq m

12. (i) 750 sq m (ii) 2754 sq m 14. 24 sq m 17. Rs 7476 20. 7 m 15. 17 m, 9 m 18. 400 sq m 21. 1.1319 sq m

Area of Plane Figures

213

Surface Area and Volume of Solids

213

24
Surface Area and Volume of Solids
24.1 INTRODUCTION In the previous lesson you have studied about plane figures i.e., figures which completely lie in a plane like squares, rectangles, triangles, circles etc. But the objects like a brick, a glass tumbler, a box, a football, an ice cream cone, etc. are not plane figures. They are called Solids. In this lesson we will study about these types of solids. 24.2 OBJECTIVES After studying this lesson, the learner will be able to :
z z z

identify different solids explain the meaning of surface area of a solid find the surface area of a cube, cuboid, cylinder, cone, sphere and a hemisphere using the respective formulae. find the area of four walls of a room explain the meaning of volume of a solid find the volume of a cube, cuboid, cylinder, cone, sphere and a hemisphere using respective formulae solve problems from day to day life situations based on the above concepts.

z z z

24.3 EXPECTED BACKGROUND KNOWLEDGE


z z z

Area of plane and rectilinear figures Circumference and area of a circle Four fundamental operations on numbers

24.4. SOLID FIGURES In Fig 24(i), we have a paper cut in the form as shown. It is a plane figure.

214

Mathematics

But when we fold the paper along the dotted lines, we can make a box (like a chalk box) as shown in Fig 24.1(ii).

(i) Fig. 24.1

(ii)

As the box occupies some part of the space, it has more than two dimensions. Such objects which occupy space (i.e., they have three dimensions) are called solids. The sum of the areas of the plane figures making up the boundary of a solid figure is called its surface area, [For example, the area of paper in Fig 24.1(i) is the surface area of box] and the measure of part of space occupied by a solid is called its Volume. Now, we shall take some solids like a cuboid, a cylinder, a cone and a sphere and learn to find their surface areas and volumes. 24.5 CUBOID A brick, chalk box, geometrical box, match box and a book are all examples of cuboids. Fig 24.2 represents a cuboid. It can be easily seen from the figure that a cuboid has six rectangular plane surfaces called faces. [ABCD, ABEF, BCHF, EFHG, ADGE and DCHG], and the opposite faces [like ABCD and EFHG] are congurent. Two adjacent faces meet along a line segment called an edge. For example, faces ABCD and BCHF meet along the edge BC. A cuboid has 12 edges in all. A cuboid has 8 corners called the vertices. A, B, C, D, E, F, G and H are the vertices of the cuboid represented by Fig 24.2.
Fig. 24.2

It can be seen that at every vertex, there are three edges meeting [called coterminous edges]. One of these edges is taken as length, the other the breadth and the third as height and are denoted by l, b and h respectively. The line-segment joining the vertex A to the vertex H is called a main diagonal of the cuboid.

Surface Area and Volume of Solids

215

Let AB= l, AE = b and AD = h, then the total surface area of cuboid = sum of the areas of six faces ABCD, EFGH, ADGE, BCHF, ABFE and DCHG = (lh + lh + bh + bh + lb + lb) = 2(lb + bh + hl)
Total surface area of a cuboid = 2(lb + bh + hl)

Main diagonal of a cuboid =

l 2 + b2 + h2

Cube : A cube is a special case of a cuboid when all the edges are equal i.e. l = b = h = a (say)
The surface area of a cube = 6a2

Main Diagonal of a cube =

3a

To find the volume of a cuboid, we first define the unit of measurement of volume, which is unit cube. A unit cube is the volume of a cube of side 1 unit. So, if the side of a cube is 1 cm, its volume is 1 cubic centimeter (or 1 cu. cm) and if the side is 1 m, the volume is 1 cu m. To find the volume of a cuboid, we are to find the number of unit cubes contained in it. Let us have a cuboid with sides 5 cm, 4 cm and 3 cm. In Fig. 24.3, you can easily find that the number of unit cubes in the cuboid are 5 4 3 = 60. So, the volume of the cuboid = 60 cu. cm.

Fig. 24.3

You can see that Volume = 60 cu. cm. = 5 cm 4 cm 3 cm = length breadth height

216

Mathematics

In general, we can deduce that Volume of a cuboid = length breadth height and Volume of cube = (edge)3 Let us now take some examples to illustrate the above formulae : Example 24.1 : Find the surface area and volume of a slab of stone measuring 3 m in length, 2 m in breadth and 25 cm in thickness. Solution : Here l = 3 m, b = 2 m and h = 25 cm = Surface area = 2(lb + bh + hl) 1 1 = 2 3 2 + 2 4 + 3 4 25 1 m= m 100 4

FH F 1 3I = 2 H6 + 2 + 4K

IK

sq m

sq m = 14.5 sq m. 1 cu m = 1.5 cu m. 4

Volume = l.b.h = 3 2

Example 24.2 : If the surface area of a cube is 96 sq cm, find its volume. Solution : The surface area of cube = 6 a2, where a is the side of cube

96 = 6 a2 a2 = 96 = 16 6 a = 4 cm

Volume of the cube = a3 = (4)3 cu cm = 64 cu cm.

Example 24.3 : A tank contains 60,000 cu. m of water. If the length and breadth are 50 m and 40 m respectively, find its depth. Solution : Volume of water in tank = 60,000 cu m Length of tank = 50 m Breadth of tank = 40 m Let depth of the tank be x m

50 40 x = 60,000 x=
60,000 = 30 m 50 40

Hence depth of the tank = 30 m.

Surface Area and Volume of Solids

217

Example 24.4 : If the volume of cube is 2197 cu cm, find the surface area and the length of the main diagonal of the cube. Solution : Volume of cube = (side)3 = 2197 cu cm = (13)3 cu cm

Side of cube = 13 cm = 6(13)2 sq cm = 1014 sq cm

Surface area of cube = 6 (side)2 sq units

In Fig 24.4, DF is diagonal of a cube


Length of the diagonal of a cube =

3 (side)

Fig. 24.4

3 .13 cm = 13 3 cm or 22.516 cm.

Example 24.5 : Five cubes each of edge 16 cm, are joined end to end. Find the surface area of the resulting cuboid. Solution : Let the cuboid obtained by joining five cubes be as shown in Fig 24.5

Fig. 24.5

l = 16 5 = 80 cm b = 16 cm h = 16 cm

Surface area of the resulting cuboid = 2(lb + bh + hl)

= 2(80 16 + 16 16 + 16 80) sq cm = 2(1280 + 256 + 1280) sq cm = 5632 sq cm. Example 24.6 : A wooden box 1.5 m long, 1.25 m wide and 65 cm deep and open at the top is to be made. Determine the cost of wood required for it, if one sq m of wood costs Rs 10. Solution : Surface area of wood required = lb + 2bh + 2lh [Q The box has five faces]

218

Mathematics

= 1.5 1.25 + 2(1.25 .65 + 1.5 .65) sq cm = 1.875 + 2 0.65 2.75 sq cm = 1.875 + 3.575 = 5.450 sq cm.
Cost of wood required for the box = Rs (5.45 10)

= Rs 54.50. Example 24.7 : A closed wooden box measures externally as 42 cm by 32 cm by 27 cm. The wood used is 1 cm thick. Find the internal capacity (volume) of the box. Solution : Here external dimensions are 42 cm, 32 cm, 27 cm Since the wood is 1 cm thick, so the internal dimension will be (42 2) cm, (32 2) cm, (27 2) cm

Volume = (40 30 25) cu cm = 30000 cu cm.

Example 24.8 : A river 10 meters deep and 100 meters wide is flowing at the rate of 4.5 km an hour. Find how many cubic meter of water runs into the sea per second. Solution : Rate of flow = 4.5 km/hr = 4500 5 m/sec = m/s 3600 4

5 Length of river bed per second = 4 m/s Breadth = 100 m and depth = 10 m

Volume of water flowing/sec =

5 100 10 cu m 4

= 1250 cu m. Example 24.9 : A field is 600 m long and 50 m broad. A tank 30 m long, 20 m broad and 12 m deep is dug in the field. The earth taken out of it is spread evenly over the field. Find the height of the field raised by it. Solution : Area of the field = (600 50) sq m = 30000 sq m Area of the tank = (30 20) sq m = 600 sq m Volume of earth taken out of tank = (30 20 12) cu m = 7200 cu m Area of the field, where the earth is to be spread = (30,000 600) sq m = 29400 sq m
Height of the field raised

12 7200 m= m. 29400 49

Surface Area and Volume of Solids

219

Example 24.10 : A cuboidal beam is 8 meters long, 50 cm broad and 20 cm thick. What is its cost at Rs 7000 per cubic meter ? 50 20 Solution : Volume of beam 8 100 100 = 4 cu m 5

FH

IK

cu m

Cost of the beam = Rs

FH 4 7000I K 5

= Rs 5600

CHECK YOUR PROGRESS 24.1 1. Fill in the blanks to make each of the following statements true : (i) Total surface area of a cuboid = ............... (ii) Surface area of a cuboid open at the top = ............... (iii) Total surface area of a cube = ............... (iv) Volume of a cuboid = ............... (v) Volume of a cube = ............... (vi) Length of diagonal of a cuboid = ............... 2. Find the volume and surface area of each of the following cuboids : Length (i) (ii) (iii) (iv) 6m 15 cm 10 m 3m Breadth 3m 10 cm 4m 2.5 m Height 2.5 m 5 cm 75 cm 1.5 m

3. Find the surface areas and the volumes of the cubes with edges (i) 5 cm (ii) 3.6 cm (iii) 1.6 m

4. Find the edges of a cube whose volume is (i) 3375 cu cm (ii) 2.197 cu m (iii) 15.625 cu cm.

5. A closed wooden box measures externally as 50 cm by 40 cm by 30 cm. The wood used in 2 cm thick. Find the capacity of the box. 6. The length of a room is 12 meters, width 8 meters and height 6 meters. How many boxes it can hold if each box occupies a space of 1.5 cu meters ?

220

Mathematics

7. A cuboidal box whose external dimensions including the lid are 32 dm, 27 dm, 12 dm is made of wood 1 dm thick. What is the capacity of the box and what is the volume of wood used in it ? 8. Find the total surface area of a wooden plank of width 3 m, thickness 0.75 m and of volume 33.75 cubic meters. 9. Three cubes each of side 8 cm are joined end to end. Find the surface area of the resulting cuboid. 10. The areas of three adjacent faces of a cuboid are a, b and c. It its volume is V, prove that V2 = abc. 24.6 RIGHT CIRCULAR CYLINDER A right circular cylinder is a solid generated by the revolution of a rectangle about one of its sides which remains fixed. Thus if the rectangle ABCD revolves about the side AB, it describes the cylinder shown in Fig 24.6. In our daily life, we come across many solids like water pipes, thin cans, beakers in the laboratory, powder box, which are right circular cylinders. We note that the ends (or bases) of a right circular cylinder are congurent circles and the line joining the centres A and B of these circles is perpendicular to the two ends. 24.6.1 Volume of a Cylinder = (area of the base) height = r2h cubic units where, r is the radius of the base and h is the height of the cylinder. 24.6.2 Curved Surface of a Cylinder Area of the rectangle obtained by cutting a hollow cylinder along any line on its surface parallel to the axis is called its curved surface area.
Fig. 24.6

Fig. 24.7

Surface Area and Volume of Solids

221

Curved surface area of a cylinder = 2 rh

If the cylinder is closed at both ends, then the total surface area = curved surface area + area of two ends = 2 rh + 2 r2 sq units = 2 r (r + h) sq units And if the cylinder is closed from one end and open form the other, then its curved surface area = 2 rh + r2 sq units. 22 Let us now take some examples to illustrate the above formulae. In all proboems use = 7 unless stated otherwise. Example 24.11 : Find the volume and the total surface area of a closed right cylinder whose radius is 7 m and height is 10 m. Solution : Here r = 7 m and h = 10 m
Volume = r2h

22 7 7 10 cu m 7

= 1540 cu m Total surface area = 2 r (r + h) =2 22 7 (7 + 10) sq m 7

= 22 34 = 748 sq m. Example 24.12 : A hollow cylindrical tube, open at both ends is made of iron 1 cm thick. If the external diameter is 12 cm and the length of tube is 70 cm, find the volume of iron used in making the tube. Solution : Here external radius (R) = 6 cm and and

internal radius (r) = 6 1 = 5 cm height (h) = 70 cm Volume of iron = External volume Internal volume = R2h r2h = h(R2 r2) =
22 70 (36 25) cu cm 7

= 220 11 cu cm = 2420 cu cm.

222

Mathematics

Example 24.13 : The diameter of a roller 1 m long is 70 cm. If it takes 200 revolutions to level a playground, find the cost of levelling at the rate of 75 paise per sq m. Solution : Here r = 35 cm = 0.35 m and h = 1 m

Curved surface area = 2 rh 22 35 1 = 2 sq m 7 100 = 2.2 sq m

Area swept in 200 revolution = 2.2 200 = 440 sq m Cost of levelling = 440 0.75 = Rs 330.

Example 24.14 : A field is 150 m long and 70 m broad. A circular tank of radius 5.6 m and depth 20 cm is dug in the field and the earth taken out of it is spread evenly over the field. Find the height of the field raised by it. Solution : Volume of earth dug out = r2h =

FH 22 56 56 20I K 7 10 10
2

cu m

= 1971.2 cu m. Area of field = (150 70) sq m = 10500 sq m

L 22 F 56I O Area of base of tank = M 7 H 10 K P sq m N Q


= 98.56 sq m
Area of the field where the earth is to be spread

= 10500 98.56 sq m = 10401.44 sq m


Height of the field raised

Volume 1971.2 = = 0.1895 m area 10401.44

= 18.95 cm. Example 24.15 : A cubic meter of iron is drawn into a wire of diameter 3.5 mm. Find the 22 length of the wire. = Use = 7 . Solution : Volume of iron melted = 1 cu m = 100 100 100 cu cm

LM N

OP Q

Surface Area and Volume of Solids

223

Let the length of the wire be x cm

7 Volume of wire = 40 =

FH IK

x cu cm

22 7 7 x cu cm 7 40 40

By the problem, 22 7 7 x = 100 100 100 7 40 40 x = 100 100 100 7 40 40 cm 22 7 7

1 40 40 = 10 km 22 7 = 103896 m (approx). Example 24.16 : A cylindrical bucket of diameter 28 cm and height 12 cm, is full of water. The water is emptied into a rectangular tub of length 66 cm and breadth 28 cm. Find the height to which water rises in the tub. Solution : Volume of water in the bucket = r2h = 22 14 14 12 cu cm 7

= 7392 cu cm. Let h be the height to which water rises in the tub.
Volume of water in the tub = 66 28 h cu cm By the problem,

66 28 h = 7392 or h= 7392 = 4 cm 66 28

i.e. water rises to a height of 4 cm in the tub. CHECK YOUR PROGRESS 24.2 1. Fill in the blanks to make each of the following statements true : (i) Volume of a right circular cylinder = ............... (ii) Surface area of a closed cylinder = ...............

224

Mathematics

(iii) Surface area of a cylinder open at the top = ............... (iv) Surface area of a cylinder open at both ends = ............... 2. Find the volumes and total surface area of the following cylinders : Radius (i) (ii) (iii) 7 cm 10 cm 5m Height 12 cm 3.5 m 1.4 m

3. A water storage tank has a cylindrical shape. If it is 2.1 m high and has a diameter 1 m, find its volume. 4. A rectangular piece of paper 33 cm long and 16 cm wide is rolled along its breadth to get a cylinder of height 16 cm. Find the volume of the cylinder. 5. The volume of right circular cylinder is 3080 cu cm and the radius of base is 7 cm. Find the curved surface area of the cylinder. 6. A hollow cylindrical tube, open at both ends, is made of iron 2 cm thick. If the external diameter be 50 cm and the length of tube be 140 cm, find volume of iron used in making the tube. 7. A well, with 10 meters inside diameter, is dug 14 meters deep. Earth taken out of it has been spread all-around it to a width of 5 meters to form an embankment. Find the height of the embankment. 8. The radii of two cylinders are in the ratio 3 : 2 and their heights are in the ratio 7 : 4. Calculate the ratios of their volumes and of the cured surface areas. 9. The diameter of a garden roller is 2.8 m and it is 1.5 m long. How much area will it cover in 100 revolutions ? 10. Find the whole surface area of a hollow cylinder open at the ends, if its length is 8 cm, the external diameter is 10 cm and the thickness is 2 cm ( = 3.1416). 24.7 SURFACE AREA AND VOLUME OF RIGHT CIRCULAR CONE A right circular cone is a solid generated by the revolution of a right angled triangle about one of the its sides containing the right angle as axis. In Fig 24.8 right triangle AOB revolves along AO to generate the cone of height h and radius r and slant height l = AC. 24.7.1 Surface Area of a Right circular cone To find the total surface of a cone, let us cut it along its slant height, and spread it into a sector ABC as shown in Fig 24.9.
Fig. 24.8

Surface Area and Volume of Solids

225

Fig. 24.9

Thus curved surface area = =

1 . radius AC . arc BC. 2 1 . l.2r = rl sq. units 2

Since area of the base is r2, so total surface area = rl + r2 = r(l + r) 24.7.2 Volume of a Right circular cone Volume of a cone = =
1 area of base height 3 1 2 r h cu units. 3

24.8 SURFACE AREA AND VOLUME OF A SPHERE A sphere is a solid generated by the revolution of a semicircle about its diameter. It can also be defined as under : A sphere is the locus of a point which moves in space such that its distance from a fixed point in space remains constant. The fixed point is called the centre of sphere and the constant distance is called the radius of sphere. In Fig. 24.10, O is the centre and OP = r is the radius. To find the surface area and the volume of a sphere, we use the following formulae. Surface area of sphere = 4 r2 Volume of sphere = 4 r 3 3
Fig 24.10

226

Mathematics

24.8.1 Hemisphere A hemisphere is obtained by cutting a sphere into two equal halves by a plane passing through its centre [Fig. 24.11] Thus, curved surface area of a hemisphere = 2 r2 Volume of a hemisphere = 2 r 3 3
Fig 24.11

Total surface area of a solid hemisphere = 2 r2 + r2 = 3 r2 Let us now take some examples :

Example 24.17 : Find the volume, curved surface area and the total surface area of a right circular cone, the radius of whose base is 14 m and the height is 9 m. Solution : Here r = 1.4 m and h = 9 m

Volume of cone = =

1 r 2 h 3 1 22 14 14 9 cu m 3 7 10 10

= 18.48 cu m l= =
r 2 + h2 =

b1.4g

+ 92
Fig 24.12

196 . + 81 = 82.96 = 9.1 m

Curved surface area = rl = 22 14 9.1 sq m 7 10

= 40.04 sq m Total surface area = rl + r2 22 14 14 = 40.04 + sq m 7 10 10 = 40.04 + 6.16 sq m = 46.20 sq m.

Surface Area and Volume of Solids

227

Example 24.18 : Find the surface area and the volume of a sphere of radius 10.5 cm. Solution : Here radius = 10.5 cm 4 3 4 22 21 21 21 Volume = 3 r = 3 7 2 2 2 cu cm = 4851 cu cm Surface area = 4 r2 22 21 21 = 4 sq cm 7 2 2 = 1386 sq cm. Example 24.19 : If the radius of a sphere is tripled, what is the ratio of the volume of original sphere to that of the second ? Solution : Let the radius of sphere = r units

Volume of sphere V1 =

4 r 3 cu units 3

When the radius is tripled, new radius = 3 r units

Volume of sphere V2 =

4 3r 3

b g

4 r 3 3 3 cu units 3

bg

Ratio is 1 : 27.

4 r 3 V1 3 = 1 V2 = 4 3 r .27 27 3

Example 24.20 : Find the diameter of a sphere whose volume is 38808 cu cm. Solution : Let r be the radius of the sphere Then its volume = By question, 4 r 3 = 38808 3

4 r 3 3

r3 =

38808 3 7 4 22 = 441 21 = (21)3 r = 21

Radius of the sphere = 21 cm.

Thus diameter of the sphere is (21 2) or 42 cm.

228

Mathematics

Example 24.21 : Find the radius of the base of right circular cone of height 10.5 cm and volume 176 cu cm. Solution : Let r be the radius of the base of the cone. Its height = 10.5 cm Then its volume = =

1 r 2 h 3 1 22 r 2 10.5 cu cm 3 7

1 22 r 2 10.5 = 176 3 7

r2 =

176 3 7 = 16 22 10.5

r=4

Hence radius of the base = 4 cm. Example 24.22 : Rain water which fills a tub of dimensions 6 m 4 m 2.75 cm is transferred to a cylindrical vessel of radius 20 cm. Find the height of water in the cylindrical vessel. Solution : Volume of rain water collected in the tub 275 cu m = 0.66 cu m = 64 10000 Let the height of the water raised in the cylindrical vessel be h m
Volume of water in cylindrical vessel = r2h

= =

22 20 20 h cu m 7 100 100 22 h cu m 175

22 h = 0.66 175 h= 66 175 = 5.25 m. 100 22 CHECK YOUR PROGRESS 24.3

1. Fill in the blanks to make each of the following statements true : (i) Curved surface area of a cone = ............... (ii) Total surface area of a cone = ............... (iii) Volume of a cone = ...............

Surface Area and Volume of Solids

229

(iv) Surface area of a sphere = ............... (v) Volume of a sphere = ............... (vi) Volume of a hemisphere = ............... (vii) Total surface area of a solid hemisphere = ............... 2. Find the volume, the curved surface area and the total surface area of right circular cone with the following dimensions : Radius (r) (i) (ii) (iii) 21 cm 14 cm 3.5 m Height (h) 28 cm 12 cm 12 m

3. How many meters of cloth 3 m wide will be required to make a conical tent, the radius of whose base is 5 m and whose height is 12 m ? 4. The area of the base of a right circular cone is 616 sq cm and its height is 9 cm. Find its volume. 5. A conical cup of height 15 cm has a base radius of 12.6 cm. The cup is full of water. The water is poured into a cylinder of base radius 7 cm. Find the height to which water will rise in the cylinder. 6. A conical tent is 6 m high with radius of base as 8 m (a) Find the cost of cloth required to make the tent, if one square meter of cloth costs Rs 30. (b) How many persons can sit in the tent if each person requires 4 sq m of space on the ground and 20 cu meter of air to breath in. [Use = 3.14] 7. Find the slant height and curved surface area of cone whose volume is 12936 cu cm and the diameter of whose base is 42 cm. 8. Find the volume and the surface area of a sphere of radius 2.1 cm. 9. The radius of an iron sphere is 3.5 cm. It is melted to form smaller spheres of diameter 1.75 cm. Find the number of smaller spheres formed. 10. A cone, a hemisphere and a cylinder stand on equal bases and have same height. Show that their volumes are inthe ratio 1 : 2 : 3. 11. Find the radius of the base of a right circular cone of height 21 cm and volume 550 cu cm. 12. Find the diameter of a sphere of volume 4851 cu cm.

230

Mathematics

24.8 AREA OF FOUR WALLS OF A ROOM Let Fig 24.15(i) represents the four walls of a room of length (l), breadth (b) and height (h). Suppose a box open on both sides, is to be made of thin cardboard and cut along its height AB. We can spread out into one continuous rectangle as shown in Fig 24.15(ii).

(i)
Fig. 24.15

(ii)

Length of this rectangle = Perimeter of room = 2(length + breadth) = 2(l + b) Breadth of this rectangle = height of room = h
Area of four walls of room = Perimeter Height

= 2(l + b) h. Let us take some examples and illustrate : Example 24.23 : A room, 7 meters long, 4 meters broad and 3 meters high, has two windows 1 1 m 1 m and two doors 2 m 1 1 m. Find the cost of papering the walls with paper 50 2 2 cm wide at Rs 15 per meter. Solution : Area of four walls 2(l + b) h = 2(7 + 4) 3 = 66 sq m Area of two windows = 2 1
1 1 = 3 sq m 2 1 = 6 sq m 2

Area of two doors = 2 2 1

Area of walls to be papered = 66 (3 + 6) = 57 sq m

Length of the paper =

57 = 114 m .5

Cost = Rs 114 15 = Rs 1710.

Surface Area and Volume of Solids

231

Example 24.24 : The length of a room is 6.5 m. The cost of painting the walls at Rs 28 per square meter is Rs 2464, and the cost of carpeting the room at Rs 112 per square meter is Rs 3276. Find the height and width of the room. Solution : Length of the room = 6.5 m Let breadth of the room be b m and height of the room = h m

Area of four walls = 2(6.5 + b).h sq m 2(6.5 + b) h 28 = 2464 (6.5 + b) h = 44 Area of floor = l b = 6.5 b sq m Cost of carpeting = Rs 6.5 b 12 ...(i)

Cost of painting the walls = 2(6.5 + b).h 28 or

6.5 b 112 = 3276 b = 4.5 ...(ii)

or,

From (i), we have (6.5 + 4.5) h = 44 or h=4 m

Hence, the height of room = 4 m and its width = 4.5 m. Example 24.25 : The area of the floor of a room is 77 square meters. The area of the two larger walls together is 88 square meters; and the two shorter walls together is 56 square meters. Find the dimensions of the room. Solution : Let length of room be l, breadth b and height h Then or and or

l b = 77 2(l h) = 88 l h = 44 2 (b h) = 56 b h = 28 l 44 = 11 = b 28 7 l= 11 b 7

...(i) ...(ii) ...(iii)

or Putting in (i) we get

11 2 b = 77 7

232

Mathematics

b2 = or

77 7 11

b=7 m l= 11 7 44 = 11 m and h = =4m 7 11

The length of the room is 11m, its breadth is 7 m and its height is 4 m.

CHECK YOUR PROGRESS 24.4 1. Find the area of four walls of the room in each of the following cases : (i) l = 8 m, b = 6 m, h = 3m. (ii) l = 20 m, b = 12 m, h = 8 m 2. A room is 6 m long, 5 m wide and 4 m high. The doors and windows in the room occupy a space of 5 square meters. Find the cost of papering the remaining portion of the walls with paper 75 cm wide, at the rate of Rs 1.20 per meter. 3. Find the cost of painting the walls and the ceiling of a room measuring 10 m 6m 3 m at the rate of Rs 1.50 per square meter. 4. A room measures 9 m 7m 3m. It has three doors, each having an area of 3 square meters and 4 windows each measuring 1.25 1 m. Find the cost of papering the remaining portion of the walls at the rate of Rs 1.50 per square meter. 5. The area of two side walls of a room is 5250 sq dm and the area of the two end walls 4550 sq dm, and the area of the floor is 4875 sq dm. Find the dimensions of the room. LET US SUM UP
z

The figures, which occupy space and have more than two dimensions, are called Solid figures. The sum of the areas of the plane figures making up the boundary of a solid object is called its surface area. The amount of space occupied by the solid object is called its Volume. Total surface area of a cuboid = 2(lb + bh + hl) Total surface area of a cube = 6a2, where a is its side. Volume of a cuboid = l b h. Volume of a cube = a3. A right circular cylinder is a solid generated by revolution of a rectangle about one of its sides, which remains fixed.

z z z z z z

Surface Area and Volume of Solids

233

z z z z

Volume of a cylinder = r2h. Total surface area of an cylinder open at one end = 2 rh + r2 Total surface area of a cylinder closed at both ends = 2 rh + 2 r2 = 2 r (h + r) A right circular cone is a solid generated by the revolution of a right triangle about one of its sides containing the right angles as axis. Curved surface area of a cone = rl, where l2 = r2 + h2 Total surface area of a solid cone = rl + r2 Volume of a cone =
1 2 r h 3

z z

A sphere is a solid generated by the revolution of a semicircle about its diameter.


z

Surface area of a sphere = 4 r2 Volume of a sphere =


4 3 r 3

z z

Curved surface area of a hemisphere = 2 r2 Total surface area of a solid hemisphere = 3 r2 Volume of a hemi-sphere =
2 3 r 3

Area of four walls of a room = 2(l + b) h. TERMINAL EXERCISE

1. Find the edge of a cube of volume equal to the volume of a cuboid of dimensions 63 cm 56 cm 21 cm. 2. Find the number of 5 cm cubes that can be cut out of a 15 cm cube. 3. Three cubes of metals whose edges are 3, 4 and 5 cm respectively are melted and formed into a single cube. If there is no waste in the process, find the edge of the new cube so formed. 4. A school room is to be built to accommodate 70 children, so as to allow 2.2 sq m of floor area and 11 cu m of space for each child. If the room is to be 14 m long, what must be its breadth and height ? 5. How many bricks 20 cm 10 cm 7.5 cm be carried by a truck whose capacity to carry load is 6 metric tons ? One cubic meter of bricks weighs 2000 kg. [1 metric ton = 1000 kg]

234

Mathematics

6. A field is 200 m long and 75 m broad; and a tank 40 meter long, 20 meter broad and 10 meter deep is dug in the field, and the earth taken out of it is spread evenly over the field. How much is the level of field raised ? 7. Four cubes each of sides 5 cm are joined end to end. Find the surface area of the resulting cuboid. 8. The sides of an open box are 0.5 cm thick and bottom is 1 cm thick. If the internal length, breadth and depth are respectively 14 cm, 10 cm ad 8 cm, find the quantity of material used in the construction of the box. 9. Find the whole surface area of a hollow cylinder open at the ends, if its length is 10 cm, the internal diameter is 8 cm and the thickness is 1 cm [Use = 3.14] 10. A cubic cm of gold is drawn into a wire 1/5 mm in diameter; find the length of wire. ( = 3.14) 11. A well with 8.4 meter inside diameter is dug 10 meter deep. Earth taken out of it has been spread all around it to a width of 4 meters to form an embankment. Find the height of the embankment. 12. Find the slant height of a cone whose volume is equal to 12936 cubic meters and the diameter of whose base is 42 meters. 13. The volume of a cone is 616 cubic meters. If the height of cone is 27 meters, find the radius of its base. 14. A conical vessel of internal radius 14 cm and height 36 cm is full of water. If this water is poured into a cylinder with internal radius 21 cm, find the height to which the water rises in the cylinder. 15. Find the diameter of a sphere whose volume is 606.375 cubic meter. 16. A room 12 meters long, 4 meters broad and 3 meters high has two windows 2 m 1 m and a door 2.5 m 2 m. Find the cost of papering the walls with paper 50 cm wide at Rs 20 per meter. 17. A hall, whose length is 15 m and breadth is twice its height, takes 250 meters of paper 2 meters wide for its four walls. Find the area of the floor. 18. The length of a room is 1.5 times its breadth. The cost of carpeting it at Rs 150 per square meter is Rs 14400 and the cost of white washing the four walls at Rs 5 per square meter is Rs 625. Find the dimensions of the room.

Surface Area and Volume of Solids

235

ANSWERS Check Your Progress 24.1 1. (i) 2(lb + bh + hl) (iv) lbh 2. (i) 45 cu cm, 81 sq cm (iii) 30 cu m, 101 sq m 3. (i) 150 sq cm, 125 cu cm (iii) 15.36 sq m, 40.96 cu m 4. (i) 15 cm 5. 43056 cm3 7. 7500 cu m, 2868 cu m Check Your Progress 24.2 1. (i) r2h (iv) 2 rh 2. (i) 1848 c cm, 836 sq cm (ii) 1100 cu cm, 848.57 sq cm (iii) 110 cu cm, 201.14 sq cm 3. 1.65 cu m 6. 48000 cu cm 9. 1320 sq m Check Your Progress 24.3 1. (i) rl (iv) 4 r2 (ii) rl + r2 (v)
4 3 r 3

(ii) 2(bh + hl) + lb (v) x3

(iii) 6x2 (vi)


l2 + b2 + h2

(ii) 750 cu cm, 550 sq cm (iv) 11.25 cu m, 31.5 sq m (ii) 77.76 sq cm, 46.65 cu cm (iii) 2.5 cm 9. 896 sq cm.

(ii) 1.3 cm 6. 384 8. 117 sq m

(ii) 2 r(r + h)

(iii) 2 rh + r2

4. 1386 cu cm 7. 3.68 m 10. 433.54 sq cn,

5. 880 sq m 8. 63 : 16, 21 : 8

(iii) (v)

1 2 r h 3 2 3 r 3

(v) 3 r2 2. (i) 12936 cm3, 2310 cm2, 3696 cm2 (ii) 2464 cm3, 811.3 cm2, 1427.3 cm2, (iii) 154 m3, 134.5 m2, 176 m2 3. 68 m (approx) 6. 50 9. 64 4. 1848 cm3 7. 35 cm, 2310 cm2 10. 12 cm. 5. 16.2 cm 8. 38.8 cm3, 55.44 cm2

236

Mathematics

Check Your Progress 24.4 1. 84 m2, 512 m2 4. Rs 123 Terminal Exercise 1. 42 cm 4. 11 m, 5 m 7. 450 cm2 10. 31.84 m 13.
14 m 3

2. Rs 132.80

3. Rs 234

5. 75 dm, 65 dm, 35 dm.

2. 27 5. 2000 8. 365 cm3 11. 3.55 m 14.


16 m 3

3. 6 cm 6. 1.9 m 9. 2574.8 cm2 12. 35 m 15. 42 cm 18. 12m, 8 m, 3.125 m.

16. Rs 3480

17. 300 sq m

Surface Area and Volume of Solids

237

240

Mathematics

Module 5
Trigonometry
Imagine a man standing near the base of a hill, looking at the temple on the top of the hill. Before venturing to start climbing the hill, he wants to have an approximation of the distance between him and the temple. We know that problems of this and related problems can be solved only with the help of a science called trigonometry. The first introduction to this topic was done by Hipparcus in 140 B.C., when he hinted at the possibility of finding distances and heights of inaccessible objects. In 150 A.D. Tolemy again raised the same possibility and suggested the use of a right triangle for the same. But it was Aryabhatta (476 A.D.) whose introduction to the name Jaya lead to the name sine of an acute angle of a right triangle. The subject was completed by Bhaskaracharya (1114 A.D.) while writing his work on Goladhayay. In that, he used the words Jaya, Kotijya and sparshjya which are presently used for sine, cosine and tangent (of an angle). But it goes to the credit of Neelkanth Somstuvan (1500 A.D.) who developed the science and used terms like elevation, depression and gave examples of some problems on heights and distance. In this chapter, we shall define an angle-positive or negative, in terms of rotation of a ray from its initial position to its final position, define trigonometric ratios of an acute angle of a right triangle, in terms of its sides develop some trigonometric identities, trigonometric ratios of complementary angles and solve simple problems on height and distances, using at the most two right triangles, using angles of 30, 45 and 60.

Introduction to Trigonometry

241

25
Introduction to Trigonometry
25.1 INTRODUCTION The word Trigonometry is derived from the word Trigon meaning a triangle and metron meaning measurement. Thus, trigonometry is that branch of Mathemetics which deals with the measurement of sides and angles of a triangle. This branch of Mathematics has been instrumental in the development of architecture, surveying, navigation, astronomy etc. The great astronomer Hipparchus is said to have developed this branch of Mathematics. The Indian traditional texts like Surya Sidhant have developed and used the knowledge of trigonometry extensively. 25.2 OBJECTIVES After studying this lesson, the learner will be able to
z z

write the trigonometric ratios of an acute angle of a right triangle. find the sides and angle of a right triangle when some of its sides and trigonometric ratios are known. write the relationships among trigonometic ratios. establish trigonometric identities. solve problems based on trigonometric ratios and identities.

z z z

25.3 EXPECTED BACKGROUND KNOWLEDGE


z z z z z z

Concept of an angle Construction of right triangles Drawing parallel and perpendicular lines Types of anglesacute, obtuse and right Types of trianglesacute, obtuse and right Types of trianglesisosceles, equilateral.

242

Mathematics

25.4 TRIGONOMETRIC RATIOS OF AN ACUTE ANGLE Let XOX' and YOY' be rectangular axes of co-ordinates. Let A be a point on OX. Let the ray OA start rotating in the plane in an anti-clockwise direction from the initial position OA about the point O till it reaches its final position OP after some interval of time. (See Fig. 25.1). Thus, an angle POB is formed with x-axis. Let POB = . ( is a Greek letter, and we read it as theta). Draw PB OX. Now clearly PBO is right angled triangled. Let PB = p, OB = b and OP = h. we define the following trigonometric ratios for angle . sine of = cosine of =

Side opposite to angle PB p = = Hypotenuse OP h Adjacent side to angle OB b = = Hypotenuse OP h

Fig. 25.1

Side opposite to angle PB p tangent of = Adjacent side to angle = OB = b Adjacent side to angle OB b cotangent = Side opposite to angle = PB = p
secant = cosecant =

Hypotenuse = OP = h Adjacent side to angle OB b Hypotenuse = OP = h Side opposite to angle PB p

The above trigonometric ratios are written below in an abbreviated form : sine of is abbreviated as sin cosine of is abbreviated as cos tangent of is abbreviated as tan cotangent of is abbreviated as cot secant of is abbreviated as sec and cosecant of is abbreviated as cosec Notes 1. Throughout the study of trigonometry we shall be using only abbreviated form of these trigonometric ratios. 2. sin is an abbreviation for sine of angle and not the product of sin and .

Introduction to Trigonometry

243

So far you have defined six trigonometic ratios. You must have observed that these trigonometric ratios are the ratios of the sides of a right angled triangle. Thus, these trigonometic ratios depend only on the magnitude of angle and not on the size of the right triangle. Let us verify this by taking right triangles of different sizes as shown in Fig. 25.2. In Fig. 25.2, let POM = PMO ~ P'M'O Thus, and ...(AAA)

PM MO PO = = k (say) = P' M ' M ' O P' O

PM = k P'M' OP = k P'O MO = k M'O


Fig. 25.2

Now in P'M'O,
P' M ' kP' M ' PM = = sin = OP' kOP' OP

i.e.,

the value of sin is the same for both the triangles PMO and P'M'O.

Thus the trigonometric ratios are independent of the length of the sides of the right triangle. Example 25.1 : In Fig. 25.3, ABC has a right angle at B. If AB = BC = 1 cm and AC =

2 cm, find sin C, cos C and tan C.

Solution : In right angled ABC, sin C = opposite side AB 1 = = hypotenuse AC 2 adjacent side BC 1 = = hypotenuse AC 2
Fig. 25.3
2 cm

cos C =

and

opposite side AB 1 = = =1 tan C = adjacent side BC 1

Example 25.2 : In Fig. 25.4 ABC is a right angled triangle, right angled at B. If AB = 5 cm, AC = 13 cm and BC = 12 cm, find cosec C, cot C and sec C. Solution : ABC is a right angled triangle

hypotenuse AC cosec C = opposite side = AB


=
13 5
Fig. 25.4

244

Mathematics

adjacent side BC 12 cot C = opposite side = AB = 5


and

hypotenuse AC 13 sec C = adjacent side = BC = 12

Example 25.3 : In Fig. 25.5, ABC is a right angled triangle. If AB = 21 cm, BC = 20 cm and CA = 29 cm and A = , find sin , cot and sec . Solution : We know that

opposite side sin = hypotenuse


= BC = 20 AC 29

adjacent side cot = opposite side


= and AB = 21 BC 20
Fig. 25.5

hypotenuse sec = adjacent side


= = AC AB 29 21

Example 25.4 : In right angled triangle ABC in Fig. 25.6, AB = 9 cm, BC = 40 cm and CA = 41 cm. Find the values of cos C, tan A, cot C and cosec A. Solution : We know that

adjacent side 40 cos C = hypotenuse = 41 opposite side 40 tan A = adjacent side = 9 adjacent side 40 cot C = opposite side = 9
Fig. 25.6

and

hypotenuse 41 cosec A = opposite side = 40

Introduction to Trigonometry

245

CHECK YOUR PROGRESS 25.1 In Fig. 25.7, write all the six trigonometric ratios for the angle C :

(i)

(ii)

(iii)

(iv)

Fig. 25.7

25.5 GIVEN TWO SIDES OF A RIGHT TRIANGLE, TO FIND TRIGONOMETRIC RATIOS When any two sides of a right triangle are given, its third side can be found out by using the Pythagoras theorem. We can find trigonometric ratios of the given angle as learnt in previous section. Let us take some examples to illustrate : Example 25.5 : In Fig. 25.8, PQR is right angled at Q. If PQ = 5 cm and QR = 12 cm find the value of sin R, cos R and tan R. Solution : PQR is a right angled triangle. PR2 = PQ2 + QR2 ...[By Pythagoras theorem] 2 2 = (5) + (12) = 25 + 144 = 169 = (13)2 PR = 13 cm Now, for R, opposite side is PQ = 5 cm, adjacent side QR = 12 cm and the hypotenuse PR = 13 cm
PQ 5 = PR 13 QR 12 = cos R = PR 13 PQ 5 tan R = QR = 12

sin R =

Fig. 25.8

and

246

Mathematics

Example 25.6 : In Fig. 25.9, ABC is a right triangle right angled at C. If BC = 3 cm, AC = 4 cm, find the values of cot A, sec A and cosec A. Solution : ABC is a right angled triangle AB2 = BC2 + AC2 = (3)2 + (4)2 = 9 + 16 = 25 = (5)2 AB = 5 cm cot A = sec A = AC = 4 BC 3
AB 5 = AC 4 AB 5 = BC 3
Fig. 25.9

...[By Pythagoras theorem]

and

cosec A =

Example 25.7 : In triangle PQR, in Fig. 25.10, right angled at Q, PR = 25 cm, QR = 7 cm and RPQ = , find tan , cosec and sec . Solution : PQR is a right angled triangle or or or cosec and (PR)2 = (RQ)2 + (PQ)2 (25)2 = (7)2 + (PQ)2 625 = 49 + (PQ)2 (PQ)2 = 625 49 = 576 = (24)2 PQ = 24 cm RQ 7 tan = PQ = 24
Fig. 25.10

PR 25 = RQ = 7 PR 25 sec = PQ = 24
CHECK YOUR PROGRESS 25.2

1. For ABC right angled at B, if AC = 10 cm, BC = 8 cm and AB = 6 cm, find sin C, cos C and tan C.

Introduction to Trigonometry

247

2. ABC is a right angled triangle, right angled at C. If BC = 24 cm and AC = 7 cm, find sin A, cosec A and cot A. 3. In PQR, right angled at Q, PR = 10 2 cm and QR = 10 cm. Find sec P, cot P and cosec P. 4. In right angled PQR, PQ = tan R, cosec R, sin P and sec P.

3 cm, QR = 1 cm and PR = 2 cm. Find

5. In right angled triangle ABC, AC = 25 cm, AB = 7 cm and ACB = , find cot , sin , sec and tan . 25.6 GIVEN ONE TRIGONOMETRIC RATIO, TO FIND THE OTHERS If one trigonometric ratio is given, the values of the other trigonometric ratios can be easily found out. Let us take some example to illustrate this : Example 25.8 : If sin =
7 , find the value of cos and tan . 25

Solution : Draw a right angled triangle ABC in which B = 90 and C = , as shown in Fig. 25.11. We know that

opposite side AB sin = hypotenuse = AC


= Let 7 25
Fig. 25.11

AB = 7 and AC = 25 AC2 = AB2 + BC2

By the Pythagoras theorem, we have (25)2 = (7)2 + BC2 BC2 = 625 49 = 576 = (24)2 Now in ABC, cos = and tan = BC = 24 AC 25 AB = 7 BC 24 BC = 24

or or

248

Mathematics

Example 25.9 : If cot =

2 2 12 , find the value of cot .sin 5 tan

Solution : In Fig. 25.12, ABC is a right angled triangle and C = We have

adjacent side BC 12 cot = opposite side = AB = 5


Let BC = 12 and AB = 5 In right angled triangle ABC, AC2 = AB2 + BC2 = (5)2 + (12)2 = 25 + 144 = 169 = (13)2 Now AC = 13 sin = cot = and tan = AB = 5 AC 13 BC = 12 AB 5 AB = 5 BC 12
Fig. 25.12

144 . 25 2 .sin 2 cot Hence, = 25 169 5 tan 12


= 144 12 = 1728 169 5 845

Example 25.10 : In ABC, right angled at B, if tan C = sin A cos C + cos A sin C.

1 , fiind the values of 3

Solution : In Fig. 25.13, ABC is a right angled triangle and B = 90 We know that, tan C = 1 = AB 3 BC

Introduction to Trigonometry

249

Also,

AB = 1 and BC = AC2 = AB2 + BC2 = (1)2 + ( 3 )2

3
...(By Pythagoras theorem)

= 1 + 3 = 4 = (2)2 AC = 2 sin C = AB = 1 AC 2

cos C = BC = 3 AB 2 cos A = AB = 1 AC 2

Fig. 25.13

sin A = BC = 3 AC 2 Hence, sin A cos C + cos A sin C = 3+1 4 4 4 =1 4 CHECK YOUR PROGRESS 25.3 1. If sin = 2. If tan = 3. If cos A =
20 , find the values of cos and tan . 29

3. 3 + 1.1 2 2 2 2

= =

24 , find the values of sin and cos . 7


7 , determine the values of sin A and tan A. 25

m 4. If cos = n , determine the values of cot and cosec . 5. If cos = 4 , evaluate the following expression : 5
cos .cot 1 sec 2

250

Mathematics

6. If cosec =

2 , find the value of 3

sin2 cos + tan2 7. If cot B = 5 , show that cosec2B = 1 + cot2B. 4 3 find sin B and tan B. 2

8. Triangle ABC is a right triangle with C = 90. If tan A = 1 and tan B = 3

9. If tan A =

3 , show that cos A cos B sin A sin B = 0

10. If cot A =

12 , show that tan2A sin2A = sin4A sec2A 5

25.7 RELATIONSHIP BETWEEN TRIGONOMETRIC RATIOS We know that in a right triangle ABC, right angled at B, we have sin = cos = and tan = = = AB AC BC AC AB BC AB . AC AC BC AB BC AC AC

Fig. 25.14

= sin cos = Also, sin cos

1 AC 1 = AB = AB = cos ec sin AC 1 AC 1 = BC = BC = sec cos AC

Introduction to Trigonometry

251

and Thus, we have

1 BC 1 = AB = AB = cot tan BC tan = cosec = sec = sin cos 1 sin 1 cos

1 = cos tan sin Let us take some examples to illustrate : and cot = Example 25.11 : If cos = Solution : We know that cosec = 1 = 1 = 2 sin 3 3 2 1 and sin = 3 , find the values of cosec , sec and tan . 2 2

1 1 sec = cos = 1 = 2 2

and

3 sin 2 tan = cos = 1 2


=

32= 3 2 1 Example 25.12 : For a ABC, right angled at C, if tan A = 1, find the value of cos B. Solution : In Fig. 25.15, ABC is a right triangle, right angled at C We have BC tan A = 1 = AC AC = 1 = BC AB =

Let Now Hence

BC 1 cos B = AB = 2 cos B = 1 2
Fig. 25.15

252

Mathematics

CHECK YOUR PROGRESS 25.4 1. If sin = 2. If sin = 1 and cos = 3 , calculate the value of cot and sec . 2 2 3 and tan = 2
2 3 , find the value of cos + sin cot .

3. In a right angled triangle ABC, right-angled at C. If cos A = of sin A sin B + cos A cos B. 4. If cosec A = 2, find the value of sin A and tan A. 5. In a right angled triangle ABC right angled at B, tan A = tan2B sec2A (tan2A + cot2B) 25.8 TRIGONOMETRIC IDENTITIES

3 , then find the value 2

3 , find the value of

In section 25.1, you have learnt to define an angle with the help of the rotation of a ray from initial to the final position. You have also learnt to define all the trigonometric ratios corresponding to that angle. Let us recall them here also. Let XOX' and YOY' be the rectangular axes. Let A be any point on OX. Let the ray OA start rotating in the plane in an anti-clockwise direction about the point O till it reaches the final position OA' after some interval of time. Let A'OA = . Take any point P on a ray OA'. Draw PM OX. In right angled PMO, sin = cos = Squaring and adding, we get sin2 + cos2 PM 2 OM 2 + = OP 2 OP 2 = PM 2 + OM 2 OP 2 ... Q PM 2 + OM 2 = OP2
PM , OP OM OP

Fig. 25.16

OP 2 = OP 2 =1

Introduction to Trigonometry

253

Hence Also, we know that sec =

sin2 + cos2 = 1

...(1)

OP PM and tan = OM OM

Squaring and subtracting these equations, we get sec2 tan2 = OP 2 PM 2 OM 2 OM 2

OP 2 PM 2 = OM 2 = OM 2 OM 2 sec2 tan2 = 1 ... Q OP 2 PM 2 = OM 2

=1 Hence, Again, we know that cosec =


OP OM and cot = PM PM

...(2)

Squaring and subtracting, we get cosec2 cot2 OP 2 OM 2 = PM 2 PM 2 = OP 2 OM 2 PM 2 PM 2 PM 2 ... Q OP 2 = PM 2 + OM 2

=1 Hence cosec2 cot2 = 1 ...(3)

Note : You may also write identities 1, 2 and 3 in the following forms by making use of elementary algebraic operations. sin2 + cos2 = 1 can be written as sin2 = 1 cos2

254

Mathematics

or and

cos2 = 1 sin2 sec2 tan2 = 1 can be written as sec2 = 1 + tan 2

or

tan2 = sec2 1 cosec2 = 1 + cot2

Also, cosec2 cot2 = 1 can be written as or cot2 = cosec2 1

Let us make use of these identities to solve some examples : Example 25.13 : Prove that tan + cot = Solution :
1 sin cos

L.H.S. = tan + cot =


sin cos + cos sin

sin 2 + cos2 sin .cos


1 sin .cos

... Q sin 2 + cos2 = 1

= R.H.S. Example 25.14 : Prove that


sin A 1 + cos A + = 2 cosec A 1 + cos A sin A

Solution :

L.H.S =

sin A 1 + cos A + 1 + cos A sin A sin 2 A + 1 + cos A sin A 1 + cos A

sin 2 A + 1 + 2 cos A + cos2 A sin A 1 + cos A

sin 2 A + cos2 A + 1 + 2 cos A sin A 1 + cos A

Introduction to Trigonometry

255

2 1 + cos A 2 + 2 cos A = sin A 1 + cos A sin A 1 + cos A

= 2cosec A = R.H.S. Example 25.15 : Prove that


1 sin A = (sec A tan A)2 1 + sin A

Solution :

L.H.S. = =

1 sin A 1 + sin A 1 sin A 1 sin A 1 + sin A 1 sin A


2

b1 sin Ag = b1 sin Ag =
cos2 A

1 sin 2 A
2

... Q1 sin2 A = cos2 A


2

F 1 sin A IJ = G H cos A K F 1 sin A IJ = G H cos A cos A K


= R.H.S. Example 25.16 : Prove that
cos 1 sin = 1 + sin 1 + sin

= (sec A tan A)2

Solution :

L.H.S. =

1 sin 1 + sin
1 sin 1 + sin 1 + sin 1 + sin 1 sin 2 1 + sin

256

Mathematics

cos2 1 + sin
cos 1 + sin

... Q1 sin2 = cos2

= R.H.S. Example 25.17 : Prove that cos4A sin4A = cos2A sin2A = 1 2sin2A Solution : L.H.S. = cos4A sin4A = (cos2A)2 (sin2A)2 = (cos2A sin2A) (cos2A + sin2A) = cos2A sin2A = R.H.S. Also, cos2A sin2A = (1 sin2A) sin2A = 1 2 sin 2 A = R.H.S. Example 25.18 : Prove that
tan + sec 1 1 + sin = tan sec + 1 cos

... Q sin 2 A + cos2 A = 1

... Q cos2 A = 1 sin 2 A

Solution :

L.H.S. =

tan + sec 1 tan sec + 1 tan + sec sec 2 tan 2 tan sec + 1

... Q 1 = sec 2 tan 2

btan + sec g bsec + tan gbsec tan g


tan sec + 1

btan + sec g 1 bsec tan g


tan sec + 1

btan + sec g b1 sec + tan g


1 sec + tan

Introduction to Trigonometry

257

= tan + sec = = sin + 1 cos cos 1 + sin cos

= R.H.S. CHECK YOUR PROGRESS 25.6 Prove each of the following identities : 1. (cosec2 1) sin2 = cos2 2. sin4A + sin2A cos2A = sin2A 3. cos2 (1 + tan2 ) =1 4. (1 + tan2) sin2 = tan2 5. sin A + sin A = 2 cos ecA 1 + cos A 1 cos A 1 + cos A = 1 + cos A 1 cos A sin A sec A tan A = cos A sec A + tan A 1 + sin A

6.

7.

8. (sin A cos A)2 + 2sinA cosA = 1 9. cos4 + sin4 2sin2 cos2 = (2cos2 1)2 10. sin A sin B + cos A cos B = 0 cos A + cos B sin A + sin B

11. (cosec sin ) (sec cos ) (tan + cos ) = 1 12. sin A(1 + tan A) + cosA(1 + cot A) = sec A + cosec A 13. 1 cos A = cos ecA cot A 1 + cos A

14.

tan A cot A + = 1 + sec A.cos ecA 1 cot A 1 tan A

cot A + cos ec A 1 1 + cos A 15. cot A cos ec A + 1 = sin A

258

Mathematics

LET US SUM UP
z

In a right angled triangle, we define trigonometric ratios as under : sin = cos =

Side opposite to angle AB = Hypotenuse AC Adjacent side to angle BC = Hypotenuse AC

Side opposite to angle AB tan = Adjacent side to angle = BC Adjacent side to angle BC cot = Side opposite to angle = AB Hypotenuse AC sec = Adjacent side to angle = BC Hypotenuse AC cosec = Side opposite to angle = AB
z

Fig. 25.17

The following relationships exist between the trigonometic ratios (i) tan =
sin cos 1 tan 1 cos

(ii) cot =

cos sin 1 sin

(iii) cot =

(iv) cosec =

(v) sec =
z

The trigonometric identities are (i) sin2 + cos2 =1 (iii) cosec2 cot2 = 1 TERMINAL EXERCISE (ii) sec2 tan2 = 1

1. If sin A = 2. If tan A =

4 , detemine the value of cos A and tan A. 5 20 , determine the value of cosec A and sec A. 21

Introduction to Trigonometry

259

3. If tan = 4. If sec =

4 , find the value of sin + cos . 3 m , find the value of sin and tan . n

5. If cos =

sin tan 1 3 , find the value of 5 2 tan 2


5 tan , find the value of 4 1 + tan

6. If sec =

7. If tan A = 1 and tan B = 8. Prove that

3 , find the value of cos A cos B sinA sin B

(sec + tan ) (1 sin ) = cos 9. Prove that

cot 1 = cos ec 1 tan sec


10. Prove that 1 sin = 1 sin 1 + sin cos 11. Prove that tan + sin = sec + 1 tan sin sec 1 12. Prove that tan A + cot B = tan A cot B cot A + tan B 13. cos ec x + 1 cos x = cos ec x 1 1 sin x

FH

IK

14. Prove that sin3A cos3A = (sin A cos A) (1 + sin A cos A) 15. cos A + sin A = cos A+ sin A 1 tan A 1 cot A

260

Mathematics

ANSWERS Check Your Progress 25.1 (i) sin C = 5 ; 13 13 ; 5 cos C = sec C = cos C = sec C = cos C = sec C = cos C = 5 ; 3 sec C = 12 ; 13 13 and 12 4 ; 5 5 and 4 7 ; 25 25 and 7 4 ; 5 5 and 4 tan C = cot C = tan C = cot C = tan C = tan C = tan C = cot C = 5 12 12 5 3 4 4 3 24 7 7 24 3 4 4 3

cosec C = (ii) sin C =

3 ; 5 5 ; 3

cosec C = (iii) sin C =

24 ; 25 25 ; 24

cosec C = (iv) sin C = 3 ; 5

cosec C =

Check Your Progress 25.2 1. sin C = 2. sin A = 3. sec P = 4. tan R = 5. cot = 3 ; 5 24 ; 25 cos C = 4 5 and 25 and 24 tan C = cot A = 3 4 7 24

cosec A =

2;

cot P = 1 and cosec R = sin = 7 ; 25 2 ; 3

cosec P = sin P = sec =

2
sec P = 2 3

3;
24 ; 7

1 and 2

25 7 and tan = 24 24

Check Your Progress 25.3 1. cos = 21 20 and tan = 29 21 2. sin = 24 7 and cos = 25 25

Introduction to Trigonometry

261

3. sin A = 256 135

24 24 and tan A = 25 7

4. cot = 27 8

m and cosec = n 2 m2

n n 2 m2

5.

6. 2 2 , tan B = 3 13

7. sin B =

Check Your Progress 25.4 1. cot = 3 2

3 and sec =

2 3

2.

3 4
1 1 and tan A = 3 2

3.

4. sin A =

14 5. 3 Terminal Exercise 1. cos A =


7 5

3 4 , tan A = 5 3

2. cosec A =

29 29 , sec A = 20 21

3.

4. sin = 3 7

m2 n 2 and tan = m

m2 n 2 n

5.

3 160 1 3 2 2

6.

7.

262

Mathematics

26
Trigonometric Ratios of Some Special Angles
26.1 INTRODUCTION In the previous lesson, we have defined trigonometric ratios for acute angles and developed some relationship between them. In this lesson, we shall find the values of trigonometric ratios of angels of 30, 45 and 60 by using our knowledge of geometry. We will also use the knowledge of trigonometry to solve simple problems based on heights and distances taken from day to day life. 26.2 OBJECTIVES After studying this lesson, the learner will be able to :
z z z

find geometrically the trigonometric ratios for the angles of 30, 45 and 60. find trigonometric ratios of complementary angles. solve daily life problems of heights and distances, using trigonometric ratios.

26.3 EXPECTED BACKGROUND KNOWLEDGE The student must know before starting the lesson that :
z

when a ray rotates in an anti-clockwise direction about the origin, the angle formed with x-axis is positive. if the sum of two angles is 90, then the angles are said to be complementary. a triangle, with one angle of 90, is called a right-angled triangle. in a right-angled triangle ABC right-angled at B, AC2 = AB2 + BC2. a triangle with three sides equal, is said to be an equilateral triangle. recalls the conditions of congruency of two triangles. in a right triangle ABC, right-angled at B sin C =

z z z z z z

opposite side hypotenuse

hypotenuse cosec C = opposite side

Trigonometric Ratios of some Special Angles

263

cos C =

adjacent side hypotenuse

hypotenuse sec C = adjacent side adjacent side cot C = opposite side

opposite side tan C = adjacent side

26.4 TRIGONOMETRIC RATIOS FOR AN ANGLE OF 45 Let a ray OA rotate in the anti-clockwise direction and make an angle of 45 with x-axis (the initial position of the rotating line) at any interval of time as shown in Fig. 26.1. Take any point P on OA. Draw PM OX. Now PMO is a right-angled triangle We know that POM + OPM + PMO = 180 45 + OPM + 90 = 180 OPM = 180 135 = 45 In PMO, OPM = POM = 45 Let OM = PM OM = a
Fig. 26.1

Now in the right-angled triangle PMO OP2 = OM2 + PM2 = a2 + a2 Now, OP2 = 2a2 OP = (By Pythagorus theorem)

2a
cosec 45=

PM a = 1 sin 45 = OP = 2a 2 OM a = 1 cos 45 = OP = 2a 2

sec 45=

and

tan 45 =

PM = a = 1 OM a

cot 45= 1

264

Mathematics

Table for Trigonometric ratios for an angle of 45 sin 45 = cos 45 = 1 ; 2 1 ; 2 cosec 45 = sec 45 = cot 45 = 1

2 2

tan 45 = 1;

26.5 TRIGONOMETRIC RATIOS FOR AN ANGLE OF 30 Let a ray OA rotate in the anticlockwise direction and make an angle of 30 with its initial position OX. Take any point P on OA Draw PM OX. Produce PM to P such that PM = PM. Join OP. Now, in PMO and PMO OM = OM PMO = PMO PM = PM PMO PMO OPM = OPM = 60 (Common) (Each is equal to 90) (By construction) (SAS congruence) (c.p.c.t.)
Fig. 26.2

OPP is an equilateral triangle. Let Also OP = OP PM = a PP = PM + MP = a + a = 2a OP = OP = PP = 2a (sides of an equilateral triangle)

Now, in right-angled PMO OP2 = PM2 + OM2 (Pythagoras theorem)

Trigonometric Ratios of some Special Angles

265

or

(2a)2 = (a)2 + OM2 OM2 = 4a2 a2 = 3a2

Hence,

OM = sin 30 =

3a
1 PM a = = OP 2a 2 3a 3 OM = = 2 OP 2a

cosec 30 = 2

cos 30 =

sec 30 =

2 3

and

tan 30 =

a 1 PM = = OM 3a 3

cot 30 =

Table for Trigonometric ratios for an angle of 30 sin 30 = cos 30 = tan 30 = 1 ; 2 3; 2 1 ; 3 cosec 30 = 2 sec 30 = 2 3 cot 30 =

26.6 TRIGONOMETRIC RATIO OF AN ANGLE OF 60 Let a ray OA rotate in an anti-clockwise direction and make an angle of 60 with its initial position OX at any interval of time. Take any point P on it. Draw PM OX. Now PMO is a right-angled triangle. Now produce OM to M such that OM = MM. Join PM, Let OM = a Now, in PMO and PMM PM = PM PMO = PMM OM = MM PMO PMM
Fig. 26.3

(Common) (Each is equal to 90) (Construction) (SAS congruence)

266

Mathematics

POM = PMM = 60 OP = PM = OM = 2a OP2 = PM2 + OM2 (2a)2 = (PM)2 + (a)2 (By Pythagoras theorem)

POM is an equilateral triangle

In right-angled PMO

Hence,

PM2 = 4a2 a2 = 3a2 PM =

3a

sin 60 = PM = 3a = 3 OP 2a 2 cos 60 = and tan 60 = OM = a = 1 OP 2a 2 PM = 3 OM

cosec 60 = sec 60 = 2 cot 60 =

2 3

1 3

Table for Trigonometric Ratios for an angle of 60 3 sin 60 = 2 ; cos 60 = tan 60 = 1 ; 2 cosec 60 = sec 60 cot 60 = 2 = 1 3 2 3

3;

26.7 TRIGONOMETRIC RATIOS FOR ANGLES OF 0 AND 90 We have defined trigonometric ratios for acute angles 30, 45 and 60. For angles of 0 and 90 we shall use the following results, as axioms and will not give any logical proof for them Table for Trigonometric Ratios for Angles 0 and 90 sin 0 cos 0 = 0; = 1; cosec 0 = Not defined sec 0 cot 0 sec 90 cot 90 = 1 = Not defined = Not defined = 0

tan 0 = 0; sin 90 = 1; cos 90 = 0; tan 90 = Not defined;

cosec 90 = 1

Trigonometric Ratios of some Special Angles

267

26.8 AN IMPORTANT TABLE : AN AID TO MEMORY The values of trigonometric ratios for angles 0, 30, 45, 60 and 90 are quite often used in solving problems in our day-to-day life. Thus, the following table will enable us to remember the values of sin and cos more easily and we will be able to write the values of other trigonometric ratios by using the existing relation between them. Trig. Ratio sin 0 4 4 4 0 40 Not defined 1 4 3 4 1 4 1 3 43 4 1 4 3 2 4 2 4 2 42 2 42 4 2 4 2 3 4 1 4 3 43 1 4 1 4 3 4 1 4 4 0 4 Not defined 0 40 4 4 Not defined 0 30 45 60 90

cos

tan

cot

cosec

Not defined 4 4

sec

Let us now take some examples to illustrate the use of these trigonometric ratios. Example 26.1: Find the value of tan260 sin230 Solution : We know that tan 60 = and sin 30 =
1 2

tan260 sin230 =

1I d 3i FH 2 K
2

1 11 = 3 = 4 4

268

Mathematics

Hence, tan260 sin230 =

11 . 4 tan260 cosec245 + sec245 sin30

Example 26.2 : Find the value of

Solution : We know that tan 60 = sec 45 = =

3 ; cosec 45 =
1 2 ; sin 30 = 2

tan260 cosec245 + sec245 sin 30

d 3i d 2 i + d 2 i
2 2

1 2

= 3 2 + 2 =6 + 1 = 7

1 2

Hence, tan260 cosec245 + sec245 sin 30 = 7 Example 26.3 : Verify that


1 tan 45 + sec 60 2 sin 90 = cosec 30 cot 45 cos 0 2

Solution : We know that tan 45 = 1, cosec 30 =2, sec 60 = 2, cot 45 = 1 sin 90 = 1 and cos 0 = 1 L.H.S. = = = Example 26.4 : Verify that 4 cot 2 30+3 sin 2 602 cosec 2 60 3 tan 2 30 10 = 3 4 3 Solution : We know that, cot 30 = 3 3 , sin 60 = 2 , tan 45 + sec 60 2 sin 90 cosec 30 cot 45 cos 0 1 + 2 2 1 2 1 1 1 +22 1 = = R.H.S. 2 2

Trigonometric Ratios of some Special Angles

269

1 cosec 60 = 2 and tan 30 = 3 3 L.H.S. = 4 cot 2 30+3 sin 2 602 cosec 2 60 3 tan 2 30 3 4

4 = 3
=

d i FGH 23 IJK
3
2

+3

2 2 3

FG IJ H K

3 4

FG 1 IJ H 3K

4 3+ 3 3 2 4 3 1 3 4 3 4 3

9 8 1 10 = 4+ = = R.H.S. 4 3 4 3 Example 26.5 : Verify that


4 4 cot 2 60+ sec 2 302 sin2 45 = 2 2 3 sin 60+ cos 45

Solution : We know that cot 60 = sin 60 = L.H.S. = 1 2 1 , sec 30 = , sin 45 = 3 3 2

1 3 and cos 45 = 2 2
4 cot 2 60+ sec 2 302 sin2 45 sin 2 60+ cos2 45 4

FG 1 IJ + FG 2 IJ 2 FG 1 IJ H 3K H 3K H 2 K FG 3 IJ + FG 1 IJ H 2 K H 2K
2 2 2 2

4 1+ 4 2 1 3 3 2 = 4 = R.H.S. = 3 3+1 4 2
Hence, L.H.S. = R.H.S. Example 26.6 : If = 30, verify that tan 2 =

2 tan 1 tan 2

Solution : We have = 30 L.H.S = tan 2 = tan 60 =

270

Mathematics

R.H.S. =

2 tan 30 2 tan = 2 1 tan 1 tan 2 30

2 = 1

FG IJ H K

2 1 3 3 = 1 2 1 1 3 3

= = Hence, L.H.S. = R.H.S.

23 3 = 32 3

Example 26.7 : Taking A = 30, verify that sin 3A = 3 sinA 4 sin3A Solution : We know that A = 30 L.H.S. = sin 3A =sin 90 =1 R.H.S. = 3 sin A 4 sin3A = 3 sin 30 4 sin3 30 = 3 1 4 1 2 2 = Hence, L.H.S. = R.H.S. CHECK YOUR PROGRESS 26.1 1. Evaluate each of the following : (i) sin260 + cos245 (ii) 2 sin230 2 cos245 + tan260 (iii) 4 sin260 + 3 tan230 8 sin 45cos45 (iv) 4 (sin430 + cos460) 3 (cos245 2sin245) tan 45 sec 60 5 sin 90 (v) cosec 30 + cot 45 2 cos 0 31 =1 2 2

FH IK

Trigonometric Ratios of some Special Angles

271

2. Verify each of the following : (i) cosec330 cos 60 tan345 sin290 + sec245 cot30 = 8 3 7 1 1 (ii) tan 2 30+ sin 2 45+ cos2 30+ cot 2 60 = 6 2 3
2 2 2 5 2 3 (iii) 5 sin 30+ cos 454 tan 30 = 6 2 sin 30 cos 30+ tan 45

3. If A = 30, verify that (i) tan 2A =

2 tan A 1 tan 2 A

(ii) cos 2A = cos2A sin2A (iii) cos 3A = 4 cos3A 3 cos A 4. Taking 2A = 60, find sin 30 and cos 30 using cos 2A = 2 cos2A 1. 26.9 TRIGONOMETRIC RATIOS FOR COMPLEMENTARY ANGLES Let XOX' and YOY' be a rectangular system of coordinates. Let A be any point on OX. Let a ray OA rotate in an anti-clockwise direction and trace an angle from its initial position (x-axis) at any interval of time. Let POM = . Draw PM OX. PMO is a right triangle Also, POM + OPM + PMO = 180 POM + OPM + 90 = 180 POM + OPM = 90 OPM = 90
Fig. 26.4

i.e. OPM and POM are complementary angles. In right-angled PMO, we know that sin = cosec = PM OM PM , cos = and tan = OP OP OM OP OP OM , sec = and cot = PM OM PM

For reference angle (90 ), we have in right-angled OPM sin (90 ) = OM = cos OP

272

Mathematics

cos (90 ) = tan (90 ) = cot (90 ) = cosec (90 ) = and sec (90 ) =

PM = sin OP OM = cot PM PM = tan OM OP = sec OM OP = cosec PM

Let us take some examples to illustrate the above Example 26.8 : Prove that tan 11 = cot 79. Solution : We know that, tan (90 ) = cot R.H.S. = cot 79 = cot (90 11) = tan 11 = L.H.S. Example 26.9 : Evaluate sin240 cos250. Solution. We know that, cos (90 ) = sin cos 50 = cos (90 40) = sin 40 Hence, sin240 cos250 = sin240 sin240 = 0 cos 43 sec 32 Example 26.10 : Evaluate sin 47 + cosec 58 Solution : We know that cos (90 ) = sin sin 47 = sin (90 43) = cos 43 Also cosec 58 = cosec (90 32) = sec 32

cos 43 + sec 32 cos 43 + sec 32 = sin 47 cosec 58 cos 43 sec 32 =1 + 1 = 2

Trigonometric Ratios of some Special Angles

273

Example 26.11 : Show that tan 7.tan23.tan60.tan67.tan83 = Solution : We know that tan (90 ) = cot and Also, tan 60 =

tan 83 = tan (90 7) = cot 7 tan 67 = tan (90 23) = cot 23 L.H.S. = tan 7.tan23.tan60.tan67.tan83 = tan 7.tan 23.tan 60.cot 23.cot 7 = = =

3 tan 7 .cot 7 tan 23 cot 23

gb

3 1 1 3

= R.H.S. Example 26.12 : Prove that Solution : We know that, sin (90 ) = cos and cos (90 ) = sin L.H.S. =

cos + sin =2 sin 90 cos 90

cos + sin sin 90 cos 90

g
g g

cos + sin = 1 + 1 = 2 = R.H.S. cos sin

sin 90 cos 90 Example 26.13 : Prove that cosec 90 + sec 90 = 1

b b

Solution : We know that sin (90 ) = cos cosec (90 ) = sec cos (90 ) = sin

274

Mathematics

sec (90 ) = cosec sin 90 cos 90 L.H.S. = cosec 90 + sec 90

b b

g g

cos sin + sec cosec

= cos2 + sin2 =1 = R.H.S. Example 26.14 : Express tan 68 + sec 68 in terms of angles between 0 and 45. Solution : We know that tan (90 ) = cot Also, sec (90 ) = cosec and tan 68 = tan (90 22) = cot 22 sec 68 = sec (90 22) = cosec 22

Hence, tan 68 + sec 68 = cot 22 + cosec 22 CHECK YOUR PROGRESS 26.2 1. Show that (i) cos 35 = sin 55 (ii) sin211 cos279 = 0 (iii) cos251 sin239 = 0 2. Evaluate each of the following : (i) cos 75 + sin 12 cos18 sin 15 cos 78 sin 72 sin 47 I + F cos 43 I FH cos H sin 47 K 43 K
2 2

(ii)

4 cos2 45

(iii)

cos2 20+ cos2 70 cos2 0 sin 2 59+ sin 2 31

Trigonometric Ratios of some Special Angles

275

3. Prove that (i) sin cos(90 ) + cos sin (90 ) = 1 (ii) cos cos (90 ) sin sin (90 ) = 0 cos 90 1 + sin 90 (iii) 1 + sin 90 + cos 90 = 2 cosec

tan 90 (iv) sin (90 ) cos (90 ) = 1 + tan 2 90

4. Express each of the following in terms of angles between 0 and 45. (i) cos 55 + sin 68 (ii) cot 75 + cosec 75 (iii) sec262 + sec269 26.10 APPLICATIONS OF TRIGONOMETRY We have so far learnt to define trigonometric ratios of an angle. Also, we have learnt to determine the values of trigonometric ratios of the angles of 30, 45 and 60. In this lesson, we will learn how trigonometry can be used to determine the distance between the objects (particularly inaccessible ones) or the heights of the objects by taking some examples from day-to-day life. We shall first define some terms which will be needed in the study of heights and distances. 26.10.1 Angle of Elevation Whenever an observer is looking at an object which is at a greater height than the observer, he has to lift his eyes to see the object; and an angle of elevation is formed between the line of sight joining the observers eye to the object. and the horizontal line. In Fig. 26.5 , is the angle of elevation. 26.10.2 Angle of Depression On the contrary, if the observer, at a height, is looking at an object at a lesser height, the angle formed between the line of sight and the line joining eye of the observer to the object is called an angle of depression. In Fig. 26.6, is the angle of depression.
Fig. 26.5

Fig. 26.6

Example 26.15 : A ladder leaning against a window of a house makes an angle of 60 with the ground. The length of the ladder is 8 m. Find the distance of the foot of the ladder from the well. Solution : Let AC be a ladder leaning against the wall AB making an angle of 60 with the level ground BC.

276

Mathematics

Let BC = x m Now, in right-angled triangle ABC cos 60 =


BC x = AC 8

1 x = 2 8

x=4

Hence, the foot of the ladder is 4 m away from the wall.

Fig. 26.7

Example 26.16 : A balloon is connected to a meteorological ground station by a cable of length 100 m inclined at 60 to the horizontal. Determine the height of the balloon from the ground. Assume that there is no slack in the cable. Solution : Let A be the position of the balloon, attached to a string AC of length 100 m which makes an angle of 60 with the level ground BC. Let AB = x m Now, in right-angled ABC sin 60 = AB = x AC 100

3 = x 100 2 x = 50 3 m = 86.6 m
Fig. 26.8

Hence, the balloon is at a height of 86.6 m.

Example 26.17 : The upper part of a tree is broken by the action of wind. The top of the tree makes an angle of 30 with the horizontal ground. The distance between the base of a tree and the point where it touches the ground is 10 m. Find the height of the tree. Solution : Let AB represent a tree. Let C be a point from where the tree was broken by the action of the wind in two parts upper part makes an angle of 30 at D with level ground such that BD = 10 m Let BC = x m Now, in right-angled CBD tan 30 = BC = x BD 10

1 x = 10 3
Fig. 26.9

Trigonometric Ratios of some Special Angles

277

x= We know that

10 3 3

...(i)

The height of the tree = BC + CD Now we shall find CD In right-angled CBD sin 30 = BC = x DC DC

1 x = 2 DC DC = 2x 10 3 = 2 3 = 20 3 3 10 3 20 3 m+ m 3 3 [From (i)]

Height of the tree =

= 10 3 = 17.32 m. Example 26.18 : A vertical flagstaff AB stands on a horizontal plane. At a point P, which is 200 m away from its foot, the top of the flagstaff makes an angle APB = 30, where A is the top of the flagstaff. Find the length AB of the flagstaff. Solution : In right-angled ABP tan 30 = AB = x PB 200

1 x = 200 3 200 x= 3 = = 200 3 = 200 3 = 2 173.2 3 3 3 346.4 = 115.5 m (app.) 3


Fig. 26.10

Hence, the length AB of the flagstaff is 115.5 m. (app.)

278

Mathematics

Example 26.19 : At a point on level ground, the angle made by the top of the tower with it is found to be such that tan = 5 12 . On walking 192 m towards the foot of the tower, the tangent of the angle becomes 3/4. Find the height of the tower. Solution : Let AB be a tower and C, D be the two positions of the observer such that CD = 192 m. Let AB = h m and BC = x m. We are given that tan = 5 3 and tan C = . 12 4

Now, in right-angled ABC tan C = h x


Fig. 26.11

3 h = 4 x x= 4h 3 ...(i)

Again, in right-angled ABD tan = h 192 + x

5 h = 12 192 + 4 h 3 =

3h 576 + 4 h

36 h = 2880 + 20 h 16 h = 2880 h = 180

Hence, the height of the tower is 180 m. Example 26.20 : Standing on the top of a tower 100 high, Swati observes two cars on the opposite side of the tower. If their angles of depression are 45 and 60, find the distance between the two cars. Solution : Let PQ be a tower which is 100 m high. Let A and B be the position of the two cars. Let the angle of depression of the car at A be 60 and for the car at B be 45 as shown in Fig. 26.12 Now, and RPA = PAB = 60 SPB = PBQ = 45

Trigonometric Ratios of some Special Angles

279

In right-angled triangle PQB, PQ 100 tan 45 = QB = QB

100 1 = QB

QB = 100 m

Also, in right-angled PQA PQ 100 tan 60 = QA = QA


Fig. 26.12

100 3 = QA
QA = 100 100 3 100 1.732 = = = 57.74 3 3 3

Hence, the distance between the two cars = AQ + QB = (100 + 57.74) m = 157.74 m CHECK YOUR PROGRESS 26.3 1. A ladder leaning against a vertical wall makes an angle of 60 with the ground. The foot of the ladder is at a distance of 3 m from the wall. Find the length of the ladder. 2. At a point 50 m away from the base of a tower, an observer measures the angle of elevation of the top of the tower to be 60. Find the height of the tower. 3. The angle of elevation of the top of the tower is 30 from a point 150 m away from its base. Find the height of the tower. 4. The string of a kite is 100 m long. It makes an angle of 60 with the horizontal ground. Find the height of the kite, assuming that there is no slack in the string. 5. A kite is flying at a height of 100 m from the level ground. If the string of kite makes an angle of 60 with a point on the ground, find the length of the string, assuming that there is no slack in the string. 6. The angle of elevation of tower at a point is 45. After going 40 m towards the foot of the tower, the angle of elevation becomes 60. Find the height of the tower. 7. Two men are on either side of a cliff which is 80 m high. They observe the angles of elevation of the top of the cliff to be 30 and 60 respectively. Find the distance between the two men. 8. One of the equal sides of an isosceles triangle is 18 2 m. If the angle of its vertex is 90, find the length of the base.

280

Mathematics

9. From the top of a building 60 m high, the angles of depression of the top and bottom of a tower are observed to be 45 and 60 respectively. Find the height of the tower and its distance from the building. 10. A ladder of length 4 m makes an angle of 30 with the level ground while leaning against a window of a room. The foot of the ladder is kept fixed on the same point of the level ground. It is made to lean against a window of another room on its opposite side, making an angle of 60 with the level ground. Find the distance between these two rooms. LET US SUM UP 1. The following are the relation between trigonometrical ratios for the complementary angles : (i) sin (90 ) = cos (ii) cos (90 ) = sin (iii) tan (90 ) = cot (iv) cosec (90 ) = sec (v) sec (90 ) = cosec (vi) cot (90 ) = tan 2. The following table illustrates the values of trigonometric ratios for the angle such that 0 90 : Table of Values of Trigonometric Ratios Trig. Ratio sin cos tan cot cosec sec 0 30 45 60 90

0 1 0 not defined not defined 1

1 2 3 2 1 3

1 2 1 2 1 1

3 2 1 2

1 0 not defined 0 1 not defined

3
1 3 2 3 2

3
2 2 3

2 2

Trigonometric Ratios of some Special Angles

281

TERMINAL EXERCISE 1. Find the value of each of the following : (i) 4cos260 + 4sin245 sin230 (ii) sin245 tan245 + 3(sin290 + tan230) (iii) 5 sin 2 30+ cos2 454 tan2 30 2 sin 2 30 cos2 30+ tan 45

2. Prove that (i) 2cot230 2cos260


3 2 5 sin 45 4sec230 = 4 24

(ii) 2 sin230 + 2 tan260 5 cos245 = 4 (iii) cos 60 cos 45 + sin 60 sin 45 = sin 45 cos 30 + cos 45 sin 30 3. It = 30, verify that (i) tan 2 =
2 tan 1 tan 2 2 tan 1 + tan 2

(ii) sin 2 =

(iii) cos 2 = 2 cos2 1 4. If A = 60 and B = 30, verify that (i) sin (A B) sin A sin B (ii) sin (A + B) = sin A cos B + cos A sin B (iii) cos (A + B) = cos A cos B sin A sin B 5. Prove that (i) sin 20 sin 70 cos 20 cos 70 = 0 (ii) sin A sin (90 A) cos A cos (90 A) = 0 (iii)
sin 90 A cos 90 A = sin2(90 A) tan A

g b

(iv) tan 20. tan 35. tan45. tan55.tan70 = 1 cos 90 A 1 + sin 90 A (v) 1 + sin 90 A + cos 90 A = 2 sec (90 A)

282

Mathematics

6. The length of a string without slack, between a kite and a point on the ground is 150 m. If the string makes an angle with the horizontal plane such at sin = 4 15 , determine the height of the kite. 7. A ladder leaning against a vertical wall makes an angle with the ground such that tan = 4 3 . The foot of the ladder is 3 m away from the wall. Determine the length of the ladder. 8. Two pillars of equal height stand on either side of a roadway which is 150 m wide. At a point on the roadway between the pillars, the elevation of the top of the pillars are 60 and 30. Find the height of the pillars and the position of the point. 9. An observer standing 40 m from a building notices that the angles of elevation of the top and bottom of a flagstaff, which is surmounted on the building, are 60 and 45 respectively. Find the height of the tower and the flag staff. 10. From the top of a hill, the angle of depression of two consecutive kilometer stones due east are found to be 30 and 60. Find the height of the hill.

Trigonometric Ratios of some Special Angles

283

ANSWERS Check Your Progress 26.1 1. (i) 5 4 (ii) 5 2 (iii) 0 (iv) 2 (v) 0

4. sin 30 =

1 ; cos 30 = 3 2 2

Check Your Progress 26.2 2. (i) 1 (ii) 0 (iii) 0

4. (i) sin 35 + cos 22 (ii) tan 15 + sec 15 (iii) cosec228 + cosec221 Check Your Progress 26.3 1. 6 m 6. 94.64 m Terminal Exercise 1. (i) 11 4 (ii) 7 2 (iii) 40 121 2. 86.6 m 3. 86.6 m 4. 86.6 m 5. 115.46 m

7. 184.75 m 8. 36 m

9. 25.35 m 10. 5.46 m.

6. 40 m 9. 40 m, 29.28 m 10. 433 m

7. 5 m

8. 64.95 m, 37.5 m

Data and Their Representation

287

Module 6
Statistics
Since ancient times, it has been the practice by the house holders, shopkeepers, individuals, etc. to keep records of their income, expenditure and other resources. In fact, this record keeping was started by ancient kings to keep account of their warriors, armoury and other fighting materials, etc. To make readers acquainted with the methods of recording, condensing and taking out relevant information from given records (or data), they would be exposed to the lesson on Data and their Representation. Everyday through communication media like radio, television, newspapers, periodicals, magazines, etc., we come across data in the form of tables, charts, graphs, etc. on different aspects of a number of variables. The information presented is eye-catching and important. In order to read and interpret these correctly, the learners would be introduced to lesson on Graphical Representation of Data. Sometimes, we are required to describe the data arithmetically, like describing mean age of a class, mean height of a group, median score of a class or modal shoe size of a group. The learner will be introduced to these in the lesson on Measures of Central Tendency. They would also learn about the characteristics and limitations of these measures. In the last lesson of this module, the learner will get acquainted to the concept of probability as measure of uncertainty, through games of chance like tossing a coin, throwing a die, etc.

288

Mathematics

27
Data and Their Representation
27.1 INTRODUCTION In this lesson, we shall first learn about statistics as science dealing with collection, recording and condensing data to take out relevant information from them. We shall learn about different types of data-primary and secondary, raw/ungrouped and grouped. We shall also learn about some important concepts like classes, frequency, class mark, class interveral, frequency table, cumulative frequency table, etc. 27.2 OBJECTIVES After studying this lesson, the learner will be able to :
z z z z z z z

define statistics both in singular and plural form differentiate between primary and secondary data differentiate between raw/ungrouped and grouped data and cite examples define frequency and cumulative frequency of a class condense raw data into frequency table form cumulative frequency tables define class-mark, class-interval, class limits and true class limits in case of grouped data.

27.3 EXPECTED BACKGROUND KNOWLEDGE


z z z z

knowledge of writing numbers in increasing/decreasing order finding averages of given numbers classification of numbers into different classes making/using tools to collect data.

27.4 STATISTICS AND STATISTICAL DATA Statistics is the science which started with collection of data about different aspects. It may be data regarding number of students presented in a school, classwise literacy rate in different

Data and Their Representation

289

states of the country, unemployment data, income-tax rates etc. With the passage of time besides collection of data, their tabulations, interpretation and drawing of inferences came under the purview of statistics. The word statistics is used both in plural and singular sense. In its singular form, it refers to the science of statistics, while in plural sense it means the numerical facts or data/observations collected with some specific purpose in view. 27.5 PRIMARY AND SECONDARY DATA Data are of two types primary and secondary. In any study, if the investigator himself is responsible for the collection of data according to the desired plan and objective, they are called primary data. Sometimes, it is not possible for the investigator to collect data himself due to paucity of time, finances and other reasons. In that case, the investigator uses the data collected by other investigators in some other context or published data from government records, journals, books etc. Since the data collected from other sources, may have been collected with an objective different from what it is being used, they are called secondary data. CHECK YOUR PROGRESS 27.1 1. Fill in blanks with suitable word(s) so that the following sentences give the proper meaning : (a) Statistics in plural sense refers to ... collected for some definite purpose. (b) Statistics in singular sense refers to the ... . (c) The data which are collected by the investigator himself are called ... data. (d) The data taken from other sources and not collected by the investigator himself are called ... data. 27.6 RAW/UNGROUPED AND GROUPED DATA Consider the marks obtained by 20 students of a class in a class test (out of 25): 15, 10, 6, 9, 8, 25, 20, 25, 14, 14, 12, 18, 12, 19, 25, 17, 20, 16, 22, 25 ...(i)

The data in this form are called raw/ungrouped data. The data given in (i), do not give much information about the standard achievement of students. Let us try to arrange the data in ascending order. We will have 6, 17, 8, 18, 9, 19, 10, 20, 12, 20, 12, 22, 14, 25, 14, 25, 15, 25, 16, 25 ...(ii)

290

Mathematics

Data arranged in the above form (ii) are called arrayed data. The arrayed data gives somewhat better perspective of the situation. They present the minimum and maximum of the data at a glance. The difference between the maximum and minium observations of the data is called the range. A point may be made about arrayed data. When the number of observations is very large, it is very difficult and time consuming to arrange data in arrayed form. To further simplify and overcome this difficulty, we can arrange the data in tabular form as follows : Table1 Marks 6 8 9 10 12 14 15 16 17 18 19 20 22 25 Total Number of Students 1 1 1 1 2 2 1 1 1 1 1 2 1 4 20

Table1 shows the number of students getting a particular number of marks. We can say that four students have secured 25 marks each, two have secured 20 marks each, and so on. We can also say that seven students have got 20 or more marks. Similarly, eleven students have got 16 or more marks, and so on. The above data is called a frequency distribution table for ungrouped data. The quantity about which the observations (or data) are collected is called variable (or variate) and the number of times an observation is repeated is called the frequency of that observation.

Data and Their Representation

291

To bring out some more salient features of data, we further simplify the presentation of data in condensed form into classes (or groups). Let us form the groups as follows : Table 2 Classes Frequencies 57 1 810 3 1113 2 1416 4 1719 3 2022 3 2325 4

Thus, for the class 2022, 20 is called the Lower Class Limit and 22 is called the Upper Class Limit. Table 2 is called a frequency table for Grouped Data. The data presented in classes (or groups) is called Grouped Data. From this type of presentation, we can draw better conclusions about the data than before. Some of these conclusions are : (i) The number of students getting marks between 23 and 25 is 4. (ii) No student has got marks less than 5 (iii) Four students have got marks between 14 and 16 and so on. 27.7 NEED FOR CONTINUOUS CLASSES From Table 2, we see that the classes are non-overlapping because there are no fractional marks here. This may happen with marks, but there may be variables like weight, height, etc. which admit fractional measurements like 2.5 kg, 1.65 m, etc. In such cases, we need to have continuous classes so that all observations can be entered. This can be done as follows for data in Table 2. Classes : 4.57.5, 7.510.5, 10.513.5, 13.516.5, 16.519.5, 19.522.5, 22.525.5

i.e., the lower limit of each class is decreased by 0.5 and upper limit increased by 0.5, to have same class-interval (i.e., length of the class). The changed limits are called True class Limits. Thus for the class 7.5-10.5,7.5 is the True Lower class limit and 10.5 is the True upper class limit. This changes the frequency distribution table as follows : Table 3 Classes 4.5 7.5 7.5 10.5 10.5 13.5 13.5 16.5 16.5 19.5 19.5 22.5 22.5 25.5 TOTAL Frequency 1 3 2 4 3 3 4 20

292

Mathematics

For making Table 3, the following two assumptions have been made: (i) The frequency in a class is centred at its mid-point, called class-mark of that class
Class mark of a class =

Upper Class Limit + Lower Class Limit 2

or

True Upper Class Limit + True Lower Class Limit 2

The class marks of distribution in Table 3 are

6, 9, 12, ..., 24 (ii) In each class, the upper class boundary is not included. Thus, in the class 7.510.5, 10.5 is not included and in the class 22.525.5, 25.5 is not included. The observations 10.5 and 25.5 are taken in respective next higher classes. CHECK YOUR PROGRESS 27.2 1. Differentiate between raw data and arrayed data 2. Find the range of the following data : 6, 8, 9, 12, 25, 3, 14, 18, 22, 27 3. Give one example each for frequency distribution table for ungrouped data and grouped data. 4. Define : (a) class limits (b) true class limits (c) class interval (d) class-mark 5. What assumptions do you make while making a frequency table for grouped data ? 6. Write the lower and upper class limits for each of the following classes : (a) 1115, 1620, 4145 (b) 10.515.5, 15.520.5, 40.545.5 7. Explain one of the reasons for condensing raw data to grouped data. 27.8 CONDENSING RAW DATA INTO FREQUENCY TABLE For condensing raw data into grouped data we follow the following steps : Step 1 : Arrange the data in ascending order and find the range of raw data.

Data and Their Representation

293

Step 2 : We decide upon number of classes to be formed. For that we remember the following : (a) There should be classes to accommodate the minimum and maximum of data (b) The classes should not be open ended (c) There is no definite rule for number of classes. The golden rule mostly used is Not fewer than 5 and not more than 10 classes For our example, we took 7 classes, the class interval being 3. This is obtained by dividing the range by class-interval and increase that to next integral value. Step 3 : Take each observation from raw data, one at a time and put a tally mark (|) against the class to which the observation belongs, till all observations are over. (The tally marks are taken in bunches of five, the fifth one crossing the other four diagonally. For example, 6 is denoted by tally marks as |||| |) Step 4 : Count the tally marks in each class to get the frequency of that class. Step 5 : The resulting table is called Frequency Table. Let us take an example to illustrate. Example 27.1 : The marks obtained by 20 students in a class test are given below: 10, 15, 9, 16, 25, 30, 14, 8, 29, 15, 4, 7, 12, 13, 14, 18, 19, 16, 29, 7 Construct a frequency distribution tables with a class size of 4. Solution : The data in ascending order is : 4, 7, 7, 8, 9, 10, 12, 13, 14, 14, 15, 15, 16, 16, 18, 19, 25, 29, 29, 30 Here the range of data is 26. Taking the class size as 4, we get the number classes as 7. (Next integral value of 26 ) 4 1216, 1620, 2024, Table 4 Classes 4 8 12 16 20 24 28 8 12 16 20 24 28 32 Tally Marks ||| ||| |||| | |||| | ||| Frequency 3 3 6 4 1 3 20 2428, 2832

Let the classes be 48, 812, Frequency Table for Marks obtained by Students

Total

294

Mathematics

Example 27.2 : The weights of apples (in grams) picked up at random from a basket are given below : 110, 100, 105, 110, 190, 200, 210, 218, 250, 170 75, 80, 95, 100, 160, 215, 225, 270, 210, 270 Construct a frequency table from the above data, with one of the classes as 75100 (100 not included). Solution : The range of data s 200 grams. Taking one of the classes as 75100, the classes are 75100, 100125, 125150, 150175, 175200, 200225, 225250, 250275 The frequency table is given below : Table 5 Frequency table for weights of Apples Class 75100 100125 125150 150175 175200 200225 225250 250275 Total 27.9 CUMULATIVE FREQUENCY OF A CLASS The total of frequencies of a particular class and of all classes prior to that class is called the cumulative frequency of that class. A table which shows cumulative frequencies of different classes is called a Cumulative Frequency Table. Thus for Table 4, the corresponding Cumulative Frequency Table is presented in Table 6 Table 6 Class 48 812 1216 1620 2024 2428 2832 Total Frequency 3 3 6 4 0 1 3 20 Cumulative Frequency 3 6 12 16 16 17 20 Tally Marks ||| |||| || | |||| | ||| Frequency 3 5 2 1 5 1 3 20

Data and Their Representation

295

Similarly, for Table 5, the Cumulative Frequency Table is given in Table 7. Table 7 Class 75100 100125 125150 150175 175200 200225 225250 250275 Total Let us take another example : Example 27.3 : The class marks of a distribution are 34, 38, 42, 46, 50, 54 and 58, determine the class size, classes and construct frequency and cummulative frequency table. If the frequencies from first class onward are 2, 5, 7, 9, 8, 5, 4. Solution : The class size is the difference between the class marks of two adjacent classes. Therefore, Class size = 3834 = 4
The classes are

Frequency 3 5 0 2 1 5 1 3 20

Cummulative Frequency 3 8 8 10 11 16 17 20

[(34 2) (34 + 2)], [(38 2) (38 + 2)], ... ... [(58 2) (58 + 2)] i.e., 3236, 3640, 4044, 4448, 4852, 5256, 5660
The frequency table and cummulative frequency table is given below :

Table8 Classes 3236 3640 4044 4448 4852 5256 5660 Total Frequency 2 5 7 9 8 5 4 40 Cummulative Frequency 2 7 14 23 31 36 40

296

Mathematics

CHECK YOUR PROGRESS 27.3 1. Enumerate the steps needed to condense raw data to grouped data. 2. Form a frequency table and cumulative frequency table for the data on runs scored by a batsman in 20 innings, one class being 020 12, 2, 15, 25, 75, 35, 45, 16, 28, 70 102, 79, 52, 71, 15, 5, 69, 72, 15, 9 3. The class marks and their corresponding frequencies are given below : Class mark : Frequency : 23 1 28 2 33 5 38 8 43 14 48 6 53 3 58 1

Form a cumulative frequency table from the above data. 27.11 LET US SUM UP
z

Statistics is that branch of Mathematics which deals with collection, classification and interpretation of data. Statistics has meaning both in singular and plural sense. The data collected from the respondents as it is, is called raw data. The data collected by experimenter himself through his own designed tools is called primary data. The data taken from other sources and not collected by the experimenter is called secondary data. The data arranged in ascending/descending order is called arrayed data When raw data are arranged with frequencies, they are said to form a frequency table for ungrouped data. When the data are divided into groups/ classes, they are called grouped data. The classes have to be decided according to the range of data and size of class. In a class 1015, 10 is called the lower limit and 15 is called the upper limit of that class. The number of observations lying in a particular class is called its frequency and the table showing classes with frequencies is called a frequency table. The mid-points of classes are called class-marks. Sometimes the classes have to be changed to make them continuous. In that case, the class limits are called true class limits. The total of frequencies of a particular class and of all classes prior to that is called the cumulative frequency of that class and the table showing cumulative frequencies is called a cumulative frequency table.

z z

z z

z z z z

z z

Data and Their Representation

297

TERMINAL EXERCISES 1. Define statistics as a term used in singular and a term used in plural sense. Give examples to clarify the difference. 2. What are primary data ? Why are they more reliable than secondary data ? 3. Fill in the blanks by appropriate words/phrases to make each of the following statements true : (i) The data which are taken from government records, magazines, etc. is called ... data. (ii) The data, which are not collected by investigatory himself should be used ... . (iii) When the raw data are arranged in ascending/descending order, they are called ... data. (iv) When the data are condensed in classes of equal size with frequencies, they are called ... data and the table is called ... table. (v) The mid-points of a class are called ... . (vi) When the class limits are adjusted to make them continuous, the class limits are renamed as ... . (vii) The number of observations falling in a particular class is called its ... . (viii) The difference between the upper limit and lower limit is called the ... . (ix) The sum of frequencies of a class and all classes prior to that class is called ... frequency of that class. (x) Class size = Difference between successive .................. . 4. Enumerate different steps needed to convert a given raw data to grouped data and to form a frequency table. 5. Find the range of the following raw data and put it as arrayed data : 7, 13, 5, 3, 4, 12, 13, 4, 3, 4, 18, 19, 12, 4, 13, 8, 4, 9, 8, 24 Also change the above data into a frequency distribution of class size 3, one of the classes being 69 (9 not included) 6. The marks obtained by 30 students in a class test are given below : 10, 18, 42, 21, 17, 19, 28, 43, 13, 17 25, 23, 47, 18, 17, 21, 19, 25, 27, 19 15, 16, 48, 46, 31, 34, 18, 19, 12, 4 (i) Form a frequency table, with one of the classes being 1421 (21 not included) (ii) Form a cummulative frequency table for table formed in (i) (iii) Can you interpret the data and give some salient observations on it ?

298

Mathematics

7. The heights of 20 students of a class (in cm) are given below : 140, 137, 146, 152, 160, 157, 153, 140, 142, 145 150, 152, 153, 155, 160, 160, 162, 145, 148, 150 (i) From a frequency distribution table for the above data using equal classes, one of them being 136140 (140 not included) (ii) Form a cumulative frequency table for the above data. 8. For the following frequency table, answer the following questions : Classes 1520 2025 2530 3035 3540 4045 4550 Total Frequency 2 3 5 7 4 3 1 25

(i) Write the lower limit of the first class. (ii) Write the class limits of the third class. (iii) Find the class mark of the fourth class. (iv) Determine the class size. (v) Form a cumulative frequency table. 9. The class-marks, in order, of a distribution and the corresponding frequencies are given below : Class marks : Frequencies : 5 2 15 6 25 10 35 15 45 12 55 8 65 5 75 2

(i) find the frequency table (ii) find the cummulative frequency table.

Data and Their Representation

299

ANSWERS Check Your Progress 27.1 1. (a) numerical facts or observations (c) primary Check Your Progress 27.2 2. 24 6. (a) Lower class limits 11, 16, 41; Upper class limits : 15, 20, 45 Check Your Progress 27.3 2. Classes 020 2040 4060 6080 80100 100120 Total Terminal Exercise 3. (i) (iv) (vii) (ix) 5. (i) secondary (ii) carefully (iii) arrayed grouped; grouped frequency table (v) class marks (vi) true class limits frequency (viii) class limit interval or class size cummulative frequency (x) class marks. xi : fi : (ii) Classes 3 2 : 4 5 5 1 36 8 7 1 69 3 8 2 9 1 12 2 13 3 18 1 19 1 24 Total 1 20 Frequency 8 3 2 6 0 1 20 Cumulative Frequency 8 11 13 19 19 20 (b) lower limit 10.5, 15.5, 40.5 Upper limt 15.5, 20.5, 45.5 (b) science of statistics (d) secondary

912 1215 1518 1821 2124 2427 Total 1 5 0 2 0 1 20

Frequency : 6. (i) Classes 07 714 1421 2128 2835 3542 4249 Total

Frequency 1 3 12 6 3 0 5 30

Cummulative frequency 1 4 16 22 25 25 30

300

Mathematics

7.

Classes 135140 140145 145150 150155 155160 160165 Total

Frequency 1 3 4 6 2 4 20

Cummulative Frequency 1 4 8 14 16 20

8. (i) 15 (v)

(ii) 25, 30 Classes 1520 2025 2530 3035 3540 4045 4550 Total

(iii) 32.5

(iv) 5

Frequency 2 3 5 7 4 3 1 20 Frequency 2 6 10 15 12 8 5 2 60

Cumulative Frequency 2 5 10 17 21 24 25

9.

Classes 010 1020 2030 3040 4050 5060 6070 7080 Total

Cumulative Frequency 2 8 18 33 45 53 58 60

Data and Their Representation

301

Graphical Representation of Data

301

28
Graphical Representation of Data
28.1 INTRODUCTION Whenever verbal problems involving a certain situation is presented visually before the learners, it makes easier for the learner to understand the problem and attempt its solution. Similarly, when the data are presented pictorially (or graphically) before the learners, it makes the presentation eye-catching and more intelligible. The learners can easily see the salient features of the data and interpret them. There are many forms of representing data graphically. They are (i) Bar graphs (ii) Histograms (iii) Frequency polygons (iv) Ogive (v) Pictographs (vi) Pie charts In this lesson, we shall learn to read and draw Bar graphs, Histograms and Frequency polygons, and graphs related to day-to-day use, like temperature-time graph, velocity-time graph, pressure-volume graph, etc. Other form of graphs are beyond the scope of the present lesson. 28.2 OBJECTIVES After studying this lesson, the learner will be able to
z z z z

draw bar charts for given data draw a histogram and frequency polygon for given data read and interpret given bar charts and histograms read the relevant information from graphs relating to day-to-day activities, like (i) temperature-time graph

302

Mathematics

(ii) velocity-time graph (iii) pressure volume graph, etc.


z

draw graph relating to day-to-day activities, like the ones above.

28.3 EXPECTED BACKGROUND KNOWLEDGE


z z z

Knowledge of drawing and marking axes Knowledge of drawing rectangles and plotting points Practice of reading graphs.

28.4 BAR GRAPHS A bar graph is a graphical representation of frequency distributions of ungrouped data. It is a pictorial representation of the numerical data by a number of bars (rectangles) of uniform width erected vertically (or horizontally) with equal spacing between them. 28.4.1 Construction of Bar Graphs For the construction of bar graphs, we go through the following steps : Step 1 : We take a graph paper and draw two lines perpendicular to each other and call them horizontal and vertical axes. Step 2 : Along the horizontal axis, we take the values of the variables and along the vertical axis, we take the frequencies. Step 3 : Along the horizontal axis, we choose the uniform (equal) width of bars and the uniform gap between the bars, according to the space available. Step 4 : Choose a suitable scale to determine the heights of the bars. The scale is chosen according to the space available. Step 5 : Calculate the heights of the bars, according to the scale chosen and draw the bars. Step 6 : Mark the axes with proper labelling Let us take some examples to illustrate : Example 28.1 : The number of trees planted by an agency in different years is given below : Years Number of trees planted 1997 400 1998 450 1999 700 2000 750 2001 900 2002 1500 Total 4700

Solution : The bar graph is given below in Fig. 28.1 :

Graphical Representation of Data

303

Fig. 28.1

Step 1 : We draw two perpendicular lines OX and OY. Step 2 : On OX, we represent years, from 19972002 and on OY we represent the number of trees planted. Step 3 : On OY, we start with 400 and marks points at equal intervals of 200. Step 4 : The height of the bars are calculated according to the number of trees. A kink (~) has been shown on the vertical axis showing that the marking on the vertical axis starts from zero but has been shown to start from 400 as the data needs. Examples 28.2 : The data below shows the number of students present in different classes on a particular day : Classes Number of students present VI 35 VII 40 VIII 30 IX 40 X 50

Represent the above data by a bar graph. Solution : The bar graph for the above data is shown in Fig. 28.2.

304

Mathematics

Fig. 28.2

Example 28.3 : The data regarding causes of accidents in factories are given below: Causes Faulty Machinery Electrical Disturbance Delay in repairs Mechanical Failure Others Draw a bar graph to represent the above data. Solution : The bar graph representing the above data is shown in Fig. 28.3 below: Percentage of Occurrence 30% 20% 35% 10% 5%

Fig. 28.3

Graphical Representation of Data

305

28.4.2 Interpretation of Bar graphs After drawing a bar graph, we can draw some conclusions, which is called interpreting bar graphs. Let us take some examples and do the same. Example 28.4 : Read the bar graphs given in Fig. 28.1, and answer the following questions : (i) In which year the maximum number of trees were planted ? (ii) What trend the number of trees planted show ? (iii) In which years the number of trees planted differ by 50 only ? Solution : After reading the bar graph, the answers to the above questions are as follows : (i) The maximum number (1500) of trees were planted in the year 2002, as in that year the height of the bar is maximum. (ii) The number of trees planted kept on increasing year after year (iii) (a) The years 1997 and 1998 (b) The years 1999 and 2000 Examples 28.5 : Read the bar graph given in Fig. 28.3 and answer the following questions : (i) Which cause is responsible for maximum accidents in factories ? Which is for minimum ? (ii) Can you think of one of the other causes ? (iii) How many percent of accidents could have been avoided by timely action? Solution : (i) Delay in repairs is responsible for maximum (35%) of accidents. Other causes are responsible for minimum number of accidents (ii) Carelessness of workers (iii) (35 + 20)% or 55% accidents could have been avoided by taking steps for timely repairs and provision of equipment which can control electrical disturbances. CHECK YOUR PROGRESS 28.1 1. Enlist the possible forms of representing a data graphically. 2. What are the steps needed to represent a data by a Bar Graph ?

306

Mathematics

3. For the data on expenditure of a company over different heads, draw a bar graph : Head Salary of Employees Travelling Allowance Rent of Premises Machinery and materials Other expenditure Percentage of Expenditure 45% 15% 20% 10% 10%

4. Given below are data on causes of strikes in mills : Causes (i) Non fulfillment of economic demands (ii) Overwork (iii) Rivalry in unions (iv) Non-congenial working conditions (v) Others Draw a bar graph depicting the above data. 5. From the bar graph given below, answer the following questions : Percentage 45 20 20 10 5

Fig. 28.4

(i) The names of two steel plants which produced maximum steel in the country during the time period. (ii) What percentage of steel was produced in other plants ?

Graphical Representation of Data

307

(iii) The steel plant at Durgapur produced how much less steel than at Bokaro? (iv) What percentage of total steel under discussion was produced at Bhalai, Durgapur and Bokaro steel plants ? 28.5 HISTOGRAMS AND FREQUENCY POLYGONS A histogram is a graphical representation of a continuous frequency distribution i.e. grouped frequency distributions. It is a graph, including vertical rectangles, with no space between the rectangles. The class-intervals are taken along the horizontal axis and the respective class frequencies on the vertical axis using suitable scales on each axis. For each class, a rectangle is drawn with base as width of the class and height as the class frequency. The area of the rectangles must be proportional to the frequencies of the respective classes. A frequency polygon is the join of the mid-points of the tops of the adjoining rectangles. The mid-points of the first and the last classes are joined to the mid-points of the classes preceding and succeding respectively at zero frequency to complete the polygon. Let us illustrate these with the help of examples. Examples 28.6 : The following is the frequency distribution of weights of 30 students of class IX of a school. Draw a histogram to represent the data. Classes : 4550 3 5055 7 5560 12 6065 5 6570 3 Total 30

Frequency :

Solution : For drawing a histogram we go through the steps similar to those of a bar graph. They are given below : Step 1 : On a paper, we draw two perpendicular lines and call them horizontal and vertical axes. Step 2 : Along the horizontal axis, we take classes of equal width : 4550, 5055, ...... As the axis starts from 4550, we take one interval 4045 before it and put a kink on axis before that Step 3 : Choose a suitable scale on the vertical axis to represent the frequency. It can start from 0 to 12, with a step of 2, i.e., 0, 2, 4, 6, ...., 12, 14 Step 4 : Draw the rectangles as shown in Fig. 28.5.

308

Mathematics

Fig. 28.5 shows the histogram required.

Note : A frequency polygon has been shown in dotted lines, as explained in the steps shown above. Example 28.7 : The daily earnings of 100 shopkeepers are given below : Daily earnings (in Rs) No. of shops 200-300 300-400 400-500 500-600 600-700 700-800 800-900 3 12 15 30 25 12 3

Draw a histogram and a frequency polygon to represent the above data. Solution : Following the steps suggested in Example 28.6, the histogram and frequency polygon representing the above data are given below in Fig. 28.6

Fig. 28.6

Graphical Representation of Data

309

Example 28.8 : Draw a frequency polygon for the following data : Pocket allowance (in rupees) Number of students 050 50100 100150 150200 200250 250300

16

25

13

26

15

Solution : To draw a frequency polygon without-drawing a histogram we go through the following steps : Step 1 : Draw two lines perpendicular to each other. Step 2 : Find the class-marks of different classes. They are 25, 75, 125, 175, 225, 275 Step 3 : Plot the ordered pairs A (25, 16), B (75, 25), C(125, 13), D (175, 26), E(225, 15) and F(275, 5) Step 4 : Join the points A, B, C, D, E and F and complete the polygon as explained before The frequency polygon is given below :

Fig. 28.7

28.5.1 Reading a Histogram Let us explain it with the help of an example Example 28.9. The following histogram shows the monthly wages (in rupees) of workers in a factory

310

Mathematics

Fig. 28.8

(i) Find the maximum number of workers getting a wage. (ii) Find the least wage and highest wage with no. of workers earning them (iii) How many workers get a monthly wage of Rs. 8000 or less ? Solution : (i) The maximum number of workers is 25 getting wages between Rs (7000 8000). (ii) The least wage is between Rs (4000 5000) and 4 workers are getting that. The corresponding figures for highest wage are Rs (9000 10000) and four workers get that (iii) 50 workers get a wage of Rs 8000 or less as Rs (4000 5000) 4 workers Rs (5000 6000) 10 workers Rs (6000 7000) 12 workers Rs (7000 8000) 24 workers Total 50 CHECK YOUR PROGRESS 28.2 1. What is the difference between a bar graph and a histogram ? 2. Write various steps in the construction of a Histogram.

Graphical Representation of Data

311

3. Draw a histogram for the following frequency distribution : Height of students 135140 140145 145150 150155 155160 160165 (in cm) No. of students 3 5 12 7 5 3

Also draw a frequency polygon for the above data on the same sheet 4. Draw a frequency polygon for the data in Question 3 on a separate paper. 5. Draw a histogram and a frequency polygon for the following grouped data: Annual income (in ten thousand rupees) No. of families in a locality 4-6 25 6-8 20 8-10 10-12 12-14 14-16 16-18 18-20 15 15 13 7 3 2

6. Interpret the data represented by the following histogram by answering the following questions :

Fig. 28.9

Shirt sale in a week in a shop. (i) The least number of shirts were sold in which class ? (ii) The maximum number of shirts were sold in which class ? (iii) How many shirts were sold upto the 42 shirt size ? (iv) How many shirts of size 4466 were sold ?

312

Mathematics

28.6 GRAPHS RELATED TO DAY-TO-DAY ACTIVITIES In addition to histograms and frequency polygons, we are sometimes faced with graphs of other types. When a patient is admitted in a hospital with fever the doctor/nurses prepare a temperature-time graph, which can be referred to any time for reference. Similarly, the velocity time graph and pressure-volume graph are of day-to-day use. We shall learn to draw these graphs and interpret them in the sections below : 28.6.1 Temperature-Time Graph-Reading and Construction Example 28.10 : The body temperature of a patient admitted in a hospital with typhoid fever at different times of a day are given below : Time of the day Temperature (in F) 7 hrs 102 9 hrs 103 11 hrs 104 13 hrs 103 15 hrs 101 17 hrs 100 19 hrs 99 21 hrs 100 23 hrs 99

Draw a graph to represent the above data. Solution : The graph of the above data is given in Fig. 28.10. The graph has been obtained by joining the points corresponding to pairs, like (7, 102), (9, 103), ........., (23, 99) in the rectangular system of coordinates, by line-segments.

Fig. 28.10

Note : While drawing the graph it has been assumed that during the time interval in between times, the same trend was present.

Graphical Representation of Data

313

Example 28.11 : If the medicine was given to the patient at 9 hours, whose temperature-time graph is shown in Fig. 28.10, answer the following questions : (i) At what time of the day was the temperature highest ? At what time lowest? (ii) After how much time, the action of medicine had started ? (iii) What trend do you observe from the above graph ? Solution : (i) The temperature of the patient was highest at 11 hours and lowest at 19 hrs and 23 hrs. (ii) The action of the medicine started 2 hours after the medicine was given as the temperature started falling after that. (iii) The administered medicine suited the patient as the temperature constantly fell after that, with the exception of period between 19 hrs and 21 hrs when it became slightly higher at 100F but again fell after that 28.6.2 Velocity Time Graph During a journey from one place to other, the speeds of vehicles keep on changing according to traffic congestions. This can be very well shown by a velocity-time graph. Let us illustrate it with the help of example : Example 28.12. During a journey from city A to city B by car the following data regarding the time and velocity of the car was recorded :

Time of the day (in hours) Velocity (in km/hour)

10

11

12

13

14

15

16

17

60

60

45

50

60

50

45

60

50

65

40

50

Represent the above data by a velocity time graph. Solution : As before the graph can be obtained by plotting the ordered pairs (6, 60), (7, 60), ... (15, 65), ..., (17, 50) in the rectangular system of coordinates and then by joining them by line-segments.

314

Mathematics

Fig. 28.11

Example 28.13 : Read the velocity-time graph given in Fig. 28.11 and reply the following questions : 1. At what time duration of the day, the velocity of the car (i) was lowest ? was highest ? (ii) constant (iii) went on increasing (iv) went on decreasing 2. What was the average speed of the car in the journey ? Solution : 1 (i) At 16 hours; At 15 hours (ii) The velocity was constant at 60 km/hour between 6 hours and 7 hours (iii) Between 8 hours to 10 hours (iv) Between 10 hours to 12 hours 2. The average speed of the car was

FG 60 + 60 + 45 + 50 + 60 + 50 + 45 + 60 + 50 + 65 + 40 + 50IJ H K 12
=
635 or 52.92 km/hour. 12

km/hour

Graphical Representation of Data

315

28.6.3 Pressure-Volume Graph For a fixed quantity of a gas at a constant temperature, is there any relation between pressure and volume of the gas ? Let us see that from the following example : Example 28.14 : The following data pertains to pressure and volume of a fixed quantity of gas : Pressure (p) (in Newton) Volume (v) (in cm3) 60 90 90 60 45 120 30 180 75 72

Draw a graph to represent the above data. Solution :

Fig. 28.12

The graph is obtained by joining the plot of the ordered pairs (60, 90), (90, 60), ..... (75, 72) by free hand curve. Example 28.15 : Read the above graph, given in Fig. 28.12, and answer the following questions : 1. Full in the blank : (i) As volume increases, the corresponding pressure ... (ii) As pressure decreases, the volume ... (iii) Pressure Volume = ...

316

Mathematics

2. What will be the pressure when volume is 100 cm3 ? 3. What will be the volume, when the pressure is 100 Newton ? Solution : 1 (i) Decreases (ii) Increases (iii) Constant = 5400 2. We know that pv = 5400
When volume = 100, p = 54 Newtons

as can be seen from the graph at point A 3. When p = 100 Newtons, v = 54 cm3 as can be seen from the graph at the point B. CHECK YOUR PROGRESS 28.3 Represent the data given in each of the questions below graphically : 1. For a town, the maximum temperature for the following months are given below : Months Maximum temperature (in C) March April 35 38 May 38 June July August September October 42 45 40 38 35

2. The body temperatures of a patient admitted in a hospital are given below: Time of the day (in hours) Temperature (in F) 8 9 10 11 12 13 14 99 15 99 16 100 17 98

103 104 105 102 102 100

3. The speeds of a car going from station A to station B at different times of the day are given below : Time of the day (in hours) Speed (in km/hour) 7 45 8 45 9 50 10 60 11 60 12 75 13 60 14 60 15 50

4. The data on pressure and volume of a gas are given below : Pressure (in Newtons) Volume (in cm3) 60 40 80 30 50 48 30 80 40 60 20 120

Graphical Representation of Data

317

5. For question No. 1, read the graph and reply the following questions : (i) Find the range of temperature for given months (ii) Which month had the least temperature ? (iii) Which month had the highest temperature ? (iv) In which month was the temperature less than 40F ? (v) Can you predict the temperature for the next two months ? 6. Read the graph of Question No. 2 and answer the following question : (i) At what time of the day was the temperature of the patient maximum? (ii) If the medicine takes at least two hours to show the effect, at what time of the day was the medicine given ? LET US SUM UP
z z

Bar graphs are the graphical representation of ungrouped frequency data. Histograms and frequency polygons are the graphical representation of continuous grouped frequency data. The graphical representation of data from day-to-day life is the join of points corresponding to ordered pairs represented by the data. The graphical representations show the trends readily and at a glance only. TERMINAL EXERCISE

Draw the bar graph for the following data in each case : 1. Height of samplings (in m) 0.5 No. of samplings 2. Weight (in kg) No. of baskets of apples 3. Number of parcels received in a post office Weight of parcels (in kg) 1 2 3 4 5 6 15 7 4 120 0.75 18 8 5 150 1.0 25 9 7 80 1.25 1.50 1.75 2.00 40 10 8 60 12 11 5 40 8 12 4 50 7 15 3

4. Interpret the data given in Question 1 and 2.

318

Mathematics

Draw a histogram and frequency polygon for the data in each case below: 5. Weight (in kg) 4045 No. of students in the class 6. Daily earning (in rupees) No. of workers 4 4550 7 5055 8 5560 9 6065 6 6570 3

100-120 5

120-140 7

140-160 8

160-180 180-200 3 2

7. Read the graphs for Question Nos. 5 and 6 and interpret them. 8. The minimum temperatures of a town for a year are given below : Month Min. temp. (in C) Jan Feb March April 12 14 16 20 May June July 20 24 25 Aug Sep Oct Nov Dec 24 22 18 16 12

Draw a graph to represent the above data and interpret it. 9. A man left New Delhi for Lucknow by car at 7 AM. The speed of the car at different times of the day is given below : Time of the day (in hour) Velocity (in km/hour) 7 45 9 50 11 60 13 65 15 70 17 60 19 55 21 40

Represent the above data by a velocity time graph and answer the following questions At what time was the velocity of the car (i) Maximum (ii) Minimum (iii) Between (5060) km/hour (iv) Can you give hypothesis regarding the places where speed is extreme ? 10. The following data pertains to a gas in a container : Pressure (in Newton) Volume (in cm3) 100 40 80 50 50 80 40 100 125 32 200 20

Represent the above data by a pressure volume graph. What relation do you find between pressure and volume from the data ?

Graphical Representation of Data

319

ANSWERS Check Your Progress 28.1 5. (i) Bhilai and Rourkela (iii) 50 (in ten thousand tonnes) Check Your Progress 28.2 6. (i) (4446) size (iii) 700 (ii) (4042) size (iv) 50 (ii) 8.33% (iv) 62.5%

320

Mathematics

29
Measures of Central Tendency
29.1 INTRODUCTION Sometimes we are required to describe the data arithmetically. The measures with which we do that are called Arithmetical Descriptors of Data. The name itself suggests that these measures describe the data arithmetically for example, average age of a group, average height of a class, median score of the group or modal collar size of a team. Because of the fact that these measures are representative of the group they represent, they are called Measures of Central Tendency central because these are the measures around which the measures of all the members of the group gather around. Represented graphically, the graph of the observations of the group will be around and close to the measure of central tendency. In this lesson, we will study about different measures of central tendency for ungrouped data and some of them for grouped data also. We will learn to calculate them by the formulae and will also learn properties of some of them. 29.2 OBJECTIVES After studying this lesson, the learner will be able to
z z z z z

define mean of raw, ungrouped and grouped data calculate mean of raw, ungrouped and grouped data by ordinary and short-cut-methods define mode and median of raw data calculate mode and median of raw data cite properties of mean and median

29.3 MEAN OF RAW DATA If x1, x2, x3, ..., xn are n observations, then their mean x is defined as Sum of all observations x = Number of observations or x =

x1 + x 2 + x 3 + ... + x n n

...(i)

Measures of Central Tendency

321

In abbreviated form, (i) can be written as

x =

i =1

xi
n

Let us take some examples to illustrate : Example 29.1 : Find the mean of first 10 natural numbers. Solution : We are required to find the mean of first ten natural numbers Mean = or Sum of first 10 natural numbers 10

x = 1 + 2 + 3 + ... + 10 10 = 55 = 5.5 10

Thus, the mean of first 10 natural numbers is 5.5 Examples 29.2 : Find the mean of first five prime numbers Solution : First five prime numbers are 2, 3, 5, 7 and 11 2 + 3 + 5 + 7 + 11 Their mean x = 5 = 28 = 5.6 5

Thus, the mean of first five prime numbers is 5.6. Example 29.3 If the mean of 5, 7, 9, x, 11 and 12 is 9, find x. Solution : We know that the mean x of observations is given by x = As, x is given to be 9

Sum of observations Number of observations

9=

5 + 7 + 9 + x + 11 + 12 6

or
The value x is 10

54 = 44+ x x = 54 44 = 10

322

Mathematics

29.4 MEAN OF UNGROUPED DATA Let the frequency distribution for ungrouped data be given as below : Observations Frequency : : x1 f1 x2 f2 x3 f3 ... ... x10 f10 ... ... xn fn

The above data shows that observation x1 is repeated f1 times, x2, f2 times and so on.
The sum of all these observations is given by

f1x1 + f2x2 + ..... + f10x10 + ...... + fnxn and the total number of observation is given by f1 + f2 + ...... + f10 + ..... + fn
x , the mean of observations, is given by

x =

f1x1 + f2 x 2 + ... + f10 x10 + ... + f n x n f1 + f2 + ... + f10 + ... + f n


n

...(ii)

In abbreviated form (ii) is written as

x =

i =1 n

fi x i
i =1 n

fi

or

x =

i =1

fi x i
n

, where n = fi
i =1

Let us take some examples to illustrate Example 29.4 : Find the mean of the following data : xi fi : : xi 2 5 8 10 12 15 2 4 5 5 fi 4 5 3 5 2 1
f i = 20

8 3

10 5 fi . xi 8 25 24 50 24 15

12 2

15 1

Solution : We make the tabular form of data as follows :

146 fi x i

Measures of Central Tendency

323

We know Mean x is given by

bg

fi x i x = f i
=
146 = 7.3 20

Example 29.5 : If the mean of the following data is 7, find x xi fi : : 4 2 x 4 6 6 7 10 9 6 11 2

Solution : Let us put the data, as usual, in tabular form. We get the following: xi 4 x 6 7 9 11
fi

fi 2 4 6 10 6 2 = 30

fi . xi 8 4x 36 70 54 22 (190 + 4x) f i x i

We know that x = 7

7=

190 + 4 x 30

or or

210 190 = 4x x=5

The value x is 5.

29.5 MEAN OF GROUPED DATA We know that for grouped data, the frequency in a class is centered at its class mark. Therefore, the first step in finding mean of grouped data is to find the class marks, say, x1, x2, x3, .... xk for k classes with class frequencies f1, f2, f3, ....., fk. Further, the steps are the same as for mean for ungrouped data. Thus, the mean x for grouped data is given by x =

f1x1 + f2 x 2 + ..... + f k x k f1 + f2 + .....+ f k

....(iii)

324

Mathematics

where, x1, x2, ....., xk are class marks of k classes and f1, f2, ....., fk are their frequencies. In abbreviated form (iii) can be written as

x =

i =1 k

fi x i
i =1

fi

Let us take some examples to illustrate : Example 29.6 : Find the mean of the following grouped data : Classes Frequencies 010 1 1020 3 2030 5 3040 7 4050 5 5060 3 6070 1

Solution : Let us put the data in tabular form Classes 010 1020 2030 3040 4050 5060 6070 Frequencies (Class marks) (fi) xi 1 3 5 7 5 3 1
f i = 25

fixi 005 045 125 245 225 165 065 875 f i x i

5 15 25 35 45 55 65

fi x i 875 Mean x = f = = 35 25 i

You can very well see that if the class marks and frequencies are large, it is difficult to find the mean as it involves lots of calculations. To simplify the procedure, we follow the following method, called Assumed Mean Method or Short cut Method. The method involves the following steps : Step 1 : Find the class-marks Step 2 : Take any convenient class mark as assumed mean A (usually the central one or with maximum frequency is taken)

Measures of Central Tendency

325

Step 3 : Find the deviations (xi A) Step 4 : Find the step deviations

xi A , where c is the interval and call them di c

Step 5 : Find fi d i and use the following formula to find x = A+

fi d i c fi

Let us try to apply this method on Example 6. Example 29.7 : Find the mean of the following grouped data using short cut method : Classes Frequencies : : 010 1020 2030 3040 4050 5060 6070 1 3 5 7 5 3 1

Solution : Let the assumed mean be 35. Classes 010 1020 2030 A 3040 4050 5060 6070 fi 1 3 5 7 5 3 1 xi 5 15 25 35 45 55 65 di =

xi A c
3 2 1 0 1 2 3

fidi 3 6 5 0 5 6 3 0 fidi

x = 35 +

fi d i c fi

=35 + 0 = 35 which is same as found in Example 29.6. Example 29.8 : Classes Frequencies : : Find the mean of the following data 0100 100200 200300 300400 400500 500600 15 25 40 30 10 5

326

Mathematics

Solution : Let the assumed mean be 250. Classes 0100 100200 200300 300400 400500 500600 fi 15 25 40 30 10 5 xi 50 150 250 350 450 550 di =

xi 250 100
2 1 0 1 2 3

f id i 30 25 0 30 20 15 10 f i x i

fi = 125

fid i x = A+ f c i
10 100 = 258 = 250 + 125 CHECK YOUR PROGRESS 29.1

1. Write the formulae for calculating mean of (i) Raw data (ii) Ungrouped data (iii) Grouped Dataordinary method (iv) Grouped DataShortcut method. 2. Find the mean of first 6 multiples of 5 3. Find the mean of all multiples of 7 which are less than 100. 4. Find the mean of following data : xi fi : : 7 3 9 4 12 5 15 8 21 3 24 2

5. Find the mean of following grouped data (1) by ordinary method, (ii) by short cut method and confirm your results. (a) Classes Frequencies (b) Classes : : 20-40 3 40-60 6 60-80 80-100 100-120 120-140 140-160 9 12 9 6 5

: 0-500 500-1000 1000-1500 1500-2000 2000-2500 2500-3000 14 16 20 30 12 8

Frequencies :

Measures of Central Tendency

327

29.6.

MEDIAN OF RAW DATA

Median is that value of the variate which divides the raw data into two equal halves, i.e., equal number of observations on its both sides, when arranged in order (ascending or descending) For finding the median of raw data, we first arrange the observations in ascending (or descending) order of magnitude and then follow the following : + 1I FH n 2 K th observation F nI Fn I (ii) If the number of observations n is even, the median is the average of H 2 K th and H 2 + 1K th (i) If the number of observations n is odd, the median is observations Let us illustrate it with examples. Example 29.9 : Find the median of the following data : (a) 5, 6 , 9 , 3, 18, 16, 10 (b) 16, 4, 18, 25, 17, 19, 12, 20, 15, 7 Solution : (a) Here the number of observations is odd (7 here). Arranging the data in ascending order, we have 3, 5, 6, 9, 10, 16, 18 The median is
Median = 9

+ 1I FH 7 2 K th or 4th observation

(b) Here n is even (10) Arranging the data is ascending order, we have 4, 7, 12, 15, 16, 17, 18, 19, 20, 25 The median is average of 10 th and 2

FH IK

FH 10 + 1I K th observations i.e. 5 2

th

and 6th observations

16 + 17 = 16.5 2 Example 29.10 : If the median of the following data is 20, find x : Median = 19, x, 16, 18, 22, 17, 24, 23, 21 Solution : As the median is 20, and there are four observations more than 20 and four observations less than 20, x must be 20 Arranging the data in ascending order, we have 16, 17, 18, 19, x, 21, 22, 23, 24

328

Mathematics

CHECK YOUR PROGRESS 29.2 1. Fill in the blanks suitably : (i) Median is that value of the variable ............... (ii) When the number of observations n is odd, median is given by ............... observation (iii) When the number of observations n is even, median is given by ............... observation. 2. Find the median of the following : (a) 5, 12, 16, 18, 20, 25, 10 (b) 18, 80, 42, 28, 15, 72, 69, 52, 46 (c) 15, 18, 7, 12, 19, 25, 17, 20 (d) 6, 12, 9, 10, 16, 28, 25, 13, 15, 17 3. If the median of the following data arranged in ascending order is 17, find x 12, 15, 16, x, 18, 19, 20, 22 29.7 MODE OF RAW DATA Mode is that value of the variable which occurs most frequently in the data. The method involved is to find the distinct values of observations in the data and find the frequency of occurrence of each. The observation with the maximum frequency is called mode. Let us take some examples to illustrate. Example 29.11 : Find the mode of the following data : 15, 11, 13, 11, 15, 12, 16, 18, 15, 19, 11, 15, 16, 8, 12 Solution : Here the distinct observation are 8 1 11 3 12 2 13 1 15 4 16 2 18 1 19 1 with their respectively frequencies as Thus, the observation 15 has maximum frequency (4 here)
Mode of the data is 15

Example 29.12. Find the value of x so that the mode of the data is 11 12, 11, 18, 20, 25, 11, 13, x, 13, 18, 13, 11, 19, 20 Solution : The distinct values of observations are 11, 3 12, 13, 1 3 18, 2 19, 1 20, 2 25, 1 x with frequencies The mode of data is 11, then 11 has to have maximum frequency, that is more than 3. Therefore, x = 11.

Measures of Central Tendency

329

Example 29.13 : If two of the 20s in Example 29.12, where x = 11, are changed to 13 each find the mode of the new data. Solution : Now the data becomes xi : fi : 11 4 12 1 13 5 18 2 19 1 25 1(* As x = 11)

The new mode is 13.

CHECK YOUR PROGRESS 29.3 1. Define mode of raw data. 2. Does the mode depend on number of observations ? 3. Find the mode of the following data : (a) 15, 25, 18, 16, 25, 19, 18, 25, 16, 19, 25 (b) 1, 9, 22, 16, 15, 28, 9, 14, 16, 9, 28 (c) 28, 7, 28, 17, 7, 19, 15, 7, 28, 7, 15, 18, 7 4. (a) Find the value of x so that the mode of the following data is 37 : 12, 33, 37, 18, 19, 37, x, 33, 12, 33, 18, 37 (b) What will be the new mode if one of the 37 after x is replaced by 33. 5. Find the mode of the following data : 28, 22, 21, 29, 12, 13, 18, 22, 14, 16, 28, 29, 13, 28, 22, 14, 22, 16, 19, 16, 15, 18, 22, 29 29.8 PROPERTIES OF MEAN 1. If two groups with observations n1 and n2 have their means x1 and x 2 , then the combined mean x of these is given by x =

n1x1 + n 2 x2 n1 + n 2

2. If x1, x2, x3, ..., xn observations have a mean x , then x1 x + x 2 x + x 3 x + ... + x n x = 0


n

i.e.

i =1

g b g b b x xg = 0
i

3. If each observation in the data is increased by a, then the mean is increased by a. 4. If each observation in the data is decreased by b, then the mean is decreased by b.

330

Mathematics

5. If each observation in the data is multiplied by some constant, then the mean gets multiplied by the same constant. 6. If each observation in the data is divided by some non-zero constant, then the mean gets divided by the same non-zero constant. 29.9 PROPERTIES OF MEDIAN 1. Median is the middlemost observation of the data i.e., it divides the data exactly in two halves, when arranged in order (ascending or descending). 2. Median, unlike mean, is not affected by extreme values. 3. Median may not lie in the data itself. 4. Median can be determined graphically also while mean can not. LET US SUM UP
z z

The three measures of central tendency are mean, median and mode. Mean is that value in the group of observations which describes it arithmetically.

z z

Mean x of raw data is given by x = Mean x of grouped data is given by


i =1 k

i =1

xi
n

(i) x =

fi x i
i =1

fi
k

, where k is the number of classes and xis are class marks

i =1 (ii) x = A + k

fi d i
i =1

fi
xi A c

where A is the assumed mean di = and c is the length of interval.


z

Median of raw data is given by nI FH 2 K th observation if the number of observation (n) is odd. F nI Fn I (ii) average of H 2 K th and H 2 + 1K th observations of n is even. (i)

Mode is that value of variate which occurs most frequently.

Measures of Central Tendency

331

TERMINAL EXERCISE 1. Find the mean of (i) 2, 9, 19, 12, 15 and 9 (ii) 3, 6, 9, 12, 15, 18, 19, 21 (iii) 5, 2, 19, 15, 13, 18, 13, 11 2. If the mean of 8, 12, x , 14, 18 is 12, find x. 3. If the mean of 13, 12, 18, 14, p, 19, 17, 14 is 14 find the value of p. 4. Find the mean of the following : (a) xi : fi : 5 3 9 5 13 12 20 5 17 8 22 7 22 7 24 5 25 5 26 3

(b) xi : 16 18 fi : 1 3

5. Find the mean of the following data : (a) Classes : 10-20 2 20-30 3 30-40 5 40-50 7 50-60 5 60-70 3

Frequencies : (b) Classes

: 100-200 200-300 300-400 400-500 500-600 600-700 3 5 8 6 5 3

Frequencies :

6. Find the mean of data given in 5(a) and 5(b) by assumed mean method. 7. Find the median of the following data : (a) 3, 19, 17, 15, 8, 5, 9 (b) 15, 13, 8, 22, 29, 12, 14, 17, 6 8. If the mode of the following observation is 12, find x (a) 16, 13, 8, 12, 19, 17, 12, 16, x, 19 (b) 15, 16, 19, 12, 13, 16, 12, 12, 12, 16, x 9. Find the mode of the following data : (a) 8, 5, 2, 5, 3, 5, 3 (b) 19, 18, 17, 16, 17, 15, 14, 17, 15 10. If the median of the follows data is 25, find x 16, 18, 19, 23, x, 29, 31, 35.

332

Mathematics

ANSWERS Check Your Progress 29.1

1. (i) x =

i =1

xi
n

(ii) x =

i =1 n

fi x i
i =1

fi
fi d i c fi

(iii) x = 2. 17.5 5. (a) 92.4

i =1 k

fi x i
i =1

fi
3. 52.5 (b) 1420

(iv) x = A + 4. 13.92

Check Your Progress 29.2 1. (i) Which divided the data into two equal halves. (ii) n + 1 th 2

(iii) Average of 2. (a) 16 3. 16

n n th and 2 + 1 th 2 (b) 46

FH

IK

(c) 17.5

(d) 14

Check Your Progress 29.3 1. 2. 3. 4. 5. The value of the variable which occur most frequency is defined mode of raw data No (a) 25 (b) 9 (c) 7 (a) x = 37 (b) New mode = 33 22

Terminal Exercise 1. 2. 4. 5. 6. 7. 8. 9. (i) 8 (a) (a) (a) (a) (a) (a) 11 15.775 42.6 42.6 9 12 5 (ii) 3. (b) (b) (b) (b) (b) (b) 12.875 5 21.75 396.7 396.7 14 12 17 (iii) 12

10. 27

Introduction to Probability

333

30
Introduction to Probability
30.1 INTRODUCTION In our day-to-day life, we sometimes make the following statements : It may be foggy today. The train may reach in time. Ram is likely to get good marks in his examination. Probably, he will come today. The usage of terms of the type may, likely or probably convey the meaning that the event, we are talking about, is not certain to occur. Under the head of probability, we will study the measure of this uncertainty under given conditions. If we go by the dictionary meaning of the word probable it says likely though not certain. The word probability has been derived from the same and is intended to be used to have a measure of uncertainty in making statements or taking decisions in such situations. Thus, probability measures the degree of uncertainty involved and hence measures the degree of certainty of occurrence of events. Interestingly, the origin of the theory of probability has been the study of games of chance and is connected with events like tossing a coin, throwing a die, selecting a card at random from a well shuffled pack of cards. 30.2 OBJECTIVES After studying this lesson, the learner will be able to :
z z z z

define an experiment, an outcome and an event define different types of events define probability of occurrence of an event solve problems based on tossing a coin, throwing a die, and drawing a card from a well shuffled pack of cards.

334

Mathematics

30.3 EXPECTED BACKGROUND KNOWLEDGE We assume that the learner is already familiar with :
z z z z

the terms connected with coin-head and tail ) the terms connected with a dice the dots on six faces as (, , , , , four fundamental operations on numbers terminology connected with playing cards

30.4 SOME DEFINITIONS Before saying something about probability, it will be desirable to have knowledge of basic terms being used in the text. We will take them one by one below 30.4.1 Experiment An activity which ends in some well defined result is called an experiment. Drawing a card from a pack of cards, throwing a die and tossing a coin are examples of experiments. 30.4.2 Trial and Outcome Performing an experiment once is called a trial and the result of the trial is called as outcome. 30.4.3 Random Experiment An experiment in which all possible outcomes are known before but will be the result of a trial can not be surely predicted, is called a random experiment. For example, tossing a fair coin or throwing a fair die Note : By the word fair, we mean that no trick has been played with the object being tossed, otherwise we can always predict the result. If both the faces of a coin have head, the result will always be head. 30.4.4 Event All the possible outcomes of a trial are called events. Any specific of these is called an event 30.5 TYPES OF EVENTS Let us now study about some types of events and sample space : 30.5.1 Sample space The collection of all possible outcomes of an experiment constitute its Sample Space. For example, when a coin is tossed, the sample space consists of Head (H) and Tail (T). When a die is thrown, the sample space consists of , , , , called 1, 2, 3, 4, 5 and 6) , , (sometimes

Introduction to Probability

335

30.5.2 Favourable Event The cases which ensure the occurrence of an event, are called favourable to that event. For example, the favourable cases to the event occurrence of an odd number, when a die is thrown are 1, 3, 5. 30.5.3 Mutually Exclusive Events If two events cannot occur together, they are said to be mutually exclusive events. For example, In a throw of a coin, head and tail are mutually exclusive events. 30.5.4 Equally Likely Events Two events are said to be equally likely events if they have equal chance of occurrence For example, in a fair toss of a coin, occurrence of head and tail are equally likely events. 30.6 PROBABILITY OF OCCURRENCE OF AN EVENT

The probability P(E) of occurrence of an event E is defined as P(E) = Number of outcomes favourable to the event Total number of possible outcomes 1 . 2

Thus, the probability of occurrence of a head (H) when a coin is tossed is

Suppose there are m cases (outcomes) which are favourable to an event capital X and n is the total number of outcomes, then the probability of occurrence of the event X is given by P(X) = We know that
m n

...(i) ...(ii)

mn 0 P(X) 1

From (i) and (ii), we can say that

When do we say that the probability of occurrence of event x is zero ? When there is no outcome favourable to the event. Such an event is called an Impossible event. For example getting a number greater than 6 when a die is thrown once is an impossible event. In the case, when P(X) = 1, all the possible out comes are favourable to the event. In that case, the event is said to be a Sure event. For example, getting a number less than 7, when a die is thrown once is a sure event. Let us now introduce another concept, called probability of non-occurrence of an event.

336

Mathematics

m If P(X) = n , then there are (n m) outcomes which are not favourable to the occurrence of X. Then the probability of non-occurrence of the event X, called P( X) is given by P( X) = nm = n m n n n

m = 1 = 1 P(X) n i.e., the probability of non-occurrence of an event equals [1 Probability of occurrence of the event] Note : X is called an event complementary to event X. Let us now take some examples to illustrate. Example 30.1 : Write the sample space if a die is thrown once.

Solution : We know that when a die is thrown once, the likely outcomes are or 1, 2, 3, 4, 5, 6 , , , , , ,

This is the required sample space. Example 30.2 : 3 males and 4 females appear for an interview, of which one candidate is to be selected. Write the sample space. Solution : Let the three male candidates be denoted by M1, M2, M3 and four female candidates be denoted by F1, F2, F3 and F4. The sample space consists of M1, M2, M3, F1, F2, F3, F4. Examples 30.3 : Find the probability of occurrence of a tail (T) when an unbiased coin is tossed once. Solution : When a coin is tossed once, the sample space is H, T There is only one outcome favourable to a tail (T).
The probability of occurrence of a tail = 1 2

Example 30.4 : A die is tossed once, find the probability of occurrence of (i) 2 (ii) an even number Solution : When a die is tossed, the sample space is 1, 2, 3, 4, 5, 6

Introduction to Probability

337

(i) Probability of occurrence of 2, i.e., P(2) = 1 6

because only one outcome is favourable to the event out of 6 possible outcomes. (ii) Probability of occurrence of an even number The even numbers are 2, 4, 6 The number of favourable outcomes = 3 3 1 P (even number) = 6 = 2 Example 30.5 : Find the probability of getting an odd number when a die is tossed once. Solution : The number of odd numbers is 3 (1, 3, 5 are odd numbers) If E is the event, called, occurrence of an odd number, then P(E) = 3=1 6 2

You could have found P(E) as P(E) = 1 P (even number) = 1


1 1 = 2 2

Example 30.6 : In case of Example 2; find the probability of selection of a (i) male candidate (ii) female candidate Solution : Total number of candidates = 7 Numbers of males = 3 3 (i) P (selection of a male candidate) = 7 (ii) P (selection of a female candidate) = 4 7

Alternatively P (selection of female candidate) = 1 P (selection of a male candidate) = 1 3=4 7 7

338

Mathematics

Example 30.7 : From a well shuffled pack of cards, if a card is drawn, find the probability of occurrence of a (i) red card (ii) black king (iii) a queen Solution : (i) Since there are 26 red cards in a pack of 52 cards, the probability of occurrence of a red card, P (R), is given by P(R) = 26 = 1 52 2

(ii) Out of 52 cards, there are two black kings


The probability of occurrence of a black king P(B) is given by

P(B) =

2 = 1 52 26

(iii) Similarly, the probability of occurrence of a queen P (Q) is given by P(Q) = 4 = 1 52 13

Examples 30.8 : A bag contains 4 red, 3 black and 2 white balls. A ball is drawn from the bag. Find the probability (i) that it is a red ball (ii) it is not black. Solution : (i) No. of red balls = 4 Total no. of balls = 9

P (a red ball) =

4 9

(ii) No. of favourable cases = 6 (4 red + 2 white) or P (getting a black ball) = 3=1 9 3

1 2 P (not getting a black ball) = 1 3 = 3 2 P (not getting a black ball) = 3 CHECK YOUR PROGRESS 30.1 1. Define (i) Sample space (ii) Experiment and a random experiment

Introduction to Probability

339

(iii) Event and outcome (iv) Types of events (v) Probability fo occurrence of an event 2. Write the sample space for the following : (i) when a fair coin is tossed once (ii) when a fair die is tossed once (iii) when 6 boys and 5 girls appear for a selection. 3. Find the probability of occurrence of (a) a multiple of 2 when a die is tossed (b) a multiple of 3 when a die is tossed (c) a number greater than 4 when a die is tossed (d) a number greater than 6 when a die is tossed (e) a multiple of 4 which is less than 3 4. Find the probability of selection of (a) a boy (b) a girl for Question 2(iii) 5. An urn contains 6 black and 4 red balls. One ball is selected from the urn at random. Find the probability of selecting a (i) black ball (ii) red ball 6. A group of people have following distribution of members according to age Number 5 10 15 Age 30 years and below (31 40) years (41 60) years

A person is selected at random from the group. Find the probability of getting a person with age (i) less than or equal to 40 yrs (ii) beyond 40 years (iii) less than or equal to 30 years.

340

Mathematics

LET US SUM UP
z z

Probability is a measure of index of uncertainty of occurrence of an event. An activity that ends in some well defined result is called an experiment. An experiment in which the sample space of outcomes is known but which one will occur cannot be predicted is called a random experiment. Performing an experiment once is called a trial and its result is called an outcome. The collection of all possible outcomes is called sample space. Outcomes which ensure the occurrence of an event are called favourable cases to the occurrence of that event The events which cannot occur together are called mutually exclusive events Events which have equal chances of occurrence are called equally likely events. Probability of occurrence of an event. =
Number of outcomes favourable to the event Total Number of outcomes

z z z

z z z

TERMINAL EXERCISE 1. Define probability of occurrence of an event. 2. Define sample space. Write the sample space for (i) selection of a candidate having 8 male and 6 female candidates (ii) when a die is tossed (iii) An urn with 4 red and 3 black balls 3. Find the probability of the following events when a die is tossed once: (i) occurrence of 4 (ii) occurrence of a number greater than 2 (iii) occurrence of a number less than 3 (iv) occurrence of multiple of 4 (v) occurrence of a number greater than 6 (vi) occurrence of a number greater than or equal to 1 (vii) occurrence of a number which is prime.

Introduction to Probability

341

4. An urn contains 6 red, 4 black and 5 green balls. A ball is selected at random. Find the probability of selection of a (i) red ball (ii) black ball (iii) green ball (iv) non-red ball (v) non black ball 5. Find the probability of a non-leap year (365 days) containing 53 Sundays. 6. In an interview for the selection of a candidate, 6 boys and 2 girls have been called. Find the probability of selection of a (i) boy (ii) girl (using the concept of complementary events)

342

Mathematics

ANSWERS Check Your Progress 30.1 2. (i) H, T (ii) 1, 2, 3, 4, 6 (iii) B1, B2, ....., B6, G1, G2, ....., G5 3. 4. 5. 6. (a) (a) (i) (i) 1 2 6 11 3 5 1 2 (b) (b) (ii) (ii) 1 3 5 11 2 5 1 2 (iii) 1 3 (c) 1 3 (d) zero (e) zero.

Terminal Exercise 3. (i) 1 6 (ii) (vii) (ii) 2 3 1 2 4 15 (iii) 1 3 (iv) 3 5 (v) 11 15 (iii) 1 3 (iv) 1 6 (v) zero

(vi) 1 4. 5. 6. (i) 1 7 (i) 3 4 2 5

(ii)

1 4

Introduction to Probability

343

APPENDIX A DIE A die is a stable cube with its six faces marked with dots from 1 to 6. (See Fig. 30.1). The plural of the word die is dice. Whenever, we roll a die and ask for a number, if that number comes on the top, the attempt is described as win. If you ) has carefully see the figure on the side, the number 6 ( come on the top and if the player had asked for that number, he would have won in that trial.

Fig. 30.1

Remmeber, for an unbiassed die, each number 1 to 6 is equally likely to come on the top (occur) A PACK OF CARDS A pack of card consists of 52 cards divided into 4 suits (colours) called (i) Spades (ii) Hearts (iii) Diamonds (iv) Clubs

Each suit consists of 13 cards Ace, 2, 3, ..., 10, Jack, Queen and King. Jack, Queen and King are called face cards If we draw a card from a well shuffled pack of cards, each of the 52 cards have equal chance of being drawn. Probability (of selection of a card) = Particularly, probability (Jack of spades) = 1 52 1 52

Black colour Red colour Red colour Black colour

Practice Work

205

Secondary Course Mathematics


Practice WorkAlgebra

Maximum Marks : 25 Instructions : 1. Answer all the questions on a separate sheet of paper. 2. Give the following informations on your answer sheet.
z z z z z

Time : 45 Minutes

Name Enrolment number Subject Topic of practice work Address

3. Get your practice work checked by the subject teacher at your study centre so that you get positive feedback about your performance. Do not send your practice work to National Institute of Open Schooling. 1. Fractions are called : (A) positive integers (B) positive rationals (C) negative rationals (D) negative integers 2. Which of the following is not an integer ? (A) 4 (B) 25 (C) 36 (D)
4 5

206

Mathematics

3. Which of the following algebraic expressions is not a polynomial ? (A) x2(x + 1) (B) 5 (C)

3x + x3

(D) 3 x + x2 4. The reciprocal of

FH 2 I 3K

is :

(A)

FH 2 I 3K FH 2 I 3K
3I FH 2 K 3I FH 2 K
3

(B)

(C)

(D)

5. The tenth term of the A.P. 2, 4, 6, ... is : (A) 20 (B) 18 (C) 20 (D) 22 6. If * is an operation defined on a and b as follows : a * b = a + b (3), then find (4) * 3

7. Find two numbers nearest to 1000 which are exactly divisible by 4, 5, 6, 7 and 10 each. 2 8. Express 0. 67 as a rational number. 2

Practice Work

207

9. Simplify :

3.2 x 2 x 1 . 2 x 1 2 x 2

10. The sum of the first four and the first five terms of an A.P. are 26 and 40 respectively. If the first term is 2, find the common difference. 2 11. If mth, nth and rth term of an A.P. are x, y and z respectively, prove that x(n r) + y(r m) + z(m n) = 0. 4

12. The area of a rectangle gets increased by 4 m2 if its length is increased by 4 m and breadth decreased by 2 m. If the length is increased by 2 m and breadth decreased by 2 m, the area is decreased by 8 m2. Find the dimensions of the rectangle. 6

Practice Work

289

Secondary Course Mathematics


Practice WorkCommercial Mathematics

Maximum Marks : 25 Instructions : 1. Answer all the questions on a separate sheet of paper. 2. Give the following informations on your answer sheet.
z z z z z

Time : 45 Minutes

Name Enrolment number Subject Topic of practice work Address

3. Get your practice work checked by the subject teacher at your study centre so that you get positive feedback about your performance.

Do not send your practice work to National Institute of Open Schooling. 1. A saree is available for Rs 450 whose marked price is Rs 600. Rate of discount given by the shop is : 1 (A) 15% (B) 20% (C) 25% (D) 30% 2. A customer has to pay Rs 20 more, if the discount is reduced from 20% to 15%. The marked price of the item is : 1 (A) Rs 420 (B) Rs 400 (C) Rs 380 (D) Rs 385

290

Mathematics

3. A man sold an article for Rs 187.50. If he had spent Rs 12.50 on transportation and had a gain of 25%, the cost price of the article was : 1 (A) Rs 175.00 (B) Rs 137.50 (C) Rs 150.00 (D) Rs 140.00 4. A sum of money becomes Rs 2000 in 2 years and Rs 2250 in 4 years at the same rate of simple interest. The sum is : 1 (A) Rs 2000 (B) Rs 1850 (C) Rs 1750 (D) Rs 1600 5. Which one is a better investment ? (A) 12% per annum interest compounded yearly. (B) 6% per half year interest compounded half yearly. (C) 3% per quarter interest compounded quarterly. (D) 1% per month interest compounded monthly. 6. The sides of a triangle are in the ratio 1 : 1.5 : 2. If the perimeter is 13.5 cm, find the length of each side. 2 7. A watch is sold for Rs 405 at a loss of 10%. Another watch is sold for Rs 540 at a gain of 8%. Find the total gain or loss in both transactions. 2 8. The cost price of 15 pens is equal to selling price of 12 pens. Find the profit percent. 2 9. A shopkeeper allows 25% discount on his articles in a sale. However he has to charge 10% sales tax on goods sold. Find the marked price of an article, if a customer has to pay Rs 330 inclusive of sales tax, to buy it. 2 10. The population of a village in the year 2000 was 8000. Its population increased by 5% during the year 2001 and 4% during the year 2002. Find the total population at the end of the year 2002. 2 11. The difference between the compound interest and simple interest as a certain sum of money at 10% per annum for 2 years is Rs 155. Find the sum, if the interest is compounded yearly. 4 12. A T.V. set is available for Rs 7500 cash or Rs 2000 as cash down payment followed by 6 monthly instalments of Rs 1000 each. Find the rate of interest charged under instalment plan. 6 1

190

Mathematics

Secondary Course Mathematics


Practice WorkGeometry

Maximum Marks : 25 Instructions : 1. Answer all the questions on a separate sheet of paper. 2. Give the following informations on your answer sheet.
z z z z z

Time : 45 Minutes

Name Enrolment number Subject Topic of practice work Address

3. Get your practice work checked by the subject teacher at your study centre so that you get positive feedback about your performance. Do not send your practice work to National Institute of Open Schooling. 1. Lines AB and CD intersect each other at O as shown in the adjacent figure. A pair of vertically opposite angles is : 1 (A) 1, 2 (B) 2, 3 (C) 3, 4 (D) 2, 4 2. Which of the following statements is true for a ABC ? (A) AB + BC = AC (B) AB + BC < AC (C) AB + BC > AC (D) AB + BC + AC = 0 1

Practice Work

191

3. The quadrilateral formed by joining the mid points of the pair of adjacent sides of a rectangle is a : 1 (A) rectangle (B) square (C) rhombus (D) trapezium 4. In the adjacent figure, PT is a tangent to the circle at T. If BTA = 45 and PTB = 70, Then ABT is : 1 (A) 110 (B) 70 (C) 45 (D) 25 5. Two points A, B have co-ordinates (2, 3) and (4, x) respectively. If AB2 = 13, the possible value of x is : 1 (A) 6 (B) 0 (C) 9 (D) 12 6. In ABC, AB = 10 cm and DE is parallel to BC such that AE =
1 AC. Find AD. 2 4

7. If ABCD is a rhombus, then prove that 4AB2 = AC2 + BD2.

192

Mathematics

8. Find the co-ordinates of the point on x-axis which is equidistant from the points whose co-ordinates are (3, 8) and (9, 5). 2 9. The co-ordinates of the mid-point of a line segment are (2, 3). If co-ordinates of one of the end points of the segment are (6, 5), then find the co-ordinates of the other end point. 2 10. The co-ordinates of the vertices of a triangle are (3, 1), (10, 7) and (5, 3). Find the co-ordinates of its centroid. 2 11. In an acute angled triangle ABC, ADBC. Prove that AC2 = AB2 + BC2 2BC. BD equal in area. 4 6

12. Prove that parallelograms on equal (or same) bases and between the same parallels are

Secondary Course Mathematics


Practice WorkMensuration

Maximum Marks : 25 Instructions : 1. Answer all the questions on a separate sheet of paper. 2. Give the following informations on your answer sheet.
z z z z z

Time : 45 Minutes

Name Enrolment number Subject Topic of practice work Address

3. Get your practice work checked by the subject teacher at your study centre so that you get positive feedback about your performance. Do not send your practice work to National Institute of Open Schooling. 1. If a is the side of an equilateral triangle, then its altitude is : (A)
3 2 a 2

(B)

3 2a 2
3 a 2

(C)

(D)

3 2a

238

Mathematics

2. The sides of a triangle are 30 cm, 40 cm and 50 cm. Its area is : (A) 120 cm2 (B) 600 cm2 (C) 750 cm2 (D) 1200 cm2 3. The perimeter of a square of side l is given by : (A) l2 (B) 4l (C) l 2 (D) 2l

4. The base of an isosceles triangle is 8 cm and one of the equal sides is 5 cm. The height of the triangle is : 1 (A) 5 cm (B) 4 cm (C) 3 cm (D) 2 cm 5. The edge of a cube, whose volume equals that of a cuboid of dimensions 63 cm 56 m 21 cm is : 1 (A) 21 cm (B) 28 cm (C) 36 cm (D) 42 cm 6. How long will a man take to walk round the boundary of a square field of area 90000 sq. m at the rate of 6 km an hour ? 2 7. One of the diagonals and a side of a rhombus are 8 cm and 5 cm respectively. Find the area of the rhombus. 2 8. The radii of two right circular cylinder are in the ratio 3 : 4 and their heights are in the ratio 5 : 2. Find the ratio of their curved surface areas. 2 9. Three equal cubes are placed end-to-end in a row. Find the ratio of the total surface area of the new cuboid to that of the sum of the surface areas of the three cubes. 2

Practice Work

239

10. A room is 7 m long and 4 cm wide. It has two doors of 2 m 1 m each and two windows 1 m 1m each. The cost of painting the walls at the rate of Rs 5 per square meter is Rs 300. Find the height of the room. 2 11. Spherical marbles of 1.4 cm diameter are dropped into a cylindrical vessel of diameter 7 cm containing water. Find the number of marbles that should be dropped, if the water level in the cylinder rises by 5.6 cm. 4 12. A cubic meter of iron is melted to form a wire of diameter 3.5 mm Find the length of the wire.

FG Use = 22 IJ H 7K

284

Mathematics

Secondary Course Mathematics


Practice WorkTrigonometry

Maximum Marks : 25 Instructions : 1. Answer all the questions on a separate sheet of paper. 2. Give the following informations on your answer sheet.
z z z z z

Time : 45 Minutes

Name Enrolment number Subject Topic of practice work Address

3. Get your practice work checked by the subject teacher at your study centre so that you get positive feedback about your performance. Do not send your practice work to National Institute of Open Schooling.

p 1. If sin = q , then cos is : q (A) p p (B) 1 q


q 2 p2 q p2 q 2 q

(C)

(D)

Practice Work

285

2. In ABC, right angled at B, AB = 5 cm, AC = 13 cm and C = , then cos is equal to : 1 (A)


5 13 13 5 5 12 12 13 3 , then the value of 1 + cosec is : 5

(B)

(C)

(D)

3. If sin = 8 5 8 3 9 5 5 9

(A) (B) (C) (D)

6 sin 5 cos 4. If 2 tan = 3 , then the value of is : 4 sin + cos

(A) 2 (B)
4 5 13 7 4 7

(C)

(D)

286

Mathematics

5. Representation of sin interms of tan is : (A)

tan 1 + tan 2 tan 1 tan 2 1 tan 1 + tan 2


1 tan 1 tan 2
2 tan A . 1 + tan 2 A

(B)

(C)

(D)

6. Show that 2 sin A cos A =

2 2 2

7. Prove that sin4A cos4A = sin2A cos2A. 8. Eliminate from

b g y = bbsec tan g
x = a sec + tan

9. For what value of , cos and sec are equal ? 10. If x = a sin + cos

2 2

b g y = bbsin cos g , prove that


x2 y2 + =2 a 2 b2

11. Prove that

tan + sin sec + 1 = . tan sin sec 1

12. The angles of elevation of the top of a light house from two boats, in the opposite direction of light house, in the sea are 45 and 60. If the distance between the boats is 60 m, find the height of the light house. 6

Vous aimerez peut-être aussi